Covering letter, 2nd FAQ and questionnaire re the proposal to

Society of Critical Care
Anesthesiologists
Residents’ Guide
To Learning in the
Intensive Care Unit
The information presented in these guidelines was obtained by the Committee on Resident Education.
The validity of the opinions presented, drug dosages, accuracy and completeness of contents
are not guaranteed by SOCCA.
Copyright В© 2013 by the Society of Critical Care Anesthesiologists, Park Ridge, Illinois.
All rights reserved. Contents may not be reproduced without prior written permission of the publisher.
Fourth Edition.
Society of Critical Care Anesthesiologists
520 N. Northwest Highway
Park Ridge, IL 60068-2573
$25.00
ii
SOCCA Resident’s Guide to the ICU 2013
Contents
Preface to Fourth Edition
vi
Preface to Third Edition
vii
Preface to Second Edition
viii
Preface to First Edition
ix
Contributors
x
General ICU Topics
1
2
3
4
5
6
7
8
9
10
The ICU Experience
Sheri Berg, MD and Edward A. Bittner, MD, PhD
1
ICU Management
Lauryn Rochlen, MD
4
Family Support and Ethics Issues
Anthony Delacruz, MD, Sherif Afifi, MD, FCCM, FCCP, and Karen J.
Schwenzer, MD
12
Cardiopulmonary Resuscitation (CPR)
Hossam Tantawy, MD
17
Transport of the Critically Ill
David Boldt, MD and Adebola Adesanya, MB, MPH
21
Sedation of the Critically Ill
Hesham R. Omar, MD and Enrico M Camporisi, MD
25
Analgesia for the Critically Ill
Enrico M. Camporesi, MD, Devanand Mangar, MD and Naga
Pullakhandam, MD, FGTBA
30
Neuromuscular Blockade
Naga Pullakhandam, MD, FGTBA, Devanand Mangar, MD, and Enrico
M. Camporesi, MD
35
Acid-Base Balance
Khaldoun Faris, MD, and Nathanael Slater, DO
38
Metabolic and Endocrine Abnormalities
Hossam Tantawy, MD
46
iii
SOCCA Resident’s Guide to the ICU 2013
11
12
13
14
15
Fluid Management
Ronald W. Pauldine, M.D.
49
Gastrointestinal Bleeding
Maria A. De la PeГ±a, MD and Miguel Cobas, MD
56
Thromboembolic Disease
Gustavo Angaramo, M.D.
61
Bleeding in the ICU
Anthony Delacruz, MD
68
Specialized Nutrition Support in Critically Ill Patients
R. Dean Nava, Jr, MD
75
Monitoring Topics
16
17
18
19
Routine Monitoring in the ICU
Daniel R. Brown, MD, PhD
80
Rational Use of the Pulmonary Artery Catheter (PAC)
Gregory E. Kerr, MD, MBA, FCCM and James A. Osorio, MD
83
Pulse Wave Variability Monitoring
Lalitha Sundararaman, MD and Miguel A. Cobas, MD
88
Ultrasound and Echocardiography in the ICU
Abbas Al-Qamari, MD
93
Respiratory System Topics
20
21
22
23
24
Airway Management in the ICU
Todd W. Sarge, M.D.
98
Chronic Airway Management
Sherif Afifi, MD, FCCM, FCCP
105
Management of Mechanical Ventilation
Daniel W. Johnson, MD and Edward A. Bittner, MD, PhD
110
Lung Protection Strategies
Georges Cehovic, MD
119
Weaning from Mechanical Ventilation
Theofilos P. Matheos, MD and Stephen O. Heard, MD
122
iv
SOCCA Resident’s Guide to the ICU 2013
Cardiovascular System Topics
25
26
27
28
Shock and Support of the Failing Circulation
R. Eliot Fagley, MD and Michael Wall, MD, FCCM
128
Diagnosis and Treatment of Dysrhythmias
Maria L. Mendoza, MD and Jose L. Diaz-Gomez, MD
134
Diagnosis and Treatment of Myocardial Ischemia
Gerardo Rodriguez, MD
142
Valvular Heart Disease
Sriharsha D. Subramanya, MD and Jose Diaz-Gomez, MD
149
Sepsis and Infections Diseases Topics
29
30
31
32
The Systemic Inflammatory Response Syndrome (SIRS), Sepsis and
Multiple Organ Dysfunction Syndrome (MODS)
Shiva Birdi, MD and Marc J. Popovich, MD, FCCM
156
Infections and Antibiotic Therapy in the ICU
Ronald W. Pauldine, M.D.
168
Antibiotic Prophylaxis
Ronald W. Pauldine, M.D.
177
Management of the Immunocompromised Patient
Zdravka D. Zafirova, MD and Jennifer Hofer, MD
181
Neurologic Topics
33
34
35
Neurologic Critical Care
Bobby L. Tsang, MD and Ihab Dorotta, MD
187
Management of Increased Intracranial Pressure (ICP)
Lauryn Rocklen, MD
193
Traumatic Brain Injury
Scott Wolf, MD
199
Renal Topics
36
Renal Protection
Daniel R. Brown, M.D., Ph.D.
204
v
SOCCA Resident’s Guide to the ICU 2013
37
Renal Replacement Therapy
Shahriar Shayan, MD
207
Miscellaneous Topics
38
39
40
41
42
Toxicology and Support of Patients with Drug Overdoses
Matthew D. Koff, MD, MS
212
Solid Organ Transplantation
Zdravka D. Zafirova, MD and Jennifer Hofer, MD
216
Organ Donation and Procurement in the ICU
Jocelyn A. Park, MD
223
Trauma Management
Edgar J. Pierre, MD and Shawn M. Cantie, MD
225
Obstetric Critical Care
Gustavo Angaramo, M.D.
233
List of Tables and Figures
Answers
vi
SOCCA Resident’s Guide to the ICU 2013
Preface to the Fourth Edition
Many things have changed since the last edition was released.
Perhaps the most obvious is the name of the society. The society has long had strong
international connections, and after deliberating, we changed the name of the
organization to the Society of Critical Care Anesthesiologists - SOCCA for short.
In the short time since the last edition was published, we have seen a variety of
practices fall into and then out of favor, drugs be released and then withdrawn,
monitoring technologies proliferate and the electronic health record become an
overriding concern for all patient care. Even our options for publication are changing.
This is likely to be the last edition that is created as a static electronic document.
About half of the authors are new for this edition. Where they have revised older
chapters, the original authors are again acknowledged at the end of the chapter.
Sherif Afifi, M.D.
Chicago, Illinois
Miguel Cobas, M.D.
Miami, Florida
Christine A. Doyle, M.D.
Campbell, California
vii
SOCCA Resident’s Guide to the ICU 2013
Preface to the Third Edition
Intensivists have witnessed profound changes in the delivery of critical care medicine in
the last decade. A large number of randomized, prospective clinical trials or “before and
after” studies have impacted the way we practice critical care. Examples include the use
of drotrecogin alfa activated for the treatment of severe sepsis and septic shock; low
tidal volume ventilation in patients with acute lung injury and the acute respiratory
distress syndrome; implementation of ventilator and catheter “bundles” to reduce
nosocomial infections; and the use of alpha-2 agonists for sedation of the critically
patient. It was clear that the “Anesthesiology Residents’ Guide to Learning in the
Intensive Care Unit” was in need of revision. We and the American Society of Critical
Care Anesthesiologists (ASCCA) are pleased to provide this revised guide to
supplement the critical care reading material used by anesthesia residents and fellow.
The third edition has been expanded to include several important topics including
“Echocardiography”, “Traumatic Brain Injury” and “Organ Donation and Procurement in
the ICU”. Similar to the second edition, we have retained the short case presentations
and the self-study questions. The reading lists have been updated.
The majority of the authors who participated in the revision of this edition are new. We
would like to thank the previous authors who were either the original authors or who
helped revise the second edition. We have acknowledged their contributions at the end
of each chapter. The ASCCA is dedicated to timely revisions of this guide and plans are
already underway on the 4th edition.
Daniel Talmor, M.D.
Stephen O. Heard, M.D.
viii
SOCCA Resident’s Guide to the ICU 2013
Preface to the Second Edition
Since the publication date of the first edition in 1995, the scope and practice of critical
care medicine have continued to change. Examples of new issues encountered during
this time include controversies over pulmonary artery catheter use and pressures over
efficient management of critical care services as managed care demands are realized.
Additionally, during this time period, academic departments have grappled with
conflicting demands of resident education and efficient patient care in the new medical
marketplace. A primary mission of the American Society of Critical Care
Anesthesiologists is to assist anesthesiology residency programs to educate the future
perioperative physicians, who are today’s anesthesia residents and fellows. The
hospitalized patients of today are older and sicker than ever before, and many will travel
to locations in hospital where the primary medical caregiver will be an anesthesiologist.
With proper skills and training, these anesthesiologists will be the most appropriate
caregivers for these critically ill patients, helping maintain complex homeostasis by
thorough evaluation and management preceding, during, and following surgical
procedures.
The curriculum of the second edition was modified somewhat from the first edition by
members of the Resident Education Task Force of the American Society of Critical Care
Anesthesiologists in an attempt to bring “up-to-date” information to the hands of
residents who are caring for critically ill and injured patients. To this purpose, several
new sections have been added to the topical outline (e.g., ICU Management, Rational
Use of PA Catheters, Lung Protective Strategies); the bulk of the topic outline remains
as a presentation of the most important concepts of critical care. Short case
presentations have been added to provoke interest and increase relevance. Self-study
questions remain (with annotated answer keys) to allow the resident to evaluate his or
her understanding. Reading lists have also been annotated to emphasize the relevance
of most references.
Lucy A. Weston, Ph.D., M.D.
Acknowledgements:
Without the help and support of the members of the Resident Education Task Force,
this endeavor would not be possible. Many special thanks to Doug Coursin, Tom
Fuhrman, Gary Hoormann, and especially, Charlie Durbin, who have provided official
(and unofficial) technical, editorial, and emotional assistance and trusted this editor with
such a monumental task. Thanks to all contributors, particularly those working on new
sections and/or with tight timelines.
Many thanks to Michelle Smith for her secretarial efforts. She has been my third and
fourth hands.
ix
SOCCA Resident’s Guide to the ICU 2013
Preface to the First Edition
The field of critical care medicine continues to expand as new technologies are
developed and old technologies are refined. Anesthesiologists have been pioneers in
the development of critical care medicine. Changes in the health care environment have
led the American Society of Critical Care Anesthesiologists to define the
anesthesiologists as the perioperative physician. To achieve this role, anesthesia
residents must be competent in critical care medicine. Assessing critically ill patients
prior to operative procedures, management of intraoperative anesthetic care, treating
postoperative pain, and management of organ function after a surgical procedure is
best done by an anesthesiologist.
The members of the Resident Education Task Force of the American Society of Critical
Care Anesthesiologists have developed this curriculum guide to help residents achieve
competence in caring for the critically ill and injured patients. This guide is not meant to
be an exhaustive curriculum for an intensivist but useful for all residents completing
anesthesiology training. These were developed into outlines of the most important
concepts with a reading list for each. Additionally, self-study questions are included with
most sections to allow the resident to evaluate his or her current understanding.
This guide may also be used by residency program directors to evaluate or improve
their own curriculums. It is not our intention that all aspects of critical care mentioned in
the guide be part of the two month critical care rotation required by the American Board
of Anesthesiology. Other experiences, including cardiac anesthesia, consult services,
and postoperative recovery room would certainly contribute to mastery of these topics.
Charles G. Durbin, M.D., FCCM
x
SOCCA Resident’s Guide to the ICU 2013
Contributors
Adebola Adesanya, M.D., M.P.H., Associate Professor, Department of Anesthesiology
and Pain Management, University of Texas Southwestern Medical Center, Dallas, Texas
Sherif Afifi, M.D., F.C.C.M., F.C.C.P., Associate Professor, Department of
Anesthesiology and Critical Care Medicine, Northwestern University Feinberg School of
Medicine, Chicago, Illinois.
Abbas Al-Qamari, M.D., Assistant Professor, Department of Anesthesiology and Critical
Care Medicine, Northwestern University Feinberg School of Medicine, Chicago, Illinois.
Gustavo Angaramo, M.D., Assistant Professor, Department of Anesthesiology and
Critical Care Medicine, University of Massachusetts Medical School, Worcester,
Massachusetts
Sheri Berg, M.D., Instructor, Department of Anesthesia and Critical Care Medicine,
Massachusetts General Hospital, Boston, Massachusetts
Shiva Birdi, M.D., Staff Physician, Department of Anesthesiology, Cleveland Clinic,
Cleveland, Ohio
Edward A. Bittner, M.D., Ph.D., Assistant Professor, Department of Anesthesia and
Critical Care Medicine, Harvard Medical School, Boston Massachusetts
David Boldt, M.D., Clinical Fellow, Department of Anesthesiology and Pain
Management, University of Texas Southwestern Medical Center, Dallas, Texas
Daniel R. Brown, M.D., Ph.D., F.C.C.M., Associate Professor, Department of
Anesthesiology, Mayo Clinic, Rochester, Minnesota
Enrico M. Camporesi, M.D., Emeritus Professor, Department of Anesthesiology and
Critical Care, University of South Florida College of Medicine, Tampa, Florida
Shawn M. Cantie, M.D., Resident, Department of Anesthesiology, Miller School of
Medicine, University of Miami, Miami, Florida
Daniel Castillo, M.D., Assistant Professor of Clinical Anesthesiology, Miller School of
Medicine, University of Miami, Miami Florida
Georges Cehovic, M.D., Assistant Professor, Department of Anesthesiology and
Critical Care Medicine, Northwestern University Feinberg School of Medicine, Chicago,
Illinois
xi
SOCCA Resident’s Guide to the ICU 2013
Miguel A. Cobas, M.D., Associate Professor of Clinical Anesthesiology, Miller School of
Medicine, University of Miami, Miami Florida
Anthony Delacruz, M.D., Critical Care Medicine Fellow, Department of Anesthesiology
and Critical Care Medicine, Northwestern University Feinberg School of Medicine,
Chicago, Illinois
Maria De la Pena, M.D., Resident, Department of Anesthesiology, Miller School of
Medicine, University of Miami, Miami, Florida
Jose Diaz-Gomez, M.D., Staff Physician, Department of Cardiothoracic Anesthesiology,
Anesthesiology Institute, Cleveland Clinic, Cleveland, Ohio
Ihab Dorotta, M.D., Associate Professor, Department of Anesthesiology, Loma Linda
University and Medical Center, Loma Linda, California
Christine A. Doyle, M.D., Partner, Coast Anesthesia Medical Group, Campbell,
California
R. Eliot Fagley, M.D., Assistant Professor, Department of Anesthesiology, Washington
University School of Medicine, St. Louis, Missouri
Khaldoun Faris, M.D., Associate Professor, Department of Anesthesiology, University
of Massachusetts Medical School, Worcester, Massachusetts
Ala Sami Haddadin, M.D., F.C.C.P., Assistant Professor, Cardiothoracic Anesthesia
and Adult Critical Care Medicine, Yale University School of Medicine, New Haven,
Connecticut
Stephen O. Heard, M.D., Professor, Department of Anesthesiology, University of
Massachusetts Medical School, Worcester, Massachusetts
Jennifer Hofer, M.D., Critical Care Fellow, Department of Anesthesia and Critical Care,
The University of Chicago, Chicago, Illinois
Daniel W. Johnson, M.D., Instructor, Department of Anesthesia and Critical Care
Medicine, Massachusetts General Hospital, Boston, Massachusetts
Gregory E. Kerr, M.D., M.B.A., F.C.C.M., Associate Professor, New York Presbyterian
Hospital - Weill Cornell Medical College, New York, New York
Matthew D. Koff, M.D. M.S., Assistant Professor of Anesthesiology, Department of
Anesthesiology and Critical Care, Dartmouth Medical School and Dartmouth-Hitchcock
Medical Center, Lebanon, New Hampshire
xii
SOCCA Resident’s Guide to the ICU 2013
Isaac P. Lynch, M.D., Instructor, Department of Anesthesiology and Critical Care,
Washington University School of Medicine, St. Louis, Missouri
Devanand Mangar, M.D., Immediate Past Chief of Staff, Tampa General Hospital,
Tampa, Florida
Theofilos Matheos, M.D., Assistant Professor of Anesthesiology, University of
Massachusetts Medical School, Worcester, Massachusetts
R. Dean Nava, Jr., M.D., Instructor, Department of Anesthesiology and Critical Care
Medicine, Northwestern University Feinberg School of Medicine, Chicago, Illinois
Hesham R. Omar, M.D., Internal Medicine Department, Mercy Hospital and Medical
Center, Chicago, Illinois
James A. Osorio, M.D., Assistant Professor, New York Presbyterian Hospital, Weioll
Cornell Medical College, New York, New York
Jocelyn A. Park, M.D., Staff Anesthesiologist, Sacred Heart Medical Center,
Springfield, Oregon
Ronald W. Pauldine, M.D., Clinical Professor & Chief of Critical Care Medicine,
Department of Anesthesiology and Pain Medicine, University of Washington School of
Medicine, Seattle, Washington
Edgar J. Pierre, M.D., Associate Professor, Department of Anesthesiology, Miller
School of Medicine, University of Miami, Miami, Florida
Marc Popovich, M.D., F.C.C.M., Staff Physician, Department of Anesthesiology,
Cleveland Clinic, Cleveland, Ohio
Naga Pullakhandam, M.D., Department of Anesthesiology and Critical Care Medicine,
University of South Florida College of Medicine, Tampa General Hospital, Tampa,
Florida
Lauryn R. Rochlen, M.D., Clinical Lecturer, Department of Anesthesiology, University
of Michigan, Ann Arbor, Michigan
Gerardo Rodriguez, M.D., Assistant Professor, Department of Anesthesiology, Boston
University School of Medicine, Boston, Massachusetts
Karen J. Schwenzer, M.D., Associate Professor of Anesthesiology and Critical Care
Medicine, Department of Anesthesiology, University of Virginia Health System,
Charlottesville, Virginia
xiii
SOCCA Resident’s Guide to the ICU 2013
Shahriar Shayan, M.D., Assistant Professor, Department of Anesthesiology and Critical
Care Medicine, Northwestern University Feinberg School of Medicine, Chicago, Illinois
Nathanael Slater, D.O., Critical Care Fellow, Department of Anesthesiology, University
of Massachusetts Medical School, Worcester, Massachusetts
Sriharsha Subramanya, M.D., F.R.C.A., Resident, Department of Cardiothoracic
Anesthesiology, Anesthesiology Institute, Cleveland Clinic, Cleveland, Ohio
Hossam Tantawy, MD., Assistant Professor of Anesthesiology, Yale University School
of Medicine, New Haven, Connecticut
Bobby L. Tsang, M.D., Resident, Department of Anesthesiology, Loma Linda University
and Medical Center, Loma Linda, California
Michael Wall, M.D., F.C.C.M., Professor of Anesthesiology and Cardiothoracic Surgery,
Washington University School of Medicine, St. Louis, Missouri
Scott Wolf, M.D., Assistant Professor, Department of Anesthesiology and Critical Care
Medicine, Northwestern University Feinberg School of Medicine, Chicago, Illinois
Zdravka D. Zafirova, M.D., Assistant Professor, Department of Anesthesia and Critical
Care, The University of Chicago, Chicago, Illinois
xiv
SOCCA Residents Guide 2013
1. The ICU Experience
Sheri Berg MD and Edward A. Bittner MD, PhD
A 79 year old male, who is postoperative day 3 after a colectomy for colon cancer, is transferred to the
intensive care unit (ICU) with abdominal pain, fever to 102, increasing WBC count, and hypotension. In
the ICU he is resuscitated with intravenous fluids and a vasopressor is started for blood pressure support.
His signs and symptoms are consistent with septic shock from an intra-abdominal source and broad
spectrum antibiotics are administered. An abdominal CT is obtained which reveals an intra-abdominal
abscess with a probable anastomotic leak. He is taken to the OR for an exploratory laparotomy, abscess
drainage and anastomotic repair. Postoperatively he is readmitted to the ICU for ongoing treatment of
septic shock.
The practice of anesthesiology has evolved into a medical specialty that not only provides traditional intraoperative care, but plays an integral role in entire spectrum of perioperative medicine. This includes caring
for patients who are critically ill. Anesthesiologists, who have pursued advanced training in critical care
medicine, are exceptionally well prepared to care for critically ill patients.1 The expertise of the critical
care anesthesiologist is of particular benefit to patients in the perioperative period; however, the critical
care anesthesiologist’s specialized skills, knowledge and judgment are valuable to all critically ill patients.
Anesthesiologists played a major role in the creation of critical care medicine; however they only represent
a small portion of critical care providers in the United States.2 The demand for critical care providers will
likely increase as the US population ages, and as external forces exerts pressure for intensivists to direct
care for all critically ill patients. These factors will provide an opportunity for anesthesiologists to regain
presence in ICUs across the country.
Critical care medicine is a multidisciplinary and collaborative effort. The Society of Critical Care Medicine
(SCCM) maintains that the best care for the critically ill is provided by an integrated team of dedicated
experts directed by a trained physician credentialed in critical care medicine.3 The core of the ICU team
consists of intensivists, critical care nurses, respiratory therapists, and pharmacists. In addition, primary
care physicians, consultant medical specialists (such as cardiologists and nephrologists), physician
assistants, nurse practitioners, social workers, dieticians, ethicists, and other professionals are often part of
the team.3 The ICU team works collaboratively utilizing the unique expertise of team members, to
provide timely, safe, effective and efficient patient centered care.
The multidisciplinary, team based approach that encompasses patient care in intensive care units provides
an ideal environment for the resident in anesthesiology to develop and master the six core competencies of
medical education as defined by the ACGME (patient care, medical knowledge, practice based learning and
improvement, interpersonal and communication skills, professionalism, and systems based practice).2 In
addition, the ICU affords residents an excellent opportunity to practice evidence based medicine, which
requires the integration of the best research evidence and clinical expertise with unique patient values and
circumstances for optimal clinical decision making.
During the ICU rotation, the resident should take advantage of the opportunity to learn the important
aspects of critical care medicine and refine their technical skills. The ICU offers abundant opportunities to
develop communication skills through presentations on rounds, engaging in conversations with other
members of the care team and comforting families. The ICU also provides an opportunity to observe, and
possibly participate in, the challenges of helping patients and families making end of life care decisions. In
circumstances where continued care is not desired, deemed futile and perhaps inappropriate, the intensivist,
in conjunction with other care providers, will be available and able to discuss the ethical withdrawal of
supportive care.
Traditionally ICU’s have been classified as closed or open, depending on whether or not the ICU team has
“total control” of the care of the patients, or they act as consultants, with the primary physician being the
one in “charge”. These are now obsolete concepts as the literature supports the “team approach”, in which
a collaborative effort is undertaken in order to provide integrated patient centered medical care. 2 The team
based approach has been shown to improve outcomes in the ICU..3,4 An additional benefit of the integrated
team based approach to ICU care is a significant cost saving for the hospital. This cost saving is important
considering that the ICU consumes a large portion of hospital resources.
1
SOCCA Residents Guide 2013
The practice of critical care medicine does not adhere to “normal working hours”. In some centers, ICU’s
are staffed with an in-house intensivist 24 hours a day, 7 days a week. High-intensity intensivist staffing
models such as “24-7 coverage” have been associated with improved staff satisfaction, enhanced
compliance with evidence based care, decreased ICU complication rates and reduced hospital length of
stay.3,4,5 The explanation for improved outcome with this high intensity staffing includes the intensivists'
availability and skills to prevent, recognize early, and treat life-threatening complications.9 In addition,
improved continuity of care and close attendance to patients may also bolster improved patient and family
satisfaction.
Despite the evidence that high-intensity intensivist staffing improves patient outcome in the ICU, the
majority of the ICUs in the US provide low-intensity or no intensive care coverage.8 Due to a shortage of
intensivists, high intensity staffing is currently not possible in all ICUs and the imbalance between the
demand for and supply of intensivists is expected to worsen in the future. Alternatives such as ICU
telemedicine have been introduced to fill this void. 3 Opportunities for our specialty to provide care to the
sickest patients in the hospital will continue to grow both within and outside of the operating room. Even
if critical care is not your subspecialty career path, the experience of caring for critically ill patients in the
ICU makes an anesthesiologist a better perioperative physician.
The chapter is a revision of the original chapter authored by Ruben J. Azocar, MD
REFERENCES:
1.
2.
3.
4.
5.
6.
7.
American Society of Anesthesiologists. Guidelines for the practice of critical care by anesthesiologists.
Approved October 27, 2004, last affirmed October 21, 2009.
Hanson CW 3rd, Durbin CG Jr, Maccioli GA, et al. The anesthesiologist in critical care: past, present, and
future. Anesthesiology 2001; 95:781-8.
http://www.sccm.org/AboutSCCM/Mission/Pages/default.aspx.(last accessed 8/30/2010).
http://www.acgme.org/outcome/comp/compmin.asp.(last accessed 8/30/2010).
Pronovost PJ, Holzmuller CG, Clattenburg L, et al. Team care: beyond open and closed intensive care
units. Curr Opin Crit Care 2006;12:604-8.
Pronovost PJ, Angus DC, Dorman T, et al. Physician staffing patterns and clinical outcomes in critically
ill patients: a systematic review. JAMA 2002;288:2151-62.
Reader TW, Flin R, Mearns K, Cuthbertson BH. Developing a team performance framework for the
intensive care unit. Crit Care Med 2009;37:1828-9.
8.
9.
Gajic O, Afessa B. Physician staffing models and patient safety in the ICU. Chest 2009;135:1039-44.
Fuchs RJ, Berenholtz SM, Dorman T. Do intensivists in ICU improve outcome? Best Prac Res Clin
Anaesthesiol 2005;19:125-35.
10. Gajic O, Afessa B, Hanson AC, et al. Effect of 24-hour mandatory versus on-demand critical care
specialist presence on quality of care and family and provider satisfaction in the intensive care unit of a
teaching hospital. Crit Care Med 2008;36:36-44.
11. Sapirstein A, Lone N, Latif A, Fackler J, Pronovost PJ, Tele-ICU: paradox or panacea? Best Prac Res Clin
Anaesthesiol 2009;23:115-26.
12.
QUESTIONS:
1.1 The
A.
B.
C.
D.
best approach for ICU care is:
Closed ICU
Open ICU
Semi-closed
Team approach
1.2 The
A.
B.
C.
D.
dedicated ICU team provides all of the following benefits except:
Decreases mortality
Decreases length of stay
Decreases staff satisfaction
Decreases resident education
2
SOCCA Residents Guide 2013
1.3 The
A.
B.
C.
D.
following statements regarding ICU care are true except:
Anesthesiologists play a major role in perioperative medicine
Anesthesiologists played a major role in the creation of critical care medicine
In the future, there will likely be too many critical care providers and not enough patients
Telemedicine offers an alternative to having an intensivist in an ICU 24/7
1.4 In the ICU, residents can:
A. learn the important aspects of critical care
B. develop their communication skills
C. master all six of the core competencies as defined by the ACGME
D. all of the above
3
SOCCA Residents Guide 2013
2. Intensive Care Unit(ICU) Management
Lauryn R. Rochlen, MD
A 55 year-old male with no significant past medical history is admitted to the ICU for acute
upper GI bleeding. He subsequently develops Transfusion Related Acute Lung Injury
following transfusion of 1 unit of packed red blood cells and requires intubation and
mechanical ventilation. He is successfully weaned from mechanical ventilation and
extubated 3 days later. The ICU team determines he is stable for transfer to a general care
floor, however his primary care physician is concerned and would like him to remain in the
ICU for one more day.
I.
Structure
A. Staffing models(1)
1. Closed vs. open ICU
a) Closed ICU: Intensivist is responsible for day-to-day management
b) Open ICU: Primary physician is responsible for day-to-day management; may be
no full time intensivist, or intensivist may involved at the discretion of the
primary physician
2. High-intensity vs. low-intensity ICU
a) High-intensity
(1) Intensivist is responsible for all patient care in a closed ICU
(2) Mandatory intensivist consultation(open ICU but it is mandatory to consult
the intensivist
b) Low-intensity
(1) Elective intensivist consultation
(2) No intensivist available
c) Intermediate-intensity
(1) <80% or patients managed by an intensivist
d) High-intensity staffing is associated with lower hospital mortality and reduced
ICU and hospital lengths of stay
(1) Availability of intensivists and necessary skills to prevent, recognize and
treat life-threatening complications
(2) Patients are more likely to receive evidence-based care
3. Choice vs no-choice ICU
a) Choice: open ICU and consulting intensivist is at the discretion of the primary
physician
b) No choice
(1) Closed ICU or mandatory intensivist consult
(2) No intensivist available
B. Organization of multi-disciplinary team(2, 3)
1. Intensivist
a) Favorable impact on ICU and hospital length of stay
b) Associated with decreased costs
c) Estimated that 30-50% fewer patients would die if an intensivist rounded daily on
all critically ill patients
d) Optimizing medical, psychological and economic outcomes
2. Nurses
a) Direct influence on preventable adverse events(ie, self-extubation, central venous
catheter infections)
(1) Staffing levels(nurse-to-patient ratios)
(a) Ideal ratio undetermined
4
SOCCA Residents Guide 2013
(2) Level of experience
3. Pharmacist
a) Many errors that occur in the ICU are related to drug choice, dose or delivery
method
b) Preventable adverse drug events are at least 2 times likely to occur in the ICU
compared to a general care floor
c) Addition of a pharmacist to the ICU care team has reduced the rate of adverse
drug related events
d) Other advantages include: cost savings of using equivalent and cheaper
medications, optimizing dosing intervals, reducing the duration of sedation and
occurrence of drug-induced coma, identifying drug interactions
4. Respiratory Therapist
a) Use of patient-driven protocols for withdrawal of ventilatory support
b) Proven benefit of non-invasive ventilation
c) Identification and management of risk factors for ventilator associated
pneumonia
C. Leapfrog Initiative(1, 3-5)
1. Leapfrog Group
a) Fortune 500 Company of healthcare providers and purchasers
b) Controls billions of dollars of healthcare expenditures
(1) ICU care in the US accounts for ≥20% of acute care hospital costs and >90
billion annually
2. ICU physician staffing initiative
a) Major initiative to improve patient safety
b) Evidence that Leapfrog staffing model improves outcomes and saves cost to
hospitals
(1) Potential to save >50,000 lives annually in the United States
3. ICU physician staffing model
a) Intensivists are present during daytime hours and provide care exclusively in the
ICU
b) When intensivists are not present on site or via telemedicine:
(1) Return pages within 5 minutes at least 95% of the time
(2) Arrange for FCC(fundamentals of critical care certified)- physician or
physician extender to reach the patients at least 95% of the time
4. Compliance
a) Estimated that 4-10% of ICU’s in United States meet Leapfrog Initiative criteria
for intensivist staffing
b) Barriers include: Shortage of intensivists, changing practice beliefs
D. Regionalization(6)
1. Consolidating scarce and expensive resources
2. System already in place for trauma and burn patients and neonatal intensive care
a) Evidence of improved outcomes for these patient populations
E. Technology
1. Telemedicine(6)
a) Use of supplemental monitors and management of ICU patients via telemedicine
has been shown to improve survival and decrease ICU length of stay in tertiary
centers without a full-time on-site intensivist.
II.
III. Processes
A. Admission criteria(7)
1. Due to utilization of expensive resources, ICU admission should be reserved for
patients with reversible medical conditions who have a reasonable prospect of
substantial recovery.
5
SOCCA Residents Guide 2013
2. Severity of Illness Indices(see below) have not been validated as preadmission
screening tools.
3. Prioritization Model
a) Priority 1: Critically ill, unstable patients in need of intensive treatment and
monitoring that cannot be provided outside of the ICU(ie, mechanical
ventilatory support, continuous infusions)
b) Priority 2: Patients who require intensive monitoring, and may require immediate
intervention
c) Priority 3: Unstable patients with reduced likelihood of recovery
d) Priority 4: Patients not appropriate for ICU admission
(1) No anticipated benefit from ICU based on low risk interventions that can be
implemented outside the ICU
(2) Terminal and irreversible illness facing imminent death
4. Diagnosis Model
a) Cardiac: Acute MI with complications; Cardiogenic shock; Complex
arrhythmias; Acute CHF with respiratory failure and/or requiring hemodynamic
support; Hypertensive emergencies; Unstable angina; S/p cardiac arrest; Cardiac
tamponade or constriction with hemodynamic instability; Dissecting aortic
aneurysm; Complete heart block
b) Pulmonary: Acute respiratory failure requiring ventilator support; PE with
hemodynamic instability; Respiratory deterioration; Nursing/Respiratory care
requirements not available on other unit; Massive hemoptysis; Imminent
intubation
c) Neurologic: Acute stroke with altered mental status; Coma(metabolic, toxic,
anoxic); Intracranial hemorrhage with potential for herniation; Acute
subarachnoid hemorrhage; Meningitis with altered mental status or respiratory
compromise; CNS or neuromuscular disorder with deteriorating neurologic or
pulmonary function; Status epilepticus; Brain-dead or potentially brain-dead
patients while determining organ donation status; Vasospasm; Severe head
injury
d) Drug ingestion and overdose: Hemodynamically unstable; Altered mental status
and inadequate airway protection; seizures
e) GI: Life threatening GI bleeding(hypotension, angina, continued bleeding,
comorbidities); Fulminant hepatic failure; Severe pancreatitis; Esophageal
perforation with or without mediastinitis
f) Endocrine: DKA complicated by hemodynamic instability, altered mental status,
respiratory insufficiency or severe acidosis; Thyroid storm or myxedema coma
with hemodynamic instability; Adrenal crises with hemodynamic instability;
Severe electrolyte disturbances with altered mental status or hemodynamic
instability
g) Surgical: Postoperative patients requiring hemodynamic monitoring, ventilator
support and/or extensive nursing care
h) Miscellaneous: Septic shock with hemodynamic instability; Hemodynamic
monitoring; Conditions requiring ICU-level nursing care; Environmental
injuries; New/experimental therapies with potential for complications
5. Objective Parameters Model
a) Vital Signs
(1) Pulse <40 or >150 beats/min
(2) Systolic BP <80mmHg or 20mmHg below patients baseline
(3) MAP<60mmHg
(4) Diastolic BP>120mmHg
(5) Respiratory rate >35 breaths/min
b) Laboratory values(acute)
(1) Serum sodium <110 mEq/L or >171mEq/L
6
SOCCA Residents Guide 2013
(2) Serum potassium <2mEq/L or >7mEq/L
(3) PaO2 <50mmHg
(4) pH<7.1 or >7.7
(5) Serum glucose >800mg/dL
(6) Serum calcium >15mg/dL
(7) Toxic level of drug or other chemical
c) Radiography/Ultrasonography/Tomography(acute)
(1) Cerebral vascular hemorrhage, contusion or subarachnoid hemorrhage with
altered mental status or focal neurologic signs
(2) Ruptured viscera, bladder, liver, esophageal varices or uterus with
hemodynamic instability
(3) Dissecting aortic aneurysm
d) Electrocardiogram
(1) MI with complex arrhythmias, hemodynamic instability or CHF
(2) Sustained ventricular tachycardia or ventricular fibrillation
(3) Complete heart block with hemodynamic instability
e) Physical findings(acute)
(1) Unequal pupils in an unconscious patient
(2) Burns covering >10% body surface area
(3) Anuria
(4) Airway obstruction
(5) Coma
(6) Continuous seizures
(7) Cyanosis
(8) Cardiac tamponade
B. Triage(7)
1. Initial triage based on prioritization admission model
2. ICU Director has the responsibility and authority to admit/discharge patients
3. Triage decisions may be made with patient consent
4. Triage decisions may be made despite anticipated untoward outcome
C. Discharge criteria(7)
1. Physiologic status has stabilized
2. Physiologic status has deteriorated and active interventions are no longer planned
D. Severity of Illness Indices(8)
1. Acute Physiology and Chronic Health Evaluation (APACHE)
a) Objective: Risk stratification
b) APACHE II
(1) Scoring
(a) 12 Routine physiologic measurements plus age and previous health
status
(b) Scoring is based on most abnormal values in first 24hours in the ICU
(c) Coefficients to adjust for 45 different operative and non-operative
diagnostic categories
(2) Limitations
(a) Does not control for pre-ICU management
(b) Not intended to assess outcome in any one specific ICU diagnosis( ie,
sepsis)
c) APACHE III
(1) Differences
(a) Impact of treatment time and location before ICU admission
(b) Disease categories increased to 78(from 45)
(c) Outcome predictions(AIDS, leukemia/multiple myeloma,
immunocompromised state, lymphoma, solid tumor with metastasis,
hepatic failure and cirrhosis)
7
SOCCA Residents Guide 2013
(d) Predictive equation – Score and proprietary reference disease data to
determine individual risk estimates
(2) Sequential scoring can update the risk estimate
(a) Single most important factor in determine daily risk of in-hospital
mortality is the updated APACHE III score
(3) Used for predicting ICU mortality, length of stay, need for interventions and
nursing workload
2. Mortality Probability Models(MPM)
a) Objective: assign weights to variables predicting hospital mortality
b) Scoring
(1) Uses data at ICU admission and the end of first 24-hour period
(2) 15 variables for admission data, 5 of those plus 8 additional for 24-hour
data
(3) Includes age and chronic health
(4) Available at ICU admission
(5) Independent of ICU treatment
(6) Does not require specifying a diagnosis
c) Limitations
(1) Based on older data
3. Simplified Acute Physiology Scores I and II(SAPS)
a) Objective/Outcome measure: vital status at hospital discharge
b) Scoring
(1) Most abnormal variables in first 24 hours of ICU admission
(2) 17 variables
(3) Does not require specifying a diagnosis
c) Limitations
(1) Based on older data
4. Uses
a) Performance assessment
(1) Quality improvement and benchmarking
b) Predicting and planning resource utilization
c) Clinical research
d) Individual patient management
5. Comparisons between models
a) Each scoring system excludes pediatric, burn, coronary and cardiac surgery
patients from their databases
b) Newer versions of each index have been shown to be sufficient for assessing
prognosis, comparing ICU performance and stratifying patients for clinical trials
c) No consistent accuracy-of-outcome prediction between different trials
d) No consistent method of calibration between different trials
6. Pitfalls
a) Misuse – prognosis, interventions
b) Application to databases and statistical analysis– data entry errors,
misapplication, use of mortality as sole criterion of outcome, failure to account
for sample size
c) Timing of ICU admission is not standardized
E. Bundles
1. A group of interventions that when implemented together act synergistically to
improve outcome
2. Ventilator-Associated Pneumonia
a) Elevate head of bed >30В°
b) GI ulcer prophylaxis
c) DVT prophylaxis
d) Daily sedation holiday and assessment for extubation
8
SOCCA Residents Guide 2013
3. Central line
a) Hand hygiene
b) Maximum barriers during insertion
c) Chlorhexidine
d) Daily assessment for line necessity and removal as soon as possible
4. Sepsis
a) 2008 Surviving Sepsis Campaign Guidelines(9)
b) Sepsis Resuscitation Bundle – completed within 6 hours
(1) Measure and follow serum lactate levels
(2) Obtain blood cultures prior to antibiotic administration
(3) Begin empiric broad-spectrum antibiotic coverage
(4) Treat hypotension with fluids
(5) Use vasopressors for ongoing hypotension
(6) Maintain adequate central venous pressure
(7) Maintain adequate central venous oxygen saturation
c) Sepsis Management Bundle – completed within 24 hours
(1) Low-dose steroids
(2) Activated Protein C
(3) Glycemic control
(4) Low to normal inspiratory plateau pressures
IV.
V. Outcomes & Quality improvement process(10)
A. Mortality/Length of Stay
1. Predictors
a) Pre-admission quality of life
b) Illness severity scores
2. Focus for new interventions
3. Focus for cost analyses
B. Quality of Life
1. High incidence of post-traumatic stress disorder following critical illness
2. Delirium from sedation/critical illness affects independence following discharge
C. Economics(10, 11)
1. Economic evaluation
a) Cost-minimization: compares costs of 2 options, assuming they have the same
outcome
b) Cost-effectiveness: compares the cost per unit of effectiveness of 2 or more
options that have different outcomes
c) Cost-utility: compares the cost per quality-adjusted life-year(QALY)
2. Cost-benefit: compares the costs and benefits of 2 or more options that have varying
outcomes Costs of drug administration or therapy
a) Direct: cost of staff, purchase of drug or equipment
b) Indirect: patient’s loss of income
c) Intangible: patient’s pain or distress
3. Patient-related costs
a) Mechanically ventilated patients cost more than non-ventilated patients
(1) Need to analyze patient outcome and improve the value of mechanical
ventilation
b) Septic patients cost more than non-septic patients
(1) Sepsis bundles can improve mortality rates and decrease hospital cost
4. Staffing
a) Accounts for average of 56% of ICU costs
b) Balance between resources and providing optimal patient care
9
SOCCA Residents Guide 2013
Discussion:
The economic and logistical burden of caring for critically ill patients in the United States
is will continue to increase. The ideal team model and structure for the ICU remain controversial
and may vary depending on local practice. In the current environment where there is great
pressure to decrease healthcare expenditures, it will become even more vital to deliver efficient
and evidence-based care. Recommendations and guidelines such as those put forth by the
Leapfrog Initiative will be helpful in achieving these goals.
References:
1.
Gajic O, Afessa B. Physician staffing models and patient safety in the ICU. Chest. 2009 Apr;135(4):
1038-44.
2. Durbin CG, Jr. Team model: advocating for the optimal method of care delivery in the intensive care unit.
Crit Care Med. 2006 Mar;34(3 Suppl):S12-7.
3. Pronovost PJ, Needham DM, Waters H, Birkmeyer CM, Calinawan JR, Birkmeyer JD, et al. Intensive care
unit physician staffing: financial modeling of the Leapfrog standard. Crit Care Med. 2004 Jun;32(6):
1247-53.
4. Angus DC, Clermont G, Linde-Zwirble WT, Musthafa AA, Dremsizov TT, Lidicker J, et al. Healthcare costs
and long-term outcomes after acute respiratory distress syndrome: A phase III trial of inhaled nitric
oxide. Crit Care Med. 2006 Dec;34(12):2883-90.
5. Manthous CA. Leapfrog and critical care: evidence- and reality-based intensive care for the 21st century.
Am J Med. 2004 Feb 1;116(3):188-93.
6. Fink MP, Suter PM. The future of our specialty: critical care medicine a decade from now. Crit Care Med.
2006 Jun;34(6):1811-6.
7. Guidelines for intensive care unit admission, discharge, and triage. Task Force of the American College of
Critical Care Medicine, Society of Critical Care Medicine. Crit Care Med. 1999 Mar;27(3):633-8.
8. Fink MP, Abraham E, Vincent J-L, Kochanek PM. Textbook of Critical Care. Fifth ed. Philadelphia:
Elsevier Saunders; 2005.
9. Dellinger RP, Levy MM, Carlet JM, Bion J, Parker MM, Jaeschke R, et al. Surviving Sepsis Campaign:
international guidelines for management of severe sepsis and septic shock: 2008. Crit Care Med. 2008
Jan;36(1):296-327.
10. Barbieri C, Carson SS, Amaral AC. Year in review 2007: Critical Care--intensive care unit management.
Crit Care. 2008;12(5):229.
11. Pittoni GM, Scatto A. Economics and outcome in the intensive care unit. Curr Opin Anaesthesiol. 2009
Apr;22(2):232-6.
Questions:
A=1,2,3
B=1,3
C=2,4
D=4
E=all of the above
2.1 Which of the following are part of the ventilator-associated pneumonia bundle:
1.
2.
3.
4.
Elevate the head of bed >30В°
Daily sedation holiday
GI ulcer prophylaxis
DVT prophylaxis
2.2 Which of the following are models for ICU admission criteria:
1.
2.
3.
4.
Objective parameters model
Diagnosis model
Priority model
Insurance model
10
SOCCA Residents Guide 2013
2.3 Which of the following are recommendations set forth by the Leapfrog Initiative:
1.
2.
3.
4.
An intensivist must be in the hospital 24/7
If not present, intensivist must return pages within 5 minutes 95% of the time
All ICUs must be “closed” units
If an intensivist is not always present, a critical-care certified physician or physician extender must
respond at least 95% of the time
11
SOCCA Residents Guide 2013
3. Family Support and Ethics Issues
Anthony delaCruz, MD; Sherif Afifi, MD; Karen J. Schwenzer, MD
A 72-year-old man was taken to the emergency department by his unmarried
cohabitant of over 20 years for chest pain. The patient was found to have an
ascending aortic aneurysm. He consented to emergency surgical repair.
Operative repair of the aneurysm was complicated by stroke. Indefinite need
for ventilatory support is anticipated and the patient is likely to benefit from
tracheostomy. The patient is unable to consent for himself, but no advanced
directives or power of attorney was established prior to these events. The
patient has an adult son from a previous relationship, but no other blood
relatives. When contacted, the son stated he has not spoken to his father in
years, but was willing to consent for the tracheostomy. The patient’s
unmarried cohabitant claims that the patient’s verbal advanced directives
were not to be kept alive if he would never be free from a ventilator.
Decision making in the ICU is a complex and dynamic process because it often invokes clinical ethics and
end of life issues. Ethical consideration and incorporation in the practice of Critical Care is crucial in order
to improve the care of patients and family through the transition of critical illness or through the dying
process.
Consider the basics of medical ethics as it pertains to patient care, the nuances in applying these ethics, and
considerations through death and dying.
I.
Tenets of Medical Ethics in Patient Care
A. Autonomy
1. Non-coercion
2. Patient’s capacity for understanding and decision making
3. Parent-child relationship
4. Family role as support, advisor of surrogate
5. Informed Consent
B. Beneficence
1. Intent of doing good
2. Preservation of life
3. Curing and Healin
4. Preservation and restoration of quality of lif
5. Disease and injury preventio
6. Relieving and alleviating suffering
C. Nonmaleficence
1. Avoiding unnecessary procedures
2. Withdrawal of death delaying suppor
3. Futility
4. Minimizing risk
5. Double Effect
D. Justice
1. Resource allocation
2. Stewardship
3. Relationship of managed care organization, physician and patient
4. Futility
E. Dignity
1. Respectful treatment of the patient
2. Patient cultural and/or religious variations
3. End-of-Life goals
4. Confidentiality and Privacy
F. Truthfulness and Honesty
1. Mutual trust between patient and physician
12
SOCCA Residents Guide 2013
2.
3.
4.
5.
6.
II.
Professional integrity of the physician
Disclosing uncertainty and futility
Disclosing therapeutic errors
Disclosure vs. information transfer
Benevolent deception
Nuances in Ethics
A. Cultural difference and language barriers
1. Deferring all medical decisions to the physician
B. Spirituality is not equivalent to Religion
C. Decision making for incapacitated patients
1. Substituted judgment standard
a) patient designated proxy
b) durable power of attorney of health care decisions
2. Best interest standard
a) family members vs. unmarried cohabitants
3. State defined legal hierarchy
a) next of kin
b) legally appointed guardian
D. Decision making for minors
1. Parents are surrogate decision makers
2. Consent vs. Assent in older children
3. State and Federal law
4. Withdrawal of life sustaining interventions
5. Imposing Religious practices on children that may restrict beneficial therapy
E. Patient and Family-Centered Care and Decision Making
1. Aids in resolving disputes
2. Sources of guidance for surrogate decision making
a) written advance directives
b) living will
c) verbal advanced directives
F. Conflict resolution
1. Futility of requested treatment
2. Institutional support mechanisms
a) Institutional Ethics Committee
b) Clergy, Chaplains
c) Social workers
d) Legal council
e) Patient Relations Committee
G. Informed Consent
1. Informational Elements
a) current and suspected diagnoses
b) nature and purpose of proposed procedure or treatment
c) explanation of risks and benefits
d) alternatives to proposed procedure or treatment
e) consequences of no treatment
2. Consent elements
a) voluntary consent
b) capacity to consent
c) assent vs. consent
d) understanding of the informational elements
H. Ethics vs. Statutes
1. Unlawful practices may be unethical
a) euthanasia
2. Lawful practices are not all ethical
a) physician assisted suicide
b) physician involvement in lethal injection
13
SOCCA Residents Guide 2013
III.
End-of-Life Care
A. Withholding and withdrawing of life support are equivalent
B. Allowing to die is not killing
C. Double Effect is imminent and recognizable
1. The good intention vs. foreseen consequences
2. Narcotics intended for pain relief may hasten death
D. Do Not Resuscitate (DNR) is an order made by a physician
1. Is not a transition from curing goals to comfort care
2. Good-quality care may be hindered by refusal of specific life sustaining treatment
a) DNR but not Do Not Intubate
b) refusing endotracheal intubation but accepting escalating non-invasive ventilation
E. Withdrawal of life-sustaining intervention is not withdrawal of care
F. Technique variation in discontinuing life sustaining therapy
G. Terminal weaning of non-comfort measures is not advisable
1. Prolongs dying process
2. contributes to patient distress
IV.
Palliative Care
A. Symptom and pain relief
B. Palliation occurs regardless of stage of illness
C. Transition from curing to palliation
D. Distress during terminal extubation
1. Anticipatory dosing of narcotics vs. titration only
2. Double effect palliative care
V.
Time of Death
A. Notification of death
1. Should be clear and concise
2. Avoid euphemisms
3. Physician as the leader of bereavement and support
a) nurses
b) chaplain
c) social worker
B. Referral to Organ Procurement Organization
1. Ensure integrity of the process
2. Consider cultural and religious variation
3. Non-consented exams and therapies after the decision to donate
4. Local statutes for mandatory referral
C. Referral for autopsy or medical examiner’s case
Discussion
As seen in the case above, family support and ethical issues in the context of patient care needs to be
continuously re-evaluated based on the patient’s current condition. Use the following discussion as a
proposed guide on how to address family support and ethics using Patient and Family-Centered Care and
Decision Making during this patient’s admission.
The patient’s autonomy changed and the decision making process has become more complex since the day
one of admission. No power of attorney was established prior to the patient becoming incapacitated.
Consider the nuances in decision making for incapacitated patients. According to state defined legal
hierarchy, the patient’s son would make decisions for him. However, according to the substituted judgment
standard, the patient’s partner may be able to answer, “what would the patient want in this situation?”
Discussion with both parties stressing decision-making according to the values of the patient will help
alleviate conflict.
Agreement has been made to transition from goals of a cure to palliation. Consider what resources your
institution has in place to aid in this transition. Involvement of chaplains, social workers and other
members of the team can help during the process. Chaplains may have both more time and experience in
dealing with families with dying loved ones. Social workers can also help with after death arrangements.
The role of a physician at this time remains as a leader and coordinator of these resources.
Agreement has been made to withdraw life-sustaining treatment. The patient has needed intravenous
14
SOCCA Residents Guide 2013
infusion including: fentanyl, norepinephrine, normal saline, and antibiotics. Consider the futility and need
for some of these medications in the role of palliation. Pressors, antibiotics and resuscitative fluids do not
aid in pain relief or anxiolysis. Therefore non-palliative therapies can be discontinued without weaning. In
this patient, an anxiolytic may be added if distress is anticipated. The Doctrine of Double Effect recognizes
that therapies of palliation risk hastening death.
According to state law where this patient will die, a referral must be made to an organ procurement
organization (OPO). The OPO may request some exams and labs. Consider how this may affect the
palliation process. How would non-consented therapies or exams violate the patient’s autonomy? It is the
physician’s responsibility to ensure integrity of the organ donation referral process and the dignity of the
patient. According to the Uniform Anatomical Gift Act revision of 2006 therapies and exams requested by
the OPO are acceptable unless the patient had advanced directives specifically refusing these for the
purpose of making an anatomical gift.
Conclusion
The practice of ethics and the treating of patients and families is the art of medicine. Adhering to the tenets
of medical ethics may be the most challenging aspects of critical care. The litany of nuances regarding the
care of patients and families is a continuously evolving parallel to medical technology. Medical ethics may
be a unique because it requires the incorporation of statues unrelated to physiology and pathology. The
future of practicing good medical ethics relies on intent, actions, outcomes and law.
References
1.
2.
3.
4.
5.
6.
Lois Snyder, JD, (2012). for the American College of Physicians Ethics, Professionalism, and Human
Rights Committee. Annals of Internal Medicine. American College of Physicians Ethics Manual, Sixth
Edition. Ann Intern Med. 156:73-104.
Billings JA (2012). Humane terminal extubation reconsidered: The role for preemptive analgesia and
sedation. Critical Care Medicine. Feb 40:625-630.
Scheunneman, LP, White DB (2011). The Ethics and Reality of Rationing in Medicine. Chest. Dec;140(6):
1625-32.
Iltis, Ana S. et al (2009) Organ donation, patient’s rights, and medical responsibilities at the end of life.
Critical Care Medicine. 37(1):310-315
Truong RD, et al. (2008 ). Recommendations for end-of-life care in the intensive care unit: a consensus
statement by the American College of Critical Care Medicine. Critical Care Medicine. May;36(5):1699.
Gavrin JR (2007). Ethical considerations at the end of life in the intensive care unit. Critical Care
Medicine. 2(Suppl):S85-S94.
Questions
3.1 A capable and informed adult refuses life-saving blood transfusions based on his religious beliefs and practice.
Honoring this decision is conforming to which of the following tenets of medical ethics?
A.
B.
C.
D.
E.
F.
Autonomy
Benificence
Nonmaleficence
Justice
Dignity
Truthfulness and Honesty
3.2 A capable and informed 16 year old minor refuses her 6th round of chemotherapy and prefers palliation invokes what
nuance of ethics?
A.
B.
C.
D.
Autonomy
Consent vs Assent
State and Federal law
Understanding informational elements
15
SOCCA Residents Guide 2013
3.3 Pertaining to patients who have declared the intention of organ donation: The Uniform Anatomical Gift Act (UAGA)
revision of 2006 purpose was to:
A.
B.
C.
D.
Restrict organ procurement organization’s (OPO) authority to impose unconsented-to interventionsprevent
Prevent OPO’s from making clinical decisions before death
Prevent OPO’s from prolonging patients life for examination even after the decision was made to withdraw life
sustaining measures
Enhance the availability of organs
3.4 When a patient becomes incapacitated in order to participate in their health care decision-making, who should
become the proxy decision maker?
A.
B.
C.
D.
E.
Patient’s capable mother
Patient’s oldest child
Appointed durable power of attorney for health care decision
Patient’s twin sibling
Patient’s most frequent visitor
3.5 The Doctrine of Double Effect considers:
A.
B.
C.
D.
Absolute outcomes of treatments and therapies should drive therapy beyond good intent
If intent is both good and evil, good overrules.
A beneficial treatment with good intent should not be avoided despite the risk of a bad outcome
Justifies euthanasia as an ethical practice
16
SOCCA Residents Guide 2013
4. Cardiopulmonary Resuscitation
Hossam Tantawy, MD
Introduction
The intensive care unit is unique in any institution where life support equipments, medications and staff are
readily available. The ICU staff is also very good in working as a team where there is adequate manpower
to respond to emergencies in a timely manner. Training the staff in CPR provides a great asset to have
them work as a team in a pre-defined algorithm, where someone picks up where the other left off.
Establishing the diagnosis cardiac arrest may not be as easy as it sounds. A pulseless patient with change in
mental status is the hallmark of cardiac arrest. It may be difficult to establish pulse or detect heart sounds in
morbidly obese patients who is unresponsive however, this should not delay the diagnosis. If not sure,
initiate the code.
CPR is about doing almost everything at the same time where once the code is initiated and the call is
made, someone is securing the airway while others initiating the compressions and another getting rhythm
on a monitor as well as securing a venous access although some medications can be administered through
the ETT. At the same time, medications are being prepared. The only pauses in this process will be either to
check for the rhythm/pulse if difficult or to pass ETT that should not last more than few seconds.
72 year old man was found down in his driveway by neighbors. They call 911
but they are not sure when he fell. They think they saw him walking 5
minutes prior to seeing him on the ground.
2 minutes later, emergency personnel are on the scene and the monitor
shows Ventricular Fibrillation. They initiate chest compressions.
Initial Response: C-A-B
Revisions in the ACLS protocol have changed the long-standing “ABCs” to “CAB” which acknowledges
the increased importance of maintaining circulation, particularly diastolic pressure.
I.
Circulation
A. Rate and rhythm. Check carotid/femoral/radial pulse (most superficial) and obtain ECG rhythm. If
no pulse and no ECG available, then assume VF and deliver shock. Obtain rhythm as soon as
possible
B. Closed Chest compressions Rescuers should push hard, push fast (rate of 100 compressions per
minute without pauses for ventilation. In Adults, The rescuer should compress in the center of the
chest at the nipple line, approximately 1.5 to 2 inches, using the heel of both hands in adults and
that depress the chest of the infant and child by one third to one half the depth of the chest. 30:2
(one or two rescuers) in adults and in children, 30:2 (single rescuer) or 15:2 (2 rescuers).
17
SOCCA Residents Guide 2013
C. Open chest compressions Via thoracotomy or median sternotomy
D. CPB
E. Adequacy of resuscitation. - Palpable pulse- Arterial diastolic pressure -ETCO2.
Emergency personnel start establishing airway and initiate bag mask
ventilation while they attach the defibrillator.
II.
Airway management
A. Establish unresponsiveness
B. call for help
C. administer oxygen
D. establish an airway. Jaw thrust/head tilt chin lift / use of an oral or nasal airway oral airway, LMA
E. secure airway ETT
III.
Breathing
A. Ventilation rate depends on the age of the patient: 10-12 in adults and 12-20 in children.
B. Mechanisms to establish an airway:
C. Mouth to mouth or to mask
D. Bag mask
E. LMA
F. ETT
G. Optimal airway management is with an ETT due to the risk of gastric distension and either
vomiting aspiration or worsening lung compliance due to pressure from the distended stomach.
They deliver the first shock at 150.
Resume compressions and mask ventilation and rhythm shows ventricular
fibrillation. They elect to intubate the patient while the (AED) is charging and
they are ready to deliver the second shock at 120.
IV.
Defibrillation
When any rescuer witnesses an adult cardiac arrest and an AED is immediately available on site, the
rescuer should use the AED as soon as possible. Single shock followed by CPR. The initial selected
shock dose for adults is 150-200 for biphasic truncated exponential waveform or 120 or rectilinear
biphasic waveform. The second should be the same or higher. Defibrillation should be started
immediately for best results and if not available, within 8 minutes of the arrest.
Second shock results in normal sinus rhythm with multifocal premature
ventricular contractions.
18
SOCCA Residents Guide 2013
Pharmacology
I.
II.
III.
IV.
V.
VI.
Epinephrine: may be given after the first or the second shock and repeated every 3-5 minutes standard
dose (0.01 mg/kg to 0.03 mg/kg IV/IO) of epinephrine for the first and for subsequent doses. There is
no survival benefit t from routine use of high dose (0.1 mg/kg IV/IO) epinephrine, and it may be
harmful particularly in asphyxia (Class III). High-dose epinephrine may be considered in exceptional
circumstances such as ОІ-blocker overdose. If epinephrine is administered by endotracheal route, use a
dose of 0.1 mg/kg. Consider epinephrine infusion 2 to 10 Вµg/min for pacing.
Atropine: 0.5 mg IV and may repeat to a total dose of 3 mg.
Dopamine:. 2 to 10 Вµg/kg per minute while awaiting a pacer or if pacing is ineffective.
Vasopressin: One dose of vasopressin (40 U IV/IO) may be given instead of either the first or second
dose of epinephrine.
Amiodarone: When VF or pulseless VT persists after 2-3 shocks plus CPR and vasopressors, consider
Amiodarone.
Lidocaine: For stable VT or hemodynamically compromising PVC (second to Amiodarone).
The patient is started on an Amiodarone infusion while transporting him to
the medical center.
ACLS
I.
II.
III.
VT/ VF: Shock – CPR
PEA: Treatment of the cause (Hypovolemia – Hypoxia – Hydrogen Ions – Hypo /Hyperkalemia –
Hypoglycemia – Hypothermia – Tamponade - Tension pneumothorax – Thromboembolism – Trauma
– Toxins) –CPR
Asystole: Epinephrine, Atropine, Pacing
In the ambulance, they initiate the hypothermia protocol and start cooling
down to 34 В°C
Hypothermia
Unconscious patients after cardiac arrest should be cooled to 32ВєC to 34В°C for 12 to 24 up to 72 hours
when the initial rhythm was VF. Similar therapy may be beneficial for patients with non-VF arrest. In
patients who have been successfully resuscitated after cardiac arrest due to ventricular fibrillation,
therapeutic mild hypothermia increased the rate of a favorable neurologic outcome and reduced mortality.
55 % Vs 39 % had a favorable neurologic outcome with six months mortality was 41 % in the hypothermia
group as compared with 55% in normothermic4 .
Post cardiac arrest Syndrome
Post– cardiac arrest brain injury is a common cause of morbidity and mortality. In 1 study of patients who
survived to ICU admission but subsequently died in the hospital, brain injury was the cause of death in 68%
after out-of-hospital cardiac arrest and in 23% after in-hospital cardiac arrest5.
In 1 series of 148 patients who underwent coronary angiography after cardiac arrest, 49% of subjects had
myocardial dysfunction manifested by tachycardia and elevated left ventricular end-diastolic pressure,
19
SOCCA Residents Guide 2013
followed _6 hours later by hypotension (MAP _75 mm Hg) and low cardiac output (cardiac index _2.2 L В·
min_1 В· m_2)6. This global dysfunction is transient, and full recovery can occur.
Ischemia reperfusion injury. The whole-body ischemia/reperfusion of cardiac arrest with associated oxygen
debt causes generalized activation of immunologic and coagulation pathways, which increases the risk of
multiple organ failure and infection.
Persistent Precipitating Pathology The pathophysiology of post– cardiac arrest syndrome is commonly
complicated by persisting acute pathology that caused or contributed to the cardiac arrest itself
Take-home message
Early diagnosis of cardiac arrest and alerting first responders.
Effective chest compressions without interruption.
Do not waste time trying to feel pulse.
Transport to medical facility to initiate cooling if indicated.
This chapter is a revision of the chapter edited by Todd Sarge
References
1.
2.
3.
Mild therapeutic hypothermia to improve the neurologic outcome after cardiac arrest.N Engl J Med 2002
May 30;346(22):1756
Laver S, Farrow C, Turner D, Nolan J. Mode of death after admission to an intensive care unit following
cardiac arrest. Intensive Care Med. 2004;30:2126 –2128.
Laurent I, Monchi M, Chiche JD, Joly LM, Spaulding C, Bourgeois B, Cariou A, Rozenberg A, Carli P, Weber S,
Dhainaut JF. Reversible myocardial dysfunction in survivors of out-of-hospital cardiac arrest. J Am Coll Cardiol.
2002;40:2110 –2116
Questions
4.1 Which comes first when a code is initiated.
A. Mask ventilation
B. Endotracheal intubation
C. Cooling
D. Compressions
4.2 Induced Hypothermia is:
A. Cooling to 35 degree for 1-3 days
B. Cooling to 32 degree for 1-3 days
C. Cooling to 28 degrees for 1-3 days
D. Cooling for 18 degrees for 1-3 days
4.3 Chest compression to ventilation in adult should be at the rate of
A. 30 / 2
B. 20 / 3
C. 20 / 2
D. 10 / 1
20
SOCCA Residents Guide 2013
5. Transport of the Critically Ill
David Boldt, MD, Adebola Adesanya, MB, MPH
21-year-old helmet wearing motorcyclist is brought to the emergency
department (ED) of a suburban community hospital following a motorcycle
collision. On arrival to the ED, he is awake and alert and his vital signs are:
Pulse 130, BP 100/60, RR 28, SPO2 94% and Temp 98.50F. He complains of
pain in the right arm, neck pain, chest pain and shortness of breath. His
physical examination is remarkable for decreased breath sounds in the right
chest and tenderness in the right arm. Arterial blood gases showed pH 7.35,
pO2 78 and pCO2 25. Base deficit was minus 10 and HCT 30%. CXR
revealed a large right hemo-pneumothrax and multiple posterior rib
fractures. A right arm x-ray showed a non displaced fracture of the right Ulna
and Radius bones.
OUTLINE:
•
•
•
•
•
•
Review the major diagnosis and management in a critically ill patient needing transport
Identify life threatening and potentially life threatening diagnosis
Decide whether patient’s condition is stable for transport
Complete patient and equipment checklist
Transfer and hand over
Legal issues
Critically ill patients tend to become unstable with movement. The abnormal physiology even after
normalization with resuscitation can deteriorate quickly with movement. Patients may require invasive
monitoring and organ system support during transport. Vehicles used for transport are not conducive to
active intervention and additional help is usually not available. The vehicles are also vulnerable to
collisions and may be exposed to temperature and pressure changes. As a result, every effort should be
made to ensure that a patient is hemodynamically stable before transport.
The decision to transfer a patient should be made by an attending physician after full assessment and
discussion with the receiving hospital. Urgency of transfer for certain groups of patients such as acute
myocardial infarction; stroke or head injury is based on national guidelines. For other patients, the balance
of risk and benefit should be carefully assessed.
I.
Types of transport
A. Primary transport versus secondary transport: Primary transport is the movement of the patient
from the incident site to the medical facility while secondary transport is the movement between
hospitals usually for further diagnosis and treatment.
B. Intra-hospital transport: usually occurs between hospital departments, between ICU’s or between
the ICU and the operating room, radiology suite or the ward.
C. Interhospital transport (Transfer): most hospitals have guidelines or arrangements for referral
between hospitals. Good communication to ensure appropriate coordination and integration of
services is essential.
II.
Transfer vehicles
A. designed to ensure good access, lighting, and temperature control. Sufficient space for medical
attendants, medical gases and electricity, storage space, and good communications are also built
in.
B. method of transport (road, air or sea) should take into account urgency, mobilization time,
geographical factors, weather, traffic conditions and cost. Road transfer is satisfactory for most
patients. It also has the advantages of low cost, rapid mobilization, less weather dependency, and
easier patient monitoring. Air transfer should be considered for longer journeys (over 50 miles or 2
hours). The apparent speed must be balanced against organizational delays and transfer between
vehicles at the beginning and end. Helicopters are recommended for journeys of 50-150 miles or if
access is difficult. They provide a less comfortable environment than road ambulance or fixed
wing aircraft and are also expensive, and have a poorer safety record. Fixed wing aircraft should
21
SOCCA Residents Guide 2013
be used for transfer distances over 150 miles.
III.
Equipment
A. should be robust, lightweight, and battery powered for back up.
B. Equipment for establishing and maintaining a safe airway as well as a portable mechanical
ventilator with disconnection alarms which can provide variable inspired oxygen concentrations,
tidal volumes, respiratory rates, levels of positive end expiratory pressure, and inspiratory :
expiratory ratios. The vehicle should carry sufficient oxygen to last the duration of the transfer and
a reserve for 1-2 hours.
C. A portable monitor with an illuminated display is used to record heart rhythm, oxygen saturation,
blood pressure by non-invasive and invasive methods, end tidal carbon dioxide, and temperature.
Monitor alarms should be visible as well as audible because of extraneous noise during transfer.
D. Suction equipment and a defibrillator should be available.
E. The vehicle must contain several medication delivery pumps with long battery life and appropriate
drugs.
F. warming or cooling devices are advantageous
G. All transport personnel should know the location of equipment and be familiar with their use.
Ambulance stretchers should allow equipment to be secured to a pole or shelf above or below the
patient rather than carried by hand or laid on top of the patient.
IV.
Staff
Transport staff should include a minimum of two attendants in addition to the vehicle's crew.
Ambulance personnel are either paramedics or emergency medical technicians with training to
respond to a wide variety of conditions including serious or life-threatening conditions.
V.
Preparation should emphasize stabilization of the patient before transfer. Patient stabilization is the
key to avoiding complications during transport. A patient transfer checklist and departure checklist
should be reviewed.
Table 5-1: Patient Transfer Checklist
Respiration
Head
Airway safe?
Heart rate <120 beats/min?
Oxygen saturation>95%?
Intravenous access adequate?
Glasgow coma score? Trend?
Circulating volume replaced?
Focal signs?
Blood needed?
Pupillary response?
Adequate Urine output?
Skull fracture?
Continuing bleeding? Site?
Other injuries Cervical spine, chest, ribs?
Laboratory
Circulation Systolic blood pressure >120 mm Hg?
Intubation and ventilation
required?
Sedation and analgesia adequate?
Perfusion OK?
Monitoring Electrocardiography?
Pneumothorax?
Pulse oximetry?
Bleeding—intrathoracic or
abdominal?
Long bone or pelvic fractures?
Blood pressure?
Stable laboratory results?
Temperature?
Adequate treatment?
Central venous pressure, pulmonary
artery pressure, or intracranial pressure
needed?
Blood gases, chemistry, and
hematology sent?
Correct radiographs (including
CT’s) taken?
End tidal carbon dioxide pressure?
Adapted from Wallace and Ridley. Transport of critically ill
patients. BMJ. 1999 Aug 7; 319(7206):368-71
22
SOCCA Residents Guide 2013
Table 5-2: Departure checklist
Travel arrangements discussed with relatives?
Adequate sedation?
Transport team has knowledge of case and
appropriate experience?
Appropriate equipment and drugs?
Medications, pumps, lines rationalized and
secured?
Contact numbers known?
Batteries checked?
Arrival Bed confirmed? Exact location?
Oxygen tanks checked?
Estimated time of arrival notified?
Medical records, Radiographs, Laboratory results
collected?
Monitoring attached and working?
Money or credit cards for emergencies?
Return arrangements checked?
Patient still stable after transfer to mobile
equipment?
Adapted from Wallace and Ridley. Transport of critically ill patients. BMJ. 1999 Aug 7; 319(7206):368-71
VI.
Transfer:
The level of care should be maintained at the same level as in the intensive care unit. Oxygen
saturation, heart rate and rhythm, temperature, and arterial blood pressure should be continuous. Noninvasive blood pressure measurement is often affected by movement and can be inaccurate. It is more
important for the transfer to be undertaken smoothly rather than at high speed. A record must be
maintained during transfer. If despite careful preparation unforeseen clinical emergencies occur; the
vehicle should be stopped at the first safe opportunity to facilitate patient management.
VII. Handover:
there must be direct communication between the transfer team and the team who will assume
responsibility for the patient on arrival. A record of the patient's history, treatment, and important
events during transfer should be added to the notes. Radiographs, scans, and other laboratory reports
should be passed on. The transfer team should retain a record of the transfer for future process
improvement.
VIII. Legal Issues- COBRA/EMTALA legislation
A. Hospitals must examine and stabilize all patients regardless of ability to pay.
B. Physicians must certify that the benefits of transfer to an outside facility outweigh the risks.
C. Receiving facility must have the personnel/space and agree to accept transfer.
D. Transferring facility must provide medical record.
E. Transfer is effectuated through qualified personnel with appropriate equipment.
F. Financial penalties to physician and/or hospital may occur for failure to comply.
G. Physician and/or hospital can be sued for violations that result in injury; receiving institutions that
receive “dumped” patients can sue for damages/obtain injunctions.
H. Transferring facility is responsible for emergency treatment and transfer decisions made by
physicians.
I. Any transfer can result in litigation if the patient deteriorates during transfer.
This chapter is a revision of the chapter authored by Daniel Castillo, MD, Claudia Chidiac, MD and Miguel Cobas, MD.
References
1.
2.
3.
4.
Wallace PG, Ridley SA. Transport of critically ill patients. BMJ. 1999 Aug 7; 319(7206):368-71.
Shirley PJ. Transportation of the critically ill and injured patient. Hosp Med. 2000 Jun; 61(6):406-10. .
Gebremichael M, Borg U, Habashi NM, Cottingham C, Cunsolo L, McCunn M, Reynolds HN.
Interhospital transport of the extremely ill patient: the mobile intensive care unit. Crit Care Med. 2000
Jan; 28(1):79-85.
Reynolds HN, Habashi NM, Cottingham CA, Frawley PM, McCunn M. Interhospital transport of the adult
mechanically ventilated patient. Respir Care Clin N Am. 2002 Mar; 8(1):37-50.
23
SOCCA Residents Guide 2013
5.
Warren J, Fromm RE Jr, Orr RA, Rotello LC, Horst HM; American College of Critical Care Medicine.
Guidelines for the inter- and intrahospital transport of critically ill patients. Crit Care Med. 2004 Jan;
32(1):256-62.
Questions
5.1 Choose the right answer regarding the patient presented at the beginning of this chapter:
A. The patient should be transported to the nearest trauma center right away
B. A thoracostomy tube should be inserted before transport.
C. Venous access and fluid resuscitation should begin before transport
D. A thoracostomy tube should be inserted, venous access should be obtained and fluid resuscitation begun
before transport
5.2 A decision is made to transfer the patient to a level one Trauma Center 30 miles away for further diagnostic work up
and possible surgery. The patient should be transported by:
A. Road
B. Helicopter
C. Fixed wing aircraft
D. Sea
5.3 Emergency department physicians in the level one trauma center were contacted by the treating physicians in the
referring suburban community hospital and a transfer was arranged per protocol. Under the COBRA/EMTALA statute:
A. The transferring hospital may transfer the patient immediately since he is uninsured and does not show any
evidence of ability to pay for hospital services.
B. The receiving hospital can refuse the transfer since the patient is uninsured and does not show any evidence of
ability to pay for hospital services.
C. Both transferring and receiving hospitals must examine and stabilize all patients regardless of ability to pay
24
SOCCA Residents Guide 2013
6. Sedation in the ICU
Hesham R. Omar, MD and Enrico M. Camporesi, MD
A 78 year old female patient with a history of diabetes mellitus, hypertension and
Alzheimer’s dementia presented to the hospital with worsening shortening of
breath. Chest X-ray revealed areas of consolidation confirming the diagnosis of
acute bronchopneumonia. On the second hospital day the patient started acting
strangely, became severely agitated and is having visual and auditory
hallucinations. Physical examination revealed a hemodynamically stable patient and
arterial blood gas analysis revealed a SO2 of 99% on 5 L oxygen supplied by a nasal
cannula. What is the best medication to control agitation in this patient?
Introduction
Sedation in the ICU is an imperative tool used to provide comfort and minimize pain, anxiety, delerium and
other forms of distress, some of which are the result of ICU interventions.1 and since survival from sepsis
and ARDS is increasing, the issue of post-traumatic stress disorder after care in the ICU needs to be
considered. Sedation strategies must initially recognize and manage all possible causes for pain and anxiety
before attempting to start sedating a patient. Selecting the ideal medication for each particular patient and
administering the lowest possible effective dose for the shortest possible time is mandatory. The adverse
effects of sedating drugs including idiosyncratic or dose-related side effects, as well as problems related to
the immobility and loss of protective reflexes should be considered.2,3
Indications of sedation
Proper sedation in the critical care is a challenging task. Reports regarding ICU sedation reveal that while
most patients are properly sedated, some are oversedated or undersedated during the course of their care.
Despite their comfortable stay in the ICU, oversedation translates to increased ICU stay, increased need to
perform extra tests, delayed weaning from ventilators and therefore increased overall costs. On the other
hand, undersedation can increase agitation and serious injuries in those patients. The various indications for
ICU sedation include the need for mechanical ventilation in a conscious patient, patient’s dys-synchronacy
with the ventilator, blunting adverse sympathetic responses during intubation, patients undergoing painful
diagnostic or therapeutic procedures, agitated patients and those with acute confusional states (after ruling
out any underlying medical etiology) and patients experiencing severe anxiety or pain not responding to
analgesics.
Continuous vs. intermittent
Patients who require prolonged ICU stays are often placed on continuous infusions in contrast with patients
who require a minute to minute monitoring of their conscious level for proper identification of the onset
and cause of any ensuing new event which can affect future management of these patients (e.g
neurosurgical patients and trauma victims). In hemodynamically unstable patients, continuous infusions
also prevent an acute decrease in blood pressures that often occurs with bolus doses of medications. Poor
hepatic or renal clearance is a second reason to consider intermittent dosing to avoid building up of the
drug level or its metabolite and subsequent toxicity in the form of hemodynamic instability or depression of
mental status.
Drugs used for sedation
There are various drug groups used for accomplishing sedation. Several factors influence the choice of the
appropriate agent for each particular patient. These include preexisting medical conditions such as liver
failure, renal failure, alcohol or substance abuse, chronic sedative use, psychiatric illness, neurosurgical
patients and previous exposure to medications. The characteristics of an ideal sedative include its minimal
hemodynamic alterations, rapid onset of action, short elimination half time, being easily titratable, minimal
depressive effect on the respiratory center and ease of arousability after discontinuation. Table 6-1
25
SOCCA Residents Guide 2013
demonstrates the different groups of drugs utilized for sedation in the ICU.
Monitoring of sedation
Using sedation scales can improve sedation titration and avoiding sedative toxicity or undersedation.
Several sedation scales have been validated for use in the ICU as shown in Table 6-2. However, their
widespread use has not yet been widely adopted. Surveys and observational studies indicate that no more
than one-half of practitioners, when queried during the late 1990s through to 2004, reported routine use of a
sedation scale.(12,13) The Richmond Agitation Sedation Scale (RASS); a 10 level scale from +4
(combative) to -5 (unarousable) is found to have an excellent inter-rater reliability.
Table 6-2. Subjective sedation assessment scales; A comparison of their scoring
Ramsay Scale
Agitated 1 Anxious, agitated,
both
Calm
Sedation Agitation
Scale
Richmond
Agitation
Sedation Scale
(RASS)
7 Dangerously agitated 6 Dangerously agitated,
+4 Combative
6 Very agitated
uncooperative
+3 Very agitated
5 Agitated
5 Agitated
+2 Agitated
4 Restless but cooperative +1 Restless
4 Calm and cooperative 3 Calm and cooperative
0 Alert and calm
2 Cooperative,
orientated & tranquil
3 Patient responds to
command only
Sedated 4 Brisk response to light 3 Sedated
glabella tap
2 Very Sedated
5 sluggish response to 1 Unarousable
light glabella tap
6 No response
Motor Activity
Assessment Scale
2 Responsive to touch,
name,
both
1 Responsive only to
noxious
stimuli
0 Unresponsive
-1
-2
-3
-4
-5
Drowsy
Light sedation
Moderate
Deep sedation
Unarousable
26
SOCCA Residents Guide 2013
Table 6-1. Drugs Used for Sedation
Pharmacology
Adverse effects
Precautions
Propofol Intravenous non-babiturate anesthetic
agent acts by potentiation of GABA
receptor activity and also as a sodium
channel blocker(5) Onset of action is
usually within 1 - 2 minutes for a
duration of 10 minutes and a terminal
half-life of 6 hours. Hepatic and renal
diseases have little impact on the
pharmacokinetics of propofol.
Benzodi
azepine
Haloper
idol
Dexmed
etomidi
ne
1. Hypotension (through
1. Propofol should be administered
vasodilation) and transient only where appropriately trained staff
apnea after bolus dose.
and facilities for monitoring are
2. Pain in injection sites;
available, as well as proper airway
can be mitigated by
management and supplemental oxygen
pretreatment with lidocaine. supply. A number of people have died
3. Propofol infusion
following self-administration of the
syndrome: hypotension,
drug due to lack of support of
metabolic acidosis,
respiratory.
rhabdomyolysis and renal 2. Monitoring trigleceride levels.
failure.(6)
4. Hypertrigleceridemia.(7)
Benzodiazepines are sedatives and
Benzodiazepines have a
1. Caution should be exercised in
hypnotics eliciting its effects through
high degree of safety when patients with liver disease, as they risk
activation of the inhibitory
compared to other sedatives toxic side effects when prescribed
eurotransmitter GABA resulting in a
and minimal hemodynamic benzodiazepine.
reduction of neuronal excitability. The effects. Benzodiazepines do 2. Benzodiazepines are labeled by the
metabolism of most available
have the potential to cause FDA as a category D or X drug.
benzodiazepines occurs within the liver. respiratory depression
Pregnant patients exposed to
Midazolam has a rapid onset and a short especially in patients with benzodiazepine risk low birth weight
half life and thus the preferred agent for liver disease.
children, as well as neuroacute events. Lorazepam is a longer
2. Lorazepam is a risk factor developmental problems.
acting benzodiazepine utilized when
for development of
3. Flumazenil is a competitive
sedation is required for longer periods. delirium(8) and propylene antagonist at the benzodiazepine
glycol(9) toxicity at high
receptor can fully reverse its effects.
doses (anion gap metabolic Caution should be exercised in patients
on long-term use as flumazenil can
acidosis and renal
cause severe withdrawal symptoms in
insufficiency)
the form of tachycardia, hypertension,
and convulsions.(10)
Dopaminergic receptor blocker used for 1. QT prolongation and
1. High dose IV therapy can lead to
acute management of agitation and
subsequent torsade de
prolongation of QT interval. So
psychosis. It is characterized byabsence pointes.
patients on IV therapy QT interval and
of hemodynamic alterations and without 2. Extrapyramidal adverse electrolytes (potassium and
depressive effects on the respiratory
effects including
magnesium) should be monitored.
center.
neuroleptic malignant
Discontinue if QT> 450 msec or >
syndrome (less with
25% baseline.
atypical antipsycotis).
2. Postural hypotension maybe acute
and severe after IM inj.
Dexmedetomidine is an alpha-2
1. Dexmedetomidine has the 1. Its preferred to use the drug for less
adrenergic receptor agonist with
potential to cause
than 24 hours to avoid the risk of a
sedative, analgesic and sympatholytic bradycardia, heart block,
withdrawal Phenomenon.
effects. Its relative selectivity for the
hypotension and asystole 2. Caution should be exercised when
О±-2a adrenoceptor subtype is responsible from unopposed vagal
dexmedetomidine is concomitantly
for providing more effective sedation
stimulation.
administered with agents causing
than clonidine. It exhibits a rapid
2. Dexmedetomidine has the cardiac depression, bradycardia and/or
distribution phase with a half life of 6 potential to cause a
and vasodilation.
minutes. Its popularity is due to its
withdrawal phenomenon
3. Expensive in comparison with other
unique form of sedation which
similar to that of clonidine. sedating drugs
minimally alters cognitive function, and
minimal respiratory depression. It is
principally used in short cases and for a
period of less than 24 hours because of
its potential to cause a withdrawal
Phenomenon similar to that of
clonidine(11)
27
SOCCA Residents Guide 2013
Conclusion
In conclusion, sedation is an imperative tool utilized by ICU physicians and is evolving rapidly with newer
classes of medications and novel management strategies. The goals of providing patient comfort and
tolerance of the ICU environment while avoiding excessive or prolonged sedation is crucial. The use of
sedation scales for better titration and avoiding toxicity is mandatory with special emphasis towards shorter
patient time on mechanical ventilation (when feasible) and faster discharge from the ICU to avoid
prolonged sedation. Finally, it is critical to emphasize that distress and agitation can be caused by pain,
delirium, anxiety, dyspnea, medications, sleep deprivation or other conditions that may be the result of
underlying medical and surgical problems. Addressing these factors is crucial before deciding to sedate
your patient.
References
1. Sessler CN. Comfort and distress in the ICU: scope of the problem. Semin Respir Crit Care Med 2001;
22:111–113.
2. Jacobi J, Fraser GL, Coursin DB, et al. Clinical practice guidelines for the sustained use of sedatives and
analgesics in the critically ill adult. Crit Care Med 2002; 30:119–141.
3. Kress JP, Pohlman AS, Hall JB. Sedation and analgesia in the intensive care unit. Am J Respir Crit Care
Med 2002; 166:1024–1028.
4. Krasowski MD, Jenkins A, Flood P, Kung AY, Hopfinger AJ, Harrison NL. General anesthetic potencies of
a series of propofol analogs correlate with potency for potentiation of gamma-aminobutyric acid (GABA)
current at the GABA(A) receptor but not with lipid solubility. Journal of Pharmacology and Experimental
Therapeutics. 2001 Apr;297(1):338-51
5. Haeseler G, Karst M, Foadi N, Gudehus S, Roeder A, Hecker H, Dengler R, Leuwer M. High-affinity
blockade of voltage-operated skeletal muscle and neuronal sodium channels by halogenated propofol
analogues. British Journal of Pharmacology. 2008 Sep;155(2):265-75.
6. Kam PC, Cardone D. Propofol infusion syndrome. Anaesthesia. 2007 Jul;62(7):690-701.
7. Devlin JW, Lau AK, Tanios MA. Propofol-associated hypertriglyceridemia and pancreatitis in the intensive
care unit: an analysis of frequency and risk factors. Pharmacotherapy. 2005 Oct;25(10):1348-52.
8. Pandharipande P, Shintani A, Peterson J, Pun BT, Wilkinson GR, Dittus RS, Bernard GR, Ely EW:
Lorazepam is an independent risk factor for transitioning to delirium in intensive care unit patients.
Anesthesiology 2006, 104:21-26.
9. Zosel A, Egelhoff E, Heard K. Severe lactic acidosis after an iatrogenic propylene glycol overdose.
Pharmacotherapy. 2010 Feb;30(2):219.
10. Whitwam JG, Amrein R. Pharmacology of flumazenil. Acta Anaesthesiol Scand Suppl. 1998; 108:3-14.
11. Gertler R, Brown HC, Mitchell DH, Silvius EN. Dexmedetomidine: a novel sedative-analgesic agent. Proc
(Bayl Univ Med Cent). 2001 Jan;14(1): 13-21.
12. Mehta S, Burry L, Fischer S, Martinez-Motta JC, Hallett D, Bowman D, Wong C, Meade MO, Stewart TE,
Cook DJ: Canadian survey of the use of sedatives, analgesics, and neuromuscular blocking agents.
13. Rhoney DH, Murry KR: National survey of the use of sedating drugs, neuromuscular blocking agents,
and reversal agents in the intensive care unit. J Intensive Care Med 2003, 18:139-145.
Questions
6.1 A 73 year old diabeteic and hypertensive patient presented to the ER with aphasia, right sided weakness and
confusion. After admission to the ICU the patient experienced severe prolonged tonic clonic contractions which did not
resolve on administration of 10 mg of diazepam and another drug was administered that successfully controlled the
seizures and was started on a maintenance therapy. 2 days later severe hypotension and lactic acidosis developed.
Laboratory analysis revealed trigleceride level of 780 mg/dL, creatine kinase 8960 U/L and HCO3 of 9 mEq/L. what is the
responsible drug?
D.
E.
F.
G.
Midazolam
Fosphenytoin
Propofol
Lorazepam
6.2 Benzodiazepine pharmacological effects is mediated through which of the following mechanisms?
A.
B.
C.
D.
Facilitation of GABA-mediated increase in chloride ion conductance.
Acts as a partial agonist at the 5HTC receptor.
Blocking the NMDA receptor.
Agonist on the mu-1 and mu-2 receptor subtype.
28
SOCCA Residents Guide 2013
6.3. Which of the following medications can be safely used in hepatic patients without excessive CNS depression?
A.
B.
C.
D.
Diazepam.
Midazolam.
Lorazepam.
Alprazolam.
6.4. A 63 year old alcoholic male patient was started on dexmedetomidine infusion for sedation following a major
vascular surgery; 24 hours later the patient started experiencing nervousness, agitation, headaches and hypertension.
Which of the following is a cause for her symptoms?
A.
B.
C.
D.
Drug withdrawal symptoms.
Agonist affect of etomidate on alpha-2 receptors.
Alcoholic withdrawal.
Post-operative delirium.
29
SOCCA Residents Guide 2013
7. Analgesia in the ICU
Enrico Camporesi, MD; Devanand Mangar, MD and Naga Pullakhandam, MD, FGTBA
A 25-year-old male sustained severe blunt chest trauma after colliding with a pole at high
speed in his motorcycle. He is awake and alert and initial CXR reveals multiple left rib
fractures and a fractured sternum. A chest CT confirms those findings with the addition of an
area of consolidation in the upper left lung that correlates with a pulmonary contusion. He is
admitted to the ICU for observation and he complains of severe pain on his left chest. On
exam, he has tachycardia, hypertension and tachypnea.
Pain is commonly encountered by intensive care unit patients. Pain is not limited to trauma or surgery
but is associated with common procedures in the ICU such suctioning and turning. The presence of an
endotracheal tube or other cannulae or catheters is also a source of pain. Puntiilo reported that procedural
pain was often described as sharp, stinging, stabbing, shooting, and awful and than less than 20% of
patients received opiates before procedures. Furthermore, patients with chronic pain syndromes may
have their baseline pain aggravated by their position or immobility while in the ICU.
Unrelieved pain may lead to physical and psychic suffering. Puntilio reported that 63% of patient in the
ICU rated their pain as being moderate to severe in intensity.2 Schelling et al reported that 40% of post ICU
ARDS patients recall having pain while in the ICU and that those patients have a higher frequency of
chronic pain issues when compared with control. Furthermore, these ICU patients had a higher posttraumatic stress disorder scores than controls.3
Pain is frequently the primary cause of agitation in the ICU patient and physiologic changes
associated with pain may adversely affect patients' ICU course. Inadequate analgesia often results in
increased sympathetic tone, causing tachycardia, hypertension and increased systemic vascular
resistance. These changes may lead to increased myocardial oxygen demand and myocardial
ischemia if oxygen delivery cannot match the demands. Postoperative pulmonary complications are also
more likely in patients with inadequate pain control. "Splinting," or limiting the depth of respiration as a
result of pain after surgery causes a reduction in tidal volume and functional residual capacity. Paired
with inhibition of cough, postoperative pulmonary complications such as hypoxemia, atelectasis
and respiratory infection are more likely. Patients with head injury will experience increased
intracranial pressure and therefore increase their morbidity/mortality risk. Deep venous thrombosis
and pulmonary embolism may develop secondary to immobility because of pain or the resulting need
for increased sedation in patients with poor pain control. Other consequences of inadequate analgesia are
increased secretion of certain hormones such as antidiuretic hormone, prolactin, and cortisol that can
negatively impact the patient's metabolic state.
Similar to sedation, appropriate pain management starts with a systematic and consistent assessment
of pain. The most reliable indicator of pain, however, is the patient's self-report. Patients are often
able to communicate the location, characteristics, aggravating/alleviating factors, and the intensity of
their pain. There are also a number of scales (visual analog scale, verbal rating scale, and numeric
rating scale) which have been employed in the assessment of pain.
A Visual Analogue Scale (VAS) is a measurement instrument that tries to measure a characteristic
or attitude that is believed to range across a continuum of values and cannot easily be directly measured.
For instance, the amount of pain that a patient feels ranges across a continuum from none to an extreme
amount of pain. From the patient's perspective, this spectrum appears continuous. The purpose of this scale is
to capture this idea of an underlying continuum. Operationally, a VAS is usually a horizontal line, 100 mm
in length, anchored by word descriptors at each end, as illustrated in Fig. 7.1
No Pain _______________________________________________Worst Pain Ever
Figure 7.1: The patient marks on the line the point that they feel represents their perception of their current state. The
VAS score is determined by measuring in millimeters from the left hand end of the line to the point that the patient
marks.
30
SOCCA Residents Guide 2013
A Verbal Rating Scale (VRS), involves asking the patient to verbally rate his or her level of perceived pain
intensity on a numerical scale from 0 to 10, with the zero representing one extreme (e.g. no pain) and the 10
representing the other extreme (e.g. the worst pain possible).
The Numeric Rating Scale (NRS) is a zero to ten point scale that patients are given. The patient then
chooses the point on the scale that most correlates with their pain level. Figure 2 is an example of a numeric
rating scale.
Figure 7.2: Numeric Pain Rating Scale.
Unfortunately, the assessment of pain using such scales becomes difficult in the critically ill patient who is
sedated, anesthetized, or receiving neuromuscular blockade. Therefore, based on current guidelines from
the SCCM Sedation and Analgesia Task Force 4, pain assessment and response to therapy should be
performed using a scale appropriate to the patient population. For patients who cannot verbally
communicate their pain level, subjective observation of the patient may be useful. Patients experiencing
pain often make certain facial expressions, body movements, and posturing. Physiologic changes such as
increased heart rate, blood pressure, and respiratory rate are also good indicators of pain. However it is
important that this assessment is consistent across the observers to develop a standardized pain evaluation
tool. The goal is to have a tool that is easy replicable, considers the patient cultural differences and that can
be performed quickly.
Agents for Analgesia
Almost all critically ill patients, particularly those receiving mechanical ventilation, will receive an
analgesic agent. A wide variety of pharmacological agents are available for analgesia and, while
recommendations have been made regarding the 'best' analgesic for ICU patients, practice varies widely
between and within ICUs. The choice of agent can be based on many factors, including the relative needs
for analgesia, the pharmacodynamics and pharmacokinetics of the drug in question, route and ease of
administration, the tolerance profile and the cost. While many studies have been conducted comparing the
effectiveness of various agents, there is relatively little published information on variations in analgesic
drug use among units or across national and international boundaries.
Opioids
Currently, the mainstay of analgesic therapy in the ICU patient is intravenous opioids. Opioids work by
stimulating opiate receptors in the spinal cord and CNS, and possibly the periphery. Multiple subtypes
have now been identified with some receptors more involved in analesia than others. Different agents have
been shown to stimulate these receptors to varying degrees. The agents are classified as naturally occurring
opium alkaloids, such as morphine and codeine, semisynthetic derivatives, such as oxycodone,
hydromorphone, and heroin, and the synthetic opioids, such as fentanyl, meperidine, and methadone. Side
effects of opioids are varied, with the degree of side effects the limiting factor in therapy. The dose-limiting
side effect of most opioids is respiratory depression. However, large amounts of opioids can be safely
administered to critically ill patients who are mechanically ventilated if weaning from mechanical
intubation is not being considered.
Morphine
Morphine has both analgesic and sedative effects and can be conveniently administered by bolus dose or
continuous infusion in patients in whom prolonged analgesia and sedation are necessary. Morphine can
cause hypotension either as a consequence of drug-induced bradycardia or histamine release. Morphine
induced bradycardia occurs as a result of stimulation of the vagal nucleus in the medulla. Systemic
31
SOCCA Residents Guide 2013
vasodilation as a consequence of morphine-induced histamine release can be seen with doses as low as 0.1
to 0.2 mg per kg. This is more pronounced in volume depleted patients. The depressant effect on the
medullary respiratory center is difficult to predict. Initially, respiratory rate slows more significantly than
tidal volume is reduced, but as the morphine dose is increased, a profound depression of total minute
ventilation can result. Morphine is metabolized by the liver, and its metabolites are normally excreted in
the urine. The elimination half-life of one of its main metabolites, morphine-6-glucuronide, is 1.5 to 4.0
hours, but in patients with impaired renal function, drug effect can persist for more than 24 hours.
Fentanyl
Fentanyl is a synthetic opioid with high lipid solubility, which permits rapid penetration into the central
nervous system and equilibration between blood and brain drug levels. Fentanyl has a more rapid onset of
action and shorter duration of action than morphine and is 75 to 200 times more potent. Because of its lipid
solubility, it can accumulate in fat stores and, after repeated doses, may have a significantly longer
elimination half-life. When administered in the ICU by continuous infusion, it offers little advantage over
morphine. It is a useful drug for short-term, painful ICU procedures, particularly when used in
combination with a benzodiazepine. Systemic hemodynamic effects resulting from vasodilatation are less
significant with fentanyl than with morphine, because less histamine is released. Fentanyl can be also
delivered via a patch and this route of administration may be helpful in stopping the infusion or intermittent
intravenous doses in patients with chronic pain or opioid dependency.
Meperidine
Meperidine is a synthetic opioid analgesic that is one-eighth as potent as morphine when administered
parenterally. It is metabolized in the liver to normeperidine, an active metabolite. Meperidine has an
elimination half-life of three to four hours. Normeperidine has an elimination half-life of 15 to 30 hours.
Often, toxic levels of normeperidine can be responsible for alterations in a patient's mental status and can
cause seizures. Therefore, meperidine should be used with caution in patients with decreased hepatic or
renal function and in patients who require repeated doses. As a consequence, it is not widely used in ICU
patients.
In summary, opioids can be titrated to a desired effect for analgesia. The most effective route of
administration in the ICU is continuous intravenous infusion. Repeated bolus injections of opioids tend to
result in peaks and troughs of oversedation and inadequate analgesia. Instead, it is more efficient to
supplement baseline continuous infusion with short-acting formulations for severe pain associated with
procedures in the ICU.
Non-steroidal anti-inflammatory drugs (NSAIDs)
Non-steroidal anti-inflammatory drugs (NSAIDs) are valuable in post-operative pain management by
decreasing the need for opioids, and thus probably decreasing the potential for opioid-related complications
such as post-operative ileus and respiratory depression. NSAIDs work by blocking the cyclooxygenase
enzymes (COX 1 & COX 2) thereby reducing prostaglandin production throughout the body. As a
consequence, ongoing inflammation, pain, and fever are reduced. Since the prostaglandins that protect the
stomach and support platelets and blood clotting also are reduced, NSAIDs can cause ulcers in the stomach
and promote bleeding. Ketorolac is a very potent NSAID and is used for moderately severe pain in
combination with a narcotic. It can be administered parenterally and, thus, is used often in the ICU setting.
However, ketorolac causes gastric ulcers more frequently than any other NSAID and is, therefore, not used
for more than five days. The Cox 2 inhibitors seemed promising in the ICU in both oral and intravenous
form but recent controversies with regard to side effects have retarded their use in the ICU.
Adjuncts
The NMDA-receptor antagonist ketamine, and the alpha-2-receptor agonists clonidine and
dexmedetomidine have analgesic and sedative effects and can be used as adjuncts to opioids. Using such
therapy, the amount of opioids can be decreased and some of their side effects minimized. Ketamine has
been found to share or at least have synergetic effects with the opioid receptors at the spinal cord level. It
may be administered IV. Clonidine and dexmedetomidine both produce a synergistic anti-nocioceptive
effects. Clonidine can be given via PO, IV and intrathecal/epidural/regional routes. It is unclear if topical
administration (i.e. CatapresВ° patch) has the same effects.
Dexmedetomidine
Dexmedetomidine is given via IV, intrathecal/epidural or nasal routes. It is a more selective (alpha)-2
agonist, approved for use as a sedative with analgesic-sparing activity for short-term use (<24 hours) in
patients receiving mechanical ventilation. More recent reports have suggested that longer utilization is
possible without increasing deleterious effects. Patients remain sedated when undisturbed, but arouse
32
SOCCA Residents Guide 2013
readily with gentle stimulation. It produces anxiolytic effect comparable to benzodiazepines. In a recent
meta-analysis the limited evidence suggested that dexmedetomidine might reduce length if ICU stay in
some critically ill patients, but the risk of bradycardia was significantly higher when both a loading dose
and high maintenance doses(>0.7mcg/Kg/h) were used6. In a randomized trial, dexmedetomidine treated
patients experienced less delirium and developed less tachycardia and hypertension in dexmedetomidine vs
midazolam for sedation of critically ill patients7. In subgroup analysis of septic patients from the MENDS
double- blind randomized controlled trial; patients receiving dexmedetomidine had more days free of brain
dysfunction and mechanical ventilation and were less likely to die than those that received a lorazepambased sedation8.
Epidural Analgesia
In recent years, the administration of local anesthetics and opioids in the epidural space has become an
increasingly popular method of post-operative analgesia. It has been shown to facilitate the early return of
function (ambulation, coughing, deep breathing, and enteral alimentation) in patients after abdominal
surgery. In the trauma population, patients with multiple rib fractures and/or pulmonary contusion benefit
the most from this type of therapy by minimization of splinting allowing the patient to take larger breaths
and to cough and by allowing the institution of aggressive pulmonary toilet. A continuous infusion of
bupivacaine and morphine (in low doses), and placement of the epidural catheter close to the dermatomes
of surgery, will allow the use of the lowest doses of each medication, maximize the concentration of
analgesics at the site where the nociceptive fibers enter the spinal cord, and minimize the side-effects and
complication of the therapy. In addition, epidural local anesthetics probably have two beneficial effects:
they decrease the need for opioids (which reduces the gut-slowing opioid side-effects) and may have direct
stimulatory effects on the bowel by decreasing the sympathetic tone.
Epidural analgesia has many benefits, but it is not without complications. Thus, the decision to place an
epidural catheter must take into account the potential benefits of epidural analgesia and the inherent risks.
Some of the complications include but are not limited to: accidental dural puncture, intravenous injection,
hypotension, high spinal, and epidural abscess and hematoma formation. The appearance of cerebrospinal
fluid through either the epidural needle or catheter signifies a dural puncture. If this occurs, the epidural
may be attempted again at a different interspace. The risk of postdural puncture headache is increased
because of the large needle size used. Large intravascular injections can be prevented by pre-injection
aspiration of the epidural catheter and the use of a 3 to 4 ml test dose of local anesthetic with epinephrine.
Sometimes, catheters migrate intravascularly hours after the initial placement in the epidural space.
Inadvertent intrathecal injection of an epidural dose of local anesthetic results in a high spinal block. The
use of a test dose before injection of the initial local anesthetic bolus, incremental injections, and aspiration
of the catheter before re-injection aid detection of an intrathecally placed catheter and reduce the risk of a
high or total spinal.
Vasodilation and subsequent hypotension produced by the sympathetic blockade is possible, it is dependent
on the level of sympathectomy and the patient's volume status. Increasing the patient preload and
administering the local anesthetics in small doses while monitoring the hemodynamics should prevent large
variations in the blood pressure.
Epidurals are generally avoided in patients who are grossly septic, have a localized infection at the site of
needle placement, or are therapeutically anticoagulated in order to prevent epidural abscess and hematoma
formation. These medications are not limited to anticoagulants such unfractionated heparin, low molecular
weight heparin or warfarin but anti-platelet drugs such as clopidogrel as well. The American Society of
Regional Anesthesia guidelines should be followed in those cases if the benefits of epidural anesthesia will
significantly improve the patient's outcome.
The data showing a clear improvement in clinical outcome and/or decreased cost with use of post-operative
epidural analgesia are limited to a few categories of patients and surgical procedures. However, the studies
where a definite advantage has been shown are quite helpful to the clinician because they suggest areas
where post-operative epidural analgesia is clearly indicated after the risk and benefits of this technique have
been considered. Several studies of patients randomized to receive either general anesthesia plus
intravenous opioids versus epidural anesthesia plus post-operative epidural analgesia have shown better
outcomes when epidural anesthesia/analgesia is used. For instance, for peripheral vascular surgery, the use
of an epidural was associated with improved graft survival. Cancer patients undergoing major abdominal
surgery who had epidurals showed decreased incidence of tachycardia, myocardial ischemia, myocardial
33
SOCCA Residents Guide 2013
infarction, shorter time to tracheal extubation, earlier discharge from ICU, earlier discharge from hospital,
and lowered total cost than those who did not. Patients undergoing partial colectomy and/or radical
retropubic prostatectomy had decreased time to recovery of bowel function and lowered hospital charges
when epidurals were used for analgesia than not.
This chapter is a revision of the original chapter authored by Ruben J. Azocar, M.D.and Reginald Neymour, M.D. and
Thomas M. Fuhrman, M.D.
REFERENCES
1. Puntillo KA, White C, Morris AB,et al Patients' perceptions and responses to procedural pain: results from
Thunder Project II.Arn J Crit Care. 2001;10:238-51.
2. Puntillo KA. Pain experiences of intensive care unit patients. Heart Lung. 1990;19:526-33
3. Schelling G, Stoll C, Haller M, et al Health-related quality of life and post-traumatic stress disorder in survivors of
the acute respiratory distress syndrome. Crit Care Med. 1998;26 :651-9.
4. Jacobi J, Fraser GL, Coursin DB,et al ; Task Force of the American College of Critical Care Medicine (ACCM) of the
Society of Critical Care Medicine (SCCM), American Society of Health-System Pharmacists (ASHP), American
College of Chest Physicians, Clinical practice guidelines for the sustained use of sedatives and analgesics in the
critically ill adult. Crit Care Med. 2002;30:119-41.
5. Giovannoni MP, Ghelardini C, Vergelli C, et al. Alpha-2-agonists as analgesic agents. Med Res Rev. 2009 Mar;
29(2):339-68.
6. Tan JA, Ho KM. Use of Dexmedetomidine as a sedative and analgesic agent in critically ill adults patients: a metaanalysis. Intensive Care Med (2010) 36:926-939.
7. Riker RR, Shehabi Y, Bokesch PM. Dexmedetomidine vs Midazolam for sedation of critically ill patients. JAMA.
2009;301(5):489-499
8. Pandharipande PP, Sanders RD, Girard TD. Pandharipande et al., Effect of dexmedetomidine versus lorazepam on
outcome in patients with sepsis: an a priori-designed analysis of the MEDS randomized controlled trial Critical Care
2010, 14:R38.
QUESTIONS
1. Epidural analgesia:
A. Is devoid of complications
B. Requires a careful assessment of the patient
C. Is not useful in trauma patients
D. Limits the patient ability to cough
7.2. Opioids produce all of the following effects EXCEPT:
A. Respiratory depression
B. Ileus
C. Amnesia
D. Pruritus
7.3 Ketamine:
A. Is an alpha-2 receptor antagonist
B. Has no role in the treatment of pain In the ICU
C. Depresses the respiratory drive
D. Can serve as an adjunct for pain management in the ICU
7.4. Characteristics of pain in the ICU include the following EXCEPT:
A.
B.
C.
D.
It can be iatrogenic In nature
It is easy to assess
May lead to PTSD In ICU survivors
It Is one of the common causes of agitation
7.5. Deleterious effects of pain include the following EXCEPT:
A.
B.
C.
D.
Elevated oxygen consumption
Agitation
Urinary retention
Atelectasis
34
SOCCA Residents Guide 2013
8. Neuromuscular Blockade
Naga Pullakhandam, MD, Devanand Mangar, MD and Enrico M Camporesi, MD
A 22-year-old male victim involved in a rollover accident, vomits and aspirates upon
attempted intubation in the field. In addition he has a right sided flail chest. Upon arrival to
the ICU he is severely hypoxemic and different modes of ventilation fail to improve his
oxygenation. A decision to prone the patient in a rotating bed is made and he is paralyzed
with vecuronium to minimize potential interference with the ventilator and or sudden
movements while in the prone position.
He remains paralyzed for 7 days and as his oxygenation improves, the paralysis is stopped.
However 24 hours after stopping the paralytic agent, his physical exam reveals generalized
muscle weakness and minimal adductor pollicis brevis muscle response to post-tetanic
stimulation.
The decision to treat a patient in the intensive care unit (ICU) with neuromuscular blocking agents
(NMBs) (for reasons other than the placement of an endotracheal tube) is a difficult one that is guided
more commonly by individual practitioner preference than by standards based on evidence-based
medicine. Furthermore, neuromuscular blockade can lead to deleterious effects in patients.
Indications for NMBs for an adult patient in an ICU frequently include management of difficult
mechanical ventilation, management of increased ICP, treatment of muscle spasms (tetanus), and
decreasing oxygen consumption when all other means have been tried without success.1
Neuromuscular blocking agents can be divided by their chemical structure into either
benzylisoquinolines or steroidal nucleus group; by their duration of action into ultrashort, short,
intermediate or long; and by the type of block into depolarizing or nondepolarizing. (Table 8.1) The
choice of a neuromuscular blocking agent for sustained paralysis in the intensive care unit must be
guided by an understanding of the drug's properties and by a cost-benefit analysis? The practitioner
should be familiar with the relevant pharmacologic features for each neuromuscular blocking agent
including but not limited to structure, ED95, usual bolus dose, infusion dose range, onset time, duration
and recovery times, major route of elimination, activity of major metabolites, autonomic interactions,
and other major side effects.
Table 8.1 Neuromuscular blocking agents by mechanism of action and duration.
Ultra-Short
Depolarizing
Succinylcholine
Non- Depolarizing
Rapacuronium*
Short
Mivacurium*
Intermediate
Long
-
-
Atracurium cisAtracurium
Rocuronium
Vecuronium
d-Tubocurarine
Doxacurium
Metocurine
Pancuronium
Pipecuronium
*no longer available
Many drugs interact with the actions of neuromuscular blocking agents. Some drugs such as
aminoglycoside antibiotics and magnesium potentiate their actions and could be involved in the
pathophysiology of muscle weakness after the paralysis course. Others such as phenytoin may
antagonize their effects. (Table 8.2)
35
SOCCA Residents Guide 2013
Table 8.2 Drugs that interact with neuromuscular blocking agents
Drug
Effect
Antibiotics
Potentiation
(aminoglycosides, vancomycin, clindamycin,
tetracycline, bacitracin)
Anticonvulsants
Resistance
(phenytoin, carbamazepine)
Antidysrhythmics
Potentiation
(lidocaine, calcium channel blockers, quinidine,
procainamide)
Antihypertensives
Potentiation - pancuronium only for
(trimethaphan, nitroglycerine)
nitroglycerine
Dantrolene
Potentiation
under 10 mcg/kg - Potentiation 1-4 mg/
Furosemide (dose related)
kg - Resistance
Ketamine
Potentiation
Local anesthetics
Potentiation
Magnesium Sulfate
Potentiation
Steroids
Potentiation
In terms of the drug selection, the only drug shown to be beneficial when used within 48 hrs of ARDS
onset in a small randomized trial is cis-atracurium.6 In this last randomized trial the paralyzed patients
a significantly had sustained a significantly improvement in P/F ratio even after the paralytic infusion
was stopped. Vecuronium is recommended by the 2002 SCCM guidelines, although these are currently
being revised.
The need for NMB should be reassessed daily and administration should be stopped as early as
possible. Despite the lack of evidence that monitoring prevents adverse effects and the lack of a
standardized method of monitoring, assessment of the depth of neuromuscular blockade in ICU patients
is recommended. Monitoring the depth of neuromuscular blockade may allow use of the lowest NMB
dose and may minimize adverse events.1 By using Train of Four (TOF) monitoring, the rate of infusion
can be adjusted to achieve one or two twitches. However, the TOF might be difficult to assess in a
swollen and/or diaphoretic patient. Another option is to stop the paralytic on daily basis ("drug
holiday") to reassess the need for the continuing the drug and assure rapid recovery from the drug
effects.
The complications of the neuromuscular blockade in the Intensive Care Unit can be categorized as
short term (ventilator disconnect, accidental extubation and acute hyperkalemia with succinylcholine
use), midterm (edema, hypostasis, bedsores, venous thrombosis) and long term (muscle weakness).
Additionally, the possibility of awareness and its deleterious consequences is also present. Awareness
and muscle weakness will be discussed in some detail.
NMBs are not sedatives therefore, before initiating neuromuscular blockade, patients should receive
appropriate sedative and analgesic drugs to provide adequate sedation and analgesia and to avoid
awareness and probably post-traumatic stress disorder (PTSD). The use of depth of anesthesia monitors
to assure a proper level of sedation while patients are paralyzed seems logical although there is not
enough evidence to support their use.3
Although thought to be multifactorial, skeletal muscle weakness in ICU patients is closely related to
the use of NMBs.4.5 A confusing list of names and syndromes, including acute quadriplegic myopathy
syndrome (AQMS), floppy man syndrome, critical illness polyneuropathy (CIP), acute myopathy of
intensive care, rapidly evolving myopathy, acute myopathy with selective lysis of myosin filaments,
acute steroid myopathy, and prolonged neurogenic weakness have been reported.1 However, the term
critical illness myopathy (CIM) has become the more frequently used name for this entity. Although the
exact mechanisms of this problem are unknown, the common factor seems to be damage to the
neuromuscular membrane. The use of steroid based NMBs (pancuronium, vecuronium) and/or the
concomitant use of steroids have been clearly associated in the development of muscle weakness.
However, benzylisoquinolinium based NMBs (cis-atracurium, atracurium) have also been reported to
produce this problem. In addition, muscle weakness has been associated with the persistent presence of
the drug or its metabolites in plasma. Alterations in clearance mechanisms such as hepatic and/or renal
failure can contribute to this problem. Unintentional overdose, drug interactions (Table 8-2), electrolyte
36
SOCCA Residents Guide 2013
imbalances (hypermagnesemia, hypophosphatemia), acidosis, hypothermia and underlying muscle
disorders (polyneuropathy of critical illness, myasthenia gravis) are also involved.
In summary, NMBs should be use as a last resource in the critically ill since the effects on the
neuromuscular membrane can lead to severe complications. If used, monitoring to minimize the dosing
and reassessment of the indications for NMB need to be done continuously. If prolonged use is needed
"drug holidays' should be instituted.
This chapter is a revision of the original chapter authored by Richard Prielipp, and Ruben J Azocar, MD
READING LIST:
1. Murray MJ, Cowen J, DeBlock H, et al. Task Force of the American College of Critical Care Medicine
(ACCM) of the Society of Critical Care Medicine (SCCM), American Society of Health-System Pharmacists,
American College of Chest Physicians. Clinical practice guidelines for sustained neuromuscular blockade
in the adult critically ill patients. Crit Care Med. 2002; 30: 142-56.
2. Ortega R, Azocar R: Neuromuscular Blocking Agents in Surgical Intensive Care Medicine. Edited by
O'Donell J ,Nacul F. Kluwer Academic Publishers, Norwood MA 2001 pp163-180
3. Arbour R. Impact of bispectral index monitoring on sedation and outcomes in critically ill adults: a case
series. Crit Care Nurs Clin North Am. 2006; 18:227-41
4. Murray MJ, Brull SJ, Bolton CF. Brief review: Nondepolarizing neuromuscular blocking drugs and critical
illness myopathy. Can J Anaesth. 2006;53:1148-56.
5. Friedrich O:Critical illness myopathy: what is happening? Curr Opin Clin Nut Metab Care. 2006 ;
9:403-405.
6. Forel , JM., Roch , A., Marin , V., Michelet , P., Demory , D., Blache , JL., Perrin , G., Bondgrand , P., &
Papazian , L. (2006). Neuromuscular blocking agents decrease inflammatory response in patients
presenting with acute respiratory distress syndrome. Crit Care Med. 2006; 34; 2749-57.
QUESTIONS:
8.1 All the the following drugs potentiate the effects of NMBA except:
A. Quinidine
B. Phenytoln
C. Amino glycosides
D. Magnesium
8.2 Which of the following NMB drugs are NOT appropriately "paired," based their duration of action?
A. Vecuronium: intermediate
B. Mivacurium: short
C. Pipecuronium: short
D. Rocuronium: intermediate
8.3. Complications of use of nondepolarizing NMB drugs in the ICU may include all of the following EXCEPT:
A. Awareness
B. Venous Thrombosis
C. Myopathy
D. Hyperkalemla
8.4. Potential causes of prolonged weakness in the ICU patient after NMB drug administration include all the following
EXCEPT:
A. Electrolyte disturbances
B. Drug or drug metabolites accumulation
C. Propofol interaction with the neuromuscular junction
D. Concomitant use of steroids
8.5. The
A.
B.
C.
D.
following are true statements in relation to monitoring NMBA effects, EXCEPT:
The train of four is always easily obtainable and measurable
Monitoring is important in the prevention of myopathy
Drug holidays may be needed to assess effects and length of recovery
Adjustment of the dose to 2 twitches with the TOF is suggested
37
SOCCA Residents Guide 2013
9. Acid-base Balance
Khaldoun Faris, M.D., Nathanael Slater, D.O.
A 68 year old male with a history of hypertension and severe COPD is now postoperative day 5 from an exploratory laparotomy for a perforated diverticulitis. The
surgery was complicated by significant blood loss with subsequent fluid resuscitation.
Once off pressors, he was placed on a furosemide drip and is presently weaning from
the ventilator. His morning lab results are: Na 138 mEq/L, K 4.0 mEq/L, Cl 104
mEq/L, HCO3- 34 mEq.L, BUN 27, Creatinine 1.2 mg/dl. His ABG results are: pH
7.41, PaCO2 54 mm Hg, PaO2 85 mm Hg, HCO3- 33 mEq/L, BE +8 mEq/L.
-----------------------------------------------------------------------------------------------------------Acid-Base disturbances are almost universal in critical ill patients. Clinicians must be prepared to
anticipate, diagnose and treat such conditions. A patient’s natural buffers and compensatory mechanisms
for mild acid-base perturbations are often overwhelmed by such insults as infection, trauma, hypovolemia,
and toxic ingestion. Furthermore, such insults are less tolerated by those with co-morbidities that directly
affect our natural compensatory mechanisms, such as chronic pulmonary disease, renal failure, or
malnutrition. The following outline will delineate the fundamental acid-base topics that need to be
understood by every critical care clinician. It is important to recognize that many patients present with
complex and mixed acid-base abnormalities.
Normal acid-base balance
I.
Henderson-Hasselbach Equation
A. Classic Equation: pH = pK + log [HCO3-] / [CO2-]
1. This equation expresses how pH changes in response to alterations in the concentration of
HCO3- and H+.
2. pH is inversely proportional to [H+] and directly proportional to [HCO3-].
3. To use this equation clinically, multiply the pCO2 by 0.03 (solubility constant of CO2).
B. Modified Equation: [H+] x [HCO3-] = K x pCO2
II. Non-renal Buffering Mechanisms
A. Hemoglobin: The buffering effect is via a histidine side group which has a pK very close to
physiologic pH, making it an excellent buffer.
B. Plasma Proteins: The most abundant protein in this group is albumin. The histidine side group
binds free H+, as it does with hemoglobin.
C. Phosphate (H3PO4): The pK of the diprotic form (H2PO4-) is 6.8, making it a good buffer at
physiologic pH. However, the buffering capacity is small compared to hemoglobin and albumin.
III. Bicarbonate buffer system: CO2 + H2O ↔ H+ + HCO3A. Addition of H+ drives the equation to the left, producing CO2 that is removed by the lungs. If the
additional CO2 was not removed, the pH would fall dramatically.
B. For example, adding 5 mmol of HCL to the equation would drop the [HCO3-] by 5mmol and
produce 5 mmol of CO2, producing a pH of 6.6. Please review the following example.
1. pH = pK + log [HCO3-]/[CO2]
2. Physiologic levels are [HCO3-]: 24 mmol/L, [CO2]: 1.2 mmol/L, and pK: 6.1
3. The addition of HCl will lead to: pH = 6.1 + log (19/6.2) = 6.6
4. If the CO2 is removed from the equation by ventilation, then the effect is ameliorated and
results in a pH compatible with life: pH = 6.1 + log (19/1.2) = 7.3
C. The effect of adding acid to this buffer system is blunted even further by the renal absorption of
HCO3- from the glomerular filtrate.
IV. Renal Buffering Mechanisms
A. Bicarbonate reabsorption: Approximately 4320 mmol of HCO3- are filtered from the blood every
24 hours into the lumen of Bowman’s capsule. Roughly 85% of HCO3- is reabsorbed in the
proximal convoluted tubule, 15% in the collecting tubule.
1. The Na+ - H+ exchanger secretes H+ into the lumen of the proximal tubule. It reacts with
filtered HCO3- via carbonic anhydrase to produce H2O and CO2. CO2 passively diffuses
across the cell membrane from the tubular lumen, reacts with H2O and carbonic anydrase
intracellularly, producing HCO3-. The Na+ - HCO3- cotransporter moves three bicarbonate
38
SOCCA Residents Guide 2013
ions across the cell membrane into the bloodstream along with one Na+ ion. Excretion of H+
into the tubule via the Na+ - H+ exchanger is limited by the pH of the filtrate. Excretion stops
below a pH of 6.5-6.8
2. Reabsorption is similar in the distal tubule; however HCO3- is moved into the bloodstream in
exchange for Cl- instead of being cotransported with Na+. H+ is pumped into the lumen by a
proton pump, and can move against a steeper gradient (down to pH of 4.5).
B. Excretion of titratable acids: Phosphate, urate, creatinine, and beta-hydroxybutyrate are filtered by
the kidney into the lumen of Bowman’s capsule. H+ that is transported across the cell membrane
in the proximal and distal tubules is bound by these non-bicarbonate buffers. Production of HCO3intracellularly is the same as with bicarbonate reabsorption.
C. Formation and excretion of ammonium: Two molecules of ammonium (NH4+) are produced by
the metabolism of glutamine to alpha-ketoglutarate. Alpha-ketoglutarate is a divalent anion and
consumes two protons as it is metabolized to glucose or CO2 and H2O. NH4+ is excreted via a Na+
- H+ exchanger into the lumen by substituting for H+.
V. Renal Response to acid-base imbalance
A. Metabolic acidosis: Acute metabolic compensation is accomplished primarily by increasing
respiratory removal of CO2. Additional buffering is accomplished by hemoglobin, plasma
proteins, and phosphate, and by carbonate from bones. The kidneys reabsorb 100% of filtered
HCO3- and generate additional HCO3- by excretion of titratable acids and formation of ammonium.
B. Respiratory acidosis: The kidneys respond to chronic elevation of pCO2 by increasing production
of ammonium. A response to an acute respiratory acidosis takes at least 72 hours to begin.
C. Metabolic alkalosis: The kidneys’ ability to adjust their resorptive threshold of HCO3- typically
allows for elimination of excess bicarbonate without accumulation leading to alkalosis. A
condition that leads to metabolic alkalosis must reset the threshold for reabsorption in addition to
producing excess amounts of HCO3-. Three inter-related mechanisms can lead to metabolic
alkalosis in the critically ill patient.
1. Hypokalemia causes intracellular K+ to move across the cell membrane into plasma. In order
to maintain electrical neutrality, an HCO3- ion diffuses out as well. The decrease in
intracellular HCO3- is seen by the cell as acidosis, stimulating an increase in renal resorption
of bicarbonate.
2. Aldosterone secretion in response to a decrease in effective circulating volume increases Na+
reabsorption from the tubular lumen, creating an electrically negative (relative) charge. This
causes H+ to move into the lumen along an electrical gradient, facilitating HCO3- reabsorption
into the plasma. The electrical gradient also pulls K+ into the lumen, maintaining
hypokalemia.
3. Volume depletion combines the effects of the above processes. Aldosterone secretion is
stimulated, leading to HCO3- reabsorption and hypokalemia. Hypokalemia can be
exacerbated by diuretic use leading to further HCO3- reabsorption. Diuretics cause loss of
fluid, not loss of bicarbonate (unless using a carbonic anhydrase inhibitor). This loss of fluid
volume while maintaining the same amount of HCO3- leads to a “contraction” alkalosis.
D. Respiratory alkalosis: Respiratory alkalosis is typically due to acute pulmonary physiology
leading to hyperventilation. Renal compensation in this situation is to decrease the amount of
HCO3- that is reabsorbed.
VI. Role of electrolytes
A. Renal management of potassium levels:
1. Approximately 756 mEq/day of K+ is filtered into Bowan’s capsule. 65% is reabsorbed in the
proximal convoluted tubule and 25-30% in the thick ascending loop of Henle. The distal
convoluted tubule and collecting tubule can either absorb or secrete K+ depending on serum
[K+].
2. Principal cells in the distal and collecting tubules are responsible for secreting K+. A Na+ - K+
ATP-ase pumps K+ into the cell in exchange for Na+. K+ then passively diffuses into the
tubular lumen. Secretion of K+ is stimulated by aldosterone, produced in response to
hyperkalemia.
B. Sodium and chloride reabsorption:
1. Na+ and Cl- are actively reabsorbed throughout the length of the nephron with the exclusion of
the thin descending limb of the loop of Henle. The majority of reabsorption occurs in the
proximal convoluted tubule, followed by the thick ascending limb of the loop of Henle. This
is essential for establishing the “countercurrent” mechanism by which the kidney produces
39
SOCCA Residents Guide 2013
2.
3.
concentrated urine.
The primary mechanism for regulating plasma osmolarity via [Na+] is secretion of ADH
(vasopressin) by the posterior pituitary. The distal convoluted tubule and collecting tubule
become permeable to H2O in the presence of ADH, producing concentrated urine.
Diabetes insipidus: Central diabetes insipidus is caused by inability to secrete ADH.
Nephrogenic diabetes insipidus occurs when the kidneys do not respond to ADH. Both result
in excretion of large volumes of dilute urine, rapidly leading to dehydration if water deficits
are not replaced.
Clinical Disorders
Metabolic acidosis
Metabolic acidosis refers to any condition in which serum HCO3- is abnormally low. Severe metabolic
acidosis can be defined as a serum HCO3- ≤ 8 mmol/L. Differential diagnosis of metabolic acidosis is
produced based on analysis of the anion gap.
I.
Anion gap: The anion gap quantifies unmeasured anions in the blood. It is the mathematical
difference of serum Na+ minus the sum of HCO3- and Cl-. Addition of acid releases H+ that binds
one molecule of HCO3- and adds the conjugate base of that acid to the quantity of unmeasured anions,
increasing the anion gap.
A. Anion gap = [Na+] – ([HCO3-] + [Cl-])
B. Normal gap = 6-12
II. Elevated anion gap acidosis differential diagnosis (MUDPILES)
A. Methanol
B. Uremia (acute renal failure)
C. Diabetic/Alcoholic ketoacidosis
D. Paraldehyde
E. Ischemia (bowel, limb)
F. Lactic acidosis (sepsis, circulatory/respiratory failure)
G. Ethylene glycol
H. Salicylates
III. Normal anion gap acidosis differential diagnosis
A. GI losses (vomiting, diarrhea)
40
SOCCA Residents Guide 2013
B. Acute renal failure (occasionally)
C. Renal tubular acidosis
1. Type I: defective distal H+ secretion
2. Type II: decreased proximal reabsorption of HCO3IV. Base Deficit
A. The base deficit is calculated as part of the standard ABG panel. It represents the amount of base
that would be required to titrate one liter of whole blood to a pH of 7.4.
1. Normal
+2 to -2
2. Mild elevation
-2 to -5
3. Moderate elevation -6 to -14
4. Severe elevation
≤ -15
B. In bleeding patients, the base deficit can indicate the degree of tissue acidosis due to hypoxemia
and therefore reflect overall volume status. Rapid correction of the base deficit has been
correlated with better outcomes in bleeding patients as opposed to more gradual correction.
V. Consequences of severe acidosis
A. Cardiovascular
1. Decreased contractility
2. Arteriolar dilatation, venoconstriction, and centralization of blood volume
3. Increased pulmonary vascular resistance (worsens with hypoxemia)
4. Decrease in cardiac output, arterial blood pressure, hepatic and renal blood flow
5. Lower threshold for reentrant arrythmias and ventricular fibrillation
6. Poor cardiovascular response to catecholamines
B. Respiratory
1. Hyperventilation
1. Respiratory muscle fatigue and loss of strength
2. Dyspnea
C. Metabolic
1. Increase in metabolic demand
2. Insulin resistance
3. Inhibition of anaerobic glycolysis
4. Reduced ATP synthesis
5. Hyperkalemia
6. Increase in protein degradation
7. Lack of efficacy of sedatives, narcotics, and supportive medications
D. Cerebral
1. Inhibition of metabolism and cell-volume regulation
2. Delerium, obtundation, and coma
VI. Management:
A. Treatment needs to be directed at the underlying mechanism of disease. For example, with
ethylene glycol ingestion, removing accumulated glycolic acid with dialysis, stopping the
metabolism of ethylene glycol with fomepizole, and aggressive fluid resuscitation are the mainstay
of therapy. Alkali therapy is reserved for pH below 7.2. The goal of akali therapy is to ameliorate
the severe drop in [HCO3-] and thereby prevent or reverse the consequences of acidosis listed
above.
B. HCO3- deficit = Lean body mass (0.5)(24 – [HCO3-])
1. 1 ampule has 50 mmol each of Na+ and HCO32. To replete give 1.5 ampules of NaHCO3 in 1/2NS or 3 ampules of NaHCO3 in D5W.
C. Consequences:
1. Buffering of H+ releases CO2, which causes an additional acid load on the already acidemic
tissue (H+ + HCO3- в†’ CO2 + H2O).
2. Excessive NaHCO3 may cause an “overshoot” alkalosis.
3. NaHCO3 can stimulate 6-phosphofructokinase activity and the production of organic acids
which limits the utility of NaHCO3 therapy.
D. Alternative agents: Limited data available on efficacy
1. Carbicarb
2. THAM (may be difficult to obtain)
VII. Commonly encountered problems in the ICU:
A. Lactic acidosis
41
SOCCA Residents Guide 2013
1.
Lactate is produced by anaerobic metabolism and is cleared from the body by the liver and, to
a lesser extent, the kidneys. Oxygen is required for metabolism of lactate to glucose or its
breakdown to CO2 and H2O. Consequently, any clinical setting which decreases delivery of
oxygen to tissues increases the production of lactate and decreases its consumption.
Treatment is aimed at reversing the underlying cause.
2. Common causes:
3. Acute hypoxemia
a) Shock (septic, cardiogenic, hemorrhagic)
b) Impaired oxidative metabolism
c) Impaired gluconeogenesis
d) Thiamine deficiency
B. Diabetic ketoacidosis
1. Lack of insulin or insulin resistance due to infection or another stressor prevents glucose from
entering cells and storage and metabolism of glucose. Glucagon secretion is increased and
gluconeogenesis is stimulated in response to the lack of energy source for cellular
metabolism. Lipolysis releases fatty acids into the bloodstream which the liver metabolizes
into ketone bodies.
2. Diagnosis: Serum glucose of 400-800 mg/dL and concomitant metabolic acidosis
a) [Na+] is lowered by 1.6 points per 100 mg/dL rise in serum glucose above normal; if [Na
+] is high, the patient is severely dehydrated.
b) Patients will initially present with hyperkalemia, but will become hypokalemic with
treatment. K+ is shifted out of cells by acidosis and lost with osmotic diuresis.
Hypovolemia and vomiting can worsen hypokalemia.
c) Serum HCO3- is lost due to buffering of ketone bodies.
3. Treatment: Aggressive fluid resuscitation and insulin therapy are the mainstay of treatment.
a) Hydrate with 0.9% saline initially, check electrolytes frequently, and add potassium to
fluid once serum levels begin to decrease.
b) Begin insulin therapy only after fluids have been started. Initial rate of infusion should
be 5-10 units per hour, increasing as needed every hour to gradually drop blood glucose.
Serum glucose should not drop below 200 mg/dL within the first 24 hours of therapy.
c) Glucose should be added to IV fluid once serum glucose is less than 200 mg/dL.
d) Insulin should never be stopped completely. It is required for metabolism of circulating
ketone bodies and proper glucose metabolism.
e) Switch to subcutaneous dosing of insulin once the patient’s HCO3- levels have returned to
normal. Stop insulin infusion 2-3 hours later.
Metabolic alkalosis
Metabolic alkalosis refers to abnormally elevated serum HCO3-. Severe alkalemia is defined as serum
HCO3- ≥ 45 mmol/L
I.
Types:
A. Volume responsive (also known as chloride-responsive): The more common of the two types of
alkalemia encountered in the ICU, and usually due to vomiting, gastric drainage, diuretic use, or
administration of NaHCO3-.
1. Urinary chloride < 15 mmol/L or sodium < 10-15 mmol/L
2. Bulimia/anorexia and surreptitious diuretic use; diuretic abuse will falsely elevate urine Na
and Cl
B. Volume resistant: Alkalosis in the absence of vomiting, gastric drainage, diuretic use, or
exogenous NaHCO3- should point to a volume/chloride resistant alkalosis.
1. Urinary chloride > 35 mmol/L
2. Bartter’s Syndrome, primary hyper aldosteroninsm and corticosteroid use
II. Consequences of severe alkalosis
A. Cardiovascular
1. Arteriolar constriction
2. Decrease in coronary blood flow
3. Decrease in threshold for angina
4. Lower threshold for refractory cardiac arrhythmias
B. Respiratory
1. Hypoventilation
2. Hypercapnea and hypoxemia
42
SOCCA Residents Guide 2013
C. Metabolic
1. Stimulation of anaerobic glycolysis and organic acid production
2. Hypokalemia
3. Decreased plasma ionized calcium concentration
4. Hypomagnesemia
5. Hypophosphatemia
D. Cerebral
1. Decreased cerebral blood flow
2. Tetany, seizure, lethargy, delirium, and stupor
E. Management
1. Volume responsive metabolic alkalosis:
a) Stop gastric suctioning
b) Treat nausea to prevent vomiting
c) H2-blockers to decrease gastric acid secretion
d) Decrease dose of, or discontinue diuretics
(1) In patients who continue to need diuresis, acetazolamide, Diamox, can be given;
either 250mg or 500mg q8 hours x3 doses then assess effect.
(2) Add spironolactone to prevent K+ loss and to minimize hypokalemic contribution to
alkalosis.
e) Treat chloride deficit with saline solution
f) Replete potassium deficit if present
2. For refractory alkalosis consider
a) Infusion HCL
b) Dialysis with a chloride-rich solution
F. Volume resistant:
1. Consider tapering corticosteroids
2. Primary hyperaldosteronism
a) Surgery if necessary
b) Spironolactone (blocks effects of aldosterone)
c) Potassium supplementation
3. Bartter syndrome:
a) Na+, K+ replacement
b) Spironolactone
c) ACE inhibitor
Respiratory acidosis
Respiratory acidosis refers to abnormally elevated PaCO2. Hypercapnea is due to either inadequate
ventilation or increased CO2 production.
I.
Etiology:
A. Hypoventilation
1. Central depression
a) Drugs such as opioids and benzodiazepines
b) CNS disorder
2. Thoracic
a) Neuromuscular disorders
b) Obesity
c) Airway obstruction
3. Iatrogenic
a) Inadequate ventilator setting
b) Permissive hypercapnea (acute lung injury and status asthmaticus)
B. Increased CO2 production
1. Hypermetabolic states
a) Fever
b) Sepsis
c) Malignant hyperthermia
2. Excess dextrose in TPN
II. Management
A. Treat the underlying disorder
B. Non-invasive ventilation, BiPAP
43
SOCCA Residents Guide 2013
C. Intubation and mechanical ventilation
D. Tolerate hypercapnea in certain conditions
1. Acute lung injury: Tidal volume is lowered in order to minimize baro/volutrauma and further
lung injury
2. Status asthmaticus: Respiratory rate is decreased to allow longer expiratory time.
Respiratory alkalosis
Respiratory alkalosis refers to abnormally decreased PaCO2, resulting from either excess ventilation or
decreased CO2 production.
I.
Etiology
A. Hyperventilation
1. Central
a) Drugs such as salicylates and chatecolamines
b) CNS disorders
c) Hypoxemia
d) Sepsis
e) Pregnancy
2. Iatrogenic, during mechanical ventilation
B. Decreased CO2 production
1. Hypothermia
2. Pharmacological paralysis
II. Management
A. Treat the underlying disorder
B. Adjust ventilatory setting by decreasing tidal volume, respiratory rate or both
C. In ventilated patients, the centrally mediated respiratory alkalosis does not respond to the
adjustment of the ventilatory setting. If the respiratory alkalosis is severe, sedatives may be used.
III.
Discussion
The patient described above could be presumed to have a severe metabolic alkalosis based on his chemistry
panel and present treatment with a furosemide drip. However, based on his history of severe COPD and
review of his blood gas revealing a normal pH, it is clear that he has a mixed disorder, a metabolic alkalosis
and concomitant respiratory acidosis. Therefore, it may be ill-advised to treat the metabolic alkalosis by
forcing the excretion of bicarbonate in the setting of chronic pulmonary disease.
Reading list
1.
2.
3.
4.
5.
Androgue HJ, Madias NE. Management of Life-Threatening Acid-Base Disorders (Part I). N Engl J Med
1998;338:26-34
Androgue HJ, Madias NE. Management of Life-Threatening Acid-Base Disorders (Part II). N Engl J Med
1998;338:107-11
This two part article is an excellent review of common acid-base emergencies and their treatments.
Corey HE. Fundamental principles of acid-base physiology. Crit Care Med 2005; 9:184-92
The review describes new and more accurate approaches to the diagnosis of acid-base disturbances in the
critically ill patient. It includes calculation of strong-ion gap and strong-ion difference.
Kellum JA. Disorders of acid-base balance. Crit Care Med 2007; 35:2630-36
This review addresses the need for a different approach to acid-base disturbances in the critically ill patient.
It advocates for the use of strong-ion difference, pCO2, and total weak acid concentration as the basis for
diagnosis.
Clive MC. Metabolic acidosis and metabolic alkalosis. In: Irwin RS, Rippe JM, eds. Manual of intensive
care medicine, 4th ed.Lippincott Williams and Wilkins, 2006; 341-46
This chapter provides a quick review of metabolic acidosis and alkalosis.
44
SOCCA Residents Guide 2013
Questions
9.1 The
A.
B.
C.
D.
most common acid-base abnormality associated with aspirin/salicylate toxicity is the following:
Metabolic Acidosis
Respiratory Acidosis
Metabolic Alkalosis
Respiratory Alkalosis
9.2 According to Winters formula, the expected PaCO2 (plus or minus 2) observed in a patient with an acute metabolic
acidosis and HCO3- of 18 mmol/L is:
A. 28 mmHg
B. 30 mmHg
C. 35 mmHg
D. 40 mmHg
E. 45 mmHg
9.3 Acute alkalemia would have which of the following effects:
A. Increased cerebral blood flow
B. Decreased affinity of oxygen for hemoglobin
C. Reduces calcium binding to proteins
D. All of the above
E. None of the above
9.4 Which of the following effects can be seen with Alkali (Bicarbonate) administration?
A. Hypernatremia
B. Hypercapnia
C. Volume overload
D. Cerebrospinal acidosis
E. All of the above
9.5 Which of the following answers is true regarding the use of the “anion gap” to narrow the differential diagnosis for a
metabolic acidosis in the intensive care unit:
A. Hypoalbuminemia decreases the “normal” anion gap whereas hypophosphatemia increases the “normal” anion
gap.
B. Hypoalbuminemia and hypophosphatemia both decrease the “normal” anion gap range observed.
C. Hypoalbuminemia and hypophosphatemia both increase the “normal” anion gap range observed.
D. Albumin and phosphorous levels have no effect on the anion gap.
45
SOCCA Residents Guide 2013
10. Metabolic Disorders
Hossam Tantawy, MD
70 year old male with PMH of CAD (CABG in 1999), CHF, HTN, NIDDM, CRI
(Cr 1.8 mg% baseline), PVD (bilateral LE bypass), presents to an outside
hospital with mental status change, DKA, hypotension, elevated liver
enzymes and a gangrenous left foot.
Initial management includes IV fluids, insulin and empiric antibiotic therapy.
A pulmonary artery catheter reveals septic physiology.
Norepinephrine is started to keep MAP > 70 mmHg. He develops Atrial
Fibrillation with rapid ventricular response (RVR) and Troponin increases to
11.7 ng/ml. He is now oliguric and creatinine has increased to 4.1mg%
Introduction
The intensive care setting is a dynamic environment. Current practice is changing nearly every day, as new
evidence emerges. Rapid change cycle management is crucial as new protocols are placed based on this
evicence. Not all changes are without complications, and some are adjusted or even withdrawn as newer
and broader based evidence comes to light. There are still many areas with little if any evidence or
conflicting evidence to guide practice.
Early Goal Directed Therapy (EGDT)
The choice of vasopressor (norepinephrine or vasopressin) was studied by Russell7 et al. This multicentre,
randomized, double-blind trial, where patients who had septic shock and receiving a minimum of 5 Вµg of
nor epinephrine per minute to receive either low-dose vasopressin (0.01 to 0.03 U per minute) or
norepinephrine (5 to 15 Вµg per minute) in addition to the norepinephrine already infusing. All vasopressors
infusions were titrated and tapered according to protocols to maintain a target blood pressure. The primary
end point was the mortality rate 28 days after the start of infusions with secondary outcomes of 90 day
mortality, organ dysfunction during the first 28 days and major adverse events.
There was no difference shown in any of the outcomes measures. However, there are several issues with
this study. First was the excluded groups - acute coronary syndrome, and severe heart failure were both
excluded. This may have skewed the studied population towards a lower mortality group. Second was the
timing of therapy institution, which averaged 12 hours. Potential benefits may have been negated by the
delay in initiating treatment.
In the hypotensive patient, Rivers8 and colleges used an algorithm including fluid resuscitation,
vasopressors and monitoring techniques. Part of the algorithm involves initiating therapy in the emergency
department rather than waiting for transfer to the ICU. It also involves starting antibiotics as soon as
cultures are obtained. They showed that early (less than 6 hours) goal directed therapy had better outcome
than those who were delayed for more than 6 hours.
In widespread, general use, it appears as if it is the timing of the therapy that is more important than the
choice of vasopressors. This, of course, is in addition to source control and early appropriate antibiotic
management.
Glucose Management
The 2001 study published by Van Den Berghe, et al4, showed a remarkable decrease in morbidity and
mortality by tightly controlling the blood glucose. Despite concerns about hypoglycemia, this quickly
became the “norm” and protocols involving tight glucose control (goal sugar 80-110 mg/dL) proliferated.
With widespread use, the findings were not reproducible (even by the same researchers in a different ICU),
and the incidence of hypoglycemia was significant. In 2009, the NICE-SUGAR study randomized over
6000 patients to two groups: “intensive” control (81-108mg/dL) or “conventional” control (144-180mg/
dL). Mortality was slightly higher in the intensive control group (27.5% vs 24.9%, P=0.02).
46
SOCCA Residents Guide 2013
Hypoglycemia (<40 mg/dL) was 6.8% vs 0.5% in the conventional group (P<0.001). However, there were
no differences in ICU days, hospital days, mechanical ventilation days or renal replacement therapy.
Most facilities have now instituted active management protocols, with goals ranging from 110-180mg/dL,
with greater use of insulin infusions, more frequent testing and point-of-care testing.
Steroids
There has long been interest in the role of glucocorticoids in critical illness. It is clear that normal diurnal
variation is adversely affected in critical illness. In addition, half-life of circulating cortisol may be
increased by renal dysfunction,or by a decrease in binding globulins. Inflammatory mediators may
increase receptor affinity, decrease degradation or increase conversion to cortisol. Drugs (etomidate,
phenytoin, ketoconazole) may impair synthesis.
Typically, a patient will have paired blood samples drawn before and after a dose of cosyntropin is given - a
“stim test.” Elevated baseline cortisol and failure to respond to the stimulus are felt to have a worse
prognostic implication. However, there are some significant limitations to the laboratory assay, and
argument about the appropriate dose of the stimulant to give. Historically, no treatment would be given
until the lab results were available, then it was more common to start treatment while awaiting the results.
Some current protocols eliminate the stim test all together.
Annane, et al6 performed a meta-analysis of 20 trials ranging from 1955 through 2008. There appeared to
be a beneficial effect of prolonged low-dose corticosteroid therapy (as done in studies from 1998 and later)
on short-term mortality. In addition, there seems to be some distinction between early versus late initiation
of treatment (much as with EGDT).
Currently, given the above meta-analysis, there is general support of use of low dose corticosteroids in
critically ill patients.
Sick Euthyroid Syndrome
Any abnormal thyroid function test (TFT) in the setting of non-thyroidal illness (NTI) or systemic stress
has been denoted Sick Euthyroid Syndrome7. Most recently the preferred term is “Non-thyroidal illness
syndrome - NTIS” as some patients may, in fact be hypothyroid. This entity is more frequent in critically ill
patients and the abnormalities can be detected as early as two hours after the onset of stress. Different
clinical scenarios have been described in the syndrome.
A low T3 level is the most commonly encountered abnormality. In these cases, T3 levels fall rapidly within
30 minutes to 24 hors of onset of illness, while rT3 levels increase. The finding of increased rT3 levels
differentiates this syndrome from the true hypothyroidism in which rT3, T3 and T4 levels would most
likely all be low. Thyrotropin (TSH), total and free T4 levels are usually normal. Low T3 syndrome is
thought to be due to a decrease in conversion of T4 to T3 by the hepatic deiodinase system. An exception is
patients with advanced AIDS, in whom baseline rT3 levels are already low.
Low T3 and low T4 may be encountered in patients who are moderately ill. This syndrome has been
described in up to 20% of patients treated in the ICU. Free thyroid hormone levels are usually normal but
may be decreased in patients treated with dopamine or corticosteroids. TSH levels may also be low to
normal. The mechanism involved may be a deficiency in TBG, which leads to low total thyroid hormone
levels, or the presence of a thyroid hormone binding inhibitor, which lowers total thyroid hormone levels.
Simultaneously low T3, low T4 and TSH is the most severe non-thyroidal illness. Although most of these
patients have TSH levels at the low end of normal, TSH may be undetectable in some, even when third
generation assay are used. This finding suggests altered pituitary or hypothalamic responsiveness to
circulating thyroid hormone levels. During the recovery period, TSH levels return to normal or may even
rise transiently before returning to normal.
An elevated T4 with normal or slightly elevated TSH and T3 is typically indicative of increased thyroid
binding globulin (TBG) synthesis and release. This may be seen in primary biliary cirrhosis and acute and
chronic active hepatitis. Drugs such as amiodarone, propranolol and iodinated contrast agents may also
47
SOCCA Residents Guide 2013
elevate T4 levels by inhibiting peripheral conversion of T4 to T3.
The decision to treat or not treat NTIS is controversial. Although we understand the laboratory
abnormalities, it is unclear if “caloric sparing” hypothyroidism is beneficial (the traditional teaching) or if it
will provide benefits (reduction in vasopressor need, for example). While research is ongoing to study this
problem, it is clear that resolution of the primary critical care disease is a key of treatment.
This chapter is a revision of the chapter edited by Ruben Azocar
References
1.
2.
3.
4.
5.
6.
7.
Russell JA, Walley KR, Singer J, et al. Vasopressin versus norepinephrine infusion in patients with septic
shock. N Engl J Med 2008;358:877-87.
Rivers E, Nguyen B, Havstad S, et al. Early goal-directed therapy in the treatment of severe sepsis and
septic shock. N Engl J Med 2001;345:1368-77.
Holmes CL, Walley KR, Chittock DR, Lehman T, Russell JA. The effects of vasopressin on hemodynamics
and renal function in severe septic shock: a case series. Intensive Care Med 2001;27:1416-21.
van den Berghe G, Wouters P, Weekers F, Verwaest C, Bruyninckx F, Schetz M, Vlasselaers D,
Ferdinande P, Lauwers P, Bouillon R: Intensive insulin therapy in the critically ill patients. N Engl J Med
2001 , 345:1359-1367
The NICE-SUGAR study investigators, Intensive versus conventional glucose control in critically ill
patients. N Engl J Med 2009 Mar 26; 360:1283-1297.
Annane D. Bellissant E. Bollaert PE. Briegel J. Confalonieri M. De Gaudio R. Keh D. Kupfer Y. Oppert M.
Meduri GU. Corticosteroids in the treatment of severe sepsis and septic shock in adults: a systematic
review. JAMA. 301(22):2362-75, 2009 Jun 10.
Docter R, Krenning EP, de Jong M, Hennemann G.В The sick euthyroid syndrome: changes in thyroid
hormone serum parameters and hormone metabolism.В Clin Endocrinol (Oxf).В NovВ 1993;39(5):499-518
Questions
10.1 In Sepsis:
A. Vasopressors Must be started within 3 hours
B. FFP is the fluid of choice
C. Maintain Hb > 10 /dL
D. Mixed Venous O2 should be > 80%
E.
Use of Vasopressin is associated with more adverse cardiac events.
10.2 In Treatment of Hyperglycemia:
A. In intensive Insulin therapy, Mortality is higher in conventional BGL 144-180 Than Intensive 81-108 mg/dL
B. Hyperglycemia is benign as long as BGL is lower than renal threshold.
C. NICE-SUGAR study showed BGL 144-180 mg/dL had same outcome of ICU, hospital days and CRRT compared
to 81-108 mg/dL.
D. Outcome of Intensive Insulin therapy (81-108) is equal across all ICU patient
10.3 Steroids in sepsis:
A. Methyprednisone is associated with less mortality than Hydrocortisone
B. Data is conflicting but suggest early low dose steroids are beneficial.
C. All sepsis patients must be on small dose or oral prednisone.
D. Steroids are well studied in sepsis and of no value and increase mortality so, they are contraindicated.
10.4 Thyroid function in sepsis:
A. Thyroid replacement is not indicated in sepsis as conversion of T3 to T4 is increased.
B. Altered TFTs in sepsis is an indicator of intrinsic Thyroid pathology.
C. Patients with Sick Euthroid syndrome will need lifelong hormonal replacement.
D. Abnormality in TFTs with acute illness could be detected as early as 24 hours after illness.
48
SOCCA Residents Guide 2013
11. Fluid and Electrolyte Management
Ronald W. Pauldine, M.D.
A 42 year old roofer falls from 3 stories. On arrival to the emergency department he
is obtunded, tachycardic, hypotensive and has a positive FAST for free fluid in the
peritoneal cavity.
-----------------------------------------------------------------------------------------------------------Fluid administration is one of the most common interventions in hospitalized patients. The properties of
specific fluids may provide advantages in certain clinical situations and have to the potential to cause harm
in others. Fluids may be thought of as providing a maintenance function or be used for volume expansion
as part of resuscitation in a variety of pathological states. Many factors including critical illness, trauma and
surgery affect the volume, composition and distribution kinetics of fluids within the intracellular and
extracellular compartments. A basic understanding of the distribution and composition of fluids along with
an appreciation of the factors commonly causing shifts in these important variables is vital to caring for the
critically ill.
Composition of Body Fluids and Water Balance
A. Body Fluid Compartments
1. Total Body Water (TBW) = 0.6 X Lean Body Weight
a) TBW is the sum of: Intracellular Volume (ICV) + Extracellular Volume (ECV)
(1) IVC = 0.4 * Lean Body Weight
(2) ECV = 0.2 * Lean Body Weight
(3) Approximately 20% of the ECV is composed of Plasma Volume (PV)
(4) The remaining 80% represents the Interstitial Fluid (IF)
B. Fate of Infused Fluids
1. The quantity of infused fluid initially remaining in the plasma volume can be estimated by
considering the distribution volume of the infused fluid:
PV increment = (volume infused * normal PV)/distribution volume
2.
Example: For a given volume of D5W with a distribution volume approximating TBW,
roughly 7% of total volume infused will remain in the plasma volume. For a given volume of
lactated ringers or normal saline with a distribution volume approximating the ECF roughly
1/3 will remain.
3. This calculated increase is transient as it does not consider ongoing fluid excretion.
C. Several adaptive responses are activated in response to hypovolemia. These include:
1. Decreased GFR
2. Redistribution of Renal Blood Flow
3. ADH
4. Renin- Angiotensin-Aldosterone
5. Altered secretion of ANP
D. Several factors affect distribution of fluid between the ECV and ICV
1. Osmolality
a) Osmolality is defined as the number of osmotically active particles per 1000ml of fluid
and is usually reported in milliosmoles per liter of water.
b) Osmolality can be measeured directly in lab or estimated by the following equation:
Osmolality = ([Na+] * 2.0) + (Glucose/18) + (BUN/2.3)
2.
3.
c) Normal osmolality of blood is 285-295 mOsm/l H2O
Tonicity is the relative osmolality when two solutions are separated by a membrane that
allows movement of water but not solutes.
The Starling Equation defines movement of fluid as influenced by several variables:
49
SOCCA Residents Guide 2013
Q = kA[(Pcapillary - Pinterstitial) + Пѓ(ПЂcapillary - ПЂinterstitial)]
where: Q = fluid filtration, k = capillary filtration coefficient, A = the area of the capillary
membrane, P = hydrostatic pressure, Пѓ = reflection coefficient, ПЂ = colloid osmotic pressure
Assessment of Fluid Requirements
Fluid choices for maintenance and resuscitation should consider potential advantages for fluids of varying
compositions as determined by the underlying clinical scenario. Frequent considerations include
hypovolemia from any cause including blood loss or dehydration, major burn injury, relative hypovolemia
in situations such as sepsis, and conditions resulting in increased intracranial pressure. Even for
experienced clinicians, clinical estimation of the intravascular volume can be difficult.
I.
Maintenance Fluids
A. Maintenance fluid requirements address the need for free water to meet losses including
evaporation and losses in urine and stool.
B. Maintenance fluids also address the daily requirements for sodium and potassium.
C. Maintenance fluids often include dextrose and may offer advantages in decreasing protein
catabolism and gluconeogenesis but may result in increased lactate production under conditions of
hypoperfusion.
II. Resuscitation Fluids
A. Resuscitation fluids are frequently used in large volumes and targeted at specific physiological
endpoints to treat or prevent shock.
1. Endpoints of resuscitation are controversial and no single endpoint can be universally targeted
for the variety of pathophysiologic states encountered.
2. Classic endpoints have included hemodynamic variables, markers of occult hypoperfusion,
and functional assessment of hemodynamic trends in response to intervention to name a few.
3. Some commonly used variables to assess adequacy of resuscitation are reviewed in Table 1.
III. Colloids:
A. Colloids are preparations of noncrystalline molecules dispersed throughout a water based diluent.
B. Albumin
1. Albumin is a protein that is a normal constituent of plasma and is biologically active as a
transport protein and buffer.
2. It is available in 5% and 25% concentrations for infusion
3. 5% Albumin can be used as a volume expander
4. 25% Albumin is hyperoncotic and causes shift of interstitial fluid to the plasma compartment
5. Clinical uses include volume replacement during large volume paracentesis and as part of
protocols for burn resuscitation.
C. Starches
1. Starches are synthetic colloids characterized by their molecular weight.
2. Examples include 6% hydroxyethyl starch available suspended in normal saline or balanced
salt solution
3. Infusion of large volumes of high molecular weight starch is associated with increased
bleeding risk due to inhibition of von Willebrand factor and Factor VII and impaired platelet
function
D. Dextrans
1. Dextrans are prepared from glucose polymers and are known to impair platelet function. This
has led some to advocate use perioperatively to decrease thrombosis risk after vascular
surgery.
IV. Isotonic Crystalloid
A. Normal saline
B. Normal saline has a sodium and chloride content of 154 mEq/l
1. Infusion of large volumes can lead to hyperchloremic metabolic acidosis
C. Ringer’s lactate
1. Ringer’s lactate has a sodium concentration of 130 mEq/l
2. It also contains potassium (4mEq/L) and calcium (3mEq/l)
3. Due to the addition of lactate, the chloride content is 109 mEq/l so large volume infusion is
not associated with hyperchloremia
D. Plasmalyte
50
SOCCA Residents Guide 2013
1. Plasmalyte is buffered to pH 7.4.
E. Colloid vs. crystalloid
1. Ongoing debate
a) Large meta-analyses and RCTs demonstrate no advantage for one over the other in fluid
resuscitation
b) Colloids have a higher cost
F. Hypertonic Solutions
1. Hypertonic saline
a) No evidence of benefit is acute resuscitation
2. May be used to establish hyperosmolar conditions for treatement of increased intracranial
pressure.
Electrolyte Replacement
A. Potassium
1. Internal Balance determined by acid-base status, insulin, mineralocorticoids, and
catecholamines.
2. External Balance determined by renal and gastrointestinal potassium excretion.
3. Hypokalemia
a) Hypokalemia is commonly observed in the ICU. Associated condition include, diuretics,
renal potassium wasting, hypomagnesemia, Cushing’s syndrome, and
hyperaldosteronism
b) Treatment involves repletion with attention to large deficits and corresponding
requirements with serum level less than 3 mEq/l and the need for concomitant repletion
of magnesium.
4. Hyperkalemia
a) Hyperkalemia is associated with renal failure, rhabdomyolysis, multiple medications and
mineralocorticoid deficiency,
b) Treatment includes:
c) Membrane stabilization with intravenous calcium and
d) Shift/removal of potassium via insulin/glucose administration, bicarbonate,
catecholamines, potassium-binding resins (kayexalate), and dialysis.
B. Sodium
1. Hyponatremia
a) The diagnostic approach requires assessment of:
(1) ECV
(a) Hypovolemic hyponatremia is the result of loss of sodium in excess to free
water as may be seen with fluid losses that are replaced with hypotonic fluids.
Urine sodium may distinguish between losses that primarily occur in the
kidney (diuretics, adrenal insufficiency, cerebral salt wasting) and those from
other sources (diarrhea, vomiting)
(b) Isovolemic hyponatremia is caused by a relative gain in free water as may be
seen in SIADH or polydipsia. Urine osmolality can be used to distinguish.
(c) Hypervolemic hyponatremia is caused by an excess retention of salt and
water with water retained in excess to salt. Common causes include renal
failure, congestive heart failure and cirrhosis.
b) The approach to treatment is
(1) Based on etiology and
(2) Rate of correction
(3) In general the rate of correction should proceed at the same rate that the condition
evolved to prevent demylinating syndromes such as central pontine myelinolysis.
c) Indications for hypertonic saline
(1) Severe symptomatic hyponatremia manifested by stupor, coma, seizures
2. Hypernatremia
a) The diagnostic approach includes assessment of:
(1) Extracellular volume
(a) Low ECV may result from loss of fluid from over diuresis, vomiting or
diarrhea
(b) Normal ECV may be observed with loss of free water (as may occur in
51
SOCCA Residents Guide 2013
(2)
diabetes insipidus or with diuretics) when the free water loss is replaced with
normal saline
(c) High ECV is seen with administration of hypertonic fluids.
(d) Urine osmolarity may be useful in establishing diagnosis of diabetes
insipidus.
Approach to treatment
(a) Based on etiology
(b) Usually caused by free water deficit as opposed to excess salt
(c) The rate of correction for symptomatic hypernatremia should be no faster
than 1-2 mEq/hr and 0.5 mEq/hr for asymptomatic patients to avoid
neurological complications such as cerebral edema
C. Calcium
1. Hypocalcemia
a) Calcium is best assessed by measurement of the ionized calcium and has a prevalence of
15% in the ICU population
b) Causes of hypocalcemia include parathyroid hormone (PTH) deficiency, vitamin D
deficiency, calcium chelation (citrate), alcohol, hypomagnesemia, pancreatitis,
hyperventilation, renal or hepatic disease,
c) Treatment is repletion of calcium. Over zealous repletion can however lead to
vasoconstriction and intracellular calcium overload.
2. Hypercalcemia
a) Hypercalcemia is most frequently seen in malignancy. Other causes include excess PTH,
immobilization and granulomatous diseases
b) Treatment includes volume expansion to address hypovolemia followed by forced
diuresis. Pharmacological agents target bone resorption and include calcitonin and
bisphosphonates.
D. Magnesium
1. Hypomagnesemia
a) Hypomagnesemia is very common with a prevalence in the ICU of 60%. Because Mg is
predominantly intracellular, normal serum values do not preclude Mg depletion. Primary
losses occur through renal mechanisms and diarrhea. Medications associated with
hypomagnesemia include aminoglycosides, amphotericin, cisplatin and cyclosporine.
b) Magnesium depletion is associated with potassium depletion, hypocalcemia, and
hypophosphatemia.
c) Serious complications include cardiac dysrhythmia (torsade de pointes) and neurologic
problems
2. Hypermagnesemia
a) Occurs almost exclusively in renal disease but may be seen as an iatrogenic complication
of exogenous Mg+2 administration.
b) Serious complications include hyporeflexia and heart block. Dialysis may be required in
the setting of severe toxicity
E. Phosphate
1. Hypophosphatemia
a) Hypophosphatemia is common in the ICU and results from intracellular shifts of
phosphorus, increased excretion or decreased absorption.
b) Clinical manifestations may include weakness, hemolytic anemia, and impaired tissue
oxygen delivery related to decreases in 2,3 DPG
c) Treatment is directed at repletion of phosphorus. role of calcium
2. Hyperphosphatemia
a) Hyperphosphatemia is most commonly observed as a result of renal failure but may occur
with rhabdomyolysis or tumor lysis syndrome.
b) Clinical findings include metastatic calcification with acute hypocalcemia resulting.
c) Treatment includes the use of enteric phosphate binders and dialysis for patients with
renal failure.
Blood and Blood products
A. Blood products are available as prepared fractionated components.
1. Whole blood
52
SOCCA Residents Guide 2013
a) Not routinely available
Packed red blood cells
a) Each unit contains approximately 300cc volume with a hematocrit of 70.
3. Washed red blood cells
a) Use may be considered for patients with a history of hypersensitivity transfusion
reactions.
4. Leukocyte-depleted red cells
a) Use may be considered for patients with a history of non-hemolytic transfusion reactions.
5. Fresh Frozen Plasma
a) May be considered for acute reversal of warfarin therapy and correction of
coagulopathies
6. Cryoprecipitate
a) Fibrinogen deficiencies
b) von Willebrand’s disease
c) Consumptive coagulopathy
7. Platelet concentrate
a) Indicated for microvascular bleeding with platelet count less than 50K. This threshold is
higher for patients with intracranial bleeding
b) Prophylactic transfusion may be considered for platelet counts less than 10K.
B. A number of adverse affects are associated with blood transfusion
1. Effect on oxygen transport and the oxygen-hemoglobin dissociation curve results from
depletion of 2,3 DPG.
2. Hemolytic transfusion reactions result from ABO incompatibility.
a) They are rare events.
b) Clinical manifestations include fever, dyspnea, chest pain, low back pain and
hypotension.
c) Treatment includes stopping the transfusion and providing hemodynamic support.
3. Allergic reactions
4. Febrile reactions
5. Transmission of Infectious Diseases
a) Viruses (HIV, CMV, Hepatitis)
b) Bacteria (Transfusion related sepsis)
6. Transfusion – Related Acute Lung Injury (TRALI)
7. Immunomodulation
8. Problems Associated with Massive Transfusion
a) Dilutional thrombocytopenia and dilutional coagulopathy
b) Hypothermia can be prevented by use of blood warmers
c) Citrate intoxication
d) Metabolic complications including hypomagnesemia, hypocalcemia, acid-base
disturbances
C. C. When should a patient receive a blood transfusion?
1. Use of transfusion triggers
2. An arbitrary trigger to administer packed red cells remains a controversial topic. However, the
best evidence suggests that patients without active coronary ischemia tolerate hemoglobin
levels as low as 7mg/dl without adverse outcome. Transfusion of blood in most ICU
populations is an independent risk factor for increased morbidity and mortality.
D. Damage control resuscitation and massive transfusion
1. An exception to conservative transfusion strategies is the patient presenting with hemorrhagic
shock. Mounting evidence suggests that these patients should be aggressively resuscitated
with packed cells, fresh frozen plasma and platelets in conjunction with surgical control of
bleeding and measures to maintain normothermia and prevent acidosis.
2. Once bleeding is controlled and shock has been treated resuming a conservative transfusion
strategy is indicated.
2.
53
SOCCA Residents Guide 2013
Table 11-1: Some commonly used methods to assess intravascular volume status and
adequacy of resuscitation.
Noninvasive
Laboratory
Invasive Monitoring
Capillary refill
BUN
Skin color
BUN:serum creatinine
CVP, pulmonary arterial and wedge
pressures
Cardiac index
Pulse rate
Urine osmolality
Frank-Starling Curve
Blood pressure
Urine:plasma creatinine
Oxygen delivery
Pulse pressure
Fractional excretion of sodium
Oxygen consumption
Orthostatic blood pressure and
heart rate changes
Mental status
Hematocrit
Mixed venous oxygen consumption
Acid-base balance
Gastric tonometry
Urine output
Serum lactate
Right ventricular ejection fraction
Temperature
Transesophageal and transthoracic
echocardiography
Discussion
The case presented offers several challenges in fluid management. The patient likely has
severe blunt injury to major abdominal organs including the spleen and liver. Further
evaluation is needed to identify other potential injuries. There could be major vascular
injury or major orthopedic trauma such as pelvic or long bone fractures as well. This
patient is presenting with signs of hemorrhagic shock and may be at risk for requiring
massive transfusion. Initial laboratory evaluation should assess for markers associated
with major transfusion requirements (decreased hematocrit on presentation, base deficit,
increased INR) If these are present in combination with hypotension and hypothermia a
massive transfusion protocol may need to be initiated. Also present is a significant closed
head injury. The initial priority will be to maintain cerebral perfusion pressure and
oxygenation to prevent secondary brain injury. After initial control of bleeding and
resuscitation the nature of the head injury will need to be evaluated. Depending on the
findings, consideration may need to be given to invasive ICP monitoring and ICP
management with hypertonic fluids. Significant electrolyte abnormalities can be
anticipated given the nature of the injury and interventions required. Finally as the patient
transitions from a resuscitative phase to a stabilization phase maintenance requirements
will need to be addressed in light of his identified injuries.
References
1.
2.
3.
4.
5.
Adrogue HJ, Madias NE. Hyponatremia. (2000) New England Journal of Medicine 342, 1581-1589.
Adrogue HJ, Madias NE. Hypernatremia. (2000) New England Journal of Medicine 342,1493-1499.
Finfer S, Bellomo R, Boyce N, et al. (2004) A comparison of albumin and saline for fluid resuscitation in
the intensive care unit. New England Journal of Medicine 350, 2247-2256.
Hebert PC, Wells G, Blajchman MA, et al. (1999) A multicenter, randomized, controlled clinical trial of
transfusion requirements in critical care. transfusion requirements in critical care investigators,
canadian critical care trials group. New England Journal of Medicine. 340, 409-417.
Kaplan LJ and Kellum JA. (2010) Fluids, pH, ion and electrolytes. Current Opinion in Critical Care. 16,
323-331
54
SOCCA Residents Guide 2013
Questions
11.1 All of the following would indicate hypovolemia EXCEPT:
A.
B.
C.
D.
BUN: Creatinine >20
Urine osmolality >350 mOsm/L
Urinary sodium <20 mEq/L
Fractional excretion of sodium (FENa) <1%
11.2 All of the following are associated with hyperkalemia EXCEPT:
A.
B.
C.
D.
Alkalosis
ACE inhibitors
Rhabdomyolysis
Hypoaldosteronism
11.3 Euvolemic hyponatremia is most likely associated with:
A.
B.
C.
D.
E.
Congestive heart failure
Cirrhosis
Aldosterone deficiency
SIADH
Diabetes insipidus
11.4 The need for a massive transfusion contributes to overall mortality by all of the following EXCEPT:
A.
B.
C.
D.
Reflects that the patient is in shock
Causes microaggregate-induced acute lung injury
Blood is immunosuppressive
Bradykinin antagonism results in irreversible hypotension
11.5 The available evidence for the use of albumin in critically ill patients suggests all of the following EXCEPT:
A.
B.
C.
D.
Overall lack of superiority of albumin over crystalloid
Higher cost for albumin
Potential for harm with use of albumin in subgroups with head trauma
Clear clinical superiority of crystalloid over albumin
55
SOCCA Residents Guide 2013
12. Gastrointestinal Bleeding
Maria A. De la PeГ±a, M.D and Miguel Cobas, M.D
70y/o female ICU day number 5, history of subdural hematoma, intubated, presents
increased bloody nasogastric tube (NGT) output with hemodynamic instability.
Resuscitation is initiated and endoscopy shows diffuse superficial gastric ulcerations,
not actively bleeding.
---------------------------------------------------------------------------------------------Objectives:
• Determine risks factors and pathophysiology associated with upper gastrointestinal bleeding in the
critical care setting.
• Recognize different prophylactic measures with benefits and disadvantages.
• Identify diagnostic tools, initial approach and management of acute upper gastrointestinal bleed.
Gastrointestinal (GI) bleeding may either result in admission to the ICU or develop during the course of the
ICU stay. Most commonly it results from upper gastrointestinal bleeding (UGIB). Use of GI prophylaxis as
well as improvement in ICU care such as fluid resuscitation, nutrition and cardiac care, have led to a better
maintenance of microcirculation and tissue oxygenation on critically ill patients. The incidence of UGIB is
increased in high-risk patients presenting with coagulopathy, severe burns, mechanical ventilation, head
injury, prior history of UGIB.
Prospective studies have classified overt bleeding (blood in nasogastric tube, melena, mild drop in
hemoglobin over several days, present in 5-25% of critically ill patients without prophylactic therapy) from
clinically significant bleeding (overt bleeding with hemodynamic changes, >2g/dl fall in hemoglobin
requiring blood transfusion, present in 1.5%-3% of ICU patients). The first step in management is
maintenance of hemodynamic stability. Once resuscitation is underway, medical history and physical
examination should be performed to determine the source of bleeding. Hemostatic abnormalities must be
corrected. Esophagogastroduodenoscopy (EGD) is performed for suspected UGIB and may be diagnostic
and therapeutic. Colonoscopy is done for most cases of lower gastrointestinal bleeding (LGIB).
Evaluation for small bowel source of bleeding using enteroclysis (fluoroscopic X-ray of the small
intestine), enteroscopy or capsule endoscopy is indicated if the initial procedures are negative. Bleeding is
controlled by local measures. Angiographic and surgical interventions are reserved for refractory cases.
Recombinant factor VIIa may be employed when all conventional measures fail.
I.
Epidemiology
A. Incidence: 1.5% of critically ill patients. Currently decreasing due to improved ICU care,
increased use of GI prophylaxis, aggressive resuscitation, maintenance of hemodynamics, early
enteral feedings.
B. Causes
1. UGIB
a) Peptic ulcer disease ( gastric, duodenal ulcers)
b) Erosive esophagitis
c) Stress related mucosal damage
d) Diffuse superficial gastritis
e) Variceal bleeding
f) Mallory Weiss tears
g) Vascular malformations
h) Carcinomas
i) Hemobilia
j) Aortoenteric fistula
k) Pancreatic pseudocyst or pseudoaneurysm
2. 2. LGIB
a) Diverticular disease
b) Hemorrhoids
c) Vascular ectasia
d) Angiodisplasia
56
SOCCA Residents Guide 2013
e)
f)
g)
h)
Ischemic/inflammatory colitis
Neoplasm/ polyps
Anorectal (hemorroids, fissures)
Upper GI source
II. Risk Factors
A. Prolonged mechanical ventilation (>48h), positive pressure may induce splachnic hypoperfusion
B. Coagulopathy
C. Use of vasopressor: dopamine, epinephrine, norepinephrine, vasopressin etc
D. Severe burns ( more than 35% of body surface)
E. Neurologic trauma
F. Multiple trauma
G. Multiple organ failure specially hepatic and renal
H. Posttransplant
I. Sepsis
J. Corticosteroid use (more than 250mg/day of hydrocortisone or equivalent)
K. Prolonged duration of NG tube placement
III. Prophylaxis
A. Optimization of hemodynamic status, systemic perfusion and oxygenation
B. Early provision of enteral nutrition
C. Use of pharmacologic prophylaxis
1. Indicated in ICU patients presenting:
a) Coagulopathy: platelets <50,000, INR >1.5, PTT >2times control value
b) Mechanical ventilation for more than 48h
c) History of GI ulceration or bleeding within the past year
d) Two or more of these factors: Sepsis, ICU admission >1week, occult GI bleed for
>6days, steroid therapy (>250mg/day hydrocortisone or equivalent)
2. Options for prophylaxis: Histamine blockers, proton pump inhibitors, sucralfate, antacids
a) Histamine Blockers: effective, may cause alteration in drug metabolism (cytochrome
P450), altered platelet function, confusion, may develop tolerance. Famotidine is the
most potent
b) Proton pump inhibitors: Most potent anti-secretory agents, safe, less drug interactions.
c) Antacids: not preferred because need frequent administration, can interact with oral
medications, diarrhea, phosphate binding, potential magnesium toxicity
d) Sucralfate: Can reduce bioavailability of oral medications, may clog feeding tubes
3. Adverse effects of acid suppression: Increased incidence of nosocomial pneumonia, increase
gastric PH facilitates bacterial growth
D. Avoidance of ulcerogenic medications: Corticosteroids, slow release potassium, non-steroidal antiinflammatory agents among the most common causative agents.
IV. Diagnosis:
A. History
a) Hematemesis: indicates UGIB
b) Melena: UGIB or LGIB from right colon
c) Hematochezia: brisk UGIB (10%) or LGIB (90%)
B. Gastric lavage: Negative only if no blood returns in effluent that contains bile.
1. In absence of bile, bleeding from a duodenal source with a competent pylorus cannot be
excluded.
C. Esophagogastroduodenoscopy (EGD)
D. Tagged red cell scans
E. Mesenteric angiogram
F. Enteroclysis/Enteroscopy/Capsule endoscopy
G. Colonoscopy/ Anoscopy
H. Digital rectal examination
V.
V. Treatment
A. Assess hemodynamic stability
B. In unstable patients:
57
SOCCA Residents Guide 2013
C.
D.
E.
F.
G.
H.
I.
J.
Initiate resuscitation, assess/secure airway, ensure ventilation, establish intravenous access, begin
fluid resuscitation, obtain and send blood for laboratory studies, arrange for the necessary blood
products.
Insert NGT and start gastric lavage, Obtain history with physical examination and initiate studies
to localize the source of bleeding.
1. IV access: preferred: 2 large bore (14-gauge) peripheral catheters in the antecubital fossa.
Alternative: Central venous introducer. Do not use: triple lumen central venous catheter
2. Laboratory studies: sent at time IV access obtained. Hematocrit, electrolytes, creatinine, liver
function test, coagulation tests, platelets, type and cross matching
3. Fluid resuscitation: Initially use isotonic crystalloid. If need for resuscitation persists use
specific blood components based on hematocrit, prothrombin time and platelet count.
a) Goals of transfusion: Hemodynamic stability. Hematocrit > 25%, platelets >50,000,
prothrombin time near normal always considering comorbidities.
In stable patients: Obtain history, perform physical exam and initiate workup to localize source of
bleeding. H.pylori screening mandatory.
Pharmacologic therapy: proton pump inhibitor infusion: esomeprazole 80mg IV bolus then 8mg/
hr infusion. If no rebleeding in 24h, switch to oral pantoprazole 40mg/day or esomeprazole
20mg/day.
Esophagogastroduodenoscopy (EGD): Recommended within 24h of UGIB
1. Benefits:
a) May establish diagnosis and control of bleeding through injection of epinephrine,
sclerosants, bipolar electrocoagulation, band ligation, thermal therapy (80% successful)
b) May predict risks of rebleeding
2. Disadvantages:
a) Need for sedation, may compromise ventilation
b) Need for a stomach empty of food and blood, Erythromycin 3mg/kg usually given
30minutes prior to procedure
3. Rebleeding:
a) Usually occurs within 48 hours In 15-20% of patients.
b) Endoscopic findings: active bleeding vessel (55% recurrence), non-bleeding visible
vessel (43% recurrence), ulcer containing visible vessels, ulcers with overlying clot,
arterial bleeding involvement.
c) Ulcer location: High on lesser curve (left gastric artery), posterior-inferior wall of
duodenum (gastroduodenal artery)
d) Treatment of rebleeding: repeat endoscopy, surgery, angiographic occlusion.
4. Failure to establish diagnosis: bleeding that has stopped, bleeding source beyond the reach
of the endoscope.
Balloon tamponade: for varices, utilices the Sengstaken-Blakemore tube
Adjunctive pharmacotherapy for variceal bleeding: Octreotide, Vasopressin
Surgical options:
1. Indications:
a) Severe life-threatening hemorrhage not responsive to resuscitative efforts
b) Failure of medical therapy and endoscopic hemostasis with persistent recurrent
bleeding
c) Coexisting reason for surgery: perforation, obstruction, neoplasm
d) Second hospitalization for peptic ulcer hemorrhage
2. Gastrectomy for stress gastritis
3. Porto-systemic shunts (most often trans-internal jugular, or TIPPS) for esophageal varices
Options for treatment of LGIB
1. Colonoscopy with laser or thermal coagulation
2. Angiographic infusion of vasoconstrictors/ embolization
3. Surgical resection
Factor VIIa: For refractory bleeding unresponsive to conventional measures
VI. Outcome:
A. Poor prognostic factors:
1. Severe hemorrhage
2. Shock at presentation
58
SOCCA Residents Guide 2013
3. Presence of coagulopathy
4. Unidentified source of bleeding
5. Advanced age
6. Multiple organ failure: hepatic, renal
B. Mortality: 6-10%
VII. Future therapies
A. Use of growth factors: EGF, transforming growth factor alpha formulations, Endothelin-1
antagonists
B. Early detection of hypoperfusion: measurement of blood flow via NGT as indicator of bowel
ischemia
Discussion
UGIB is an entity in which an ounce of prevention is worth a pound of cure; it is important to identify
patients at risk, those on mechanical ventilation and brain injured patients being especially susceptible.
Its incidence has decreased due to early recognition of these risks factors, enteral nutrition, adequate
prophylaxis and improved ICU care.
Different diagnostic tools, as well as medical and invasive treatments are available, yet EGD remains the
most commonly used diagnostic and therapeutic tool. Rational use of blood products, and correction of
coagulopathy are fundamental in the care of these patients. The goal of management is to restore
hemodynamic stability sometimes as a means to begin performing diagnostic and more invasive
procedures.
READING LIST
1.
2.
3.
4.
5.
6.
7.
Ali T, Harty R. Stress induced ulcer bleeding in critically ill patients. Gastroenterol Clin N am 38 2009
245-265.
Sesler JM. Stress related mucosal disease in the intensive care unit: an update on prophylaxis. AACN adv Crit
Care 2007; 18: 119-126.
Lin CC, Lee YC, Lee H, Ho WI, Chen TH, Wang HP. Bedside colonoscopy for critically ill patients with acute
lower gastrointestinal bleeding. Intesive care Med 2005 31;743-46.
Spirt MJ, Guth PH, Randall G, Leung FW. Gastroduodenal perfusion and mortality in mechanical
ventilation-dependent patients with systemic inflammatory response syndrome. Dig Dis Sci 2004;
49:906-913.
Faisy C, Guerot E, Diehl JL, Iftimovici E, Fagon JY. Clinically significant gastrointestinal bleeding in critically
ill patients with and without stress-ulcer prophylaxis. Intensive Care Med 2003; 29:1306-1313.
Kahn JM, Doctor JN, Rubenfeld GD. Stress ulcer prophylaxis in mechanically ventilated patients: Integrating
evidence and judgment using a desicion analysis. Intensive Care Med 2006; 32:1151-1158.
Daley RJ, Rebuck JA, Welage LS, Rogers FB. Prevention of stress ulceration: Current trends in critical care.
Crit Care Med 2004; 32:2008.
Questions
12.1 Factors resulting in a reduced incidence of stress gastritis in the ICU setting include:
A. Greater awareness of the patophysiology of the disease
B. Increased use of prophylactic agents
C. Early use of enteral feeds
D. Improvement in resuscitation
E. All of the above
12.2 Factors involved in the patophysiology of stress-induced ulcer bleeding:
A. Ischemia reperfusion injury
B. Oxydative stress / free radicals
C. Alteration of COX-2 defense regulation
D. Mucosal blood flow
E. All of the above
12.3 65 year old female on mechanical ventilation in the ICU with an episode of bright blood output via NGT 3 days ago.
Endoscopy was negative. Currently presenting recurrence of UGIB with hemodynamic instability. What would be the next
step:
A. Repeat endoscopy
B. Surgical treatment
C. Angiography
59
SOCCA Residents Guide 2013
D.
E.
Initiate resuscitation
Colonoscopy
12.4 An 86-year old female is admitted to the ICU after presenting with hematochezia. An EGD is negative.
Colonoscopy is unsuccessful due to the presence of large amount of blood and stool in the colon. The patient remains
hypotensive despite agressive resuscitation. A mesenteric angiogram was attempted but is unsuccessful due to severe
atherosclerotic disease of the mesenteric vessels. The next step in management of this patient is:
A. Subtotal colectomy
B. Administration of DDAVP
C. Initiation of vasopressin infusion
D. Bolus 300 micrograms of octreotide followed by an infusion
E. Administration of recombinant factor VIIa
12.5 A 49-year old alcoholic is admitted to the ICU after developing an UGIB following severe retching during a binge
episode. After resuscitation, an EGD is performed which shows linear tears on the gastric side of the gastroesophageal
junction. The most important aspect of management is:
A. Expectant observation
B. Administration of antiemetics
C. Raising the gastric PH
D. Endoscopic thermal coagulation of the tear
E. Esophageal resection
60
SOCCA Residents Guide 2013
13. Thromboembolic Disease
Gustavo Angaramo, MD
CASE: An otherwise healthy 28 years old man is admitted to the trauma unit due to a
thoracic spine injury with paraplegia (T10) after a fall from a ladder. Three days after
admission, he complains of shortness of breath. His chest radiograph (CXR) and arterial
blood gas analysis are normal. A ventilation- perfusion (V/Q) scan is obtained, and is read
as low probability of embolism. Three days after admission, he develops sudden respiratory
distress with hypotension and hypoxia. A helical computed tomography (CT) scan reveals a
right inferior pulmonary artery embolus (PE).
INTRODUCTION: Although the exact incidence of PE is uncertain, it is estimated that 600,000 episodes
occur each year in the United States (U.S), resulting in 100,000 to 200,000 deaths. The majority of
preventable deaths associated with PE are related to a missed diagnosis rather than to a failure of existing
therapies. Additionally, screening studies for venous thromboembolism (VTE) yet lack sensitivity and
specificity. Several noninvasive diagnostic techniques have been developed to improve the accuracy of the
diagnosis; however, no single noninvasive diagnostic test is sensitive or specific enough for the diagnosis in
all patients. The cornerstone of management involves identification of high risk groups and treatment with
adequate prophylactic measures.
I.
Scope of the Problem
A. Incidence:
1. Underestimated, based on unselected autopsy studies–over 70% of those who expire with PE
are not suspected of PE prior to death
2. The prevalence of VTE is 3% on intensive care admission and 10% over the intensive care
unit stay.
3. Best recent estimate (2003): 23 per 100,000 leading to overall estimate of 100,000 cases per
year in U.S.
4. Mortality of untreated PE is approximately 30%, once diagnosed, treatment mortality is 2,5%.
II.
Risk Factors for VTE (Table 13-1)
A. Age greater than 40 years old
B. History of VTE
C. Surgery requiring > 30 minutes of anesthesia
D. Prolonged immobilization
E. Cerebrovascular accident
F. Congestive heart failure
G. Cancer
H. Fracture of pelvis, femur, or tibia
I. Obesity
J. Pregnancy or recent delivery
K. Estrogen therapy
L. Inflammatory bowel disease
M. Genetic or acquired thrombophilia: deficiencies of antithrombin III, protein C, or protein S,
including recently described activated protein C resistance present in up to 7% of the
asymptomatic population (occurs secondary to a mutant factor V that doesn’t bind protein C)
61
SOCCA Residents Guide 2013
Table 13-1. Rules for Predicting the Probability of Embolism
Variable Risk Factors
Points
Clinical signs and symptoms of deep venous thrombosis
3
An alternative diagnosis deemed less likely than pulmonary embolism
3
Heart rate >100 beats/min
1.5
Immobilization or surgery in the previous 4 wk
1.5
Previous deep venous thrombosis or pulmonary embolism
1.5
Hemoptysis
1
Cancer (receiving treatment, treated in the past 6 mo, or palliative care)
1
Clinical Probability
Low
< 2.0
Intermediate
2.0-6.0
High
> 6.0
Adapted from Wells et al.
III.
Pathophysiology
A. Virchow’s triad: venous stasis, hypercoagulable state, and endothelial injury
B. Involved vessels: usually lower extremity including pelvis, may involve upper extremity when
associated with central venous catheterization.
IV.
Diagnosis of VTE/PE
A. Clinical findings (low sensitivity and specificity):
1. VTE–distal limb edema/pain, differential limb circumference, Homan’s sign, distended
collateral veins, increased temperature if infection present.
2. PE–dyspnea, pleuritic pain, tachypnea (3 most common symptoms), tachycardia, hypoxemia,
hypocarbia, hemoptysis, infiltrate on CXR.
3. The use of empirical standardized assessments of probability allows patients to be classified
into three groups on the basis of the prevalence of PE. (Table 1)
4. Low clinical probability (prevalence of 10% or less), intermediate probability (30%), high
probability (70% or higher).
B. Diagnosis of VTE in lower and upper extremities.
1. Contrast venography
a) The reference standard for the diagnosis of VTE.
b) Noninvasive tests have supplanted the venogram
2. Impedance plethysmography (IPG)
a) Overall sensitivity and specificity of 83% and 92% respectively.
b) False positive results with tensing of the leg muscles, reduces arterial flow and
compression by an extravascular mass.
3. Ultrasonography with color Doppler flow (duplex scan)
a) Sensitivity for proximal DVT is 97%; specificity is 99%
62
SOCCA Residents Guide 2013
b) Can not distinguish between occlusion from external pressure vs. thrombosis
c) Less sensitive in identifying asymptomatic, isolated calf vein and recurrent thrombosis.
d) Predicted value is greater than that of IPG.
4. Ventilation perfusion scanning
a) V/Q scanning has had a central role in the diagnosis of embolism for almost three
decades and is a valuable tool when the results are definitive.
b) A normal V/Q scan essentially rules out the diagnosis of embolism, and a high
probability one is strongly associated with the presence of embolism.
c) However large trials have demonstrated that most patients with suspected PE who
undergo a V/Q scan do not have findings that are considered definitive.
d) High clinical suspicion and high probability lung scan: PE in 96%
e) Low clinical suspicion and low probability lung scan: PE in 4%
5. Contrast enhanced helical CT scan
a) The reported sensitivity ranges from 57 to 100% and its specificity ranges from 78 to
100%.
b) The sensitivity and specificity vary with the location of the emboli, ranging from 90% for
emboli involving the main and lobar pulmonary arteries to much lower rates for
segmental and subsegmental pulmonary vessels.
c) A normal CT scan may indicate a substantially reduced likehood of embolism but cannot
be used to rule out the possibility of embolism with the same degree of certainty that a
negative V/Q scan provides.
6. D-dimer: degradation products of cross linked fibrin, endogenous marker for fibrinolysis
a) Highly sensitive but nonspecific screening test for suspected VTE.
b) Elevated levels are present in nearly all patients with embolism but are also associated
with many other situations, including advance age, pregnancy, trauma, postoperative
period, inflammatory states, and cancer.
7. Troponin T (cTnT)
a) In a small series, elevated troponin T identified a higher risk group among normotensive
PE patients; those with elevated cTnT had more RV dilation, higher end diastolic RV/LV
ratios by echocardiography and suffered all the deaths so indeed a cTnT concentration >
0,01 ng/ml has a marked impact on short term prognosis.
8. Brain Natriuretic Peptide (BNP)
a) BNP is released from cardiac ventricular cells in response to high ventricular filling
pressures, so is an indicator of myocardial wall stress and hypoxia.
b) In one series, serum BNP was elevated above normal reference values for age and sex in
80% of patients with acute PE and significant RV overload.
9. Echocardiography
a) Massive PE is associated with right ventricle (RV) enlargement, RV free wall hypokinesis
with preservation of apical contractility, dilation of pulmonary arteries, and elevated RV
pressure.
b) According to the International Cooperative Pulmonary Embolism Registry, RV
hypokinesis predicted an increased risk of death within 30 days in patients with systolic
systemic pressure > 90 mm Hg. 30 day survival rates in patients with or without RV
hypokinesis were 84% and 91% respectively.
10. Pulmonary angiography–the gold standard
a) It requires expertise in performance and interpretation, is invasive, and has associated
risks.
b) Angiography is reserved for a small subgroup of patients in whom the diagnosis of
embolism cannot be established by less invasive methods.
V.
Prophylaxis
A. Pharmacologic agents
1. Heparins:
a) Formulations:
(1) Low-dose unfractionated heparin (LDH)
(a) Doses: 5000 IU SQ Q 8-12 hr
(b) Monitoring: not needed for anticoagulation
(2) Low molecular weight heparin (LMWH)
63
SOCCA Residents Guide 2013
(a) Doses: 30 mg SQ Q 12 hr or 40 mg SQ QD (enoxaparin)
(b) Monitoring: not needed for anticoagulation
b) Complications
(1) Bleeding–5% risk of major bleeding with continuous IV or SQ unfractionated
(2) Thrombocytopenia–occurs in 3% with unfractionated heparin(less with LMWH);
secondary to immune IgG-mediated response; may lead to arterial thrombosis.
(3) Resistance to heparin–antithrombin III deficiency.
(4) Osteoporosis–in 30% treated with long-term unfractionated heparin therapy
2. Oral anticoagulation
3. Warfarin
a) Doses: 2-10 mg po daily or every other day; adjust to keep PT ≥1.5 control
4. Complications:
a) Bleeding
b) Skin necrosis–in patients with protein C or S deficiency
B. Mechanical agents:
1. Early ambulation, leg elevation, physiotherapy
2. Graduated compression (elastic) stockings
3. Intermittent pneumatic compression (sequential compression device, SCD)
4. Prophylactic IVC filter–in those for whom anticoagulation is contraindicated, or who
experience recurrences despite adequate anticoagulation.
VI.
Treatment
A. Local measures–elevation of extremity, warm compresses; to be used as adjunct to other measures
B. Anticoagulation (should begin before diagnostic studies if PE is intermediate or high probability):
1. Adjusted-dose intravenous unfractionated heparin
a) 80 U/kg IV bolus; 18U/kg/hr IV.
b) Adjust to keep PTT ≥1.5 - 2.0 times control
c) Follow with warfarin within 24 hours for at least 3 months
2. Low molecular weight heparin–fixed dose, SQ regimens proven as effective treatment for
VTE and PE, usually not monitored
a) Fondaparinux: subcutaneous(sc) qd; 5 mg if < 50 kg, 7,5 if 50 to 100 kg, 10 mg if >
100 kg.
b) Enoxaparin: sc, 1 mg/kg q12 hours, adjusted to creatinine clearance.
c) Dalteparin: sc, 200U/kg q24hours.
d) Tinzaparin: sc, 175 mgU/kg q24hours.
3. Lepirudin: 0,1 mg/kg/hr IV, no bolus, follow aPTT
4. Argatroban: 2 mcg/kg/min, no bolus, follow aPTT.
C. Thrombolytic therapy
1. Considered in patients deteriorating despite aggressive medical therapy; and normotensive
patients with evidence of RV impairment.
2. Agents: Streptokinase; Urokinase; Alteplase.
D. Inferior Vena Cava filter
1. For those patients who have absolute contraindications to anticoagulant therapy, who develop
serious bleeding problems with anticoagulation, or experience recurrent emboli despite
adequate anticoagulation.
2. Length of filter therapy is still unknown, but recent evidence suggests that some filters can be
removed after as long as 350 days.
E. Pulmonary embolectomy
1. For those with serious systemic symptoms from PE, and in whom thrombolysis is
contraindicated
2. Historically accompanied by a high mortality rate (30%)
3. Recent series with better case selection have reported survivals rates between 80 to 89%.
F. Percutaneous catheter devices
1. mechanically reestablishing pulmonary blood flow with percutaneous devices.
2. None of the available devices has regulatory agency approval for PE or has been tested in
randomized controlled trials.
VII. Current recommendations:
64
SOCCA Residents Guide 2013
Table 13-2A: Current Recommendations - general
Risk Category
Details of Group
Prophylaxis Recommended
Low risk general
surgery
• Minor surgery
• Age <40
• No clinical risk factors
Early ambulation
Moderate risk
general surgery
• Major surgery
ES, LDH (2 hr before and 12 hr after surg) or IPC;
• Age >40
should use ES and PIC during operation and
• Additional clinical risk factors postoperative period
High risk general
surgery
• Major surgery
LDH (Q 8 hr) or LMWH (or IPC for those prone to
• Age >40
wound complications)
• Additional clinical risk factors
Very high risk
general surgery
• Major Surgery
Perioperative warfarin (INR 2.0-3.0)
• Age > 40
• Pre-existing embolism
LDH = low-dose unfractionated heparin, LMWH - low-molecular weight heparin, IPC = intermittent
pneumatic compression, ES = graduated compression (elastic) stockings
Table 13-2B: Current Recommendations - specific
Clinical Situation
Total hip replacement surgery
Recommended Prophylaxis
LMWH (BID), warfarin (INR 2.0-3.0), or adjusted dose heparin; adjuvant
ES or IPC may provide additional benefits
Total knee replacement Surgery LMWH (BID) or IPC
Hip fracture surgery
LMWH or warfarin (INR 2.0-3.0; IPC may provide additional benefit
Intracranial neurosurgery
IPC В± ES; LDH acceptable alternative; combo may be more effective
Acute spinal cord injury
Adjusted dose heparin or LMWH
Multiple trauma patients
LMWH preferable to IPC or warfarin if complex pelvic/lower extremity
injuries; IVC for very high risk groups; may consider serial duplex
ultrasonography surveillance
Myocardial infarction
LDH or full dose anticoagulation
Ischemic stroke
LDH or LMWH; IPC and ES probably OK
General medical patients with risk LDH or LMWH
factors
Patients with long-term indwelling Warfarin to prevent axillary-subclavian thrombosis
central venous catheters
LDH = low-dose unfractionated heparin, LMWH - low-molecular weight heparin, IPC = intermittent
pneumatic compression, ES = graduated compression (elastic) stockings
VIII. CONCLUSIONS and RECOMMENDATIONS
Suspected PE in the stable critically ill patient algorithm - See Figure 13-1.
If patient is unstable, obtain lower extremities duplex, if positive anticoagulate; if negative anticoagulate
anyway until stability allows transport for CT. Consider echocardiogram to rule out right ventricular strain.
REFERENCES:
65
SOCCA Residents Guide 2013
1.
2.
3.
4.
5.
6.
7.
8.
Fedullo PF, M.D., Tapson VF, M.D. The
Evaluation of Suspected Pulmonary Embolism.
N Engl J Med 2003; 349:1247-56.
Rosenow EC. Venous and pulmonary
thromboembolism: an algorithmic approach to
diagnosis and management. Mayo Clin Proc
1995; 70:45-49.
Clive Kearon, MB, PhD; Susan R. Kahn, MD;
Giancarlo Agnelli, MD;Samuel Goldhaber, MD,
FCCP; Gary E. Raskob, PhD;and Anthony J.
Comerota, MD. American College of Chest
Physicians Evidence-Based Clinical Practice
Guidelines (8th Edition).Antithrombotic therapy
for venous Thromboembolic disease. CHEST
2008; 133:454S–545S.
Geerts WH, Jay RM, Code KI, Chen E, Szalai JP,
Saibil EA, et al. A comparison of low-dose
heparin with low-molecular-weight heparin as
prophylaxis against venous thromboembolism
after major trauma. N Engl J Med 1996;
335:701-7.
David J. Carlbom, MD; and Bruce L. Davidson,
MD, MPH, FCCP. Pulmonary Embolism in the
Critically Ill. CHEST 2007; 132:313–324.
Deborah Cook, MD; Mark Crowther, MD;
Maureen Meade, MD; Christian Rabbat; Lauren
Griffith, MSc; David Schiff, MD; William Geerts,
MD; Gordon Guyatt, MD. Deep venous
thrombosis in medical-surgical critically ill
patients: Prevalence, incidence, and risk factors.
Crit Care Med 2005 Vol. 33, No. 7.
Wells PS, Anderson DR, Rodger M, et al.
Derivation of a simple clinical model to
categorize patients’ probability of pulmonary
embolism: increasing the model’s utility with
the SimpliRED D-dimer. Thromb Haemost
2000;83:416-20.
Guyatt GH et al. Executive Summary:В Antithrombotic Therapy and Prevention of Thrombosis, 9th ed:
American College of Chest Physicians Evidence-Based Clinical Practice Guidelines. CHEST 2012;141
(suppl 2): 7S-47S.
QUESTIONS:
13.1 A 66-year-old man is 5 days post repair of an abdominal AAA and develops dyspnea with pleuritic chest pain. Vitals:
T = 39C, BP - 110/60, P = 120. CXR and ECG are unrevealing. ABG on room air shows: pH = 7.48, PaCO2 = 32, PaO2 =
72. What should be done next?
A.
B.
C.
D.
E.
Intravenous heparin and no further diagnostic testing.
Intravenous heparin followed by either V/Q scanning or deep venous studies.
Subcutaneous heparin (5000 IU BID) followed by oral warfarin and observation.
Impedance plethysmography or duplex ultrasonography and, if no abnormalities detected, repeat every 2 to 4
days for 10 days.
Pulmonary arteriography.
13.2 A 68-year-old woman with acute dyspnea 10 days after hemiplegia from a non-hemorrhagic stroke has a V/Q scan
demonstrating a high probability for PE, with two segmental perfusion defects with normal ventilation to these areas. She
has a history of 90 pack-years of smoking, and has received heparin, 5000 IU SQ Q12 hrs since her admission. Which
one of these measures is most appropriate at this point?
A.
Discontinue heparin, as the dyspnea may be a complication of therapy.
66
SOCCA Residents Guide 2013
B.
C.
D.
E.
Do a pulmonary arteriogram, because a V/Q scan is unreliable in heavy smokers.
Repeat the V/Q scan in several days.
Anticoagulate with IV heparin (PTT ≥1.5 normal), then start warfarin in 5 to 7 days.
Anticoagulate with IV heparin (PTT≥1.5 normal), then start warfarin the same day.
13.3 Which one of the following statements is true about the initial clinical manifestations of acute PE?
A.
B.
C.
D.
E.
Normal findings on deep venous studies including venography of both lower extremities exclude the possibility
of PE.
Hypoxemia (PaO2 <80) is present in almost all patients with PE.
More than half of the patients with features typical of acute PE do not have it.
Hormone replacement therapy in postmenopausal women is a risk factor for venous thromboembolism.
Patients with eventually fatal PE have characteristic manifestations that suggest its presence.
67
SOCCA Residents Guide 2013
14. Bleeding in the ICU
Anthony delaCruz, MD; Scott Wolf, MD; Sherif Afifi, MD
A 53-year-old man is admitted to the ICU for acute stroke with a middle
cerebral artery thrombosis found on CT scan. Patient was outside the
therapeutic window to thrombolytic therapy. He was stabilized in the ICU
and further work up revealed an ipsilateral internal carotid aneurysm with
dissection as the likely originator of the thrombus. Three days after the
stroke, a follow up head CT demonstrated hemorrhagic conversion of the
ischemic stroke with 1.8cm midline shift. He was taken to the OR for
decompressive craniotomy.
The balance between hemostasis, thrombosis and hemorrhage is a constant challenge in the critically ill.
The physiology and biochemistry of adequate hemostasis while preventing spontaneous thrombosis is
complicated. Consider the mechanisms of hemostasis and how critical illness, comorbid illnesses, and
medication affect these mechanisms. Consider the tools available to address the specific etiology
contributing to the bleeding patient. Venous thromboembolism is discussed elsewhere in this manual.
I.
Physiology and Biochemistry of Hemostasis
A. Intact Endothelium inhibits platelet aggregation and thrombosis via
1. Nitric oxide/EDRF (endothelial derived relaxation factor)
a) reduces intracellular Ca+ thus suppresses IIb/IIIa
2. Arachidonic acid
a) Converting arachidonic acid into prostacyclin via COX 1&2 decreasing platelet AMP
levels
3. Ecto-ADPase (CD39) on intact cell surface
a) limits plasma ADP and ATP
4. Secretion of anticoagulants/ antithrombotics:
a) Antithrombin III
b) Heparin
c) Tissue plasminogen activator
B. Endothelial damage reduces the above processes and exposes collagen and tissue factor for
platelet activation and thrombus formation
1. Surgery
2. Inflammation
3. Sepsis
4. Malignancy
C. Tissue Factor pathway of platelet activation and thrombus formation
1. Protein disulfide isomerase (PDI) on the exposed vessel wall expresses Tissue Factor and
generates fibrin
2. Tissue Factor complexes with Factor VIIa starts thrombin generation
3. Thrombin captures more platelets and cleaves Par4 leading to platelet activation and release
of alpha and dense granules
4. The cycle repeats
D. Collagen Pathway of platelet activation and thrombus formation
1. Collagen (of the exposed subendothelial matrix) comes into contact with platelet glycoprotein
VI
2. vonWillebrand factor (of the exposed subendothelial matrix) comes into contact with platelet
glycoprotein Ib-V-IX
3. Both of the above lead to more platelet capture
4. Platelet-platelet interaction mediated by О±iibОІ3 to fibrinogen and vonWillebrand factor leads
to more platelet activation and the release contents of alpha and dense granules:
a) ADP
b) Serotonin
c) Calcium cations
d) Epinephrine
68
SOCCA Residents Guide 2013
e) The cycle repeats
E. Clot stabilization
1. Thrombin produced from the intrinsic and extrinsic pathways converge (common pathway) to
activate fibrin conversion and crosslinking (via factor XIII)
2. Redundancy of pathways with Tissue Factor activation
3. Platelets, rbc’s and plasma components are incorporated in the fibrin mesh to form a clot
4. Thrombin inhibits fibrin degradation
F. Fibrinolysis
1. Plasminogen is incorporated into the fibrin clot as it forms
2. Activated into Plasmin by intrinsic tissue plasminogen activator (t-PA), urokinase, and
factors: XIa, XIIa, Kallikrein
3. Contributes to the balance of hemostasis vs. thrombosis
II.
Etiologies of Bleeding
A. Technical Bleeding (Surgical Bleeding)
B. Trauma
C. Surgical incisions and anastomosis
1. Integrity of mechanical hemostasis (suture lines)
2. Acquired coagulopathies
3. Arterial bleeding vs. venous bleeding vs. oozing
D. May have to return to the O.R. for ligation
E. Spontaneous bleeding may need surgical hemostasis
1. Clipping of upper GI bleed
2. Embolization of a cerebral aneurysm
F. Medical Bleeding
1. Spontaneous bleeding
2. GI bleeding
3. Hemorrhagic conversion of ischemic stroke
4. Coagulopathies
a) Acquired
b) Congenital
c) Pharmacologic
III.
Congenital Coagulopathies
A. Hemophilia
B. Hemophilia A: Factor VIII deficiency
C. Hemophilia B: Factor IX deficiency
D. Von Willebrand disease
IV.
Acquired Coagulopathies
A. Factor Deficiency
1. Liver failure
2. Vit K deficiency
3. Factor consumption
a) DIC
b) Ongoing hemorrhage
4. Pharmacologic (anticoagulants)
a) Coumadin
(1) Increase in PT via intrinsic pathway (factor VII)
(2) Inhibits production of factor: II, VII, IX, X
b) Heparin- antithrombin III activator thus thrombin inhibitor
(1) Unfractionated
(2) Increase in aPTT via extrinsic pathway
c) Low Molecular weight-Enoxaparin (Lovenox)
(1) Factor Xa inhibitor via binding antithrombin at different site than heparin
(2) Monitored with anti-Xa assays
d) Direct thrombin inhibitors- Dabigatran (Pradaxa), Argatroban, Rivaroxaban (Xa
inhibitor)
69
SOCCA Residents Guide 2013
(1) Increase in PT and aPTT
Fibrinolytics- promote plasminogen conversion into plasmin
(1) tPA-alteplase
(2) Streptokinase-Streptase
B. Platelet deficiency
1. Consumption or destruction
a) Mechanical devices- balloon pumps
b) Autoimmune- HIT
2. Sequestration
a) Post cardiopulmonary bypass (CPB)
b) Hypersplenism
3. Decrease in production (marrow suppression)
a) Thrombocytopenic purpuras
b) Sepsis, MODS, ARDS, DIC
C. Platelet dysfunction
1. Hypothermia
2. Uremia
3. Acidosis
4. Pharmacologic
a) Heparin- antibody mediated (HIT)
b) Anti-platelets
(1) Aspirin inhibit thromboxane mediated platelet activity
(2) P2Y (2, 12) inhibitors- inhibits P2Y12 (ADP) mediated platelet activity
(clopidogril, prasugrel)
c) NSAIDS
(1) Inhibits arachidonic acid conversion by COX affecting platelet AMP mediated
platelet activation
d) Quinine
5. Post CPB
e)
V.
Blood Component Therapy and Pharmacologic hemostatic aids
A. One Unit of Packed Red Blood Cells:
1. 200ml of red blood cells, the rest is a solution to extend storage life
2. Hematocrit of approximately 60-70%
3. Will increase a 70kg adult Hg by 1g/dl
4. Citrate added as a Calcium binder to prevent clotting during storage
5. May decrease plasma Calcium until metabolized inducing hypocalcemia
B. One Unit of Plasma (FFP):
1. Coagulation factors, and components
2. Albumin
3. Fibrinolytic and complement components
4. Will decrease a PT/INR by 2-3%
5. Will increase factor levels by 10%
6. Has minimal fibrinogen, decreased Factor V and VIII
7. Will unlikely be able to correct a coagulopathy to an INR of 1
C. One pool (6 bags) of Cryoprecipitate
1. Is a concentrate of 6 Units of plasma (a pool of 6 bags)
2. Has higher levels of vWF, Factor VIII and fibronectin
3. 2100 mg of fibrinogen (350mg per bag)
4. Will increase fibrinogen by 45mg/dL
D. Platelets
1. Can be pooled for random donors
2. Can be apheresed from a single donor
3. One bag contains 4-6 units
4. Will raise a normal 70kg patient platelets 40-60k/uL
E. Vitamin K
1. Can be given to boost hepatic production of Factors II, VII, IX and X
2. Onset of action 2 hours if given IV, with maximal effect 6-12 hours later. Oral administration
70
SOCCA Residents Guide 2013
doubles the time to onset.
F. Recombinant human factor VIIa (Novo Seven)
1. Thrombin generator via the Tissue Factor pathway
2. Platelet activation via Par4 cleavage by thrombin
G. Factor VIII Inhibitor Bypassing Activity (FEIBA)
1. Activated prothrombin complex concentrate
2. Contains concentrated amounts of Factors: II, VII, IX and X
H. Antifibrinolytics (Clot Stabilizers)
1. Tranexamic acid (Cyclokapron) and Aminocaproic acid (Amicar)
2. Inhibits activation of plasminogen to plasmin thus inhibits fibrin degradation
VI.
Specific Drug Reversal Strategies
A. Aspirin
1. Consider last dose
a) ASA irreversibly binds COX activity of exposed platelets
b) Complete platelet turnover takes 10 days (10-15% new platelets/day)
c) May have overall adequate platelet activity after 30-40% of platelets are replaced i.e.
3-4days since last dose
2. Consider available Aspirin response assays
a) VerifyNow Aspirin Response Unit (ARU)
b) ARU > 550 indicates aspirin inhibition of platelets
c) Undetermined meaning when considering actively bleeding patients
3. Consider transfusing platelets
B. Clopidogril
1. Consider last dose
a) Clopidogril irreversibly inhibits P2Y12 (ADP subtype) receptor
b) Duration of action 3-7days affect on platelet activity
2. Consider available P2Y12 inhibition assays
a) VerifyNow P2Y12 whole blood assay
b) P2Y12 inhibition of >50% indicates adequate inhibition of platelets
c) Undetermined meaning when considering actively bleeding patients
3. Consider transfusing platelets
C. Coumadin
1. Rapid reversal possible with plasma
a) Associated with large volumes (only 2.5% INR correction per Unit)
b) Associated with acute lung injury
2. Slower reversal possible with Vitamin K
a) Less volume
b) Not associated with acute lung injury
c) Reversal on the order of hours
d) Effectiveness relies on intact hepatic factor synthesis
3. FEIBA (Factor VIII Inhibitor Bypassing Activity)
a) Contains concentrated Vit K dependent factors
b) Reliably able to normalize INR
D. Direct Thrombin or Xa inhibitors
1. Argatroban, Dabigatran and Rivaroxaban have no known antidotes
2. Anecdotal evidence of Novo Seven, FEIBA or a combination to be affective
3. Dabigatran is dialyzable in 2-3hrs
E. Fibrinolytics (tPA)
1. Antifibrinolytics can be used in tPA overdose
a) Tranexamic acid, aminocaproic acid
F. Protamine
G. Binds heparin to produce a protamine-heparin complex with no anticoagulant activity
1. May also bind to and disassociate heparin from the heparin-antithrombin III complex
2. 1mg of protamine will inactivate every 100U of active heparin
VII. Hemoglobin Goals in the ICU
A. Adequate tissue oxygenation is dependent on the cumulative performance of:
71
SOCCA Residents Guide 2013
1.
B.
C.
D.
E.
F.
Blood oxygen carrying content and capacity=
a) Cao2=(1.34 x hemoglobin concentration x SaO2) + (0.0031 x PaO2)
2. Oxygen delivery
a) Cardiac output x CaO2
3. Oxygen consumption requirements
a) metabolic demand in different physiologic states
4. Oxygen extraction ratio
a) Hemoglobin disassociation
Thus anemia is at least partly responsible for tissue hypoperfusion
Serious Hazards of Transfusion (SHOT) Study collected data from 1994-2004 analyzed 2630
transfusion events from over 27 million blood component transfusion
1. Incorrect blood component transfused 69.7%
2. Acute transfusion reaction 10.2%
3. Delayed transfusion reaction 9.7%
4. Transfusion related lung injury 6.2%
a) 1:2000-1:8000 incidence of plasma containing products
5. Transfusion-transmitted infection 1.8%
a) Bacterial infection most frequent
b) HIV risk 1:8 million
c) Hepatitis C risk 1:30 million
d) Hepatitis B risk 1:260,000
6. Post transfusion purpura 1.7%
7. Transfusion associated-graft versus host disease 0.5%
8. Unclassified 0.3%
TRICC trial (Transfusion Requirements in Critical Care, 1999)
1. Randomized study of 838 patients of 25 different Canadian ICU’s
2. Restrictive Hg transfusion triggers of 7g/dL vs.
3. Liberal Hg transfusion triggers of 10g/dL
4. Restrictive group received less transfusions 2.6U/patient vs. 5.6U/patient
5. Restrictive group had avg Hg of 8.5g/dL vs. 10.7g/dL
6. Decreased 30day and hospital mortality in the restrictive arm
Different pathologies may require different goals
1. Neuro Critical Care
a) Developing research may suggest that stroke patients may benefit from higher
hemoglobin than suggested in TRICC
2. Cardiac surgery patients
a) Developing research may suggest that Cardiac centers with a higher tolerance for anemia
demonstrate less complications
The International Consensus Conference on Transfusion Outcomes
1. 15 international multidisciplinary experts reviewed 494 articles regarding expected outcomes
of a variety of non-bleeding patients receiving pRBCs
2. Presented with 34 scenarios of different patient populations with a variety of comorbidities
3. Experts were asked to determine the appropriateness of transfusion’s based on scenarios and
Hg concentrations:
a) Hg 7.9g/dL or less
b) Hg 8-9.9g/dL
c) Hg greater than 10g/dL
4. Overall consensus:
a) Inappropriate to transfuse patients with Hg greater than 10g/dL
b) 70% experts agree not to transfuse patients with Hg 8-9.9g/dL
c) Absence of comorbidities was the greatest factor for inappropriateness of transfusion
d) 6.9% experts agreed not to transfuse patients with Hg 7.9g/dL or less
e) Age under 65 and absence of comorbidities were the greatest factor for inappropriateness
of transfusion
72
SOCCA Residents Guide 2013
Discussion
Evaluation of bleeding in the ICU involves continuous consideration of the patient’s current ability to
develop and stabilize a formed clot. Consider the patient in the case provided. How did the carotid
aneurysm contribute to distal thrombosis? What may have prevented thrombosis safely in the patient if the
aneurysm was found before the stroke? If the patient had been on antiplatelet therapy, consider the
appropriateness of platelet transfusion if the last dose of antiplatelet therapy was 4 days prior. Consider any
laboratory tests and assays that may direct your therapy. If the patient were to return from the operating
room anemic, consider if the anemia was due to a fixable technical cause or a medical cause. Consider
hemoglobin goals based on the variety of literature. Does this patient fall into the population of the TRICC
trial, Neuro critical care, or cardiac disease?
References
1.
2.
3.
4.
5.
6.
Shander A, Fink A, Javidroozi M, Erhard J, et al. (2011) Appropriateness of allogeneic red blood cell
transfusion: the international consensus conference on transfusion outcomes. Transfusion Medicine
Review. Jul;25(3):232-246.e53
Moskowitz DM, McCullough JN, Shander A, et al. (2010) The Impact of Blood Conservation on Outcomes in
Cardiac Surgery: is it safe and effective? Annals of Thoracic Surgery. Aug;90(2):451-8.
Naidech A, Shaibani A, Garg RK et al. (2010) Prospective, Randomized Trial of Higher Goal Hemoglobin
after Subarachnoid Hemorrhage. Neuro Critical Care. 13:313–320.
Furie B, Furie BC (2008) Mechanism of Thrombus Formation. New England Journal of Medicine.
359:938-49.
Davi G, Patrono C (2007) Platelet Activation and Atherothrombosis. New England Journal of Medicine.
357:2482-94.
Stainsby D, Jones H, et al (2006) Serious Hazards of Transfusion: A Decade of Hemovigilance in the UK.
Transfusion medicine Reviews. Oct;20(4):273-282.
Questions:
14.1 Which of the following statement regarding dabigatran is/are TRUE?
A. Inhibits thrombin and the development of fibrin
B. Is dialyzable
C. Is NOT reliably reversible with Novo Seven and FEIBA
D. May increase PT and PTT
E. All the above statements are true
14.2 Which of the following would most rapidly correct an 80kg patient’s INR of 6.2 from Coumadin overdose?
A. FFP
B. FEIBA
C. Cryoprecipitate
D. Vitamin K
14.3 Which of the following would be most appropriate to treat a post op 20-year-old bleeding patient with the following
labs: Hg 11.1; INR 1.3; PTT 22; fibrinogen 89; Platelets 230k?
A. 1 unit of pRBC’s
B. FFP
C. Cryoprecipitate
D. FEIBA
14.4 Which of the following is TRUE regarding risks of blood component therapy?
A. Greatest risk is infectious disease transmission
B. Greatest infectious risk is HIV transmission
C. Plasma does not carry infectious risk
D. 70% of adverse episodes are potentially avoidable
73
SOCCA Residents Guide 2013
14.5 Which of the following would most likely warrant pRBC transfusion?
A. Acute and ongoing blood loss
B. Hg of 8.9g/dL in patient with respiratory failure second to pneumonia
C. A cardiac patient with a Hg of 8.9g/dL
D. A post-op postpartum hemorrhage patient with a Hg of 8.5
74
SOCCA Residents Guide 2013
15. Nutritional Support in the Critically Ill
R. Dean Nava, Jr., M.D.
A 27 year old, 84 kg male with no significant past medical history is involved in an
industrial accident that included spillage of caustic chemicals; upon arrival to the ED
he is found to have chemical burns over 40% of his body surface area. Initial fluid
resuscitation is carried out and he is intubated for potential airway compromise due
to suspected chemical inhalation. You come on service 3 days after his admission to
the ICU and receive this patient from the outgoing resident. He has no enteral
access at this point and has been NPO in the 3 days since his accident. What are
your initial nutritional goals for this patient?
-----------------------------------------------------------------------------------------------------------------------In recent years, there has been a significant shift in thinking regarding critical care nutrition-from being
seen as a supportive adjunct to a primary treatment that can significantly alter the outcome of the critically
ill patient. The type and route of nutritional support for patients in the ICU is in large part determined by
the nature of their disease process, the functional limitations of the GI tract, and disease severity. The
purpose of this chapter is to provide a basic overview of general nutritional principles and how approaches
might be constructed given the framework above.
I.
Basic metabolic needs
A. The three basic categories of macronutrients are carbohydrates, lipids, and protein.
1. Carbohydrates: 3.4 kcal/g
2. Lipids: 9 kcal/g
3. Protein: 4.1 kcal/g
B. There are a large number of equations to calculate the daily caloric needs of the critically ill.
1. For non-ventilated patients the Harris-Benedict equation is commonly used
a) Harris-Benedict equation for men:
B.E.E. = 66.5 + (13.75 x kg) + (5.003 x cm) - (6.775 x age)
b) For women:
B.E.E. = 655.1 + (9.563 x kg) + (1.850 x cm) - (4.676 x age)
2. For ventilated patients a commonly used formula is the Ireton-Jones equation
Total calorie need=1784-11(A)+4(W)+244(S)+239(T)+804(B)
A=age W=weight in kg S=sex (1=male 0=female)
T=trauma (1=yes 0=no) B=burns (1=yes 0=no)
3. Certain conditions lead to an increased metabolic demand above and beyond that of the
aforementioned equations. One must increase the delivered calories to fit the metabolic
requirements of the patient in certain circumstances.
a) 25% increase- mild peritonitis, long bone fractures, mild/moderate trauma
b) 50% increase- severe infection, multisystem organ dysfunction, severe trauma
c) 100% increase- burn of 40% to 100% body surface area
4. Simpler estimations of the total daily caloric requirement can be made using the stress level
associated with the patient’s condition and their weight in kg.
a) Maintenance or minimal stress: 25-30 kcal/kg/day
b) Moderate stress: 30-35 kcal/kg/day
c) Severe stress (e.g. burns): 35-40 kcal/kg/day
5. Protein requirements also change based on the patient’s condition and severity of illness. The
baseline protein requirement is between 1.2-2.0 g/kg/day, though this may be significantly
higher in protein wasting conditions such as severe burns or in protein- calorie malnutrition.
a) This should be monitored and adjusted using nitrogen balance and monitoring for a
prerenal azotemia, indicating hypersupplementation of protein.
b) The calories from protein should be calculated first. After this, 15-30% of the total caloric
intake should be from lipid. The rest (generally 30-70% of the total daily caloric intake)
should be from carbohydrate.
C. Malnutrition
75
SOCCA Residents Guide 2013
1.
2.
Patients at risk for malnutrition
a) There is no completely reliable way to assess the baseline nutritional status of the
critically ill in the acute setting. One can presume that if the patient suffered a recent
unintentional weight loss, has been NPO for a significant amount of time previous to the
acute situation that led to there being in an ICU, and/or is significantly underweight, they
were most likely suffering from some level of baseline malnutrition.
b) Protein calorie malnutrition (PCM) is defined as a weight loss >10-15% of total body
weight or a body weight <90% of ideal.
Consequences of malnutrition
a) Malnutrition can produce a host of maladaptive processes. These include increased
morbidity and mortality, prolonged hospital stay, impaired tissue function and wound
healing, defective muscle function, reduced respiratory and cardiac function, increased
risk of infection and overall immune suppression.
b) Critically ill patients lose approximately 2%/day of muscle protein.
You calculate his estimated caloric needs, and decide to begin nutritional therapy.
After numerous attempts, you are only able to get the nasoduodenal feeding tube
into the patient’s stomach. Do you feed the patient’s stomach? Should you start TPN
instead?
II.
Total parenteral nutrition (TPN)
A. When to start TPN
1. TPN should generally not be used until the patient has been without enteral nutrition for at
least 7 days and does not have baseline protein-calorie malnutrition (PCM). Up to that point,
a dextrose containing maintenance fluid should be used. If the patient does have PCM on
admission and is not able to be fed enterally, it is appropriate to start TPN after the initial fluid
resuscitation is complete.
B. Advantages
1. Ability to provide nutrition to patients that are unable to tolerate enteral feeding
2. Patients with severe malnutrition and/or non-functional GI tracts who receive TPN in the 7
days before major surgery may have improved outcomes.
C. Disadvantages
1. The systemic infectious risk is much higher in patients receiving TPN
2. Liver complications may vary from mildly elevated liver enzyme values to steatosis,
steatohepatitis, cholestasis, fibrosis, and cirrhosis.
3. Hyperglycemia is more prevalent, which may contribute to increased infectious risk
D. Access for TPN
1. Central vs. peripheral access- most TPN formulas are hypertonic, and as a result cannot be
infused at faster rates via peripheral access. Thus, central venous access is the preferred route.
If for whatever reason the TPN must be run peripherally, signs of vein infiltration and
phlebitis must be monitored for very carefully and therapy at that site halted should either of
these appear.
III.
Enteral nutrition
A. Advantages
1. Enteral feeding helps to maintain the functional and structural integrity of the GI tract. It helps
to maintain tight junctions between intraepithelial cells, helps potentiate the release of
endogenous GI substances such as cholecystokinin, gastrin, and bile salts; and stimulates
blood flow to the intraabdominal viscera. It also helps to preserve the immune function of the
gut by maintaining IgA immunocytes and the gut-associated lymphoid tissue (GALT). The
height of the villous structures of the gut is also maintained.
a) By maintaining the integrity of the gut, bacterial challenge to the GI lymphoid tissue is
decreased, decreasing the associated inflammatory response. Systemic infection of a GI
source is also reduced and the potential for multi-organ dysfunction is decreased.
b) While there has not been a significant benefit shown in reducing mortality, enteral
feeding does reduce the amount of infectious morbidity in the ICU setting.
76
SOCCA Residents Guide 2013
B. Disadvantages
1. Enteral feeding cannot generally be carried out in the setting of high vasopressor requirements
and hemodynamic instability.
2. Though the incidence is low, aspiration is always a risk. This is especially true in patients with
GI dysmotility or obstruction.
C. Access for EN
1. Most critically ill patients are enterally fed via a nasogastric or nasoenteric (e.g. Dobhoff)
tube. For a long while, it has been standard practice to make sure that enteral feeds are given
via a post-pyloric feeding tube. The thinking behind this was that feeding the stomach would
lead to an increased incidence of aspiration. This has been shown in a number of smaller
studies and meta-analyses to not be the case. Feeding via the stomach is currently listed as a
safe alternative in most clinical situations. Should gastric feeding be carried out, it is
advisable to have the patient’s head of bed at 30-45 degrees, use promotility agents if
tolerated, and reduce the amount of narcotics if possible.
2. There are some exceptions to this. Gastric feeding requires intact gastric function. Situations
where gastric feeding might be contraindicated are in patients with gastroparesis, gastric
outlet obstruction, known gastric reflux with aspiration, or gastric fistula. Situations where
post-pyloric feeding is necessary due to the patient’s disease process include conditions such
as pancreatitis.
3. In patients requiring long term enteral nutritional support, devices such as percutaneous or
open gastrostomy tubes or jejunostomy tubes can be placed. These are also chosen based on
the functional status of the patient’s GI tract and the specific limitations presented by their
disease process.
D. Types of EN
1. There are numerous formulations of enteral nutrition available, and the number continues to
grow. They typically have a specific balance of carbohydrate, protein, lipid, essential vitamins
and minerals, and a specific number of calories per unit of volume. Some formulas are
specifically tailored to certain disease processes, i.e. trauma, renal failure, hepatic failure, etc.
The standard formula for typical ICU patients varies from institution to institution, but many
formulas have 1-2 kilocalories per milliliter.
You begin feeding the patient’s stomach and his recovery is proceeding with few
complications. After 10 days on the ventilator, he has been deemed medically stable
such that he can be weaned from mechanical ventilation and potentially extubated.
On numerous attempts at weaning, he becomes tachypneic and develops a mild
respiratory acidosis necessitating continued mechanical ventilation. What nutritional
factors may be playing into the difficulty weaning from the ventilator and how might
you quantify them?
IV.
Nutrition monitoring
A. Common markers to monitor the adequacy of nutrition are prealbumin and albumin. When
reduced, these indicate a potential malnutrition state. These cannot be looked at alone as indicators
however, because they can be reduced as long as any inflammatory, catabolic state is present. The
overall clinical picture must be assessed in concert with these numbers.
B. There is also a multitude of nutrition scales used to calculate the nutritional status of patients.
Parameters used in these scales include lab values (albumin, prealbumin, etc.), BMI, stress level,
amount of weight loss, and numerous others.
C. Metabolic cart
1. A metabolic cart is the calculation of the patient’s metabolic needs via indirect calorimetry
2. Used to make sure patient is not being under or overfed.
3. The Fick equation assumes that at steady state, the amount of oxygen absorbed is equal to the
amount of oxygen consumed- this is extrapolated from the amount of carbon dioxide
produced.
4. Patient must be stable metabolically- rapidly dynamic metabolic states will give inaccurate
results, i.e. active sepsis, immediately post-trauma, etc.
5. The device is attached to the patient’s ventilator and the amount of end-tidal carbon dioxide
produced over a period of time is measured.
6. Using the Fick equation, the amount of oxygen consumed per minute is calculated.
77
SOCCA Residents Guide 2013
7.
8.
V.
The conversion to calories is 5 kcal/liter of oxygen
Used to derive the respiratory quotient (RQ)= CO2 expired / O2 inspired
a) 0.6-0.7 = starvation/underfeeding
b) 0.84-0.86 = desired range/mixed fuel utilization
c) 0.9-1.0 = primarily carbohydrate
d) >1.0 = overfeeding/lipogenesis
e) Overfeeding leads to difficulty weaning from the ventilator due to increased carbon
dioxide production, which leads to increased mandatory minute ventilation to maintain a
normal serum pH, which leads to significant increase in the work of breathing.
Potential complications
A. Refeeding syndrome
1. In chronically malnourished patients, a baseline hypophosphatemia is often present.
2. Upon administering a carbohydrate load to these patients via the enteral or parenteral
route, there is a significant spike in serum insulin levels.
3. This leads to an increase in cellular phosphate uptake, which can drop serum
phosphate levels precipitously.
4. The resulting severe and sudden hypophosphatemia can lead to acute respiratory
failure (due to diaphragmatic weakness), muscle weakness and rhabdomyolysis,
hemolysis, altered mental status, gait disturbances, parasthesias, cardiomyopathy, and
decreased inotropy.
5. Hypophosphatemia is usually not symptomatic until serum phosphate is <1 mg/dL.
6. Phosphate levels should be monitored closely and appropriate repletion instituted
when feeding is started.
This chapter is a revision of the previous version authored by Gustavo Angaramo, M.D.
REFERENCES/READING LIST
1. American Society for Parenteral and Enteral Nutrition Board of Directors. Clinical guidelines for the use
of parenteral and enteral nutrition in adult and pediatric patients, 2009. Journal of Parenteral and
Enteral Nutrition 2009; 33, No. 3.
2. Gramlich L, et al. Does enteral nutrition compared to parenteral nutrition result in better outcomes in
critically ill adult patients? A systematic review of the literature. Nutrition 2004; 20: 843-848.
3. Doig GS, et al. Early enteral nutrition, provided within 24 h of injury or intensive care unit admission,
significantly reduces mortality in critically ill patients: a meta-analysis of randomized controlled trials.
Intensive Care Med 2009; 35:2018–2027.
4. Guglielmi FW, et al. Total parenteral nutrition-related gastroenterological complications. Digestive and
Liver Disease 2006; 38: 623-642.
5. Mizock BA. Risk of Aspiration in patients on enteral nutrition: frequency, relevance, relation to
pneumonia, risk factors, and strategies for risk reduction. Current Gastroenterology Reports 2007;
9:338–344.
6. Bongers T, et al. Are there any real differences between enteral feed formulations used in the critically ill?
Curr Opin Crit Care 2006; 12:131–135.
QUESTIONS
15.1 Initiation of parenteral nutrition should be considered in adult patients with non-functional gastrointestinal tracts
and:
A. 3% weight loss of usual body weight over 1 month
B. Inadequate oral nutrient intake for 3 days
C. 5% weight loss of usual body weight over 1 month
D. Inadequate oral nutrient intake for 7 days
78
SOCCA Residents Guide 2013
15.2 What is the minimum period that severely malnourished patients can receive pre-operative TPN and still achieve a
significant reduction in post-operative morbidity and mortality?
A. 1-2 days
B. 3-5 days
C. 7-15 days
D. 21-28 days
15.3 A metabolic cart (indirect calorimetry) is obtained on a patient and the respiratory quotient is calculated to be 1.1.
What nutritional maneuver should generally be undertaken first in this situation?
A. Nothing; keep nutritional support as it is
B. Increase overall caloric intake
C. Decrease overall caloric intake
D. Increase carbohydrate fraction of nutritional support
15.4 Advantages of enteral feeding include all of the following EXCEPT:
A. Maintenance of GI structural integrity
B. Decrease in GI visceral blood flow
C. Preservation of GI immune function
D. Decrease in infectious risk
15.5 After beginning to provide nutritional support to a chronically malnourished patient, they develop confusion and
dyspnea. The electrolyte disturbance most likely to be involved is:
A. Hyperkalemia
B. Hypokalemia
C. Hypophosphatemia
D. Hyperphosphatemia
E. Hypermagnesemia
79
SOCCA Residents Guide 2013
16. Routine Monitoring in the ICU
Daniel R. Brown, M.D., Ph.D.
A 65-year-old male has just undergone elective open repair of an abdominal aortic
aneurysm under general anesthesia. His arterial catheter was inadvertently pulled while he
was transferred to the ICU bed. Past medical history is significant for coronary artery
disease and diabetes. What monitoring is indicated for this patient in the ICU?
I.
Noninvasive monitoring in the ICU
A. Blood pressure
1. Use of noninvasive BP vs. invasive BP determined as in O.R..
2. Similar concerns as to cuff size and site of measurement.
B. ECG
1. Rhythm and rate are monitored as part of vital signs.
2. Consider 5-lead and ST-segment analysis in patients at risk for ischemia.
3. Arrhythmias may be first sign of electrolyte imbalance.
C. Pulse oximetry (SpO2)
1. A monitor of oxygenation; use of supplemental oxygen negates ability to use SpO2 as monitor
of ventilation.
2. Not a monitor of perfusion unless plethysmographic waveform is utilized; only a crude
measure if used.
3. More artifacts in ICU (vs. O.R.) related to patient movement.
D. Temperature
1. Site of measurement important, must be noted with each measurement.
2. Fever is associated with significant increase in mortality.
E. Peripheral nerve stimulator
1. Mandatory use when patient receiving pharmacologic neuromuscular blockade.
2. Response to T.O.F. to be recorded in patient chart.
F. Capnography
1. Can be used in non-intubated patients as a crude monitor of ventilation.
2. Difficulty in long-term use with humidification of ventilator circuit.
3. End-tidal CO2: arterial CO2 gradient changes can be due to ventilation or perfusion changes;
end-tidal CO2 should be noted with each blood gas.
G. Non-invasive cardiac output (CO)/cardiac function
1. Echocardiography: transthoracic (TTE) or transesophageal (TEE)
a) Accurate TTE may not be possible in edematous or obese patients.
b) Both modalities can be used for detection of ischemia, assessment of preload, and
ejection fraction.
c) TEE better for assessment of right ventricle, pulmonary circulation and aortic dissection.
2. Doppler noninvasive/thoracic impedance
a) Newer monitors which can correlate well with invasive CO monitor and can be used for
trending and rapid evaluation of changes in therapy.
b) Respiratory CO2-derived cardiac output measurements in mechanically ventilated
patients via proprietary technology
H. Radiology/Ultrasound
1. Consult with radiologist to make sure the test is the right one for the presumed diagnosis.
2. Many tests may need proper preparation of the patient (NPO, bowel prep, contrast, etc.).
3. Prepare patients with poor renal function (avoid hypovolemia, consider bicarbonate and/or Nacetylcysteine) or allergy to contrast (consider histamine blockers and/or steroids).
4. Bedside ultrasound can be used for serial evaluation and follow-up exams.
I. Gastric tonometry
1. Measures increases in gastric mucosal CO2 by allowing equilibration of CO2 partial pressures
between a fluid-filled balloon and the mucosal layers.
2. Used as an indirect monitor of tissue hypoxia assuming an increase in CO2 production
measured is a reflection of anaerobic metabolism
80
SOCCA Residents Guide 2013
II.
Invasive monitoring
A. Arterial catheters
1. Used with unstable patients or those receiving vasoactive infusions.
2. Sites: radial, femoral, dorsalis pedis, axillary, posterior tibial, temporalis
3. Check for collateral circulation (controversial)
4. Femoral site with higher risk of infection
B. Central venous catheters (see Chapter 17: Rational Use of the PA Catheter)
1. Sites: Internal/external jugular, subclavian, femoral
2. Cardiac output measurements via non-pulmonary artery catheter devices; may include
additional measurements such as lung water
3. Associated morbidity and mortality (including infection, bleeding, arrhythmias, emboli,
arterial puncture/catheterization, pneumothorax (subclavian and internal jugular), thrombosis,
catheter misdirection, or perforation of insertion vessel, more central vein or even the heart)
C. ICP monitoring
1. Indicated in closed head injury patients at risk for intracranial hypertension and herniation
2. Ventriculostomy vs. bolt: differences in infection rates, ease of placement, therapeutic
possibilities. (see Chapter 34: Management of Increased Intracranial Pressure)
Discussion
Monitoring in the ICU may exist for diverse reasons: observation for detection of changes in physiologic
status, intensive titration of therapy, and/or detection of breaches of safety (e.g., ventilator disconnection),
among others. By definition, patients are admitted into the ICU because they require a higher level of care.
Monitoring is a significant part of that care, whether it involves monitoring by human caregivers and/or
medical machines. The patient’s condition and the likelihood that this condition may rapidly change should
determine the level of monitoring. One task of the ICU physician is to continually evaluate the need of a
patient for a particular level of monitoring. Ideally, monitors should be used to confirm or further define
problems that have been discovered by careful physical examination and clinical assessment.
References
1.
2.
3.
4.
5.
Karakitsos D, et al. Real-time ultrasound-guided catheterization of the internal jugular vein: a
prospective comparison with the landmark technique in critical care patients. Crit Care. 2006; 10:R162
Marino P. The ICU Book. Philadelphia: Lippincott, Williams and Wilkins, 2007
Mark JB. Central Venous Pressure and Pulmonary Artery Pressure Monitoring. ASA Refresher Course
Lectures, 2010.
Marschall J, et al. Strategies to prevent central line – associated bloodstream infections in acute care
hospitals. Infection Control and Hospital Epidemiology. 2008 Oct;29 Suppl 1:S22-30.
Oh J. The Echo Manual. Philadelphia: Lippincott, Williams and Wilkins, 2007.
Questions
K-type:
16.1 Which of the
1.
2.
3.
4.
following statements concerning noninvasive BP monitoring are true?
Accurate pressure measurement requires a stable pulse pressure.
Cardiac tamponade limits the usefulness of non-invasive BP monitoring.
If the noninvasive BP cuff is too small, the BP will read higher than the actual pressure.
Frequent non-invasive BP measurement may decrease accurate measurement
16.2 All of the following statements concerning the CVP tracing are true?
1. “A” waves result from atrial contraction.
2. “C” waves are a result of tricuspid valve closure and ventricular contraction.
3. When a patient has a junctional rhythm, prominent “A” waves (cannon “A” waves) may result.
4. Cardiac tamponade changes the normal, biphasic CVP tracing into a monophasic tracing, obliterating
the “Y” descent.
81
SOCCA Residents Guide 2013
16.3. Which of the
1.
2.
3.
4.
following statements concerning central venous cannulation are true?
Ultrasound use eliminates inadvertent arterial catheter placement.
With an internal jugular approach, there is no risk of pneumothorax.
The internal jugular vein and carotid artery relationship is consistent between patients at the
bifurcation of the sternocleidomastoid muscle.
2% chlorhexidine alcohol-based antiseptic is preferred in adult patients.
82
SOCCA Residents Guide 2013
17. Rational Use of the Pulmonary Artery Catheter
Gregory E. Kerr, M.D., M.B.A., F.C.C.M., James A. Osorio, M.D.
Three days after he underwent a mitral valve replacement for severe mitral regurgitation, a
62 year old man developed acute shortness of breath with a mild fever and mild elevation in
his white blood cell count. Soon thereafter, he became severely hypotensive and required
norepinephrine to maintain an adequate blood pressure. On physical exam, he was noted to
have a new murmur and diffuse bilateral rales despite having been diuresed 4 liters over the
two previous days. A chest radiograph revealed diffuse white fluffy infiltrates.
I.
Introduction of Pulmonary Artery Catheter (PAC)
A. H.J.C. Swan and William Ganz invented the balloon-tipped flow-directed pulmonary artery
catheter in 1970.
B. The PAC enables the bedside clinician to directly measure the central venous pressure, the
pulmonary artery pressure, the pulmonary capillary wedge pressure and cardiac output for the
evaluation of intravascular volume status and/or cardiac function.
C. The PAC has provided information in the management of patients during critical illness for at least
the last three decades.
D. Recent clinical trials failed to demonstrate that use of the PAC improves patient outcomes such as
mortality and morbidity in managing critically ill patients. However, the PAC is still widely used.
II.
Indications for the use of the PAC
A. The PAC may be used in surgical procedures or conditions associated with significant blood loss
or fluid shifts in the setting of known significant pulmonary hypertension, right heart dysfunction
or severe COPD.
B. The PAC is the only available tool that allows the clinician to continuously and directly measure
pulmonary artery pressures. Patients and conditions where an acute rise in pulmonary pressure
may lead to acute right failure may be best managed with a PAC in the perioperative period. For
that purpose, and because of complex fluid shifts in cardiac surgery patients, the PAC is still
widely used.
III.
Clinical information and measurements
A. Central venous pressure (CVP)
1. The CVP is the pressure measured in the superior vena cava or right atrium
2. Normal value is 4 – 10 mmHg
3. Common reasons for elevated values include volume overload, right ventricular dysfunction,
tricuspid valve disease, pneumothorax, tamponade and pulmonary embolism.
B. Pulmonary Artery Pressure (PAP)
1. The PAP is measured through tip of the catheter when it is placed in the pulmonary artery.
There is a systolic component (PAS) and diastolic component (PAD)
2. Normal values are normally one fourth of systemic pressures. The PAD closely reflects the
left ventricular end diastolic pressure (LVEDP) when there is no evidence of pulmonary
hypertension, elevated airway pressures or mitral valve disease.
3. Common reasons for elevated values include volume overload, pulmonary hypertension,
pulmonary embolism, elevated airway pressures, mitral valve disease, left ventricular
dysfunction, tamponade and increased intrabdominal pressure.
C. Pulmonary Capillary Wedge Pressure (PCWP)
1. The PCWP (aka pulmonary occlusion pressure (PAOP) is measured when the PAC is
advanced with an inflated balloon to a point where the catheter is “wedged” and thus should
not be advanced further. For the most part, blood flow stops and a static fluid column is
created.
2. Normal values are 8-16mmHg. The PCWP closely reflects the LVEDP when there is no
evidence of elevated airway pressures, pulmonary vein occlusion or mitral valve disease.
3. Common reasons for elevated values include volume overload, elevated airway pressures,
83
SOCCA Residents Guide 2013
mitral valve disease, left ventricular dysfunction, tamponade and increased intrabdominal
pressure.
D. Cardiac Output (CO)
1. The cardiac output is defined as the heart rate x stroke volume.
2. It is obtained by injecting a standardized volume of solution (usually 10 ml of cold normal
saline) through the proximal port of the catheter with the blood temperature being measured
continuously by a thermistor at the distal aspect of the catheter.
3. The change in temperature over time curve is measured and plotted until full mixing of the
blood occurs to restore baseline temperature. The area under the curve is inversely related to
cardiac output.
4. Equation for cardiac output determination:
(V = Volume of injectate (ml); TB = Initial blood temperature (В°C) ;
TI = Initial injectate temperature (В°C); K1 = Density factor;
K2 = Computation constant; ∫ ∆TB(t)dt = Integral of temperature change over time)
a) Normal value is 3.5 - 5.5 L/min or calculated cardiac index above 2.2 L/min.
b) Factors and conditions that could affect accuracy of measurement:
(1) Inappropriate volume of injectate.
(2) Slow injection of the solution.
(3) Presence of intra-cardiac shunts, tricuspid regurgitation or pulmonic regurgitation.
(4) Inaccurate computation coefficient calibration
E. Mixed Venous Oxygen Saturation (SvO2)
1. The SvO2 is obtained by withdrawing a sample of blood from the distal port of the PAC and
measuring the oxygen saturation. This must be done slowly so that mixing with post capillary
oxygenated blood does not occur. It evaluates the balance of oxygen delivery versus oxygen
consumption
2. Normal value for SvO2 ranges from 65 -78 % in adults.
3. Reasons to have a low mixed venous saturation include hypoxia, low cardiac output, anemia
and increased oxygen consumption.
4. Reasons to have an elevated high SvO2 include situations where oxygen is not being extracted
as much as in a normal state such as early stages of sepsis and cyanide toxicity.
IV.
Placement of PA Catheter
A. Materials for Insertion
1. Pulmonary Artery Catheter –The most commonly used PAC includes a pulmonary artery port,
central venous port, auxiliary (infusion) port (VIP port), a balloon and a thermistor.
a) If one needs the capability to administer transvenous pacing, either a Paceport or Pacing
PAC could be placed.
b) A fiberoptic SvO2 PAC may be placed when continuous mixed venous oxygen saturation
readings are desired.
c) A CCO PAC will provide continuous cardiac output measurements.
2. Percutaneous sheath – A PAC needs to be passed through a central venous conduit. This is
called the percutaneous sheath (CordisВ® or other brands). The size will vary depending on
the size of the PA catheter, but typically ranges between 7Fr through 9Fr in size.
3. Equipment for sterile line insertion – Equipment that is used for sterile insertion of the CVL
should adhere to those standards outlined by the Joint Commission. Items needed include
84
SOCCA Residents Guide 2013
gown, mask, sterile gloves, cap, chlorhexadine prep and sterile drape.
B. Insertion Procedure
1. Site
a) Internal Jugular Vein – The right and left IJ sites are commonly used due their
accessibility and perception that it has the lowest significant complication rate. The right
is easier than the left. Ultrasound is an easily useful adjunct. Dressings can be difficult
to maintain in some patients.
b) Subclavian Vein – Both the right and left subclavian approach are often considered
preferred because of the lower infection rate (in part because the catheter is more stable
on the skin and in part because the dressings are easier to maintain). It may actually be
an easier approach in the morbidly obese. The left side is easier than the right.
Ultrasound is usually not that helpful. Many are uncomfortable with the subclavian
position due to the slightly higher incidence of associated pneumothorax.
c) Femoral Vein – This site is generally considered the last choice. It can be technically
difficult in the morbidly obese. There is higher rate of infection associated with this
position. It is more difficult to float the PAC from the IVC approach. Ultrasound is a
useful adjunct during placement. During surgery, a femoral line is usually not well
accessible.
2. Technique of insertion of percutaneous sheath and PAC
a) Prep the skin widely with recommended solution (currently chlorhexidine) and drape
using appropriate sterile technique.
b) Place patient in slight Trendelenburg position if tolerated.
c) Place a roll between the shoulders for subclavian approach if tolerated.
d) Options for determining needle insertion point.
(1) Anatomical landmarks have traditionally been used to determine needle entry point.
(2) The use of ultrasonography continues to gain in popularity to help guide placement
of the central venous catheter. It has become the standard in some institutions.
(3) Confirmation of venous access (as opposed to arterial) can be done by placing a
85
SOCCA Residents Guide 2013
16G catheter over the guide wire, removing the guide wire and then transducing the
pressure waveform that is elicited. Some recommend that this should be done if the
position is not confirmed with ultrasonography.
e) Use the Seldinger technique to insert the percutaneous sheath. The sheath may be either
8Fr or 9Fr, but will always have a side arm for infusion. Suture securely in place.
f) Connect the PAC ports to the appropriate transducers and sensors. If a CCO device,
calibrate as manufacturer instructions direct. Flush all ports to ensure patency. If you do
not have a VIP infusion setup ready, cap that port. Inflate PAC balloon only by using a
syringe which is found in the PAC packaging. Maintaining the PAC natural curve, pass
through the percutaneous sheath. Observe as the tracing (E) reveals in sequence the CVP
(20cm), RV pressure (35cm), PAP (45cm) and PCWP (50cm). (See Fig 1.) Deflate the
balloon after insertion.
3. Post insertion management
a) Secure catheter.
b) Place sterile dressing.
C. Complications associated with PAC
1. Insertion
a) Hematoma/bleeding/hemothorax
b) Carotid artery catheterization
c) Ventricular arrhythmias
d) Pulmonary artery rupture
2. Monitoring
a) Infection
b) Pulmonary infarction
c) Right bundle branch block or complete heart block
Conclusion and Future
The Pulmonary Artery Catheter:
• can be a very useful diagnostic and monitoring tool in experienced hands.
• may help in the assessment of intravascular volume status, myocardial function, the balance of
oxygen delivery and consumption in high-risk surgical and critically ill patients and thus guidance
of therapy.
Other modes of monitoring, such as echocardiography, can be helpful adjuncts when managing a critically
ill patient. Many new types of technology are being developed to help assess volume status and cardiac
output less invasively. One can expect that these new devices will be used more frequently in managing
critically ill patients.
References:
1. Funk DJ, Moretti EW, Gan TJ. Minimally Invasive Cardiac Output monitoring In the Perioperative
Setting. Anesthesia & Analgesia, 2009; 108:3.
2. Graham AS, Ozment C, Tegtmeyer K. Central Venous Catheterization; NEJM, 2007; 356:e21.
3. http://www.glowm.com/?p=glowm.cml/section_view&articleid=211
4. Richard C, Teboul JL. Early Us of the Pulmonary Artery Catheter and Outcomes in Patients with Shock
and ARDS. JAMA, 2003; 290:20.
5. Sandham JD, MD , et al. A Randomized, Controlled Trial of the Use of Pulmonary – Artery Catheters in
High-Risk Surgical Patients; NEJM,
6. 2003; 348;1.
7. Shure D. Pulmonary Artery Catheters – Peace at Last? NEJM, 2006; 354:21: 2273-2274.
8. Swan HJC, Ganz W, Forrester J et al. Catheterization of the heart in man with the use of flow directed
balloon – tipped catheter. NEJM, 1970; 283-447.
86
SOCCA Residents Guide 2013
Questions:
17.1. Accepted indications for the use of the pulmonary artery catheter include management of:
A. Acute respiratory distress syndrome
B. Cardiac surgery
C. Congestive heart failure
D. Pulmonary hypertension
17.2. In
A.
B.
C.
D.
a patient with an elevated PCWP, one might expect to find which of the following
Mitral stenosis
Left ventricular dysfunction
Mitral regurgitation
Pulmonary hypertension
17.3. A SvO2 of 48% may be found in a patient with which of the following?
A. Left ventricular dysfunction
B. Cyanide toxicity
C. Anemia
D. Early sepsis
87
SOCCA Residents Guide 2013
18. Arterial Pulse Waveform Analysis
Lalitha Sundararaman, MD and Miguel A. Cobas, MD
A 57-year-old man with a bleeding mycotic aneurysm is in the ICU after undergoing
exploratory laparotomy in the operating room (OR). He continues to bleed and is
getting blood through a rapid infuser. He is on a vasopressin infusion. An arterial
line is in place. Could we get information with regards to the stroke volume, cardiac
output, peripheral vascular resistance and volume status from just the arterial
waveform? How would we assess the reliability and limitations of the waveform
analysis?
The arterial waveform has become an integral part of hemodynamic monitoring of unstable patients in the
OR and ICU. It can provide a wealth of information ranging from volume status to systemic vascular
resistance to just simply the blood pressure. However it is of paramount importance to be able to assess the
reliability of the pressure transducer system first to validate our conclusions.
I.
What are the components of an arterial pressure transducing system?
An arterial catheter that provides access to the arterial system being monitored is designed to pick up the
pressure waves generated by cardiac contractions. The catheter is connected to a fluid-filled tubing of a
monitoring system. The fluid column in the tubing system carries a mechanical signal created by the
pressure wave to the diaphragm of an electrical pressure transducer. The transducer creates the link between
the fluid-filled tubing system and the electronic system, and converts the mechanical signal into an
electrical signal. The electrical signal is transmitted to the monitor and then is amplified and displayed as an
analog waveform and digital output.
II. How do I know that the waveform is accurate?
A good method of testing the arterial monitoring system is through the FLUSH test.
A brief flush can be applied to the catheter tubing system to determine whether the recording system is
distorting the pressure waveform or not. Most systems are equipped with a one-way valve that can be used
to deliver a flush from a pressurized system (usually at 300 mm Hg). Accordingly to the response obtained
the system can be classified as normal, overdamped or underdamped. In each case the pressure increases
abruptly when the flush is applied. However the response at the end of the flush differs in each situation.
Each flush is followed by a few oscillating waveforms. The frequency of these oscillations is the resonant
frequency of the recording system and is calculated as the reciprocal of the time period between the
oscillations. Signal distortion is minimal when the resonant frequency of the system is atleast 5 times the
arterial frequency (normally 5 Hz). When using standard strip chart recording paper with a speed of 25mm /
sec, we see the following 3 situations (Figure 18-1):
Figure 18-1: Flush Test. A = normal resonance, B = over-damped, C = under-damped.
Calculate the resonance frequency by dividing the velocity (paper speed, 25mm/s) by
the distance (width) of the first wave.
While a damped waveform maybe due to air bubbles, clots, lack of flush solution, lack of pressure in flush
system or catheter kinks, underdamping is usually due to extra long tube length (more than 200 cm) or
88
SOCCA Residents Guide 2013
noncompliant tubing.
III. Does the waveform reflect the central aortic pressure?
As we proceed from the aorta to the periphery, the systolic pressure is amplified by reflected waves from
the periphery hence increasing from the carotid to the radial artery; hence measurements at the radial artery
often overestimates the central aortic pressure.
IV.
What are the salient parts of the waveform, as seen in Figure 18-2? And, based on its analysis, can we
gauge the contractility of the heart?
The initial upswing of the arterial waveform called the anacrotic limb gives an idea of the contractility of
the heart. It is steeper with the use of ionotropes and less steep when the contractility is impaired as in the
case of post-MI or advanced sepsis, It is normal in the above graph.
V. What do the dicrotic notch and limb represent?
It signifies aortic and pulmonary valve closure. The dicrotic notch lies low on the downstroke when the
patient is dehydrated (usually associated by “arterial cycling/swing”(see below)) or when there is a high
pulse pressure as in the case of septic shock as with our patient. The valve is often flattened in
aortopulmonary valve insufficiency.
In Figure 18-3A, there is a sharp fall from the dicrotic notch that indicates rapid peripheral runoff or a low
SVR. In Figure 18-3B, there is a long drawn out fall off from the dicrotic notch to the baseline indicating a
slow peripheral run off or systemic vasoconstriction. Therefore in the second graph, the contribution to the
blood pressure is more from the systemic vascular resistance or vasoconstriction rather than the narrow area
under the graph or stroke volume.
VI. After reviewing a single pulsation, what can be studied about the arterial tracing as a whole?
What is cycling and stroke volume variation?
The systolic blood pressure reading can vary from time to time – this is known as “arterial line swing or
cycling” and its more evident in dehydrated patients. This phenomenon occurs during respiration – both
spontaneous and mechanically ventilated, although it is more pronounced during controlled mechanical
ventilation.
The mechanism for this phenomena is the following: during spontaneous respiration, at the beginning of
inspiration, the intrathoracic pressure briefly drops before building up and becoming much higher than
before due to the expansion of the lungs. This increased pressure causes a reduction in transmural blood
flow back to the heart by compressing the intrathoracic veins (reduced preload) causing stroke volume to
drop. As expiration begins, the pressure drops significantly as the air leaves the lungs, greatly increasing
transmural blood flow (increased preload) causing stroke volume to rise. This can also be noted in the
systolic blood pressure figure as it fluctuates with respiration.
During positive pressure ventilation there is always a reduced transmural blood flow – this is why blood
pressure is always lower during mechanical ventilation-. Stroke volume variation is therefore much more
pronounced in the presence of higher tidal volumes and elevated peak airways pressures.
VII. What is systolic pressure variation?
It’s a dynamic marker of volume responsiveness that can serve as one of the useful end points of volume
resuscitation. During positive pressure ventilation, in the inspiratory phase, increasing lung volume
displaces blood into the left heart and increases preload, LV filling and blood pressure. There is also a less
significant decrease in the LV afterload. The increased intrathoracic pressure decreases right ventricular
preload and increases afterload by increasing pulmonary vascular resistance. This reduced preload crosses
the pulmonary vascular bed during expiration and this results in reduced left heart filling and hence blood
pressure during expiration. Hence this is manifested as the cyclical change in the blood pressure with
positive pressure ventilation or systolic pressure variation. Keeping the end expiration apneic level as the
baseline, an increase in the pressure is marked as delta up and a dip as delta down. Normally delta up is 2-3
mm Hg and delta down 5-6 mm Hg and the sum is less than 10mm Hg. If the sum is greater than 15 mm
89
SOCCA Residents Guide 2013
Hg, then there is a high likelihood of the preload or PAOP being less than 10 mm Hg. Delta up is more
indicative of the afterload dependence of the ventricle and delta down of the preload. It has been shown that
delta down is a much better predictor of volume responsiveness and possible increase in cardiac output by
increasing stroke volume than CVP, PAOP and LV end diastolic area.
VIII. What is pulse pressure variation (PPV)?
It is another dynamic marker of volume responsiveness. Pulse pressure is the difference between systolic
and diastolic pressures. The maximal difference in pulse pressure measured over the course of a positive
pressure ventilation is divided by the average of the maximal and minimal pulse pressures over a
respiratory cycle. If the percentage is more than 13%, the preload is low. Newer methods of pulse contour
analysis use the arterial waveform and a computerized algorithm to calculate the cardiac output. This can
be predicted from the pulse plethysmograph also but many confounding factors impair the usefulness of
these calculations. The magnitude of pulse pressure variation will be influenced by the tidal volumes used
in mechanical ventilation and the peak pressures achieved (increases in airway pressures and changes in
PEEP can again alter the intrathoracic pressures and alter the PPV percentages). Hence changes in these
parameters will confound the usefulness of PPV and SPV and exact threshold values have yet to be
assertively defined.
IX. Can I use this to manage the fluid status of the patient?
Certainly! But keep in mind the limitations listed. Only once the fluid status is optimized with algorithmic
challenges should inotropes/vasopressors be considered.
There are currently two devices commercially available to use for this:
• Lithium dilution cardiac output monitoring (LIDCO®) is another method wherein a small intravenous
bolus of lithium chloride is given and the lithium dilution curve plotted by an electrode attached to a
peripheral arterial catheter which is the used to compute the cardiac output. LiDCOВ® pulse contour
analysis is a new variant that analyses the arterial trace stroke volume variance and translates this into a
visual percentage to determine if a patient will be pre-load (fluid) responsive or not. It has to be initially
calibrated with a CO measurement such as LIDCO CO method or any other conventional thermodilution
technique.
• The FloTrac® system, which consists of the Vigileo® monitor and the FloTrac® sensor which can be
attached to any peripheral arterial catheter. It uses a clinically validated algorithm to provide continuous
cardiac output (CCO), stroke volume (SV) and stroke volume variation (SVV) in real-time. The FloTrac
algorithm utilizes arterial pressure, age, gender, and body surface area to calculate SV. The patientspecific SV is then multiplied by pulse rate to provide CO. Patient-specific SV is updated as fast as every
20 seconds. However the device can only work with a good arterial waveform and changes in the
waveform will impede interpretation.
However, methods that use the arterial pulse waveform have many limitations, markedly reducing the
number of critically ill patients that may benefit from accurate readings.
Among the main limitations for these methods are:
• Need for Positive Pressure Ventilation
• Need for a Closed Chest
• Need for Heavy Sedation
• Need for Sinus Rhythm
• Need for Tidal Volumes > 8ml/kg
• Need for a properly calibrated Arterial Line System
90
SOCCA Residents Guide 2013
• Very different technologies looking at the same principle using proprietary algorithms
X.
Are there any other methods to assess volume responsiveness or the change in cardiac output in
realtime?
Yes!
Ultrasound based methods to calculate the cardiac output use the Doppler shift frequency principle of
reflected ultrasound waves to determine the blood flow velocity and thence the cardiac output.
Suprasternal cardiac output monitoring uses a transducer to measure blood flow velocity in the distal aortic
arch and hence the cardiac output.
A transducer at the tip of an esophageal stethoscope measures the blood flow velocity in the distal aorta and
calculates the cardiac output.
These methods calculate a cardiac output value that reasonably compares with the value via thermodilution
measurements but is dependent on the operator positioning the probe optimally.
Bioimpedance cardiac output monitoring measures the changes in electrical impedance and uses a formula
to calculate the cardiac output. This is reliable however only in healthy patients and the measurements are
confounded by numerous factors in critically ill patients severely limiting its usefulness.
The newest iteration of this method is the Cheetah® – a “bioreactance” based approach. It is based on the
principle that when blood flows out of the heart, phase shifts are created in alternating radiofrequency
electrical currents applied across the patients’ chest. Such phase shifts are conceptually similar to a
Frequency Modulation, or FM, as used in FM radio transmissions. The phase shifts are measured
continuously and have been shown to relate almost linearly to blood flow in the aorta. So how is it different
from the older bioimpedance method? Both technologies use sensors placed on the patient’s chest to deliver
an electrical current of known amplitude and frequency across the thorax, but the major difference
between bioreactance and bioimpedance is analogous to that between FM and AM radio: With FM radio,
detection of the radio signal is based on changes in signal frequency rather than changes in signal
amplitude, allowing for greater fidelity in the obtained signal. FM enables significant advantages in
filtering noise, for example noise coming from other electronic or physiologic emitters or in our case other
monitors. This is why FM-based systems offer superior performance compared to AM.
Another advantage of bioreactance stems from the fact that detection of frequency modulations is
insensitive to distance between the sensors, which is in sharp contrast to the shortcomings of detection of
amplitude changes. Therefore, bioreactance has the important advantage of flexibility in sensor location but
also in various clinical scenarios such as morbid obesity and pleural effusion. It is FDA approved and non
invasive. However its usefulness in a variety of trite situations in the ICU is yet to be established.
XI. So what’s the future?
• The signal abnormality index (SAI) holds great promise. The algorithm outputs
at a beat-level time resolution and intelligently detects abnormal beats by imposing a series of constraints
on physiologic, noise/artifact, and beat-to-beat variability. The normal beats are “flagged”, data collected
and analyzed through standard algorithmic models to give cardiac output and vascular data.
• Partial carbon dioxide rebreathing cardiac output monitoring is another noninvasive method that holds
great promise. Intermittently through a pneumatic valve that allows for rebreathing of exhaled gases, the
change in end-tidal carbon dioxide is noted and using a differential version of the Fick equation, the
cardiac output is calculated. However presently this method requires tracheal intubation for accurate
measurements and changing the pattern of ventilation may have unpredictable influences.
References:
Gloria Oblouk Darovic: Hemodynamic Monitoring: Invasive and Noninvasive Clinical Application
J Xun et al: The cardiac output from blood pressure algorithms trial. Crit Care Med. 2009 January; 37(1): 72–
80.
B McGhee et al: Monitoring blood pressure, what you may not know. Crit Care Nurse April 2002 vol. 22 no. 2
60-79
91
SOCCA Residents Guide 2013
Questions:
18.1 A poor dP/dT on the arterial waveform would favor selection of which of the following drugs?
A.
B.
C.
D.
Nitroglycerine
Lasix
Dopamine
Milrinone
18.2 The most reliable measured value in the arterial trace irrespective of the degree of damping would be:
A.
B.
C.
D.
E.
SBP
MAP
DBP
No value is reliable
All values are reliable in different phases of inspiration
18.3 When there is cycling, which phase of respiration would provide the most accurate stroke volume calculations?
A.
B.
C.
D.
End inspiration
Early expiration
End expiration
End inspiration
18.4 The delta down on the systolic pressure variation describes:
A. Preload
B. Afterload dependence
C. Contractility
D. Diastolic dysfunction
92
SOCCA Residents Guide 2013
19. Ultrasound in the ICU
Abbas Al-Qamari, MD
A 59 year old obese male with a history of pulmonary hypertension arrives to the intensive
care unit after coronary artery bypass grafting and mitral and aortic valve replacement. The
patient is intubated and receiving therapy with norepinephrine and epinephrine infusions but
continues to be hemodynamically unstable with a heart rate of 123, blood pressure of
83/58, and oxygen saturation of 92% on 100% oxygen. Central venous pressure is
estimated at 16 and pulmonary pressures are estimated at 57/34 with a pulmonary artery
occlusion pressure of 43.
Introduction
Ultrasonography use is becoming an indispensible tool in the practice of critical care medicine. Its safety
and portability allow for rapid noninvasive bedside assessment to aid in diagnosis and ongoing
management of critically ill patients. In particular the etiology of hemodynamic instability can be difficult
to ascertain in patients with cardiac pathophysiology without the use of this diagnostic tool. Resuscitation
efforts are frequently redirected based on ultrasound findings. Both transthoracic and transesophageal
echocardiography can be used to evaluate cardiovascular compromise. Additionally ultrasound can aid in
the placement of indwelling catheters, in the care of patients with pulmonary pathology, management of
intraperitoneal fluid collections and urinary tract assessment. Expertise in ultrasonography provides an
invaluable adjunct to the intensivists armamentarium.
The case presentation illustrates the difficulty that can be encountered when treating hemodynamic
instability. Despite both epinephrine and norepinephrine infusions the patient continues to exhibit a poor
hemodynamic status. Although a central venous and pulmonary catheter data are available the diagnosis
remains elusive. The clinical picture is consistent with left ventricular failure but is also compatible with
right heart failure, valvular dysfunction or cardiac tamponade. Echocardiography can provide real time
images to distinguish between differing etiologies.
Ultrasonography, particularly echocardiography, requires a formal education. The outline below simply
aims to provide a basic understanding of the use of ultrasonography in the critically ill patient and therefore
cannot substitute for formal training in critical care ultrasound.
I.
Cardiac Critical Care Ultrasound Examinations
A. Indications
1. Hemodynamic Instability
a) Ventricular Failure
b) Hypovolemia
c) Pulmonary Embolism
d) Acute Valvular Dysfunction
e) Cardiac Tamponade
2. Complications After Cardiothoracic Surgery
a) Infective Endocarditis
b) Suspected Aortic Dissection or Rupture
c) Unexplained Hypoxemia
d) Sources of Emboli
3. Chest Trauma with Hemodynamic Compromise
B. Indications for TEE over TTE - High Image Quality is Vital
1. Aortic Dissection
2. Endocarditis
3. Intracardiac Thrombus
4. Structures That May be Inadequately Seen on TTE
a) Thoracic Aorta
b) LA Appendage
c) Prosthetic Valves
5. Patient Conditions That Prevent Image Clarity on TTE
a) Severe Obesity
93
SOCCA Residents Guide 2013
C.
D.
E.
F.
II.
b) Emphysema
c) High PEEP
d) Surgical Drains, Incisions, Dressings
TEE Complications
1. Odynophagia 0.1%
2. Dental Injury 0.03%
3. Endotracheal Tube Dislodgement 0.03%
4. Esophageal Perforation 0.01%
Contraindications to TEE
1. Absolute
a) Esophageal Stricture
b) Esophageal Mass
c) Esophageal Diverticulum
d) Mallory-Weiss Tear
e) Dysphagia/Odynophagia Unevaluated
f) Cervical Spine Instability
2. Relative
a) Esophageal varices
b) Recent Esophageal/gastric Surgery
c) Oropharyngeal Carcinoma
d) Upper GI Bleeding
e) Severe Cervical Arthritis
f) Atlantoaxial Disease
Echocardiography Findings in Hemodynamic Instability
Hypovolemic Shock
a) Decreased End-Diastolic Area
b) “Kissing” Papillary Muscle
c) Hyperdynamic Function
2. Cardiogenic Shock
a) Failing Left Ventricle
(1) Decreased Area Change
(2) Increased End-Diastolic Area
(3) Increased End Systolic Area
b) Failing Right Ventricle
(1) Increased Right Ventricular Size
(2) Intraventricular Septum Bulges Towards Left Ventricle
(3) Pulmonary Embolus if Echogenic Density Present
c) Valvular Pathology
(1) Mitral Regurgitation
(2) Mitral Stenosis
(3) Aortic Regurgitation
(4) Aortic Stenosis
d) Cardiac Tamponade
(1) Pericardial Effusion
(2) Diastolic Collapse of Right Ventricle
Non-Cardiac Modalities of Critical Care Ultrasound Use
A. Pleural
1. Pneumothorax Identification
2. Effusion Identification, Characterization and Quantification
3. Guidance During Thoracocentesis
B. Lung
1. Identification of Aerated Normal Lung
2. Identification of Consolidated Lung With or Without Air Bronchograms
3. Identification of Pulmonary Edema
C. Abdominal
1. Identification, Quantification and Characterization of Intraperitoneal Fluid
2. Assessment of Urinary Tract
94
SOCCA Residents Guide 2013
a) Hydronephrosis
b) Distended Bladder
3. Identification of Abdominal Aortic Aneurysm and Dissection
D. Vascular
1. Identification of Deep Vein Thrombosis
2. Vascular Access
III. Images
The images below illustrate the authority ultrasonography can provide in a critical care setting. Even
though these images provide evidence of the capability of ultrasound to aid in patient management,
expertise to obtain proper ultrasonography images can only be obtained with appropriate training.
Figure 1 – TTE Four Chamber View with Right Ventricular Mass
95
SOCCA Residents Guide 2013
Figure 19-2 – TEE Four Chamber View with Stenotic Mitral Valve and Dilated Left
Atrium. The stenotic mitral valve does not open completely and the left atrium is dilated.
It should be noted that the chambers are inverted when compared to TTE images.
Figure 19-3 – Short-Axis and Long Axis View of Descending Aortic Dissection on TEE.
Conclusion
Ultrasonography provides the intensivist a tool to rapidly assess a patient’s condition. It is safe and can be
used at the patient’s bedside. With technological advances image quality has improved allowing for the
development of new applications for ultrasonography. As experience with this diagnostic modality has
increased it is clear that ultrasound use will become ubiquitous with the practice of critical care medicine.
96
SOCCA Residents Guide 2013
References
1.
2.
3.
4.
Mayo P, Beaulieu Y, Doelken P, et al. American College of Chest Physicians/La Societe de Reanimation de
Langue Francaise Statement on Competence in Critical Care Ultrasonography. Chest 2009; 135:1050-60.
Noble V, Nelson B, Sutingco A. Manual Emergency and Critical Care Ultrasound. New York: Cambridge
University Press; 2007.
Beaulieu Y, Marik P. Bedside Ultrasonography in the ICU Part 1. Chest 2005; 128:881-95.
Beaulieu Y, Marik P. Bedside Ultrasonography in the ICU Part 2. Chest 2005; 128:1766-81.
Questions
19.1 In which situation is a TTE most preferred over a TEE?
A. NPO status of less than 2 hours
B. Pregnancy
C. Esophageal Varices
D. Cervical Instability
E. Morbid Obesity
19.2 Images from a TEE are preferred over a TTE in all but which circumstance?
A. Severe ARDS Requiring High Levels of PEEP
B. Recent Sternotomy
C. Severe Hepatic Cirrhosis
D. Surveillance for Left Atrial Blood Clot before Cardioversion
E. Morbid Obesity
19.3 Which pulmonary pathology is ultrasound unable to assess?
A. Pneumothorax
B. Pulmonary Edema
C. Pulmonary Embolism
D. Pneumonia
E. Ultrasound is able to aid in diagnosis of all of these etiologies
19.4 A middle-aged man is a victim of a stab wound to the chest. He is hypotensive. A bedside TTE is performed and
the image below is obtained. What is located by the area indicated by the “X”?
A.
B.
C.
D.
E.
Left Ventricle
Right Ventricle
Lung
Pericardial Effusion
Liver
97
SOCCA Residents Guide 2013
20. Airway Management in the Intensive Care Unit
Nicole D. Martin, MD and Gabriel Sarah, MD
A 22-year-old male is involved in a motor vehicle accident and arrives to the
hospital spontaneously ventilating, alert, and oriented to person, place, and
time. He has full movement and muscle strength of the head and neck along
with decent shoulder movement. He has no elbow extension but good
flexion. No finger or wrist movement, and complete paralysis of the body and
legs. He is admitted for neurologic evaluation and further workup of his acute
injuries. The patient and his family initially refuse elective intubation, citing
his unaltered mental status and current ability to ventilate. After further
discussion, however, they agree with the intensivist’s plan to intubate. The
patient is 87 kg and 182 cm with good mouth opening. He is currently in ccollar and his Mallampati score is 2. An awake fiberoptic technique is selected
for this patient and his airway is topicalized with aerosolized and viscous
lidocaine. An infusion of dexmedetomidine is started during the topicalization
and he is placed on nasal canula at 4L/min. The patient is sedated yet
spontaneously ventilating while a fiberoptic bronchoscope is passed through
the mouth and the vocal cords are clearly visualized. An 8.0 cuffed
endotracheal tube is passed over the bronchoscope and tube placement is
confirmed with the patient subsequently sedated. Vital signs were
unchanged and oxygen saturation remained between 98% to 100% through
the entirety of the procedure.
Proficiency in airway management is a vital skill for any healthcare provider caring for critically ill
patients. Although elective airway management in the operating room is associated with a low rate of
complication, this is in contrast to emergent airway control in the ICU. Unlike the operating room, critically
ill patients have limited physiologic reserve and are much less likely to tolerate even the smallest degree of
hypoxia or hypoventilation. Furthermore, additional comorbidities such as cardiovascular instability, tissue
edema, copious secretions, injuries to the head and neck, and alterations in mental status may increase the
difficulty of airway management. The physical structure of the intensive care unit often does not allow for
easy access to the patient or their airway. In addition, the myriad of equipment available in the operating
room may not be easily accessible in the ICU. Lastly, many of the induction agents used in the elective
setting may not be suitable for the critically ill patient. Even low doses of some agents may have profound
effects on the ICU patient who may become severely hypotensive and rapidly desaturate. To summarize,
airway management in the intensive care unit can be a challenging task that requires knowledge of the
anatomy, physiology, and pathophysiology of the critically ill patient.
I.
Indications for Airway Management
A. Alterations in consciousness with inability to protect the airway
1. Loss of gag/cough reflex
2. GCS <9
3. Sustained seizure activity
B. Airway obstruction
1. Redundant tissues, obstructive sleep apnea, large, relaxed tongue, malignancy
2. Facial or neck trauma
3. Foreign body
4. Acute laryngeal edema - inhalational injury, Ludwig’s angina, epiglottitis
C. Severe pulmonary or multisystem injury associated with respiratory failure (hypoxia and
hypercarbia) requiring a need for positive pressure ventilation
D. Low c-spine injury (c5-T1) - while originally believed to only be a requirement with high c-spine
injuries evidence suggests that the immediate evaluation of low c-spine injuries and early
intubation in these patients is ideal
98
SOCCA Residents Guide 2013
E. Aggressive pulmonary toilet, copious secretions
F. Protection from aspiration of gastric contents
G. Hemodynamic instability
II.
Evaluation of the Airway
A. General Exam
1. Goal is to predict the relative ease or difficulty of airway management
2. Look externally to identify conditions that would predict a difficult airway
a) Obesity
b) Facial trauma or swelling
c) Short, thick neck, neck scarring from prior surgery or radiation, or neck deviation
secondary to malignancy
d) Small or receding mandible, pronounced overbite, or large, protruding tongue
e) Facial hair – mustache or beard may increase difficulty of bag mask ventilation and cause
difficulty securing the endotracheal tube
f) Edentulous or old age may cause redundant loose facial tissue that interfere with bag
mask ventilation
3. Mallampati Classification – performed with patient sitting upright, neck extended and tongue
protruding without phonation. Higher grades are thought to predict a more difficult intubation
a) I - Able to visualize the soft palate, fauces, uvula, anterior and posterior tonsillar pillars
b) II – Able to visualize the soft palate, fauces, uvula. The tongue hides tonsillar pillars.
c) III – Only the soft palate and base of uvula are visible.
d) IV – Only the soft palate can be seen (no uvula seen)
4. Oral opening – less than 4cm may increase the difficulty of intubation
5. Thyromental distance – less than 3 fingerbreadth (<6cm) may increase the difficulty of
intubation
6. Neck Range of motion – Limited ROM due to cervical collar, cervical spine injury or chronic
neck pathology (rheumatoid arthritis, Down syndrome) may limit ability to align oropharyngeal axis during laryngoscopy
7. Dentition – poor dentition can make laryngoscope blade insertion challenging and loose teeth
may be dislodged
8. NPO Status – trauma patients, patients on continuous tube feeds, morbidly obese, diabetic,
and pregnant patients are considered to have full stomachs with increased risk for aspiration
B. ICU considerations that increase difficulty of bag mask ventilation or intubation:
1. Altered mental status and uncooperative patients make awake intubations difficult
2. Airway and pharyngeal edema due to fluid shifts and long surgical procedures
3. Facial trauma and fractures may alter airway anatomy
4. Cervical spine precautions may limit ROM and ability to extend neck
5. Airway trauma or hemorrhage can obscure view during laryngoscopy and fiberoptic
intubation
6. Head trauma and potentially increased intracranial pressure
7. Unstable cardiovascular status
III.
Intubation in the ICU
A. Preparation
1. Urgency is essential in planning which technique of securing the airway is the most
appropriate. Consider whether intubation is emergent vs. urgent vs. semi-elective
2. Evaluate the airway as above
3. Evaluate the patient
a) Cardiovascular and respiratory status
b) Mental Status
c) Medications and allergies
d) Labs and electrolytes
4. Notify ICU staff, nurse, and respiratory therapist of impending intubation
5. Ensure all required equipment is available and functional. Ambu-bag, suction, medications,
endotracheal tube and, preferably, two functioning laryngoscope blades should be available
along with a means to measure capnography
6. Anticipate the need for special equipment and, if time allows, gather this equipment. This may
99
SOCCA Residents Guide 2013
include blankets for positioning, special blades, bougie catheters, intubating LMAs, video
laryngoscopes (e.g. Bullard, GlideScope, Airtraq) and a fiberoptic bronchoscope
7. Ensure monitors are functional and set to cycle at appropriate time intervals. Ensure alarms
are audible
B. Pharmacology of intubation in the ICU
1. ICU patients may be obtunded and require little or no medication
2. If medications are required, consider the patient’s hemodynamic status and electrolyte values.
Understand the pharmacologic profile of the medications chosen, the side effects, and how to
counter them
3. Induction agents
a) Used to provide amnesia
b) Includes: propofol, etomidate, ketamine, midazolam or pentothal
c) These drugs have cardiovascular, respiratory, neurologic and endocrinologic effects
which should be known prior to using
4. Analgesics
a) Have sedating properties
b) May reduce the need for induction agents
c) Includes: fentanyl, morphine, remifentanil and sufentanil
d) These agents may not reliably produce amnesia
e) Can mitigate pain response to laryngoscopy
5. Muscle relaxants
a) Not always necessary for intubation
b) Rapid onset agents include succinylcholine and rocuronium and can provide adequate
relaxation within 60 seconds and are preferred in patients with full stomachs
c) Slower onset agents with longer duration of actions like vecuronium, atracurium and cisatracurium can be used if patient can be ventilated by mask while waiting for adequate
relaxation, provided patient has an empty stomach
d) The disadvantage of longer acting muscle relaxants (i.e. rocuronium, vecuronium, etc.) is
that if the trachea cannot be intubated immediately, mask ventilation will need to be
adequate for a relatively long period of time until the airway can be secured or the
paralytic effects have dissipated. Mask ventilation may become inadequate over this
period
6. Miscellaneous agents
a) Local anesthetic agents can blunt the sympathetic response to intubation and can be used
to topicalize the airway for awake intubations
b) Vasoconstrictors can reduce bleeding with nasal intubations
C. Techniques for intubation in the ICU
1. General Considerations
a) Be prepared as listed above
(1) Continue to monitor patient for changes in condition that may require changes in
your airway management plan. Always have a backup plan in mind
(2) Preoxygenate the patient with 100% oxygen to increase the time that a patient will
tolerate apnea. Continue to oxygenate during the procedure if attempts at
intubation are unsuccessful. Use of noninvasive positive pressure ventilation may
increase oxygenation
(3) Patients who require urgent/emergent intubation may have high sympathetic tone.
Induction agents and restoration of oxygenation and ventilation can rapidly reduce
this tone. Be prepared for the possibility of hypotension after intubation. In
addition, induction agents and positive pressure ventilation may unmask
hypovolemia
2. Awake vs. Asleep intubation
a) An awake intubation is generally undertaken in patients where there is a significant
possibility of being unable to secure an airway after induction with amnestic or paralytic
agents. Examples are patients with significant facial and neck pathology and those with
poor mouth opening or high Mallampati score
b) To perform an awake intubation, the nerves of the airway – glossopharyngeal, superior
laryngeal and +/- the recurrent laryngeal – should be anesthetized with local anesthetics.
Inhalation of aerosolized local anesthetic and topicalization with local anesthetic may be
100
SOCCA Residents Guide 2013
sufficient
With adequate local anesthesia, an awake intubation can be performed in a myriad of
ways, including blind nasal, via direct laryngoscopy, or, most commonly, with a flexible
fiberoptic bronchoscope
d) Local anesthetic technique and the procedure, in general, take time and require a
cooperative patient for success. Also, a non-hypoxemic patient who is stable from a
respiratory standpoint is ideal for this form of intubation
e) Alpha-2 agonists, like dexmedetomidine, can provide sedation for awake intubations
without suppressing the respiratory drive
f) A low-dose infusion of remifentanil may also provide adequate sedation for an awake
fiberoptic while allowing the patient to remain able to spontaneously ventilate and follow
commands.
g) An asleep intubation is undertaken when a practitioner feels confident in their ability to
secure the airway after induction with anesthetic agents. Ideally, the patient will have an
empty stomach to prevent aspiration of gastric contents during the process of securing the
airway
h) The steps involved in asleep intubation include: preoxygenation, induction of anesthesia
with or without a paralytic agent, attempt at mask ventilation (unless rapid sequence
induction is indicated), and intubation. Securing the airway can be done in several ways,
usually through the use of direct laryngoscopy, but also with a GlideScope or McGrath
Video Laryngoscope, an intubating Laryngeal Mask Airway (LMA), an Eschmann tube
(“bougie”), or a flexible fiberoptic bronchoscope
i) With DL, the views obtained are graded as follows:
(1) Grade I: The entire glottis and vocal cords are seen
(2) Grade II: Only the posterior aspect of the glottis is seen (arytenoid cartilage)
(3) Grade III: Only the epiglottis is seen
(4) Grade IV: The epiglottis is not visualized
Rapid sequence intubation (RSI) is a variant of intubation performed with a patient considered
to have a full stomach
a) An induction agent and rapid acting paralytic are given in sequence and mask ventilation
is not performed, hopefully preventing aspiration of gastric contents. Preoxygenation, if
possible, is essential with RSI
b) Cricoid pressure (Sellick maneuver) is performed to occlude the esophagus
c) A backup plan is imperative should intubation prove to be difficult and, ideally, a surgeon
proficient in surgical airways will be present or available should attempts at intubation
fail
Management of the difficult airway
a) Understand that a difficult airway may be anticipated or unanticipated prior to induction
b) If anticipated, an awake intubation should be performed, if possible
c) If discovered after induction, the following steps should be taken:
(1) Call for help from other personnel with emergency airway management experience
(2) Attempt bag-mask ventilation, using an adjunct device (i.e. oral airway) if needed
(3) If unable to ventilate, attempt ventilation via an LMA or through the use of transtracheal jet ventilation or with a surgical airway
(4) If attempts remain unsuccessful, try to awaken the patient. This may prove difficult
in the ICU setting where many patients are intubated for emergency purposes
Oral vs. Nasal Intubation
a) Oral intubation:
(1) Often technically easier and faster
(2) Can interfere with the patient’s ability to communicate and can hinder oral care
(3) Usually permits the use of a larger endotracheal tube
b) Nasal intubation:
(1) Often better tolerated by patients – can mouth words for communication
(2) Usually requires a smaller endotracheal tube size and the angular nature of the tube
may prove difficult when suctioning secretions
(3) May be done blindly or with the aide of a laryngoscope or fiberoptic bronchoscope.
Fiberoptic usually is easier with the nasal approach
(4) May lead to an increased incidence of sinusitis
c)
3.
4.
5.
101
SOCCA Residents Guide 2013
D. Complications of Endotracheal Intubation
1. Complications during intubation
a) Trauma: dental injury, injury to lips or oral mucosa, perforation or dislocation of
structures of the pharynx, larynx, and trachea
b) Acute respiratory complications can include unrecognized esophageal intubation,
laryngospasm with or without pulmonary edema, and bronchospasm
c) Cardiovascular complications may include PVCs, VT, bradycardia, and hypo or
hypertension
d) Neurologic complications include increased intracranial pressure, spinal cord injury, or
passage of the tube into the cranial vault when a nasal intubation in attempted in a patient
with a basilar skull fracture
2. Complications with the ETT in place
a) Blockage, kinking, or movement of the tube causing mainstem intubation
b) Trauma to airway structures or ischemia to tracheal tissue
c) Sinusitis in the case of nasal intubation
3. Complications after extubation
a) Aspiration
b) Laryngospasm
c) Tracheoesophageal fistula
d) Vocal cord paralysis
e) Laryngeal edema with stridor
f) Tracheal stenosis
E. Controversies in Airway Management in the ICU
1. Long Term Intubation
a) Conversion to tracheostomy
(1) Some centers will convert to a surgical airway relatively early (i.e. less than 4 days)
when prolonged intubation is anticipated
b) Percutaneous vs. surgical tracheostomy
(1) Percutaneous tracheostomy can be performed at bedside in the ICU although some
centers will perform standard tracheostomies at bedside. Percutaneous approach
may be contraindicated in patients with poor neck flexion, large neck
circumference, or poorly defined neck structures
c) Tracheostomy decreases dead space in the ventilator circuit and allows for easier
suctioning
d) Less oro/nasopharyngeal damage and damage to the vocal cords is noted with
tracheostomy
e) Tracheostomy does carry a risk of significant complications, including pneumothorax,
tracheoinnominate artery fistula, and infection
f) There are studies that estimate up to a 1% complication rate leading to significant
morbidity or mortality due to tracheostomy. This may be difficult to assess, however, as
many tracheostomies are performed on an emergent basis which, in and of itself, may
lend to a higher rate of complications
IV.
Extubation in the ICU
A. Criteria for extubation in the ICU is more complicated than in the operating room
B. It must be determined that the patient may no longer require mechanical ventilation
C. FiO2 should be less than 0.5
D. The condition that required intubation should be resolved or medically controlled
1. Additional considerations may include stable vital signs, lack of fever, stable metabolic status,
and adequate hemoglobin
2. Patient must be able to adequately cough and control secretions
3. Patient must have adequate mental status and capacity to maintain a patent airway
4. Is there a leak around the airway? While this may rule out airway edema, the absence of a
leak should not completely exclude extubation as the ETT may have a snug fit in the trachea.
Upper airway examination with a fiberoptic may be warranted if concern is present
E. Performance of a spontaneous breathing trial is one of the most useful ways to determine the
patient’s ability to be extubated
1. Can be performed by using minimal pressure support and PEEP, CPAP alone, or by using a T-
102
SOCCA Residents Guide 2013
2.
3.
V.
piece
Watch patient for acute distress at onset of trial. If this does not occur, continue trial for a
maximum of 120 minutes
Evaluate for:
a) Negative inspiratory force greater than -20 cmH2O
b) Respiratory rate less than 30
c) Tidal volumes are greater than 5ml/kg
d) Rapid Shallow Breathing Index (RBSI) is the ratio of respiratory frequency to tidal
volume (Liters) (f/VT). A RSBI below 105 has been associated with the ability to
successfully extubate
Reintubation in the ICU
A. Repeat intubation rate is as high as 19% in extubated patients in surgical ICUs
B. A patient with a difficult airway may develop severe respiratory distress early after extubation and
require emergent reintubation. Failure to reintubate a patient can result in hypoxemia,
hypercarbia, hemodynamic instability and potentially death.
C. Airway exchange catheters (AEC) can be used to increase the safety of extubation as well as
exchanging endotracheal tubes (ex. In pt’s who present to the ICU with double lumen ETT or
whose ETT get clogged with secretions/mucous)
1. An AEC is a long, small ID, hollow, semirigid catheter that is inserted through an in situ ETT
before tracheal extubation.
2. After the ETT is withdrawn over the AEC, the AEC serves as a conduit to administer oxygen
manually, by insufflation, or by jet ventilation and as a stylet for repeated intubation.
References
1.
2.
3.
4.
5.
Buckley TA, Short TG, Rowbottom YM, Oh TE. Critical incident reporting in the intensive care unit.
Anaesthesia. 1997;52:403-409.
Levitan R, Ochroch EA. Airway management and direct laryngoscopy. A review and update. Critical Care
Clinics. 2000 Jul; 16(3):373-88.
Mort, T. Continuous Airway Access for the Difficult Extubation: The Efficacy of the Airway Exchange
Catheter. Anesthesia and Analgesia. November 2007 vol. 105 no. 5 1357-1362.
Hassid, Victor J., EA. Definitive Establishment of Airway Control is Critical for Optimal Outcome in Lower
Cervical Spinal Cord Injury. The Journal of Trauma Injury, Infection, and Critical Care. December 2008.
Vol. 65 no. 6, 1328-1332.
Thille, A, EA. Outcomes of Extubation Failures in Medical ICU Patients. Critical Care Medicine. December
2011. Vol 39 No. 12, 2612-2618
Questions
20.1 After intravenous dosing of medications for rapid sequence induction, you attempt to intubate the patient via direct
laryngoscopy. You have a grade 4 view and are unable to successfully intubate the trachea on your first intubation
attempt and the patient’s saturation is now 84%. After calling for help, what is your next step in airway management.
A. Placement of an LMA
B. Attempt to intubate with a different blade (i.e. mac to miller)
C. Fiberoptic intubation
D. Attempt face mask ventilation
20.2 Which of the following statements regarding cricoid pressure for rapid sequence intubation are true:
A. Despite adequate cricoid pressure, aspiration may still occur, particularly in patients with full stomachs or active
vomiting.
B. Cricoid pressure should be released immediately after the endotracheal tube is inserted by the person
performing the intubation.
C. When applied correctly, cricoid pressure will always prevent aspiration during intubation.
D. None of the above
103
SOCCA Residents Guide 2013
20.3 A patient presents after motor vehicle collision with blunt trauma to the face, head, and neck and decreasing mental
status. You have chosen to electively intubate this patient. Which of the following statements is true:
A. Nasal intubation is the route of choice
B. An asleep intubation with propofol and vecuronium is ideal
C. An awake fiberoptic intubation through the mouth is a safe way to intubate this patient.
D. A rapid sequence induction is not indicated for this patient should you elect to proceed with an asleep
intubation
20.4 A 37yr old female presents to the ER 2 hours after sustaining a 3rd degree burn to 45% of her body in a house fire.
On exam, you observe soot in the airway and singed nasal hairs. Pt is awake, alerted, and oriented to person, place, and
time and is able to speak clearly. Which of the following is most appropriate:
A. Clear the patient for discharge as she is at low risk for airway obstruction
B. Continue to monitor the patient in the ER as she may develop airway edema
C. Electively intubate this patient using vecuronium, since succinylcholine is contraindicated in burn patients
D. Electively intubate this patient using a RSI with succinylcholine
20.5 You are called to the ICU to intubate a patient with respiratory distress. The patient is a 27 yr old female G1P0 at
30weeks gestational age. She has a history of cystic fibrosis, chronic hypertension, and newly diagnosed gestational
diabetes. Her last meal was 10 hours ago. You prepare your equipment for intubation. Which of the following is most
accurate regarding your induction:
A. A standard induction using a slower acting paralytic agent is appropriate since the patient’s last meal was over
8 hours ago.
B. Patients with diabetes always have delayed gastric emptying and every diabetic patient should receive a rapid
sequence induction
C. Pregnancy places this patient at risk for aspiration and a rapid sequence induction is safest.
D. Pregnancy causes dilation of the airway and a larger ETT is recommended in such patients.
104
SOCCA Residents Guide 2013
21. Chronic Airway Management: The Tracheostomy Tube
Sherif Afifi, MD, FCCM, FCCP
Case Scenario
A 56 year old, 115 kg, 60 inch tall woman was admitted for squamous cell
carcinoma of the mandible, underwent tracheostomy under local anesthesia,
then, under general anesthesia, eventual extensive mandibular resection,
and flap reconstruction. The first postoperative week was uneventful, and the
first tracheostomy tube (TT) change was done where the TT was down-sized
from # 8.0 cuffed Shiley tube to a # 4.0 cuffless tube. Later that day, the
patient developed a coughing spell and dislodged the TT. Staff members in
attendance were unable to reinsert the tube after several attempts. The
patient was oro-tracheally intubated, and the position of the tip of the
endotracheal tube was confirmed within the lumen of the trachea by
fiberoptic bronchoscopy
Introduction
Care of the patient with a TT encompasses several critical care skills and knowledge including acute airway
management, pulmonary mechanics, mechanical ventilation, discontinuation of mechanical ventilation,
types of TT, techniques of tracheostomy, routine management of downsizing to the point of TT
decannulation, and, surveillance for complications from TT.
Outline
I.
II.
III.
Indications for Tracheotomy
Similar to endotracheal intubation, the conditions that require a tracheostomy fall into four broad
categories: difficulties with ventilation, airway obstruction, airway protection, and excessive
respiratory secretions.
A. Need for extended mechanical ventilator support
B. Optimize bronchial hygiene
C. Facilitate weaning from mechanical ventilator support
D. Upper airway obstruction
E. Airway and lung protection
F. Following Laryngectomy
Timing of Tracheostomy
The benefit of early (within one week) versus late tracheostomy has been controversial. Various
publications concluded that earlier tracheostomy is associated with shorter duration of mechanical
ventilation and length of stays in the ICU. Moreover, tracheostomy after 21 days of intubation has
been associated with a higher of failure to wean, longer ICU stays, and higher mortality. Project
IMPACT, a multi-institutional database adopted by the Society of Critical Care Medicine (SCCM),
attempted to identify not only optimal time to tracheotomy but also to evaluate clinical and
nonclinical factors on tracheostomy practice. With a database of nearly 44,000 patients, they found
the median time to tracheotomy was 9 days and that prolonged intubation led to increased duration of
mechanical ventilation, increased ICU length of stay, and increased hospital length of stay. Currently,
the recommended critical care practice is to withhold sedation and initiate a weaning trial with
spontaneous breathing on a daily basis. When the patient is unable to be weaned after 7–10 days, a
tracheotomy is usually considered. Early tracheostomy in carefully selected patients where long-term
ventilation is predicted may be beneficial for outcome, length of stay, outcome, and cost
Advantages of Tracheostomy
A. Avoid of tracheo-laryngeal injury from prolonged endo-tracheal intubation
1. Reversible ulceration, erythema, inflammation
2. Long-term laryngeal granuloma, vocal cord adhesions, and stricture
B. Increase efficiency of bronchial hygiene
105
SOCCA Residents Guide 2013
C.
D.
E.
F.
IV.
V.
VI.
Mobilize early
Facilitate communication
Advance towards swallowing
Improve patient comfort
Tracheotomy Procedure
A. Set-up & Safety Measures
1. The conduct of a safe tracheotomy, whether at the ICU bedside or in the operating room, is
dependent on careful patient assessment and a thorough equipment check. A pre-procedure
checklist should be reviewed prior to commencing the procedure (see table 1)
B. Open Tracheotomy: steps:
1. Incision below the cricoids membrane (between 2nd and 3rd tracheal rings)
2. Thyroid isthmus is divided and/or retracted
3. ETT cuff is deflated and ETT is withdrawn to sub-glottic position to allow TT insertion
4. Confirm presence of breath sounds and ETCO2
5. Remove ETT after tracheostomy tube is secured
C. Percutaneous Tracheotomy: steps are accomplished under continuous Fiberoptic bronchoscopic
(FOB) guidance
1. Fiberoptic bronchoscope (FOB) is placed into ETT
2. ETT is retracted to sub-glottic position
3. Tracheal lumen is accessed with an introducer needle
4. Wire is advanced into the trachea
5. Using Seldinger technique, the trachea is dilated with successive dilators
6. Once tracheotomy is established, the TT is placed and the wire is removed.
7. Confirm presence of tracheostomy tube with FOB, ETCO2 and auscultation
8. Remove ETT after tracheostomy tube is secured
9. In select patients, percutaneous tracheostomy is safe and cost-effective
Components of a Tracheostomy Tube (see Figure 21-1)
Management of a Tracheotomy Tube
A. Tracheostomy tube cuff inflation and management (low pressure, high volume)
B. Cuff Leaks: these are multi-factorial and can result from
1. leak within the tracheostomy tube cuff
2. leak around the tracheostomy tube cuff
3. Cuff is not fully inflated to seal the airway.
4. Cuff is inflated, but the tube is too small for the airway.
5. Cuff is inflated, but air and secretions are leaking past the cuff.
6. Leak is associated with changes in patient position.
106
SOCCA Residents Guide 2013
7. Elevated airway pressures or high levels of PEEP.
8. An area of tracheomalacia has developed.
C. leak within the TT itself: these are rare and predominantly result from size mis-match between
tracheostomy tube and inner canula.
VII. Sequential steps towards removal of a Tracheostomy Tube
A. Vocalization
B. Down-sizing
C. Plugging
D. De-cannulation
VIII. Complications and Emergencies of a Tracheotomy Tube
A. Early: (within the first week after Tracheostomy)
1. Accidental de-cannulation
2. Tube malposition
3. Stomal Hemorrhage
4. Pneumothorax
5. Sub-cutaneous emphysema
6. Pneumo-mediastinum
B. Late
1. Pulmonary Infections
a) ventilator-associated pneumonia
b) tracheo-bronchitis
2. Tracheal Stenosis: at site of tracheostomy tube cuff or stoma
3. Tracheo-innominate arter fistula
4. Tracheo-malacia
5. Swallowing Dysfunction
6. Stomal Erosions: traction against the stoma will create a widened area of erosion or
maceration of tissue. These wounds can become inflamed or infected and can develop
cellulitis. Infected, erosive wounds present a greater challenge. Systemic antibiotics, adequate
local treatment Mesalt, (15% sodium chloride). DuoDERM, silver-impregnated pads.
7. Stomal Infections: Most inflammation of the stoma is caused by moisture, which creates
irritation and inflammation of the skin, maceration of tissue and the development of cellulitis.
These complications can be prevented by keeping the stoma dry, frequent changes of the
gauze dressing, and absorbent dressings may be required (Lyofoam)
8. Tracheo-innominate artery fistula
9. trachea-esophageal fistula
Conclusions
Care of the patient with the tracheostomy tube includes:
• Careful surveillance for any evidence of early or late complications from the tube
• An orderly management plan that works towards removal of the tracheostomy tube (with few longterm exceptions)
• Integrating all aspects of pulmonary care, nutrition, swallowing, phonation, and skin care
• Accidental dislodgement of a tracheostomy requires a careful, immediate multi-disciplinary
assessment if it occurs early (within 7 days of the procedure), as well as confirmation of the trach
tube position within the lumen of the trachea at the bedside in all cases.
References
1.
2.
3.
Morris L, Afifi S. Tracheostomies: The Complete Guide. Springer Publications (2010)
Freeman, BD, Borecki, I. B., Coopersmith, C. M., & Buchman, T. G. (2005). Relationship between
tracheostomy timing and duration of mechanical ventilation in critically ill patients. Crit Care Med,
33(11), 2513–2520.
Mitchell, RB, Hussey HM, Setzen G, Jacobs IN, Nussenbaum B, Dawson C, Brown CA, Brandt C,
Deakins K, Hartnick C, Merati. (2012). A. Clinical Consensus Statement: Tracheostomy Care.
Otolaryngology – Head and Neck Surgery, 147 (2) supplement
107
SOCCA Residents Guide 2013
Questions
21.1 Chronic complications of tracheostomies include all of the following except:
A. a) swallowing dysfunction
B. b) tracheomalacia
C. c) stomal erosion
D. d) innominate artery – tracheal fistula
E. e) pneumothorax
21.2 The timing of tracheostomy
A. a) may impact success of weaning from mechanical ventilation
B. b) is considered late if conducted 10 days after orotracheal intubation for respiratory failure
C. c) depends on whether an open or a percutaneous tracheostomy is planned
D. d) ideally should be on the 4th day post-orotracheal intubation for respiratory failure due to pneumonia
E. e) has no relevance to patient outcome
21.3 Which of the following is not an indication for tracheostomy
A. a) a patient with myasthenia gravis unresponsive to medical therapy and has a pH of 7.20 and PaCO2 of 80; the
patient has a history of failed orotracheal intubation and refuses awake intubation.
B. b) stridor in a patient with supraglottitis
C. c) PaCO2 of 62 in a COPD patient with a respiratory rate of 24 on nasal cannula
D. d) Absence of swallow reflex in a patient with a large sub-arachnoid hemorrhage who was orotracheally
intubated 7 days ago
21.4 The following are considered emergent complications of tracheostomy tube except:
A. a) accidental de-cannulation
B. b) pneumothorax
C. c) pneumo-mediastinum
D. d) swallowing dysfunction
E. e) stomal hemorrhage
21.5 The most common sequence towards removal of a tracheostomy tube is:
A. a) weaned mechanical ventilation -> vocalization -> down-sizing -> plugging -> decannulation
B. b) down-sizing -> vocalization -> weaned mechanical ventilation -> plugging -> decannulation
C. c) plugging -> decannulation -> vocalization -> down-sizing -> weaned mechanical ventilation
D. d) weaned mechanical ventilation -> plugging -> down-sizing -> decannulation -> vocalization
E. e) decannulation -> down-sizing -> plugging -> vocalization -> weaned mechanical ventilation
108
SOCCA Residents Guide 2013
Table 21-1: Bedside Tracheostomy Pre-procedural Checklist
Personnel
в–Ў Surgeon: ______________________________
в–Ў Anesthesiologist: ______________________
в–Ў RN: __________________________________
Key Elements
в–Ў Patient is stable to undergo procedure at this time
в–Ў Bipolar vs monopolar cautery has been considered
Both CO2 and ABC Fire Extinguishers have been located with
в–Ў their triggers, and the responsible person has been pointed to
the team
в–Ў
Main O2 gas shut-off valve in the ICU has been located, and the
responsible person has been pointed to the team
в–Ў Skin Antisepsis product is confirmed to be alcohol-free
в–Ў Drape does not cover mouth, endotracheal tube, or Ambu bag
в–Ў FiO2 level setting has been coordinated with electrocautery
в–Ў Time-out has been performed and documented.
109
SOCCA Residents Guide 2013
22. Management of Mechanical Ventilation
Daniel W. Johnson MD and Edward A. Bittner MD, PhD
A 66-year-old man with a history of CAD and COPD is admitted to the ICU
after an open AAA repair. Overnight he remains on full ventilator support
(Vt=600 mL, freq=14, FiO2=0.4, PEEP=5). His fluid requirements decrease
and he is hemodynamically stable in the morning, with an ABG of 7.38 / 37 /
92, heavily sedated on propofol and morphine infusions. How would you
proceed toward extubation? SBT? PSV? SIMV?
Airway control and ventilatory support are commonly encountered in the ICU. At present, numerous
techniques exist for the initial control and subsequent support of the respiratory system. A thorough
understanding of these techniques leads to individualized treatment strategies and reduction of
complications.
I.
Airway
A. Indications for tracheal intubation
1. Hypoxemic respiratory failure
a) Intubation facilitates administration of high FiO2 and positive pressure ventilation, which
reduces trans-pulmonary shunt.
2. Hypercapnic respiratory failure
a) Positive pressure ventilation reduces the patient’s work of breathing and facilitates
elimination of CO2 when patient effort is inadequate.
3. Airway protection
a) Intubation secures the airway, reduces the risk of massive aspiration, and prevents closure
of airway (e.g. secondary to edema from inhalation injury or external compression from
hematoma).
4. Impending hemodynamic collapse
5. Facilitation of suctioning / pulmonary toilet / bronchoscopy
B. Endotracheal intubation
1. Oral
a) Orotracheal intubation is the most common route, usually performed with direct
laryngoscopy. Adequate mandible and neck mobility facilitate direct visualization of
glottis. Alternatively, fiberoptic laryngoscopy with a flexible bronchoscope or rigid
device can be used.
2. Nasal
a) Nasotracheal intubation may be performed blindly (guided by breath sounds), with direct
laryngoscopy (and forceps to guide the tip of the tube into the glottis) or with fiberoptic
devices. Nasotracheal intubation can sometimes be performed in cases when the oral
route is impossible. Disadvantages include potential for nasal hemorrhage during
placement, increased airway resistance, difficulty with passage of suction catheter and
increased risk of sinusitis.
3. Double lumen
a) Double lumen tubes are composed of two tubes; the tip of one ends in a mainstem
bronchus and the tip of the other ends in the trachea. Both tubes have cuffs so that the
lungs can be isolated from each other. Proper positioning of the double lumen tube is
essential and placement is usually confirmed with flexible bronchoscopy. Minimal
patient movement can cause dislodgement, so deep sedation and paralysis are usually
necessary.
b) Placement of a double lumen tube is indicated for surgical exposure, lung protection
(from massive hemoptysis or unilateral infection), bronchoalveolar lavage or independent
lung ventilation. Complications include difficult/impossible placement, airway trauma
and malposition (with associated lobar collapse, hypoxia and hypoventilation).
4. Bronchial blockers
a) Bronchial blockers are balloons that can be selectively inflated in a large bronchus. They
are used in situations when it is not possible to place a double lumen tube (e.g. difficult
110
SOCCA Residents Guide 2013
airway or pediatrics) or when satisfactory lung isolation cannot be achieved by other
means.
C. Surgical airway
1. Cricothyrotomy
a) Cricothyrotomy is opening of the cricothyroid membrane (by needle or surgical incision)
for access to the airway. Indications include emergency access to the airway in cases
when mask ventilation and intubation are impossible or when the pharyngeal airway is
obstructed by trauma or foreign body. It takes less time to perform than tracheostomy
and is associated with less bleeding.
2. Tracheostomy
a) Tracheostomy is opening of the trachea for establishment of a secure airway. It is
primarily indicated in patients who require long term mechanical ventilation.
Considerable controversy exists regarding the optimal timing of tracheostomy.
Advantages of tracheostomy over an endotracheal tube include increased ease of
suctioning, ease of swallowing, and the possibility of speaking.
D. Endotracheal tube management
1. After insertion, the tube is subject to dynamic changes due to patient pathology and medical
treatment. Complications can occur if the tube is not well managed. Inspissated secretions can
occlude the tube, the tube can migrate into a mainstem bronchus (e.g. with neck flexion) or
out of the trachea (e.g. with neck extension). If the cuff is insufficiently inflated, a leak
resulting in inadequate ventilation is possible. If the cuff is over-inflated, tracheal ischemia
can result. Cuff pressures should be less than 25 mmHg to minimize risk for ischemia.
II.
Ventilators
A. Positive pressure ventilators operate by applying positive pressure (via flow of O2 or air) to the
airways during inspiration. In the ICU, mechanical ventilation is almost exclusively positive
pressure ventilation.
B. Negative pressure ventilators create intermittent negative pressure around the thorax and abdomen.
The “iron lung,” popular during polio outbreaks in the 1940s-50s, is the prototypical example. In
modern ICUs, negative pressure ventilators are almost never used.
III.
Modes of Ventilation
The mode of mechanical ventilation describes the control (volume, pressure, flow, time) and phase
variables (trigger, limit, cycle) that define how ventilation is provided. The control variable remains
constant throughout inspiration, regardless of resistance. Phase variables include trigger, limit and
cycle variables. The trigger variable is adjusted to sense patient effort (by negative pressure or by
flow at the proximal airway) for the initiation of inspiration. The limit-ventilation variable rises no
higher than a given preset value or increases to a preset value before inspiration ends. Cycle is the
variable that terminates inspiration (commonly volume, time or flow).
The descriptions of modes of mechanical ventilation are listed in order of oldest (and simplest) to
newest (and more sophisticated). Please note that in the absence of patient respiratory effort (e.g. in
the setting of deep sedation or neuromuscular weakness or paralysis), there is essentially no
difference between CMV, IMV, SIMV and A-C; in this setting all of these modes are capable of
providing full ventilatory support and will mimic CMV.
A. Continuous mandatory ventilation (CMV): In this simple mode, the intensivist presets the tidal
volume (Vt), frequency (freq), flow rate, and the FiO2, and the ventilator delivers breaths with no
potential for patient-ventilator interaction. Alternatively the intensivist could set the airway
pressure, frequency, inspiratory time and FiO2 and achieve a similar profile. Because of the lack
of potential for patient-ventilator interaction and multiple advances over the last 30 years, CMV is
almost never used in the modern ICU.
B. Intermittent mandatory ventilation (IMV): In this mode, the intensivist sets the ventilator in the
same way as CMV and the ventilator delivers breaths while allowing the patient to take
spontaneous breaths at any time during the respiratory cycle. IMV was an improvement on CMV
because it allows the patient to spontaneously take a breath (in CMV, a patient’s inspiratory effort
is met with a closed valve, which is both uncomfortable and dangerous). If a patient is taking a
spontaneous breath when the IMV ventilator is scheduled to give a mechanical breath, “breath
stacking” will occur (the full preset Vt will be delivered on top of the patient’s spontaneous Vt).
Spontaneous breaths during IMV receive no support from the ventilator.
111
SOCCA Residents Guide 2013
C. Synchronized IMV (SIMV): This is IMV but with the ability to sense the patient’s spontaneous
breath so that the mandatory mechanical breaths can be delivered in synchrony with spontaneous
breaths. SIMV was an improvement on IMV because it reduces the risk of “breath stacking.”
With early SIMV ventilators, between mandatory breaths the patient was allowed to breathe
spontaneously with no support from the ventilator. Modern SIMV ventilators can provide varying
levels of pressure support (see below) for the breaths between mandatory breaths.
D. Assist-control (A-C): This is CMV except that every time the patient makes an inspiratory effort,
the ventilator delivers a fully supported breath. For example, if the A-C ventilator is set at
Vt=1000mL and freq=8 and the patient makes 30 inspiratory efforts per minute, the ventilator will
deliver 1000mL x 30, or 30 L/min of ventilation.
1. Volume control: this term is commonly used to mean assist control with Vt as the cycle
variable (though volume control can be used with SIMV or A-C). In volume control mode, the
I:E ratio is typically determined by the set flow rate (e.g. 30 L/min), which is typically
constant throughout the breath.
2. Pressure control: this term is commonly used to mean assist control with pressure as the
control variable (though pressure control can be used with SIMV or A-C). In pressure control
mode, the intensivist sets a specific airway pressure and sets an inspiratory time over which
this pressure will be maintained. Inspiratory flow during pressure control ventilation is
decelerating and is determined by the set pressure, airways resistance, and the compliance of
the lungs and chest wall. For example, the ventilator might be set at 20 cmH2O for 2 seconds
per breath. This might result in large tidal volumes in patients with compliant lungs and result
in small tidal volumes in patients with non-compliant lungs.
E. Pressure support (PSV): Unlike the above modes, pressure support does not provide full ventilator
support to an apneic patient. It is a pressure-preset, flow-cycled mode used to support the patient’s
spontaneous respiratory efforts. With each inspiratory effort the patient triggers the ventilator,
which maintains the preset pressure in the circuit throughout inspiration. The inspiratory cycle
ends when the flow rate has decreased to a pre-determined level (usually 25% of the peak flow
rate).
F. Inverse ratio ventilation (IRV): this mode increases the mean airway pressure by prolonging the
inspiratory to expiratory (I:E) ratio. The prolonged inflation time can help prevent alveolar
collapse resulting in improved oxygenation. Heavy sedation with or without neuromuscular
blockade is usually required for patients to remain on IRV.
G. Airway pressure release ventilation (APRV): APRV cycles between a high continuous positive
airway pressure (HCPAP) and a low continuous positive airway pressure (LCPAP). Transition
from HCPAP to LCPAP allows deflation and transition from LCPAP to HCPAP allows inflation.
APRV can improve oxygenation by maximizing alveolar recruitment and reducing shunt.
H. Proportional assist ventilation (PAV): PAV is synchronized partial ventilatory support in which the
ventilator generates pressure in proportion to the patient’s instantaneous inspiratory effort. In PAV,
the more effort a patient makes, the more support the ventilator provides. PAV was created to more
closely mimic the body’s innate communication between the nervous system and the respiratory
system. It is sometimes used in the ICU as an alternative to pressure support ventilation.
I. Noninvasive Positive Pressure Ventilation (NPPV): NPPV is the delivery of mechanically assisted
or generated breaths without an endotracheal or tracheostomy tube. Ventilation is delivered via
face mask, nasal mask or helmet. Advantages of NPPV include: avoiding the risks of intubation,
reduced need for sedation and lower rates of healthcare-associated pneumonia. Disadvantages
include: lack of protection against massive aspiration, less airway pressure tolerated, and lack of
access to the airways for suctioning. NPPV is most beneficial for patients with acute COPD
exacerbations and patients with acute cardiogenic pulmonary edema. Other uses include postextubation support, obesity-hypoventilation syndrome, acute post-operative respiratory failure and
for patients who are not candidates for intubation (DNI order).
1. Continuous positive airway pressure (CPAP): CPAP works by generating a continuous flow
of oxygen and/or air that maintains a continuous positive pressure to the respiratory system
during inspiration and expiration thus preventing airway and alveolar collapse. CPAP may
improve alveolar ventilation and oxygenation by reversing atelectasis, maintaining greater end
expiratory lung volume, and preventing obstruction of the airways.
2. Bi-level positive airway pressure (BiPAP): BiPAP involves independently set inspiratory
positive airway pressure (IPAP) and expiratory positive airway pressure (EPAP). Vt results
from the combination of patient effort and the difference between IPAP and EPAP. BiPAP is
112
SOCCA Residents Guide 2013
effective at reducing PaCO2.
Positive end expiratory pressure (PEEP): PEEP may increase oxygenation in lung diseases
characterized by lung collapse.
1. Extrinsic PEEP (PEEP set on the ventilator) is essentially CPAP in between inspiratory cycles.
It maintains alveolar recruitment, increases FRC, decreases pulmonary shunt, may improve
lung compliance, and may decrease patient work of breathing. The application of PEEP may
have disadvantages: it increases intra-thoracic pressure which can decrease venous return and
compromise cardiac output. In addition, since PEEP has the greatest effect on compliant
regions of the lung, overdistention can occur resulting in increased dead space and high levels
of PEEP may contribute to ventilator-induced lung injury (VILI).
2. Intrinsic PEEP or auto-PEEP results from pressure developing from gas trapping (dynamic
hyperinflation) due to insufficient expiratory time and/or excessive expiratory airways
resistance. Increasing expiratory time, reducing airways resistance, and reducing minute
ventilation (by reducing tidal volume or reducing frequency) are methods for reducing autoPEEP.
3. Recruitment maneuvers refer to the application of elevated pressures and volumes for variable
duration, magnitude and frequency in an effort to recruit atelectatic lung
4. Permissive hypercapnia is an approach to limit ventilator-induced lung injury through
deliberate tolerance of elevated PCO2 in the setting of hypoventilation. Contraindications
include elevated ICP, right ventricular failure, and severe ongoing acidemia.
K. Complications of mechanical ventilation
1. Ventilator-induced lung injury (VILI) occurs when the lung is directly damaged by the action
of mechanical ventilation. “Barotrauma” is alveolar overdistention/rupture related to high
inspiratory pressures. Pneumothorax, pneumomediastinum and pneumoperitoneum can occur
in this setting. “Volutrauma” is lung injury from excessive volume rather than excessive
pressure. “Atelectrauma” refers to the possibility of injury to the lung secondary to the cyclic
opening and closing of alveoli during mechanical ventilation. “Biotrauma” refers tothe
release of inflammatory mediators related to mechanical ventilation.
2. Oxygen toxicity: prolonged exposure to high concentrations of oxygen may cause lung
damage. Normal lung units are at highest risk for oxygen toxicity because these areas receive
the most ventilation. The FiO2 should be reduced when possible provided that arterial
oxygenation is adequate. In adults, an FiO2 of less than 0.5 is considered safe.
J.
IV.
Mechanics
A. Airway pressures
1. Peak (Ppeak) is the pressure reached at end inspiration during positive pressure volume
controlled ventilation. Ppeak is the sum of the pressure required to overcome airways
resistance and the pressure required to overcome the elastic properties of the lung/chest.
2. Plateau (Pplat) reflects the pressure required to overcome the elastic properties of the lung/
chest. The Pplat is an estimate of the peak alveolar pressure, which is an indicator of alveolar
distention. Measurement of Pplat requires the absence of patient effort and is obtained during
a short inspiratory hold.
3. Mean: the average pressure within the airway during one complete respiratory cycle. It is
related to inspiratory time, freq, Ppeak and PEEP.
B. Compliance is change in volume divided by the change in pressure.
1. Static compliance is measured when airflow is absent. It is calculated by Cstat = Vt / (Pplat PEEP). When airflow is absent, the airways resistance is not a factor. Thus static compliance
reflects the distensibility of the lung/chest only.
2. Dynamic compliance is measured when airflow is present at end inspiration. Since airflow is
present, airways resistance contributes to Cdyn. It is calculated by Cdyn = Vt / (Ppeak PEEP)
3. Comparing static and dynamic compliance can help identify the cause(s) for difficulty with
ventilation or difficulty with discontinuing the ventilator. Cdyn is decreased by conditions
where airways resistance is increased (e.g. bronchospasm), while the Cstat is unaffected by
such conditions.
4. Resistance (R), is the change in pressure divided by the flow. It is calculated by R = (Ppeak –
Pplat) / F where F is the peak inspiratory flow rate
C. Pressure- flow relationships
113
SOCCA Residents Guide 2013
1.
2.
3.
V.
Flow of air into the alveolus is driven by trans-pulmonary pressure (PL), or the difference
between alveolar pressure and pleural pressure. PL is calculated by PL = PA - Ppl where PL is
transpulmonary pressure, PA is alveolar pressure and Ppl is pleural pressure. Flow,
transpulmonary pressure and resistance relationships are summarized by Flow = PL / R, where
PL = PA - Ppl Because PA and Ppl are difficult to measure directly, they are estimated by Pplat
and esophageal pressure (Pes) respectively in clinical practice.
Esophageal pressure changes reflects pleural pressure changes (the absolute Pes does not
reflect absolute pleural pressure). Pes is measured with a thin walled balloon in the lower
esophagus. Changes in Pes can beused to assess chest wall compliance during full ventilatory
support and to assess respiratory effort, work of breathing and auto-PEEP during spontaneous
breathing and patient-triggered modes of ventilation. In severe lung disease (e.g. severe
ARDS), esophageal manometers can be used to estimate Ppl so that PL can be estimated and
ventilator settings adjusted accordingly.
Work of breathing: To achieve ventilation, work is performed to overcome the elastic and
frictional resistances of the lung and chest wall. Work of breathing can be calculated by
plotting the transpulmonary pressure (PL) against tidal volume and measuring the subtended
area.
Monitoring and physiology
A. Oxygenation
1. Pulse oximetry is a standard ICU monitor that provides noninvasive measurement of the
oxygen saturation of hemoglobin using differential light absorption characteristics in
oxyhemoglobin versus deoxyhemoglobin. Two wavelengths of light, red (660 nm) and
infrared (900-940 nm) are passed through a pulsating vascular bed (e.g. a fingertip) to a
photodetector. The relative amounts of red and infrared light reaching the photodetector
provide information about the relative ratio of deoxyhemoglobin and oxyhemoglobin. These
data are compared (by a computer) to calibration curves developed in studies of healthy
volunteers to give an oxygen saturation reading, denoted SpO2. The actual arterial oxygen
saturation, SaO2, is + 4% to 5% of the SpO2 when the SaO2 is greater than 80%. At lower
SaO2 the accuracy is diminished. Other causes of inaccurate SpO2 include
dyshemoglobinemias, dyes, pigments, low perfusion, motion, abnormal pulses, anemia and
external light sources.
2. Arterial blood gas (ABG) analysis provides (among other data) a measurement of the partial
pressure of oxygen (PaO2), which represents the amount of oxygen dissolved in arterial blood.
It is important to remember that dissolved oxygen (represented by PaO2) makes up a small
portion of the total arterial oxygen content. The oxygen content of arterial blood (CaO2)
consists of two components: oxygen bound to hemoglobin (which determines the SaO2) and
the oxygen dissolved in plasma (which determines the PaO2). CaO2 is described by CaO2 =
[1.34 * Hgb * SaO2] + [PaO2 * 0.003] where 1.34 mL O2 per g Hgb is a constant, Hgb is
hemoglobin in g/dL, SaO2 is the arterial oxygen saturation and should be in decimal form
(e.g. 98% is written 0.98), PaO2 is in mm Hg, and 0.003 mL O2 per mm Hg per dL is a
constant
a) Intermittent ABG: a sample of arterial blood is inserted into a blood gas analyzer
intermittently and results are displayed within a few minutes
b) Continuous ABG monitors detect variations in light and fluorescence to allow the
continuous display of ABG results. These monitors are subject to a variety of artifacts
and are not routinely used in the ICU.
3. Transcutaneous oxygen tension monitoring uses the polarographic principle to measure PO2 at
a warmed skin surface. This monitor is particularly useful in neonates and infants, but due to
technical and physiologic limitations in adult patients, it is rarely used in the adult ICU.
4. The shunt fraction is the proportion of the cardiac output that does not participate in gas
exchange. Normal shunt fraction is approximately 3% and is mostly due to the bronchial
circulation. The degree of shunt can be estimated by the shunt equation: Qs/Qt = (CcO2 –
CaO2) / (CcO2 – CvO2) where Qs is shunt, Qt is total cardiac output, CcO2 is end-capillary O2
content, CaO2 is arterial O2 content and CvO2 is mixed venous O2 content
5. Global oxygen delivery (DO2) is the product of arterial O2 content and the cardiac output:
DO2 = CaO2 * CO * 10 where DO2 is in mL/min, CaO2 is in mL/dL, CO is cardiac output in
L/min, and 10 converts L into dL
114
SOCCA Residents Guide 2013
6.
Oxygen consumption (VO2) is calculated by the Fick equation VO2 = CO * (CaO2 – CvO2) *
10 where VO2 is in mL/min and CO is in L/min
B. Ventilation
1. Capnometry/capnography: The capnometer is a device that measures CO2 in exhaled gas and
the capnograph provides a display and ability to track changes in CO2. Quantitative
capnometers measure CO2 using infrared spectroscopy, Raman spectroscopy or mass
spectroscopy. Quantitative capnometers and capnography are standard monitors in the
operating room and are sometimes used in the ICU to non-invasively estimate/track the
PaCO2. In the setting of lung disease, capnometry is less accurate in its estimate of PaCO2.
Non-quantitative capnometers indicate the presence of CO2 by color change of an indicator
material (e.g. for confirmation of endotracheal tube placement).
2. ABG: the arterial partial pressure of CO2 reflects the balance between CO2 production and the
alveolar ventilation. PaCO2 varies directly with CO2 production (VCO2) and inversely with
alveolar ventilation (VA) as described by: PaCO2 = VCO2 / VA Minute ventilation (VE) affects
the PaCO2 only to the extent that it affects the alveolar ventilation (VA).
3. CO2 production is a function of O2 consumption and CO2 that is liberated in the buffering of
H+ ions. In normal physiology, increased CO2 production is rapidly followed by an increase in
alveolar ventilation to eliminate excess CO2 and maintain normal PaCO2. In patients with
impaired ability to increase alveolar ventilation (e.g. sedation, weakness or lung disease), an
increase in CO2 production can result in an increase in PaCO2. Overfeeding is a recognized
cause of hypercapnia in patients with respiratory failure. Overfeeding with carbohydrates is
especially problematic because metabolism of carbohydrates generates more CO2 than do
lipids or proteins.
4. Dead space (Vd) refers to ventilation that does not participate in gas exchange. The dead space
ratio is calculated from the Bohr equation which measures the ratio of dead space to tidal
volume: Vd / Vt = (PaCO2 – PeCO2) / PaCO2 where PeCO2 is the CO2 concentration in mixed expired
gas, NOT end-tidal CO2, though end-tidal CO2 is sometimes used as an estimate The normal dead space to
tidal volume ratio is 0.3 to 0.4. High dead space ratio can be predictive of failure to
successfully discontinue mechanical ventilation.
5. Transcutaneous CO2 monitoring has been used in the neonatal ICU but has had limited
acceptance in the adult ICU.
VI.
Cardiopulmonary interactions
A. Venous return: Administration of positive pressure ventilation causes increased intrathoracic
pressure which can cause decreased venous return leading to reduction in cardiac output.
Administration of intravascular fluid may counteract the negative hemodynamic effects of positive
pressure ventilation.
B. Afterload reduction: Lung expansion increases extramural pressure which helps to pump blood
out of the thorax and thereby reduce LV afterload. In conditions where cardiac function is mainly
determined by changes in afterload rather than preload (e.g. hypervolemic patient with systolic
heart failure), positive pressure ventilation may be associated with improved cardiac output.
C. Shunt effects: In patients with right-to-left intracardiac shunt, positive pressure ventilation and
PEEP may increase right-to-left shunt and worsen systemic hypoxemia. The mechanism of
increased shunt is likely Valsalva-like activity (e.g. breathing against the ventilator) or an increase
in pulmonary vascular resistance secondary to PEEP.
VII. Ventilator discontinuation
Several criteria should be evaluated before a patient is assessed for ventilator discontinuation:
A. Resolution of underlying disease: Before mechanical ventilation can be safely withdrawn, there
must be some evidence that the disease leading to respiratory failure is improving.
B. Mental status: Patients should be awake, alert, able to manage secretions and protect their airway.
Sedating medications should be adjusted/discontinued so that the patient is able to cooperate with
reduction in ventilator support.
C. The patient should be hemodynamically stable without evidence of acute myocardial ischemia and
without clinically significant hypotension requiring large or escalating doses of vasopressors.
D. Adequate gas exchange: Some criteria to consider include PaO2 > 60 with an FiO2 < 0.5 and
PEEP < 5, and a PaCO2 such that pH is > 7.25 with a minute ventilation that the patient can
sustain.
115
SOCCA Residents Guide 2013
E. Respiratory muscle strength: Successful discontinuation of ventilation requires that the load
placed on the respiratory muscles does not exceed their capacity to manage the load. Common
causes of high load include high airways resistance, low lung compliance, and high minute
ventilation (appropriate or inappropriate). Reduced muscle strength may result from disease,
disuse, malnutrition, hypoxia, electrolyte imbalance or polymyoneuropathy of critical illness.
F. Ventilator mode: Methods of “weaning” the patient from ventilator vary by mode
1. SBT: A spontaneous breathing trial is more predictive of a patient’s readiness to breathe
without ventilatory support than is any weaning parameter (see below) reported to date. An
SBT can be performed off the ventilator with a T-piece or tracheostomy collar, or on the
ventilator with minimal pressure support and PEEP. Most patients who tolerate an SBT of
30-120 minutes can be successfully discontinued from mechanical ventilation.
2. PSV: With pressure support ventilation the level of inspiratory pressure is gradually
decreased until the patient is able to breathe without assistance (usually when the patient
receives < 10 cmH2O support).
3. SIMV: With SIMV the mandatory rate can be reduced gradually until the patient is able to
breathe without assistance (usually when the patient receives less than 4 mandatory breaths
per minute). Several studies suggest that weaning with SIMV is inferior to other approaches
and may prolong the duration of mechanical ventilation.
G. Weaning parameters are objective measures sometimes used as predictors of successful
discontinuation of ventilation. Most of these reflect only a single component of the respiratory
system and thus are limited predictors.
1. RSBI: The rapid shallow breathing index is calculated by dividing the respiratory rate by the
Vt (in liters). A low RSBI (<105) has been used to predict successful discontinuation of
mechanical ventilation.
2. MIP (or NIF): The maximal inspiratory pressure or negative inspiratory force is a measure of
respiratory muscle strength. A manometer is applied to an occluded airway to measure the
force that the patient can generate during inspiration. A MIP (or NIF) of more negative than
-30 cm H2O has been used to predict successful discontinuation of mechanical ventilation.
3. P0.1 is the airway pressure generated 0.1 seconds after initiating an inspiratory effort against
an occluded airway and is believed to reflect central respiratory drive. A high respiratory drive
(P0.1 of -4 to -10 cm H2O) has been used to predict successful discontinuation of mechanical
ventilation.
VIII.
IX. Miscellaneous
A. Hyperbaric oxygen (HBO) therapy involves breathing 100% oxygen at > 1 atmosphere of
pressure. HBO therapy is administered in specialized chambers under close patient monitoring.
Mechanisms of action of HBO therapy stem from 2 types of effects: hyperoxygenation of
perfused tissues and reduction of gas bubble volume. Indications include:
1. Air embolism: the effect of HBO is predicted by Boyle’s Law, which states that the volume of
air (mostly nitrogen) bubbles is inversely proportional to the pressure exerted on them.
Nitrogen bubble size is further reduced by replacement of nitrogen with oxygen, which is
rapidly used in cellular metabolism. Air embolism can result from procedures (e.g. central line
placement) or operations (e.g. sitting craniotomy) in which air can be entrained through a
disrupted vascular wall.
2. Decompression sickness (“the bends”): Divers breathing compressed air who return to the
surface too rapidly are at risk for decompression sickness, which occurs when bubble
formation in blood and tissues occurs as the partial pressure of inert gas (nitrogen) exceeds
that of ambient air. HBO is the primary treatment for decompression sickness through its
effects on bubble size and relief of hypoxia.
3. Carbon monoxide poisoning: HBO significantly reduces the half-life of carboxyhemoglobin,
which may prevent the late neurocognitive defects associated with severe CO poisoning.
4. Soft tissue infections: HBO has been used as an adjunct therapy for severe life or limb
threatening infections such as clostridial myonecrosis, necrotizing fasciitis and Fournier’s
gangrene.
B. Independent lung ventilation (ILV): Patients with severe unilateral lung disease may require
different ventilation strategies applied to each lung. Indications for independent lung ventilation
include: unilateral pulmonary contusion, bronchopleural fistula, massive hemoptysis from a single
lung or following single lung transplantation (though ILV is not routinely used for any of these
116
SOCCA Residents Guide 2013
conditions). A double lumen tube and two ventilators facilitate ILV.
C. Heliox is a gas mixture of helium and oxygen that is used in conditions of high airflow resistance.
Helium is less dense than air and flows more readily through regions of reduced cross-sectional
area where flow is turbulent. Heliox may be useful in acute exacerbations of asthma / COPD or in
tracheal obstruction. Heliox is generally well tolerated but its use is frequently limited by the high
concentration of helium required, which limits FiO2 delivery. Most studies use a helium:oxygen
mixture of 80:20 or 70:30 to achieve benefit. Many ICU patients can not tolerate an FiO2 of 0.2 or
0.3 due to hypoxemia.
D. High frequency ventilation achieves gas exchange by combining very high respiratory rates with
very low tidal volumes (smaller than anatomic dead space). Potential advantages include a lower
risk of barotrauma due to small tidal volumes and improved gas exchange due to more uniform
distribution of ventilation and greater alveolar recruitment.
1. High frequency jet ventilation (HFJV) delivers pulses of gas at high velocity and frequency
into the trachea through a small catheter. The high velocity jet pulse creates an area of reduced
pressure which entrains additional gas and produces a mixing effect. Exhalation is a passive
process. HFJV has been used in ARDS patients with the goal of reducing airway pressures
and ventilator induced lung injury.
2. High frequency oscillatory ventilation (HFOV) delivers low tidal volumes at very high
frequency using a pump so that airway pressure oscillates slightly about a mean airway
pressure. This allows maintenance of alveolar recruitment while avoiding high peak airway
pressure. HFOV has been used primarily in children and neonates where its use is associated
with improved oxygenation and reduced barotrauma.
X.
Future directions
A. Improvement in ventilator technology has allowed for the recent development of a variety of
modes (e.g. Neurally Adjusted Ventilatory Assist, or NAVA) with increased emphasis on patientventilator interactions. The goal is to mimic the complex interplay of the central nervous system,
peripheral nervous system and respiratory system exhibited during normal breathing.
B. Alternatives to traditional positive pressure mechanical ventilation, including pumpless extracorporeal gas-exchange devices driven by the patient’s blood pressure, continue to attract more
attention as intensivists seek to minimize ventilator induced lung injury.
Discussion
Proper management of mechanical ventilation requires a thorough understanding of respiratory and
cardiovascular physiology and pathophysiology of critical illness. While a wide variety of ventilator modes
exist, the recovery of patients in respiratory failure depends mostly on clinicians’ vigilance and ability to
modify therapy appropriately.
REFERENCES:
Bigatello LM ed. Critical Care Handbook of the Massachusetts General Hospital, 5th Ed. Philadelphia:
Lippincott Williams & Wilkins, 2010.
Fink MP, Abraham E, Vincent JL and Kochanek P eds. Textbook of Critical Care, 5th Ed. Philadelphia:
Elsevier Saunders, 2005.
Hess D, MacIntyre NR, Mishoe SC et al eds. Respiratory Care: Principles and Practice. Philadelphia: WB
Saunders, 2002.
Marino PL. The ICU Book, 3rd Ed. Philadelphia: Lippincott Williams & Wilkins, 2007.
McLean B and Zimmerman JL eds. Fundamental Critical Care Support, 4th Ed. Society of Critical Care
Medicine. Mount Prospect, IL, 2007.
Irwin RS and Rippe JM eds. Intensive Care Medicine, 6th Ed. Philadelphia: Lippincott Williams & Wilkins
2008.
117
SOCCA Residents Guide 2013
QUESTIONS:
22.1 Static compliance will be independently affected by:
A. Pulmonary edema
B. Bronchospasm
C. Kinked endotracheal tube
D. Mucus plugging
22.2 Goals of adding PEEP to routine ventilation settings might include all of the following EXCEPT:
A. Improved lung compliance ! CORRECT
B. Improved PaO2 ! CORRECT
C. Decreased work of breathing ! CORRECT
D. Reduced dead space
22.3 Which weaning mode/method is inferior to other approaches and my prolong duration of mechanical ventilation?
A. Synchronized intermittent mandatory ventilation
B. Pressure support ventilation
C. Spontaneous breathing trial
D. Proportional assist ventilation
118
SOCCA Residents Guide 2013
23. Lung Protective Strategies
Georges Cehovic, MD
A 25 year old male sustains a motor vehicle collision. After adequate fluid
and blood resuscitation he is admitted to the ICU for hypoxia. He is placed on
mechanical ventilation with the following settings: RR=12 x TV=700mL,
PEEP=15cmH2O, FiO2=80%. An arterial blood gas reveals pH=7.32, pCO2=35
mmHg, pO2=120 mmHg.
This case exemplifies the intricacies of ARDS management.
I.
PATHOPHYSIOLOGY
ARDS is an inflammatory process with an overlapping, temporally-sequential exudative and fibroproliferative phase. The exudative phase is characterized by the destruction of type 1 pneumocytes
and the leakage of fluid in the alveolar space. The fibro-proliferative phase is characterized by
proliferation of type 2 pneumocytes and fibroblasts. The latter contribute to the deposition of hyaline
membranes in the interstitial space leading to a decrease in lung compliance. The combination of an
altered alveolar capillary membrane (resulting in hypoxia) and worsening lung compliance frequently
leads to airway intubation and implementation of mechanical ventilatory support. The goals of this
support should include the following three measures: reverse the hypoxic state; alleviate the patient’s
work of breathing while minimizing lung injury exacerbation.
II.
DEFINITION
The first step in managing ARDS is pattern recognition of the disease process. The four diagnostic
criteria for ARDS are:
A. An acute onset process in the context of a reasonable etiology.
1. Typically, the urgent need for intubation and ventilation is the clinical expression of the
acuteness of the process.
2. For ease of classification, etiologies of ARDS can be classified as local and general. Examples
of local causes include: lung contusion, pneumonia and aspiration. Examples of general
causes include: general trauma, sepsis, TRALI (transfusion related acute lung injury), and
cardio-pulmonary bypass. ARDS due to trauma usually presents a shorter duration and better
outcome. The etiologic characterization of ARDS will possibly addressed in the future.
B. Hypoxia per PaO2/FiO2 criteria:
1. A PaO2/FiO2 less than 200mmHg defines ARDS. A PaO2/FiO2 less than 300mmHg defines
Acute Lung Injury (ALI), a milder form of ARDS.
C. Radiologic evidence of bilateral chest infiltrates (that can occasionally be delayed).
D. Exclusion of cardiogenic pulmonary edema. Diagnostic criteria for ARDS include: a central
venous pressure less than 4 cmH2O or a pulmonary artery occlusion pressure less than 18 cmH2O.
Definite exclusion may require complementary exams, such echocardiography or BNP levels.
III.
LUNG PROTECTIVE VENTILATORY STRATEGY
In 2000, the landmark ARDS Network trial article (1) presented the goals of a comprehensive lung
protective strategy that included: lowering tidal volumes, limiting plateau pressures, applying
recruitment maneuvers and permissive hypercapnia, while minimizing FiO2. Application of these
measures lead to a significant survival benefit (mortality of 31% vs. 39%).
A. Application of lower tidal volume to 6 mL per kg of ideal body weight (based on height solely)
prevents over-distension and barotrauma.
B. Application of limited airway pressures with a plateau pressure less than 30 cmH2O also prevents
over-distension injury. Over-distension is deleterious as it can lead to further injury and possibly
barotrauma.
C. Application of recruitment maneuvers to prevent de-recruitment injury. Recruitment maneuvers
entail keeping the lung inflated for a pre-determined duration at a pre-determined pressure,
allowing alveoli to re-expand (thus get recruited), and participate again in the gas exchange.
Maintenance of the PEEP over a certain level (optimal, best, or lower inflection point) also aims at
maintaining alveoli open. De-recruitment injury results from the shear injury incurred by the
cyclic closing and re-opening of the alveoli. De-recruitment injury will further propagate the
119
SOCCA Residents Guide 2013
injury to unaffected lung units.
D. Application of permissive hypercapnia. Its basic rationale is that by decreasing minute ventilation,
one reduces the shear injury to the alveoli. This occurs over a prolonged period of time and is done
at the cost of tolerating some degree of respiratory acidosis (up to pH <7.0). Intracranial
hypertension is an obvious contraindication to permissive hypercapnia.
E. Application of a lowest tolerable oxygen concentration. As oxygen can be toxic at higher
concentrations, its concentration should be titrated to achieve the following goals: a PaO2 between
55 and 80mmHg, or a SpO2 between 88% and 95%.
IV.
NON-VENTILATORY STRATEGIES
The following therapies improve oxygenation indices but have not demonstrated to improve survival.
Except for fluid restriction the routine usage of the following measures: Nitric oxide, prone
positioning and high frequency ventilation is not recommended. These last measures should only be
used as temporary patches.
A. FLUID MANAGEMENT
Fluid restriction is supported by a plethora of evidence. A trial part of the ARDS network (2)
looked specifically at this question. When compared to a liberal fluid strategy, the restrictive fluid
strategy had a net fluid deficit of 7 liters over a seven day period achieved with fluid restriction
and higher doses of diuretics and pressors. In the fluid restriction group, positive findings included
a shorter stay in the ICU and in the hospital and a trend although not statistically significant
towards a higher survival rate. Of note, these findings do not apply to patients with renal failure.
B. NITRIC OXIDE
Nitric oxide is administered through the inhalational route. In the alveoli, Nitric Oxide acts as a
selective pulmonary vasodilator improving cardiac output. By impacting selectively the ventilated
pulmonary units, Nitric oxide improves ventilation / perfusion mismatch and oxygenation. In
refractory hypoxemia Nitric Oxide should mainly be employed as a temporizing measure. Several
studies have looked at the efficacy off nitric oxide in ARDS with different titration protocols that
usually consist of an initial dose, an escalation algorithm and duration. While Nitric Oxide
improved oxygenation indexes, it failed to demonstrate a survival benefit (3). This and other issues
including cost, availability and secondary renal failure have limited Nitric oxide’s widespread use
C. PRONE POSITIONING
Oxygenation indexes improve in about 70% of patients with ARDS when placed in the prone
position. There are no factors to predict which patients will be responders. Mechanisms of this
oxygenation improvement are unclear. They are probably related to the recruitment of atelectatic
dorsal areas and to the redistribution of blood flow from the dorsal to the ventral areas of the lung.
The latter results in an improved ventilation / perfusion match and oxygenation. Prone positioning
requires adequate preparation as well as resources in material and educated personnel in order to
minimize complications such as unexpected extubations and loss of lines. No improvement in
survival has been demonstrated in several meta-analyses (4), although the sub-population of
patients with extreme hypoxia PaO2/Fio2 <100 mmHg, shows some promising trends.
D. HIGH FREQUENCY VENTILATION
This benefit of this mode of ventilation has been confirmed in the pediatric population. In the adult
population it is indicated as a rescue therapy. In contrast to “traditional” ventilator settings, this
modality uses a high respiratory rate (60 – 900 breaths/min), an increased airway pressure
(maintained at a pre-determined pressure) and delivers a reduced tidal volume (less than the dead
space volume). High-frequency ventilation improves oxygenation but carbon dioxide clearance is
limited. Factors limiting the wide-spread usage of this modality include: lack of proven survival
benefit, lack of clinician’s familiarity, cost and availability.
V.
MORTALITY
With an incidence around 50 per 100.000 inhabitants per year, this is comparable to the yearly rate of
patients dying from AIDS or breast cancer. ARDS carries a mortality around 40-60%. Etiologic
factors and pre-existing conditions play a major role in mortality. Multiple organ dysfunction
syndrome is frequently associated with ARDS at the time of death. Survivors frequently complain of
long term disability not related to their pulmonary insufficiency.
VI.
FUTURE DIRECTIONS
Extracorporeal membrane oxygenation has proven some benefit in sub-populations of patients with
120
SOCCA Residents Guide 2013
ARDS
VII. CONCLUSION
In our patient, possible etiologies for ARDS are trauma, lung contusion and TRALI. His oxygenation
index shows a PaO2 / FiO2 = 120/0.80 = 150 < 200mmHg, thus fulfilling that ARDS diagnostic
criteria. His oxygenation improved after changing his ventilator settings to a pressure-controlled
mode, limiting airway pressure to 35 cmH2O. This brought the tidal volumes closer to his “ideal tidal
volume” and resulted in permissive hypercapnia when his respiratory rate was decreased. PEEP was
decreased to 10cmH2O. FiO2 was decreased to 70%. Over the course of several days, the patient’s
oxygenation status worsened and he required Nitric Oxide before recovering over a period of six
weeks.
TAKE HOME MESSAGE
ARDS is a prevalent form of pulmonary inflammatory disease with a high mortality, manifested mainly by
hypoxia and non-compliant lungs. Only lung protective strategy, with pressure and tidal volume reduction,
has proven survival benefit. Patients with refractory hypoxia are candidates for temporary alternative
therapies that carry no proven survival benefit.
REFERENCES:
1.
2.
3.
4.
The Acute Respiratory Distress Syndrome Network. Ventilation with lower tidal volumes as compared
with traditional tidal volumes for acute lung injury and the acute respiratory distress syndrome. N
Engl J Med 2000; 342:1301-8.
Comparison of two fluid-management strategies in acute lung injury. N Engl J Med. 2006 Jun 15;
354(24):2564-75. National Heart, Lung, and Blood Institute Acute Respiratory Distress Syndrome
(ARDS) Clinical Trials Network, Wiedemann HP, Wheeler AP, Bernard GR, Thompson BT, Hayden D,
deBoisblanc B, Connors AF Jr, Hite RD, Harabin AL.
Effect of nitric oxide on oxygenation and mortality in acute lung injury: systematic review and metaanalysis British Medical Journal 2007 Apr 14;334(7597):779 Adhikari NK, Burns KE, Friedrich JO,
Granton JT, Cook DJ, Meade MO.
Effect of prone positioning in patients with acute respiratory distress syndrome: a meta-analysis
Critical Care Medicine 2008 February Volume 36(2),В В pp 603-609 Alsaghir, Abdullah H. MD; Martin,
Claudio M. MSc, MD, FRCPC
QUESTIONS
23.1 Of
A.
B.
C.
D.
E.
the following treatments, which one has demonstrated a systematic decrease in mortality in ARDS?
Prone positioning
High frequency jet ventilation
Extracorporeal membrane oxygenation
Nitric Oxide
none of the above
23.2 Lung protective strategy should include the following measure(s):
A. decrease the plateau pressure to 40 cm H2O
B. decrease the tidal volume to 6 ml per dry weight
C. Oxygen titration first then PEEP reduction
D. decrease the tidal volume to 6 ml per kg of ideal body weight based on height only
E. No fluid restriction
23.3 Which one element is part of the diagnostic criteria for ARDS?
A. an arterial blood gas showing an oxygenation index of Pa02 / FiO2 < 300 mmHg
B. an acute onset of hypoxia with a reasonable etiology for ARDS
C. a pulmonary artery occlusion pressure greater than 18 cm H2O
D. a proliferation of type 1 Pneumocytes
E. E a decrease in total lung compliance
121
SOCCA Residents Guide 2013
24. Weaning From Mechanical Ventilation
Theofilos P. Matheos, MD & Stephen O. Heard, MD
A 62-year-old female with severe chronic obstructive pulmonary disease
(COPD), history of deep venous thrombosis (DVT) on coumadin, obesity,
sleep apnea, and anxiety undergoes surgery for a hip fracture following a fall.
She also suffered bilateral rib fractures. The post-operative course is
complicated by a pneumothorax from an attempted subclavian catheter
placement necessitating a chest tube, transfusion related acute lung injury
(TRALI), difficulties with pain control, and delirium. The patient is unable to
maintain long periods of pressure support ventilation (PSV) secondary to
increased narcotic requirements for pain control, requires pharmacological
treatment for her agitation and delirium, and has failed a 24 hour trial with
dexmedetomidine. Prepare a plan to wean the patient from mechanical
ventilation.
Weaning is the abrupt or gradual discontinuation of mechanical ventilatory support. Although, for the
majority of patients, weaning is uncomplicated and spontaneous ventilation easily achieved, there are
patients with severe acute respiratory distress syndrome, chronic obstructive pulmonary disease, and
neuromuscular disorders in which the weaning process can account for up to 40% of the time that the
patient requires mechanical ventilation. The ability to wean is not merely a function of pulmonary status
but is a co-dependant process affected by patient co-morbidities, fluid balance, neurological status and
associated therapy (such as sedatives). There is little consensus regarding the optimal method for weaning
patients from mechanical ventilator support. The traditional and primary modes of weaning are T-piece and
continuous positive airway pressure (CPAP), synchronous intermittent mandatory ventilation (SIMV),
pressure support ventilation (PSV), and combinations of the above.
Although extubation (the removal of the endotracheal tube, tracheostomy, or artificial airway) usually
follows weaning, it is a different process, and criteria related to the structure and function of the upper
airway play an important role in the decision to extubate. When implemented in a structured and organized
approach, along with collaboration with nursing staff and respiratory therapists, and taking into account
individual patient circumstances, there may be effective decreases in ventilator days, associated ventilator
complications, and hospital costs.
I.
Weaning Classification
A. Immediate – approximately 75-80% of intubated intensive care patients
B. Acute – within 72 hours, approximately 10-15%
C. Chronic – greater than 72 hours, approximately 5-10%
D. Terminal – following decisions to withhold/withdraw non-beneficial care
II.
Weaning Readiness
A. Resolution of the pathological process – improvement in the underlying process causing
respiratory failure (pneumonia, ARDS, cardiogenic pulmonary edema)
B. Reversal of the pharmacological process – reversal of neuromuscular blockers, decreased
concentrations of opioids and sedatives
C. Treatment of patient’s co-morbidities:
1. Stable hemodynamics on minimal inotrope/vasopressor support
2. Fluid balance problems resolved
3. Acid/base balance restored to the patient’s pre-morbid status
4. Electrolytes corrected, especially potassium, calcium, and phosphate important for muscle
function
5. Patient’s mental status adequate to allow cooperation with weaning – daily cessation of
sedation to allow the patient to awaken and participate in the weaning trial
122
SOCCA Residents Guide 2013
6.
Adequate peripheral nerve function and muscle strength
III.
Weaning Criteria
A. Single index predictors – gas exchange and perfusion, ventilatory performance, and lung
mechanics
B. Multiple indices predictors (see below) – rapid shallow breathing, CROP weaning index (see
below).
1. Positive predictive value – patient will wean if the indices predict success
2. Negative predictive value – patient will not wean if the indices indicate failure
3. If a predictor has a low negative predictive value compared to its positive predictive value,
weaning may inadvertently be delayed while waiting for the predictor to improve
IV.
Single Index Predictors
A. Gas exchange and perfusion:
1. PaO2 >60 mmHg and FiO2<0.4
2. P(A-a)O2 <350mmHg on FiO2 = 1.0
3. PaO2/PAO2>238
4. PaO2/PAO2>0.47
5. Shunt fraction <20%
6. pH>7.30
7. Vd/Vt<0.6
8. PaCO2 increase post-disconnect < 8 mmHg
9. Arterial blood gases have not been shown to correlate with ability to wean despite their
suggestion of gas exchange (Reference 4)
B. Ventilatory Performance:
1. Vital capacity >10mL/kg
2. Tidal volume >5mL/kg
3. Respiratory rte <25/min
4. Minute Ventilation <10 L/min
5. Maximum minute ventilation > 2 times resting minute ventilation
C. Lung Mechanics and Work of Breathing:
1. Maximum inspiratory force <-30 cm H2O
2. Occlusion pressure P0.1<6 cm H2O
3. Dynamic compliance > 25 mL/cm H2O
4. Oxygen cost of breathing <15%
5. Work of breathing-poor predictor of weaning
6. Indirect markers include but are not limited to respiratory rate, use of accessory muscles,
tachycardia, etc.
V.
Multiple Indices Predictors
A. Rapid shallow breathing (RSB)
1. RSB = respiratory frequency/tidal volume (liters) = f/Vt
2. Tachypnea and decreases in tidal volume indicated failure
3. RSB < 105 breaths/liter predicts success
4. Positive predictive value = 0.78
5. Negative predictive value = 0.95
6. Independent of patient effort
B. CROP Index:
1. Acronym for an index combining compliance, respiratory rate, oxygenation, and airway
pressure
2. >13 breaths per minute predicts weaning outcome
3. Positive predictive value = 0.71
4. Negative predictive value = 0.72
C. Weaning Index:
1. Pressure time and gas exchange index
2. <4 breaths per minute
3. Positive predictive value = 0.95
4. Negative predictive value = 0.96
123
SOCCA Residents Guide 2013
VI.
Traditional Weaning Techniques
A. Abrupt discontinuation of ventilatory support
1. Useful in immediate and acute weaning
2. Inexpensive
3. Simple
4. Patient comfort must be kept in mind when choosing a particular support mode
B. T-piece trials and continuous positive airway pressure (CPAP)
1. CPAP:
a) A mode of ventilation
b) Constant level of positive pressure during spontaneous ventilation
c) Decreases extrinsic and intrinsic respiratory work load of the patient
d) Systems – demand flow, continuous flow, and mixed systems
2. Useful in immediate, acute, and chronic weaning
3. Observe patient for two hours
4. If patient tolerates – extubate
5. If patient does not tolerate then:
a) Daily trials, gradually increase duration of T-piece trials
b) Rest on control, assist control, or IMV
c) If tolerates 24 hour period – extubate
6. Combine with continuous positive airway pressure
C. Synchronous Intermittent Mandatory Ventilation (SIMV):
D. Ventilator breath – delivered at set rate and volume or pressure
1. Patient’s inspiratory effort – mandatory breath delivered in synchrony
2. If no inspiratory effort – mandatory machine breath, determined by preset interval
3. Triggered by patient pressure or flow
4. SIMV and Weaning
a) Initial total support is provided with SIMV
b) Decrease machine breaths (usually by 2s) and allow increase in spontaneous minute
ventilation
c) If SIMV = 2 with no respiratory distress and adequate gas exchange – extubate
d) Underlying principle is that the patient progressively exercises and reconditions their
respiratory muscles during spontaneous effort, while resting during mandatory machine
delivered breaths.
E. Pressure-support ventilation (PSV):
1. Pressure targeted ventilation is a spontaneous ventilatory mode
2. Each breath is patient initiated, and minute ventilation determined by patient’s respiratory
drive
3. Decreases work of breathing, improving spontaneous respiration as evidenced by increased
tidal volumes and reduced respiratory rates
4. No minimum minute ventilation; therefore, risk of hypoventilation
5. Modern ICU ventilators have backup minute ventilation if apnea occurs, but ventilators differ;
therefore, need to know each ventilator
6. PSV and Weaning:
(1) Set pressure level of support, aim to achieve Vt = 8 mL/kg, RR<25/min
(2) Decrease by 2-4 cm H2O, according to patient’s response
(3) Attempt decrease at least every 12 hours
(4) When PSV of 8-10 cm H2O reached, consider extubation
7. Combined modes
8. Utilizing protocol-driven techniques
VII. Weaning Trial Studies
A. PSV Weaning Better than SIMV and T-piece (Reference 5):
1. Entry criteria – patient assessed ready to wean, but has failed 2 hr trial
2. 25% of all mechanically ventilated patients entered study
3. After 21 days – significantly larger number of patients weaned by PSV
4. IMV was without PSV and T-piece was without CPAP
B. T-piece better than PSV and SIMV (Reference 6):
124
SOCCA Residents Guide 2013
1.
2.
3.
Similar entry criteria to trial quoted in reference 5
Once daily and multiple intermittent T-piece trials vs. PSV vs. SIMV
At 14 days:
a) Successful extubation rate higher with T-piece
b) Median duration of weaning shorter with T-piece
4. Higher re-intubation rate in T-piece group
C. Two hour T-piece screening better than usual respiratory care (Reference 7):
1. Prospective, randomized patients once daily respiratory assessment and T-piece trial vs. usual
care by managing physicians
2. Those with T-piece trials – shorter weaning time and duration of mechanical ventilation
3. Fewer total complications, decreased rates of reintubation, and decreased costs
D. Paired sedation and ventilator weaning protocols result in better outcomes for mechanically
ventilated patients (Reference 3):
1. Prospectively randomized patients to management with spontaneous awakening trial followed
by an spontaneous breathing trial (SBT) vs. sedation per usual care plus a daily SBT
2. Intervention group spent more time breathing without assistance, were discharged from the
ICU and hospital earlier and had a lower mortality
VIII. Failure to Wean Checklist
A. Weaning to exhaustion – rest
B. Excessive work of breathing, auto PEEP, technical problems
C. Nutritional status – malnutrition vs. overfeeding
D. Electrolyte Balance – hypomagnesemia and hypophosphatemia
E. Acid/Base Status – metabolic alkalosis, acidosis
F. Cardiovascular status – myocardial ischemia, left ventricular failure
G. Neuromuscular disorders – acquired neuropathy, myopathy
H. Infection
I. Organ dysfunction/failure
J. Cuff Leak Test – to evaluate post extubation airway patency
1. Has not been shown to be reliable
2. Addition of steroids to prevent stridor is also controversial
IX.
Future Directions/Alternative Newer Modes of Weaning
A. Proportional Assist
1. Level of assistance is proportional to patient effort
a) Positive feedback loop
b) May be more physiologic (Reference 8)
B. Tube Compensation
1. Designed to overcome the flow resistance associated with the endotracheal tube size and the
accompanying work load
2. Ventilator compensates for the pressure drop across the tube length during inspiration
3. Enhances patient comfort, ventilator synchrony, and reduces respiratory muscle fatigue
(Reference 9)
C. Airway Pressure Release Ventilation
1. Two levels of positive pressure are set for preset times
2. Higher pressure set for longer duration during inhalation, while lower pressure exists during
exhalation
a) form of inverse ratio ventilation
3. Short duration of expiratory phase does not allow lung to completely deflate
a) auto PEEP maintains alveolar recruitment
4. Patient may breath spontaneously during inspiratory phase, but no additional support is
provided
D. Bi-Level Ventilation
1. Similar to a pressure regulated SIMV mode, but additional spontaneous breaths may be
supported with addition of pressure support
2. Weaning is accomplished by reducing mandatory rate and increasing pressure level and then
switching to pressure support
E. WEAN Study – Wean Earlier and Automatically with New Technology Reference 10)
125
SOCCA Residents Guide 2013
1.
Pilot randomized controlled trial involving computerized or automated weaning that
continuously measures and adapts ventilator support and automatically conducts SBTs
a) Primary outcome to evaluate clinician compliance with proposed weaning and sedation
protocols
b) Secondary outcome is to evaluate clinician acceptance of protocols
DISCUSSION:
Patients who fail attempts at weaning constitute a unique problem in critical care. It is necessary to
understand the mechanisms of ventilatory failure in order to address weaning in this population. Although
numerous trials have been performed to develop criteria for successful weaning, a “gold standard” has not
emerged. Physicians must rely on clinical judgment as to when to begin the weaning process. What is
known is that daily screening and implementation of spontaneous breathing trials paired with daily sedation
interruptions may reduce the duration of mechanical ventilation and its associated intensive care unit costs.
Collaborative involvement with all care caregivers with the presence of protocols is also integral to success.
Tracheostomy should be considered when it is evident that the patient requires either long term ventilator
support or airway protection, despite the capacity to wean.
REFERENCES:
1.
Tobin MJ. Principles and practice of mechanical ventilation. 2nd edition. New York: McGraw-Hill; 2006.
Comprehensive textbook on the fundamentals of mechanical ventilation.
2. Smyrnios NA, Connolly A, Wilson MM, et al. Effects of a multifaceted, multidisciplinary, hospital-wide
quality improvement program on weaning from mechanical ventilation. Crit Care Med
2002;30:1224-1230. The program described in this article resulted in large reductions in the duration of
mechanical ventilation, length of stay and hospital costs despite an increase in severity of illness.
3. Girard TD, Kress JP, Fuchs BD, et al. Efficacy and safety of a paired sedation and ventilator weaning
protocol for mechanical ventilated patients in intensive care (Awakening and Breathing Controlled trial):
a randomized controlled trial. Lancet 2008;371:126-134. The authors found that a daily spontaneous
awakening trial coupled with a spontaneous breathing trial increased the number of days breathing
without assistance, reduced intensive care unit and hospital length of stay and was associated with a
lower risk of death over one year.
4. Pawson SR, DePriest JL. Are blood gases necessary in mechanically ventilated patients who have
successfully completed a spontaneous breathing trial? Respir Care 2004;49:1316-1319.
5. Brochard L, Ruass A, Benito S, et al. Comparison of three methods of gradual withdrawal from
ventilatory support during weaning from mechanical ventilation. Am J Respir Crit Care Med 1994;150(4):
896-903.
6. Esteban A, Frutos F, Tobin MJ, et al. A comparison of four methods of weaning patients from mechanical
ventilation. N Engl J Med 1995;332(6):345-350.
7. Ely E, Baker A, Dunagan D, et al. Effect on the duration of mechanical ventilation of identifying patients
capable of breathing spontaneously. N Engl J Med 1996;335:1864-1869.
8. Grasso S, Ranieri VM. Proportional assist ventilation. Respir Care Clin N Am 2001 Sep; 7(3):465-473
9. Burns KE, Meade MO, Lessard MR, et al. Wean Earlier and Automatically with New technology (the
WEAN study): a protocol of a multicentre, pilot randomized controlled trial. Trials 2009 Sep 4;10:81
10. Fabry B, Haberthur C, Zappe D et al. Breathing pattern and additional work of breathing in
spontaneously breathing patients with different ventilatory demands during inspiratory pressure support
and automatic tube compensation. Int Care Med 1997;23:545-552
QUESTIONS:
K-type: For each of the items in this section, ONE or MORE of the numbered options is correct. Choose
answer:
(A) If only 1, 2 and 3 are correct
(B) If only 1 and 3 are correct
(C) If only 2 and 4 are correct
(D) if only 4 is correct
(E) If all are correct
126
SOCCA Residents Guide 2013
24.1 Concerning the epidemiology of weaning and its problems, which of the following statements are true?
1. Time spent in the weaning process represents 40–50% of the total duration of mechanical ventilation
2. Weaning tends to be delayed, exposing the patient to unnecessary discomfort and increased risk of
complications, and increasing the cost of care.
3. Almost half of patients with self-extubation during the weaning period do not require reintubation
4. The weaning process begins with the first SBT, defined as a T-tube trial or a low-level pressure support
(≤8 cmH2O)
24.2 A 60 year-old female was intubated 10 days ago for respiratory failure secondary to ARDS due to urosepsis. She is
now on an FIO2 of 0.4 with a PEEP of 5 cm H2O. On her last SBT this morning, tidal volumes were between 160 and 200
ml and her respiratory rates rose to 35-40/min after only 10 minutes of spontaneous breathing. Other vital signs included
a BP of 145/60, HR 90, and Temp of 38.5C. Potential problems include which of the following?
1. Electrolyte abnormalities
2. Increased sputum production
3. Excessive sedation
4. Lingering effects of neuromuscular blocking agents
24.3 A 65 year-old gentleman was intubated five days ago for respiratory failure secondary to sepsis from a presumed
pneumonia. He is on appropriate antibiotics, is now off pressors, and his WBC count has declined to the normal range.
During rounds, you note that his FIO2 is down to 0.4 and he is on a PEEP of 5 cm H2O. On these settings, the ABG shows
pH 7.36, pCO2 46, PO2 75, HCO3- 26. He has a weak cough and continues with copious secretions, requiring suctioning
every 30 to 60 minutes.
With regards to respiratory function, how do you determine if the patient is capable of being separated from the
ventilator?
1. Respiratory rate < 25 breaths/minute
2. Tidal volume > 8ml/kg
3. Vital capacity > 10 ml/kg
4. Minute volume > 10 l/min
127
SOCCA Residents Guide 2013
25. Shock and Support of the Failing Circulation
R. Eliot Fagley, MD and Michael Wall, MD, FCCM
A 72 year old male arrives in the intensive care unit following aortic valve
replacement and CABG x3. Intra-op TEE showed severe LVH, and a LVEF of
30%. Vital signs on arrival are HR 90 AV paced. BP 120/80 mmHg CVP is 20
mmHg on SVO2= 40%. He is mechanically ventilated SaO2=100%, on FO2
0.4.
I.
Definition – Shock is classically described as any condition resulting in inadequate perfusion of end
organs. Inadequate perfusion may be caused by low circulating blood volume, decreased vascular
tone, poor cardiac pump function, or obstruction of outflow of blood from the heart. Diagnosis and
treatment of shock can be difficult, and incorrect diagnosis with an accompanying foray down the
wrong treatment path can result in disaster. Hence, a detailed understanding of the clinical
presentations, physiologic principles, pathophysiology, and current evidence for diagnosis and
treatment of shock in all its forms is of paramount importance to the intensivist. This chapter will
outline the important considerations for the diagnosis and management of shock
II.
Clinical Presentation
A. History – The history that precedes shock and circulatory failure varies based on etiology. The
patient may or may not have a previous history of trauma or neurologic injury, route of infection,
heart failure or coronary artery disease, or hypercoagulable state. The unifying concept, however,
is the concept of diminished end-organ function and insufficient blood supply relative to demand.
B. Signs, Symptoms, Physical Exam – Variable based on etiology.
1. Neurologic – Mental status changes, delirium, encephalopathy, focal neurologic deficits,
paralysis, hemiparesis.
2. Cardiovascular – Generally hypotension and tachycardia, though medications may mask these
findings. Look for indicators of pulmonary edema when suspicious for left ventricular failure.
Look for indicators of venous congestion (JVD, hepatomegaly, coagulopathies, etc.) when
suspicious of right ventricular failure. The most frequent cause of right ventricular failure is
left heart failure. Listen for S3 and S4 components of heart sounds, muffled heart sounds,
friction rubs, New murmurs raise suspicion for cardiogenic or distributive etiologies. Tearing
chest pain with or without trauma raises suspicion for aortic dissection, a combined
cardiogenic and hypovolemic lesion.
3. Pulmonary – Lung symptomatology and physical exam can be a very sensitive indicator for
various types of shock. Pulmonary edema and effusions are more indicative of cardiogenic,
left ventricular obstructive, or distributive etiologies, whereas a normal lung exam is more
indicative of hypovolemic or right ventricular obstructive etiology. Productive cough with
consolidation on percussion may point one toward a distributive etiology.
4. Renal – Decreased urine output is one of the most frequent symptoms of poor perfusion.
Flank pain, burning on urination, or the presence of a long-term indwelling urinary catheter
should make one consider urosepsis as a possible cause of shock, especially in patients at the
extremes of age or disability.
5. Hematological – Coagulopathies may indicate hepatic venous congestion. Late hemorrhagic
shock (hypovolemic) may present as anemia. Septic shock (distributive) may present with
disseminated intravascular coagulation and anemia.
6. Gastrointestinal – Vomiting or diarrhea may be a cause (resulting in hypovolemic or
cardiogenic shock) or symptom of shock. Ileus is not uncommon in shock. Pain out of
proportion to exam is concerning for gut ischemia and its resulting distributive and
hypovolemic picture.
7. Skin/Extremities – Warm shock and cool shock are important distinctions. Warm, pink, wellperfused extremities often indicate early distributive shock. Cool, clammy, poorly-perfused
extremities often indicate low cardiac output states.
C. Laboratory
1. Arterial Blood Gas – Primary metabolic acidosis generally indicates a systemic flow or a
128
SOCCA Residents Guide 2013
maldistribution problem. Primary respiratory alkalosis with impaired oxygenation is likely a
right ventricular obstructive problem, such as pulmonary embolism.
2. Liver Function – May be cause or effect. Elevated hepatic enzymes (AST and ALT) raise
suspicion for acute hepatic hypoperfusion, while an elevated bilirubin raises suspicion for a
more chronic course.
3. Lactate – Lactic acid is a byproduct of anaerobic metabolism, and is a non-specific indicator
of malperfusion. Generally used as a confirmatory test.
4. B-type Natriuretic Peptide (BNP)/Troponins – Very sensitive, very specific indicators of
cardiogenic shock, however they may also be elevated in distributive shock, or in
hypovolemic shock if there was a prolonged period of hypotension during the resuscitation.
Also useful as trends during the recovery phase of cardiogenic shock.
5. ScvO2/SvO2 – Usefulness is somewhat controversial. SvO2 requires placement of a PA
Catheter, which has long been debated in the literature. ScvO2 appears to be a reasonable
surrogate, though recent evidence suggests poor correlation with SvO2. Conceptually, both
reflect tissue oxygen extraction.
6. Complete Blood Count – Leukocytosis or leukopenia may indicate distributive shock.
Thrombocytopenia may develop as well in severe sepsis. Anemia should be corrected, though
to what extent is controversial. Active MI or ischemia likely warrants a somewhat more
liberal transfusion trigger, while other etiologies likely warrant a more restrictive transfusion
strategy. In traumatic injury resulting in hypovolemic shock, evidence supports a liberal
blood and plasma transfusion strategy until surgical control is achieved.
7. Coagulation Studies – May indicate profound and prolonged hepatic hypoperfusion, DIC
associated with sepsis, or massive resuscitation with resulting clotting factor and platelet
deficiencies.
8. Electrolytes – Significant derangements may be cause or effect of circulatory failure.
D. Imaging – First ensure the patient is clinically stable for imaging. Definitive therapy should not be
postponed for imaging.
1. CXR – Look for signs of pneumonia, pulmonary edema, pleural effusions, pneumothorax,
widened mediastinum, Westermark Sign (PE), subcutaneous air, free air under the diaphragm,
etc.
2. KUB – Look for pneumatosis of the bowel, free air under diaphragm, evidence of obstruction,
etc.
3. CT Scan – Again, definitive therapy or transport to the operating room for emergency surgery
should not be delayed for imaging.
a) Head – Especially useful for ruling out cerebrovascular accident and herniation in the
workup for depressed mental status. Rim-enhancing lesions may lead to consideration of
endocarditis.
b) Chest – Along with abdominal and pelvic CT scanning, very useful in the setting of
hypovolemic, cardiogenic, or distributive shock. Capable of identifying source of inury,
bleeding, or infection. Speed of scanning and continued improvement in slice resolution
makes this an incredibly useful imaging modality.
c) CT Pulmonary Angiography – For suspected pulmonary embolism, right heart
catherization and pulmonary angiography are still considered the Gold Standard, but
there is increasing acceptance of CT Scanning with radiocontrast, numerous studies
having shown at least non-inferiority.
d) Abdomen – Very useful in the diagnosis of circulatory failure, particularly in the setting
of trauma or when suspicious of intra-abdominal catastrophe.
e) Pelvis – Very useful in the diagnosis of circulatory failure, particularly in the setting of
trauma or when suspicious of bleeding or infection.
4. Ultrasound – An increasingly important monitoring tool in the hands of the intensivist. As the
technology gets cheaper and more portable, and the images get sharper, physicians will need
to become facile with this modality.
a) TTE – Especially useful in delineating hypovolemic versus cardiogenic shock. Evidence
of RV strain may signal obstructive shock. Some technical limitations in the surgical
population.
b) TEE – Long in use by Cardiothoracic Anesthesiologists and Cardiologists in the
operating theater. Outstanding imaging of dynamic ventricular performance and valvular
function. Increasing use in all types of circulatory failure.
129
SOCCA Residents Guide 2013
c)
Abdominal/FAST Exam – Pioneered by Trauma surgeons and Emergency Medicine
physicians, the FAST Exam has proven to be a speedy and efficient means of
differentiating between the various causes of shock.
(1) Subxiphoid Four-Chamber View
(2) Parasternal Long-Axis View
(3) Right Coronal View
(4) Right Intercostal Oblique View
(5) Left Coronal View
(6) Left Intercostal Oblique View
(7) Longitudinal and Transverse Pelvic Views
(8) Anterior Thoracic View
E. Monitoring
1. Arterial lines – Allow for frequent lab draws and beat-to-beat pressure monitoring, thus
increasing the granularity of the patient’s clinical picture.
a) Pay special attention to pulse pressure variation/Systolic pressure variation, as they
appear to correlate well with fluid-responsiveness.
2. Central lines – Useful for both monitoring and resuscitation.
a) CVP is more useful as a trend than as a data point.
b) Most goal-directed therapies strive toward a specific CVP.
c) Two short, large-bore IV’s are more efficient for resuscitation than most central venous
catheters.
3. Pulmonary artery catheters – Use of the PAC continues to be controversial in critical care.
There are no clear indications for their use, though they prove extremely useful in the right
hands. (See Chapter 17)
a) Pro – Continuous monitoring of changes in filling pressures (especially PA diastolic and
CVP, however flawed), SvO2, and Cardiac Output allow for rapid changes in treatment
strategy.
b) Con – Thrombogenic, prone to misinterpretation, requires leveling which lends to error,
no difference in outcome.
4. Other monitoring devices
a) CO monitors, tonometry, tissue oxygen tension
III.
Physiologic Principles
A. Oxygen Delivery
1. DO2 = CO x CaO2: Delivery of O2 is equal to the Cardiac Output in L/min multiplied by the O2
content of the blood.
a) CaO2 = (1.38 x Hb x SaO2) + (0.003 x PaO2): O2 content is equal to the amount of
hemoglobin multiplied by O2 saturation plus a small contribution from the dissolved O2
pressure.
B. Perfusion
1. Determinants of blood pressure
a) Ohm’s Law
b) V = R x I; MAP = CO x SVR
2. Determinants of Cardiac Output
a) CO = HR (rhythm) x SV
b) SV related to
(1) Preload (Starling forces)
(2) Afterload
(3) Contractility
3. Perfusion Pressures
a) Cerebral Perfusion Pressure = MAP – Intracranial Pressure
b) Coronary Perfusion Pressure = MAP – RA Pressure
c) Renal Perfusion Pressure = MAP – IVC Pressure
4. Autoregulation –
a) The brain and kidneys can autoregulate up to a pressure of 100 to 150 mm Hg, at which
point the pressure-flow relationship becomes linear. In patients who are chronically
hypertensive, the autoregulation curve may be shifted, leading to a state of relative
hypoperfusion at low-normal pressures.
130
SOCCA Residents Guide 2013
C. Oxygen Supply and Demand
1. Determinants of Supply
a) CO
b) SaO2
c) Hb
d) PaO2
2. Markers of Demand
a) Lactate/HCO3b) Balance of supply and demand
c) SvO2
d) ScvO2
IV.
Pathophysiology and Treatment
A. Hypovolemic Shock
1. ATLS Algorithm
a) Aggressive crystalloid resuscitation, and draw blood for type and cross-match. If the
patient fails to respond to initial crystalloid boluses, then transfusion with type O negative
blood should begin. As soon as type-specific blood is available, it should be used.
2. Crystalloid – Effective, long history of use, and inexpensive. Generally Ringer’s Lactate or
Normal Saline is the first option for crystalloid resuscitation.
3. Colloid – No definitive evidence of superiority, and in fact some data suggest colloid may be
detrimental. Use judiciously, especially in the septic, distributive shock population. Increasing
popularity of starch solutions bears mention.
4. Blood and blood products – Transfuse components as labs indicate.
5. If bleeding continues with poor surgical control, consider transfusion of platelets,
cryoprecipitate, desmopressin, CaCl, and recombinant Factor VII.
B. Distributive Shock
1. Surviving Sepsis Guidelines
a) Early goal-directed therapy during the first 6 hours after recognition.
b) Broad-spectrum antibiotic therapy instituted within 1 hour of diagnosis of septic shock.
(1) Gram-positive coverage, Gram-negative coverage, consider anti-fungal.
(2) Narrow coverage when appropriate.
c) Source control
(1) Surgical vs. Percutaneous drainage
d) Fluid challenge with crystalloid or colloid to goal MAP of 65 mm Hg.
e) Vasopressors (Norepinephrine or Dopamine) to maintain MAP at least 65 mm Hg.
f) Consider Inotropic therapy (Dobutamine or Epinephrine) if CO remains low despite fluid
challenge.
g) Consider Steroids if hypotension continues despite above maneuvers.
h) Target hemoglobin 7-9 g/dL (except in the face of coronary artery disease, obvious tissue
hypoperfusion, or hemorrhage).
i) Use low tidal volume ventilation and PEEP when able.
2. Early Goal Directed Therapy
a) Begin resuscitation immediately.
b) CVP = 8-12 mm Hg
c) MAP at least 65 mm Hg
d) UOP at least 0.5 mL/kg/hr
e) ScvO2 (Central venous) at least 70% (controversial), or SvO2 (Mixed venous) at least
65%
3. Other Considerations
a) Anaphylaxis
b) Adrenal crisis and thyroid storm
c) Neurogenic/Spinal shock
d) Toxins and Drugs
C. Cardiogenic Shock
1. Diagnosis and treatment of:
a) Acute myocardial infarction – ASA, nitrates, narcotics for pain.
131
SOCCA Residents Guide 2013
2.
3.
b) Classification by ECG findings
(1) STEMI – Thrombolytic therapy or PCI
(2) NSTEMI – ASA, heparin, platelet inhibiting agents (thienopyridines or GP IIB/IIIa
inhibitors)
c) Congestive heart failure/cardiomyopathy
Echocardiography
a) Regional Wall Motion Abnormalities
b) Ischemic Mitral Regurgitation
c) Ruptured papillary muscle
d) Post-infarct VSD – poor prognosis
Mechanical Support
a) Intra-Aortic Balloon Pump
b) Left Ventricular Assist Device
c) Right Ventricular Assist Device
d) Extra-Corporeal Membrane Oxygenation
D. Obstructive Shock
1. Diagnosis and treatment of:
a) Pulmonary Embolism – Classic example of obstructive shock. May be due to clot, air, fat,
or amniotic fluid.
(1) Acute increase in RV afterload results in RV pressure overload.
(2) Causes Dead Space and Shunt lesions of ventilation and perfusion. Resulting
hypoxemia worsens RV function.
(3) Chemical thrombolysis indicated early.
(4) Thrombectomy may be indicated for saddle embolus, but outcomes are exceedingly
poor.
b) Tamponade – TTE and TEE are especially useful, as they show dynamic changes in the
RA, RV, and LA anatomy due to extracardiac pressure increases.
(1) May be traumatic, spontaneous (especially in the setting of malignancy), postsurgical, or idiopathic.
c) Restrictive myocarditis and pericarditis – Due to fibrosis of the myocardium or of the
pericardial sac, usually in response to infection.
d) Obstructive cardiomyopathies
(1) HOCM, SAM – Avoid inotropic therapy, increase afterload.
(2) Sub-aortic membrane – May require inotropic therapy and decreased afterload.
e) Tension pneumothorax
References
1.
2.
3.
4.
5.
6.
7.
8.
9.
10.
11.
12.
13.
Kirkpatrick AW, Sirois M, Laupland KB, et al., J Trauma, 2004;57(2):288–95.
Mayo PH, et al., Chest, 2009;135;1050-60.
Levy MM, et al., Crit Care Med, 2010 Feb; 38(2):683-4.
Sprung CI, Annane D, et al., N Engl J Med 2008; 358:111-124.
Kushner, FG, et al. Circulation 2009;120;2271-2306.
Jessup M, et al. Circulation 2009;119;1977-2016.
Hebert PC, et al. N Engl J Med 1999 Apr 1;340(13);1056.
American College of Cardiology. http://www.acc.org.
American Heart Association. http://www.americanheart .org.
Eastern Association for the Surgery of Trauma. http://www.east.org.
Society of Critical Care Medicine. http://www.sccm.org.
Society of Critical Care Anesthesiologists. http://www.socca.org.
Surviving Sepsis Campaign. http://www.survivingsepsis.org.
132
SOCCA Residents Guide 2013
Questions
25.1 The patient described initially is found to have a cardiac index of 1.5L/min/m2. Which if the following drugs is most
appropriate?
A. Dobutamine
B. Vasopressin
C. Norepinephrine
D. Digoxin
25.2 Two hours later his cardiac index is 1.8L/min/m2, his BP is 120/80, HR 90 AV paced and CVP has risen to 25 mmHg.
His blood gas is normal. He is on 5 mg/Kg/min of dobutamine. TEE shows no tamponade, a dilated RV and an under
filled LV. Which of the following is most appropriate?
A. Nitric Oxide
B. Furosemide
C. Vasopressin
D. Norepinephrine
25.3 Three days later he is noted to have a fever, leukocytosis and left lower lobe infiltrate. His vital signs are HR 100
sinus rhythm, BP 80/40, CVP 15 CI 4.0 L/min/m2. Which of the following is the most appropriate next step?
A. Norepinephrine
B. Esmolol
C. Dobutamine
D. Furosemide
133
SOCCA Residents Guide 2013
26. Diagnosis and Treatment of Dysrhythmias
Isaac Lynch, M.D. and Michael H. Wall, M.D.
A 68 year-old woman with a history of COPD, HTN, and previous PTCA has
been in the ICU for 2 days after open repair of AAA. The nurse calls you
because the patient has had marginal BP, low urine output, and this funnylooking EKG (figure 1) for two hours, and now she seems a little sleepier…
Figure 26-1. Funny-looking EKG
I.
Introduction
A. Warning! This is not an exhaustive description of the morphology, pathophysiology and
treatment of all known and unknown arrhythmias! It is intended to quickly educate on, and guide
the treatment of, arrhythmias commonly encountered in the ICU. Please refer to a cardiac
textbook for more detailed information.
B. Dysrhythmias are very common in the ICU and complicate the management of patients with
sepsis, renal failure, pulmonary disease, coronary ischemia and heart failure. They are extremely
common after cardiac surgery, and occur frequently following other types of major surgery as
well.
C. Dysrhythmias are classified according to their origin and the resulting heart rate (e.g. sinus
bradycardia). Bradycardias are most commonly due to SA node dysfunction or conduction
blockade. Tachydysrhythmias occur via one of three mechanisms: automaticity, reentry, or
abnormal triggering (from afterdepolarizations).
1. Automaticity. Cardiac cells that are normally capable of developing spontaneous diastolic
depolarization are known as pacemakers. There are pacemaker cells at the sinoatrial (SA)
node (dominant, 60-100 bpm), the atrioventricular (AV, 40-60 bpm) junction, and the bundle
of His (ventricular, 20-40 bpm). The SA node suppresses the firing of other �downstream’
pacemakers. Abnormal automaticity can result from slowing of the SA rate (or blocking the
conduction) to the point where a downstream pacemaker takes over. It can also occur because
of enhanced automaticity of non-pacemaker cells (which have reduced resting membrane
potentials as a result of injury), which then fire at a rate greater than the SA node.
2. Reentry. This can occur when there are two conduction pathways existing in parallel, each
with a different conduction velocity (e.g., the AV node and an accessory tract). In �typical’
reentry, the electrical impulse propagates down the slow pathway (AV node) and conducts
retrograde up the fast pathway (accessory). Anterograde conduction via the accessory tract is
usually prevented due to unilateral block. In �orthodromic’ reentry occurs when conduction to
the ventricles occurs via the fast pathway, while conduction to the atria is retrograde up the
slow pathway. These arrhythmias are usually paroxysmal, with abrupt onset.
3. Triggering. Early or delayed afterdepolarizations (EADs/DADs) are changes in the
membrane potential that occur during or immediately after repolarization. EADs occur as a
result of prolongation of the action potential, and are enhanced by slower rates. DADs are
transient depolarizations that occur after repolarization, and are caused by abnormally high
intracellular Ca++ levels (e.g., catecholamines). They are enhanced by a fast rate.
D. Symptoms
1. Bradycardia. Traditionally HR less than 60 beats per minute (bpm). Patients are rarely
symptomatic until the HR is in the 40s, although well-trained athletes may have a resting HR
that low. Symptomatic bradycardia results from decreased cardiac output, and can manifest as
fatigue, exercise intolerance, dyspnea, angina, delirium, and syncope. Severe bradycardia can
134
SOCCA Residents Guide 2013
mimic cardiogenic shock, and result in hypotension, end-organ dysfunction and death.
Tachycardia. Traditionally HR greater than 100 bpm. Symptoms due to tachycardia can
result at any rate, but rarely at HR less than 150. In addition to the symptoms of bradycardia
(decreased cardiac output/cardiogenic shock), patients frequently complain of anxiety,
palpitations, and breathlessness. Patients with coronary artery disease (CAD) may have
demand ischemia and angina. Tachycardia syndromes can also result in sudden cardiac death
(SCD).
II. Pathophysiology
A. Atrial Dysrhythmias
1. Sinus Dysrhythmia. A normal variant, with heart rate changes secondary to intrathoracic
pressure differences during respiration (Bainbridge reflex).
2. Bradydysrhythmias.
a) Sinus Bradycardia (SB). Decreased automaticity of the SA node. Can be due to
increased vagal tone (or absent sympathetic tone, as in high spinal cord lesions), drugs
(Beta-blocker toxicity), coronary ischemia, and primary node dysfunction.
b) Sinoatrial disease. �Sick sinus syndrome’ (SSS). This is a disease of the conduction
system, characterized by symptomatic bradycardia, sudden changes in HR, frequent sinus
pauses, atrial standstill, junctional escape, and often alternating atrial or junctional
tachycardia and escape (the so-called �tachy-brady syndrome’). SSS has many causes,
but it can be familial or due to ischemia, rheumatic disease, as well as infiltrative or
inflammatory disorders (e.g. amyloidosis). It is often idiopathic. SSS accounts for 50%
of pacemaker implantation.
(1) Carotid Hypersensitivity. An exaggerated response to carotid sinus stimulation,
often occurs in older men, and is a contributing factor to unexplained falls
(syncope).
(2) Wandering atrial pacemaker (WAP). Multiple foci of automaticity in SA node, atria
and AV node cyclically dominate atrial pacing. Usually asymptomatic.
(3) Conduction system blockade. The conduction system can be disturbed anywhere
along its length, from the SA node to the bundle branches in the ventricles. This
can occur as a result of the impulse reaching tissue in its refractory phase, that has
an abnormally low resting membrane potential (e.g., due to drug effect or disease),
or that is physically unable to conduct an impulse (i.e., scar from prior MI). If the
block occurs above the AV node, a latent atrial pacemaker may dominate. If the
impulse cannot pass to the AV node or ventricles, distal ectopic pacemakers will
emerge.
(a) AV nodal blockade (AVB). Conduction block may be incomplete (1st and
2nd degree) or complete (3rd degree). Block may be temporary or
permanent, and is caused by a variety of drugs and diseases. Common
pharmacologic offenders are adenosine, Ca++ channel blockers, Beta
blockers, amiodarone, and digoxin. Inflammatory and infiltrative disorders
(e.g., Sarcoid), coronary ischemia, myocarditis, thyroid disorders, and
malignancy can lead to AVB. It is a frequent complication following cardiac
valve surgery (esp. aortic and mitral).
(b) 1st degree AVB is defined as a PR interval > 210msec. It is almost always
due to conduction slowing in the AV node, and may be caused by increased
vagal tone, Ca++ channel blockers, B-blockers, and digoxin. It may also
occur in structurally normal hearts. It rarely requires intervention.
(c) 2nd degree AVB usually reflects disease of the AV node (Mobitz I) or HisPurkinje system (Mobitz II).
(d) Mobitz I (Wenckebach). A progressive lengthening of the PR interval until
the QRS complex is dropped. Pattern resumes after a pause. May occur with
digoxin toxicity or MI, and is usually transient. Typically does not need
treatment.
(e) Mobitz II. Irregular pattern of complete conduction blockade, resulting in
randomly dropped QRS complexes. There is no PR lengthening. The
ventricular rate depends on the frequency of dropped beats. This type of AVB
can proceed to complete heart block, and should be treated.
(f) 3rd degree AVB is complete atrio-ventricular dissociation due to atrial
2.
135
SOCCA Residents Guide 2013
3.
impulses not reaching the ventricular conducting system. The EKG will show
independent atrial and ventricular rates, as a downstream ectopic pacemaker
arises. The ventricular rate and QRS pattern depend on the site of the ectopic
pacemaker, but often arises from the ventricles (rate 30-40). This often
produces symptomatic bradycardia, and requires immediate treatment, and
long-term pacemaker placement.
(4) Bundle-branch block (right or left), can be recognized by its typical appearance on
EKG. It is more prevalent at older ages, frequently occurs without underlying
disease, and rarely causes symptoms. However, the finding of a new bundle branch
block, especially LBBB, in a patient should elicit a workup for coronary ischemia.
c) Pulseless electrical activity (PEA) is electrical cardiac activity (may appear like NSR)
without mechanical activity. It is a non-perfusing, lethal rhythm. Get help, activate
ACLS.
d) Asystole refers to the complete absence of electrical and mechanical activity of the heart.
Get help, activate ACLS.
Tachydysrhythmias.
a) Sinus tachycardia (ST) has a rate >100 bpm (though rarely >140), and 1:1 AV
conduction. It can be differentiated by other SVT by its gradual onset and resolution. It
is usually an attempt to increase CO, such as during hypovolemia, anemia, heart failure,
and PE. May also be due to pain, fear, hypoglycemia, drug/ETOH withdrawal, and
hyperthyroidism.
b) Sinus nodal reentrant tachycardia (SNRT) is a rare condition, with a rate of 110-180 bpm,
usually 1:1 conduction, with normal-appearing P waves and QRS morphology. The
abrupt onset and paroxysmal nature differentiate it from ST. It is rarely symptomatic,
thus rarely requires treatment.
c) Premature atrial contractions (PACs) arise from areas of ectopic foci (automaticity) in the
atria. They can be associated with feelings of �fluttering’ or heaviness in the chest, and
are often caused by excessive caffeine, ETOH, stress or hyperthyroidism. They are
common in patients of all ages, and are not associated with heart disease. Differentiated
from PVCs by the narrow QRS complex and presence of P waves, and usually lack
compensatory pause.
d) Multifocal atrial tachycardia (MAT) is an irregular rhythm that features 3 or more distinct
P wave morphologies, indicating multiple areas of ectopic atrial pacemakers. It can
closely mimic atrial fibrillation, but is differentiated by distinct P waves and usually a
lower atrial rate (100-180). It occurs almost exclusively in the setting of chronic
pulmonary disease (chronic hypoxic conditions and atrial stretch), and is resistant to
pharmacologic and electrophysiologic treatment. Magnesium may be of some benefit.
e) Supraventricular tachycardia (SVT)
f) Paroxsymal atrial tachycardia (PAT). P waves will be present on EKG. The atrial rate is
130-200. If rate is >180, physiologic block at the AV node will occur. Can be from
reentry, increased automaticity or DADs.
g) AV nodal reentrant tachycardia (AVNRT). Also known as junctional tachycardia, it is
caused by a unilateral block within the AV node, allowing retrograde conduction and
initiation of the reentry circuit. There are no P waves present. The ventricular rate is 150
with a narrow-complex QRS.
h) AV reentrant tachycardia (AVRT). Requires an accessory pathway. Rate is usually
150-220 bpm with 1:1 conduction.
i) Wolff-Parkinson-White (WPW) is a form of reentrant SVT that involves an AV bypass
tract (bundle of Kent). During NSR, the conduction down the accessory tract is faster
than normal, leading to pre-excitation of the ventricles and the characteristic �delta wave’
pattern on EKG. During reentrant tachycardia, the impulse can either be orthodromic
(majority), which means conduction goes anterograde through the AV node (narrow
complex), or antidromic (anterograde down the accessory tract, wide complex). Nodal
blocking agents such as adenosine and Ca++ channel blockers will worsen conduction
down the accessory tract, and therefore should be avoided.
j) Atrial Flutter is often associated with a structural lesion (typically around the mitral
valve). The atrial rate is 300 +/- 20 bpm, with variable conduction through the AV node
(usually 2:1). Typical �sawtooth’ pattern can be seen best in lead II. If conduction block
136
SOCCA Residents Guide 2013
is present, the patient may have a slow ventricular rate. Aflutter commonly progresses to
Afib.
k) Atrial fibrillation (Afib) can be a reentrant phenomenon or fibrillatory, originating in the
atria or pulmonary veins. It can occur without structural abnormality (Lone Afib), or as a
result of atrial distension due to mitral valve disease (MR or MS), chronic pulmonary
disease, coronary ischemia, heart failure and cardiomyopathy. Afib is extremely common
after cardiac surgery. In Afib, the atrial rate is 260-450 with variable conduction and loss
of AV synchrony. The ventricular rate is usually 100-180, but can be faster in the
presence of an accessory tract. Faster rates, known as �Rapid Ventricular Response’, or
RVR, can lead to fatigue, heart failure, angina, and cardiovascular collapse, and is a
medical emergency, often requiring synchronized cardioversion. Perioperative Afib in
patients with no prior history usually resolves within 3 months, and long-term treatment
is rarely indicated. The biggest risk of chronic Afib is embolic stroke, and these patients
should be considered for anticoagulation if Afib is paroxysmal or last >48 hours. Use the
CHADS2 scoring system as a guideline. (See table 26-1)
B. Ventricular Dysrhythmias
1. Generally speaking, ventricular dysrhythmias can be distinguished from atrial ones by the
absence of associated P waves, and QRS complexes that are longer than 120 msec, often with
a bizarre morphology. Atrial rhythms with bundle branch block and those with accessory
pathways may have widened QRS complexes (rarely greater than 160msec), but will still have
associated P waves.
2. Bradydysrhythmias.
a) Ventricular escape. Due to activation of latent pacemaker sites distal to the AV node
(His-Purkinje system), secondary to upstream conducton blockade (see above).
b) Idioventricular rhythm
c) Accelerated idioventricular rhythm (AIVR). Abnormal automaticity arising from the
Purkinje fibers. Rate varies, can be >60 bpm with variable conduction, possibly with AV
dissociation.
3. Tachydysrhythmias.
a) Long QT syndrome. Prolonged repolarization of action potential predisposes cardiac
myocytes to afterdepolarizations, which can trigger PVCs. A PVC during the sensitive
repolarization period (“R-on-T” phenomenon) can lead to reentry VT and Torsades de
Pointes, precipitating sudden cardiac death (SCD). The strongest risk of induced VT/
TdP occurs when the corrected QT interval (QTc) is greater than 500msec. Drug toxicity
should be ruled out in cases of new prolonged QT interval, but Long QT syndrome can be
a familial disease. Syncope is the hallmark symptom.
b) Premature ventricular contractions (PVCs) arise from one or more ectopic foci distal to
the AV node, and are distinguished by their wide QRS complex and lack of a P wave.
The premature beat leads to a decrease in SV (and thus CO), which is balanced by an
increased in the SV of the beat following a compensatory pause. PVCs may occur in
normal hearts, but increasing frequency may indicate coronary ischemia, valvular
disease, cardiomyopathy, electrolyte disturbance (hypokalemia, hypomagnesemia), or
prolonged QT. It may also occur as a result of direct irritation from central catheters and
guidewires, or as a result of certain antiarrhythmic therapy (procainamide). Symptoms
are rare unless there are >3 PVCs consecutively. In this case, patients may complain of
palpitations or syncope.
c) Accelerated AV junctional tachycardia. From increased automaticity or abnormal
triggering, below the AV node. Rate is 80-130 with 1:1 or variable conduction.
d) Ventricular tachycardia (VT) may be defined as either monomorphic or polymorphic, and
sustained or non-sustained. VT is usually associated with underlying cardiac disease
such as chronic or acute ischemia, heart failure, cardiomyopathy, Long QT syndrome,
and cardiac surgery, where the diseased myocardium is prone to develop abnormal
automaticity. Monomorphic VT can be well tolerated by some patients, but often is
associated with symptoms of cardiac failure/shock, such as dyspnea, syncope,
hypotension, and oliguria. It should be treated regardless, but symptomatic VT should be
treated urgently, often with synchronized cardioversion.
(1) Non-sustained VT is typically less than 30sec in duration, occurring in 3-5 beat
�salvos’. Increasing frequency and duration of salvos is considered high risk for a
137
SOCCA Residents Guide 2013
potentially lethal rhythm.
Sustained VT occurs at a HR greater than 100bpm for greater than 30s. There are
rare instances of sustained monomorphic VT in structurally normal hearts, but it
usually forms in the setting of diseased myocardium. For instance, a reentrant
circuit that forms around a ventricular aneurysm following MI can lead to recurrent
sustained monomorphic VT. Patients with EF <35% are particularly at risk for
SCD from VT/VF, and should undergo prophylactic automated implantable
cardioverter-defibrillator (AICD) placement.
(3) Polymorphic VT has an irregular QRS pattern, and may be an irregular rhythm,
suggesting a shifting electrical activation pattern. Polymorphic VT is a more
dangerous electrical pattern, and more prone to produce symptoms and deteriorate
into VF. It can be seen in the setting of ischemia or myocarditis, and classically in
Long QT syndrome. All polymorphic VTs are a medical emergency.
(4) Torsades de Pointes (TdP) is technically the polymorphic VT associated with Long
QT syndrome. It literally means �twisting of the points’, and looks like a sine wave
slowly rotating on an axis. It typically occurs when the QT interval has been
excessively prolonged, which is usually iatrogenic from antiarrhythmic agents
(sotalol, procainamide) or phenothiazine drugs (droperidol). Withdrawal of the
offending agents is the best way to avoid TdP, but symptomatic episodes require
emergency treatment. Magnesium is often helpful, and cardioversion may be
necessary.
(5) Pulseless VT, regardless of the pattern or etiology, is a lethal rhythm requiring
immediate CPR and defibrillation. Get help and activate ACLS.
Ventricular fibrillation (VF) is incompatible with life. There is no associated pulse as the
ventricle does not contract in an organized manner. Only immediate defibrillation will
convert VF into a life-sustaining rhythm. Survival is highest when time to defibrillation
is less than 3 minutes. Get help, activate ACLS.
(2)
e)
III.
Management
A. When evaluating a patient with an arrhythmia, there are 5 basic questions to ask. Obtain a 12 lead
EKG and determine the following. Knowing the answers to these questions will help guide
pharmacologic and electrophysiologic treatment, and suggest underlying causes to treat. Please
refer to ACLS algorithms for rhythm-specific treatment plans.
1. Is the rhythm slow or fast?
2. Is it regular or irregular?
3. Are the QRS complexes narrow or wide (120 msec)?
4. Does every P have a QRS and does every QRS have a P?
5. Is the patient hemodynamically stable?
B. Bradycardia. Treatment for bradycardia should be reserved for patients with symptoms, as it is
often well tolerated, unless the HR is <40. Continue cardiac monitoring with telemetry, and if
patient becomes symptomatic or HR slows further, initiate therapy. The mainstay of therapy is
treating the underlying cause or removing the offending agent. Remember the H’s and T’s! In
children especially, bradycardia is often a symptom of hypoxia, and heralds respiratory collapse.
1. Glucagon should be considered in cases of B-blocker toxicity. Give 50 mcg/kg IV loading
dose followed by 1-15 mcg/hr infusion, titrated to effect. Side effects are hypokalemia,
hyperglycemia, nausea and vomiting.
2. Atropine (0.5 mg IV per dose, every 3 to 5 min), a vagolytic, can be used to temporize
bradycardia until definitive treatment. The toxic dose in adults is not known, but some
undesirable side-effects can occur such as urinary retention, bowel obstruction, and delirium
(central anticholinergic action). The CNS side-effects can be reversed with physostigmine.
Atropine is ineffective after cardiac transplant.
3. Epinephrine (2-10 mcg/min IV) is a potent beta1 agonist that increases HR and contractility,
as well as systemic vascular resistance (via alpha-receptor activity). Epi administration can
lead to tachydysrhythmias.
4. Dopamine (2-10 mcg/kg/min) is a direct beta1-agonist and also causes the release of
endogenous norepinephrine. At low doses, it is not associated with significant
tachydysrhythmia, but that is always a concern.
5. Isoproterenol (0.5-5 mcg/min) is a synthetic analogue of epinephrine that has almost exclusive
138
SOCCA Residents Guide 2013
beta1-agonist effect. It can significantly increase myocardial oxygen demand and decrease
coronary perfusion, worsening injury after MI. It can also lead to tachydysrhythmia.
Isoproterenol is not a first-line agent.
6. Electrical pacing should be considered in cases of Mobitz II and 3rd degree AVB, regardless
of current symptoms, and in cases of bradycardia refractory to initial treatment. In the ICU,
emergency temporary pacing falls into two categories, transcutaneous and and transvenous.
a) Transcutaneous pacing is indicated for patients with hemodynamic compromise from
bradycardia or high-degree AVB. It should be used only until transvenous pacing is
established, as external pacing is painful and unreliable. Place hands-free defibrillator
pads over the cardiac apex and medial to the left scapula, and turn defibrillator to
synchronized pacing mode with a HR of 80bpm. It is better to start with a high
milliamperage (mA), for example, 200 mA, and then reduce to the threshold after capture
is made. Conscious patients will likely require sedation.
b) Transvenous pacing, for ICU purposes, is best accomplished via central venous access,
ideally the right Internal Jugular vein (although some advocate the femoral approach). A
bipolar endocardial pacing catheter is advanced through an introducer sheath (6 French),
into either the right atrium or ventricle. In an emergency, the pacing catheter should be
set to 80bpm or higher, at maximum mA, and advanced until there is EKG and
mechanical evidence of capture (pulse). Then, the mA can be reduced to 2 times the
threshold. Transvenous pacing is less painful than transcutaneous pacing, but has more
complications. In addition to the complications of placing a central line (pneumothorax,
arterial puncture, hematoma), there is a risk of VT/VF, myocardial puncture, thrombus,
PE, and infection. Also, there is a continued risk of loss-of-capture, requiring continuous
telemetry.
C. Tachycardia. Again, treatment of tachyarrhythmia must begin with correction of underlying
causes, especially in the case of ST (pain, fever, hypovolemia, anemia). In the absence of
offending drug or reversible state, treatment is aimed at three things: suppressing automaticity,
prolonging the effective refractory time, and facilitating normal impulse conduction. Multiple
studies have shown that for many tachyarrhythmias, controlling the cardiac rhythm with
antiarrhythmic agents actually increases mortality, as many of these drugs are, ironically,
proarrhythmic as well. For that reason, treatment focuses on controlling the ventricular rate and
providing electrophysiologic prophylaxis against fatal arrhythmia with AICD.
1. Adenosine (6 mg rapid IV bolus) is an AV nodal blocker, slowing conduction time and
blocking reentry circuits. It will terminate reentrant SVT in 60% of patients after the first
dose, and an additional 30% on repeat dosing. It is not effective in converting Afib, but will
slow the ventricular rate to allow for better diagnostic testing. It can cause asystole, so have
resuscitation equipment at hand. Side effects are coronary vasodilatation (can lead to
coronary steal), bronchoconstriction, and flushing.
2. Beta-blockers (multiple drugs and dosages) act by blocking cardiac beta-adrenergic receptors,
ameliorating the effect of circulating catecholamines. By decreasing HR and BP they are
cardioprotective, and are mainstays in the treatment of MI. They are indicated for rate control
in patients with preserved LV systolic function with Afib/flutter, and SVT. They should be
used with caution in patients with decompensated HF and high-degree AVB. Side effects are
bradycardia, hypotension (orthostasis as well), and worsening AVB.
3. Calcium channel blockers (the non-dihydropyridines verapamil and diltiazem) are negative
chronotropes (reduce SA node firing) and dromotropes (slow conduction through the AV
node), and thus effective at controlling the rate of most SVTs. They are also negative
inotropes (diltiazem less so), and therefore undesirable in heart failure. Side effects are
bradycardia, hypotension, and varying degrees of AVB. Overdose may be reversed with IV
calcium.
4. Amiodarone (150 mg bolus followed by infusion, bolus may be repeated) acts on Na+, K+,
and Ca++ channels, as well as alpha and beta receptors. This causes prolongation of the
myocardial action potential and refractory period, as well as decreasing the effect of
circulating stress hormones (decreasing intracellular calcium). Amiodarone is effective in
controlling the rate of all SVTs, including Afib, and is indicated for refractory/recurrent VT
and VF as well. Amiodarone has a very long half-life, and must be loaded in order to reach
meaningful levels quickly. This can cause hypotension, bradycardia, and sinus arrest acutely.
Long term side effects include serious pulmonary toxicity, ventricular arrhythmias, rare
139
SOCCA Residents Guide 2013
hepatic toxicity, CNS disturbance, and thyroid dysfunction (it is an analogue of iodine). That
being said, it is incredibly useful in acute, hemodynamically unstable cases of SVT/VT.
5. Digoxin inhibits the Na+K+-ATPase membrane pump, leading to increased intracellular
sodium (and thus calcium), and positive inotropy. It prolongs the refractory period of
conduction cells and reduces conduction through the AV node. It has a delayed peak effect
(up to 6 hours), and a narrow therapeutic index (especially in the setting of hypokalemia).
Digoxin toxicity can lead to fatal ventricular arrhythmias. It can also cause GI disturbance,
visual disturbances, and heart block. It is definitely not a first line agent, but is useful in
refractory Afib, especially in the setting of heart failure. Digibind, a digoxin antidote, is
commercially available.
6. Other agents for SVT and ventricular arrhythmias include lidocaine, procainamide, sotalol,
flecainide, and dipyridamole. These agents are best left to cardiology specialists, as they have
the tendency to cause fatal arrhythmias if given in the wrong setting. Lidocaine, however,
may be used acutely in cases of refractory VT. Magnesium can be useful in TdP.
7. Synchronized cardioversion refers to DC electrical stimulation of the myocardium during
QRS, which allows the delivered shock to safely depolarize all excitable tissue at once,
resetting the refractory period. This allows the dominant pacemaker cells to resume and
hopefully suppress areas of ectopy and reentry. There are both monophasic and biphasic
devices, and the energy delivered will differ depending on the type. Biphasic cardioversion
seems to be superior to monophasic, and 50-150 Joules (J) is a safe range. Complications of
cardioversion include embolic events (especially in Afib), skin burns, myocardial dysfunction,
other arrhythmias, and transient hypotension.
D. Defibrillation refers to the non-synchronized delivery of massive amounts of energy with the
intent of depolarizing all of the myocardium simultaneously. If the energy is insufficient to
completely affect all cardiac tissue, areas of fibrillation will remain and the heart will revert back
after the refractory period. In addition, it seems that with time the fibrillation is more difficult to
convert. Therefore in cases of pulseless VT/VF, where every second counts, it is recommended
that high energy shocks (360J monophasic or 150-200J biphasic) be delivered as soon as possible.
If a defibrillator is not immediately present, begin CPR and defibrillate when able.
IV.
Future Directions
A. Development of improved diagnostic tools and mapping programs will improve understanding of
arrhythmias, and improve our ability to treat them.
B. Pharmacologic research will result in safer antiarrhythmic agents to help control Afib and
recurrent VT/VF. One such agent, dronedarone, has shown initial success.
Web Sites
1.
2.
3.
4.
American College of Cardiology. http://www.acc.org.
American Heart Association. http://www.americanheart.org.
Society of Critical Care Medicine. http://www.sccm.org.
Society of Critical Care Anesthesiologists. http://www.socca.org
References
1.
2.
3.
4.
5.
6.
Breitkreutz R. WF, Seeger F.: Focused echocardiographic evaluation in resuscitation management:
Concept of an advanced life support-conformed algorithm. Critical Care Medicine Focused Applications of
Ultrasound in Critical Care Medicine 35(5):S150, 2007
Dobrev D, Nattel S: New antiarrhythmic drugs for treatment of atrial fibrillation. The Lancet 375:1212,
2010
Kleinman ME, Chameides L, Schexnayder SM, et al: Part 14: Pediatric Advanced Life Support: 2010
American Heart Association Guidelines for Cardiopulmonary Resuscitation and Emergency
Cardiovascular Care. Circulation 122:S876, 2010
Lafuente-Lafuente C, Mahe I, Extramiana F: Management of atrial fibrillation. BMJ 339, 2009
Menke J, LГјthje L, Kastrup A, et al: Thromboembolism in Atrial Fibrillation. The American Journal of
Cardiology 105:502, 2009
Neumar RW, Otto CW, Link MS, et al: Part 8: Adult Advanced Cardiovascular Life Support: 2010
American Heart Association Guidelines for Cardiopulmonary Resuscitation and Emergency
Cardiovascular Care. Circulation 122:S729, 2010
140
SOCCA Residents Guide 2013
7.
Schirmer SH, Baumhäkel M, Neuberger H-R, et al: Novel Anticoagulants for Stroke Prevention in Atrial
Fibrillation: Current Clinical Evidence and Future Developments. Journal of the American College of
Cardiology 56:2067, 2010
Table 26-1: CHADS2 Score
Medical Condition
# Points
Congestive heart failure
1
Hypertension
1
Age >75
1
Diabetes mellitus
1
Stroke/TIA
2
Legend:
0-2 low risk of stroke (<5%/year)
3 or 4 moderate risk of stroke (5-10%/year)
5 or 6 high risk of stroke (10%/year)
CHADS Score. Circulation 2004: 110; 2287-2292
Questions
26.1 A 75 year-old man is post-operative day one following CABG and is found to be in new onset atrial fibrillation. He
has an LVEF of 25%. He is asymptomatic, his HR is 130, and his BP is 110/70. Which of the following is the most
appropriate next step?
A. Amiodarone
B. Metoprolol
C. Defibrillation at 200J (biphasic)
D. Cardioversion at 105J (biphasic)
26.2 A 56 year-old man is post-operative day three following mitral valve repair. He has an irregularly narrow complex
tachycardia with a HR of 140. His BP is 88/40 and his oxygen saturation is 88% on 100% FO2. Which of the following is
the most appropriate therapy?
A. Amiodarone
B. Metoprolol
C. Defibrillation at 200J (biphasic)
D. Cardioversion at 100J (biphasic)
26.3 A 86 year-old man is post-operative day one following CABG, he is found to have wide complex tachycardia, HR
150, his arterial and PA tracings are flat and he has no pulse. Which of the following is the next step in management?
A. Amiodarone
B. Metoprolol
C. Defibrillation at 200J (biphasic)
D. Cardioversion at 100J (biphasic)
141
SOCCA Residents Guide 2013
27. Diagnosis and Treatment of Myocardial Ischemia
Gerardo Rodriguez, MD
A 66-year-old male presents for elective shoulder surgery. Nine months
before he suffered an acute myocardial infarction (AMI) in his left anterior
descending coronary artery and underwent emergent cardiac catheterization
and placement of 3 drug eluting stents. He stopped his clopidogrel and
aspirin 8 days prior to his current surgery as instructed. A recent dobutamine
echocardiogram demonstrated no evidence of ischemia and he has excellent
exercise tolerance.
Background
Coronary artery disease (CAD) remains one of the leading causes of death in the industrial world and it is
the most common cause of death in the US. Clinically, a continuum ranges from a temporary imbalance
between coronary oxygen consumption and myocardial oxygen delivery to significant imbalance,
potentially leading to myocardial ischemia, myocardial infarction, and ultimately death. Though
preventative care is key to reducing the incidence of CAD, early recognition and therapeutic intervention of
acute myocardial ischemia is critical to reducing morbidity and mortality. Anesthesiologists routinely care
and manage patients with significant CAD risk factors both in the perioperative and critical care setting,
and are, therefore, poised to be the first to recognize and treat an early myocardial ischemic event. The
following chapter provides a brief overview of myocardial ischemia, pathophysiology, management, and
therapy.
Classification
There are many classifications based on the extent of the infarction. In a transmural MI the full thickness of
myocardium, from endocardium to epicardium, is involved. In a nontransmural MI the damage to the
myocardium is limited to the endocardium. The endocardial subendocardial zones are poorly perfused
compared to rest of heart making them more vulnerable to ischemia. Clinically, the use of the terms QWave MI (indicating the presence of Q-waves on ECG) and non-Q Wave MI are used as descriptors. A
differentiation between the presence of ST segment elevation on ECG (STEMI) or non-ST segment
elevation on ECG (NSTEMI) has been also used. Although it does not indicate the degree of myocardial
involvement, STEMI patients have higher early morbidity and mortality. However, this classification does
not predict long-term sequelae. Finally, emerging data suggest that myocardial infarction occurring in the
perioperative period (PMI) may differ from a nonperioperative myocardial infarction.
I.
Factors that influence myocardial oxygen delivery
A. Heart rate
1. When the HR increases, the diastolic time decreases. Significant coronary blood flow occurs
during diastole, particularly to the left ventricle.
B. Coronary perfusion pressure
1. The aortic diastolic BP is the primary determinant of coronary perfusion pressure.
2. Ventricular end-diastolic pressure if elevated may compromise subendocardial circulation.
C. Arterial oxygen content
1. Primary determinants
a) Arterial oxygen saturation
b) Hemoglobin concentration
D. Coronary vessel diameter
1. Small arteries increase the risk of significant coronary lesions from atheroma formation.
II.
Factors that influence myocardial oxygen demand
A. Basal requirements
1. Physical exertion, severe HTN, and severe aortic stenosis increase myocardial metabolic
demands
B. Heart rate
1. Increases in HR increase myocardial oxygen demand
C. Wall tension
1. Increases in wall tension increase myocardial oxygen demand
142
SOCCA Residents Guide 2013
D. Contractility
1. Increases in contractility increase myocardial oxygen demand
III.
Pathophysiology of Myocardial ischemia / infarction
A. Coronary artery occlusion
1. Atheromatous plaque forms over many years to decades
a) Fibromuscular cap and underlying lipid-rich core
b) Shoulder region, where cap meets vessel wall
c) Platelet-mediated thrombus formation on any disruption of vessel wall
2. Disruption of vascular endothelium with unstable atheromatous plaque stimulates an
intracoronary thrombus resulting in an acute coronary artery occlusion
a) Most common cause of MI.
B. “Demand ischemia”
1. Imbalance between myocardial oxygen delivery and consumption. Appears to be the most
likely cause in the early perioperative period.
IV.
Extent of Myocardial Damage
A. Level of coronary artery occlusion
1. More proximal thrombus, more myocardium at risk
B. Length of occlusion time
1. 20-40 min to irreversible myocardial cell damage.
C. Collateral Circulation
1. Mitigates transient myocardial perfusion limitation
V.
Postoperative Myocardial Infarction (PMI) Mechanisms
A. Risk of PMl peaks within the first three postoperative days (Days 0 to 1 is carries the highest risk)
1. Mobilization of extravascular fluid, results in an increase in preload
2. Pronounced thrombotic risk by activation of coagulation cascade during surgery
3. Increases in HR and BP associated with catecholamine surge
4. Postoperative pain increases myocardial demand
5. Recent data suggest ischemia begins at the end of surgery and during emergence from
anesthesia
6. Most cases of “early” PMI are possibly related to perioperative myocardial demand ischemia,
progressing to plaque rupture and thrombosis similar to the general population
VI.
Risk Factors for Arteriosclerosis and MI
A. Hyperlipidemia
1. Major Component of Plaque
2. High LDL levels associated with higher MI rate
B. Diabetes Mellitus
C. Hypertension
D. Smoking
E. Family History
F. Mate Gender
G. Age
VII. PMI risk evaluation and reduction
A. The 2007 ACC/AHA perioperative guidelines for noncardiac surgery
1. Guidelines provide a foundation by which physicians involved in the perioperative care of
patients undergoing noncardiac surgery can use to risk stratify, determine preoperative testing,
and modify intraoperative management (see figure 1.) Furthermore, guidelines provide an
update to past literature regarding management of patients with coronary stents and
antiplatelet medication therapy.
2. Active cardiac conditions requiring preoperative cardiology evaluation
a) Unstable coronary syndromes
(1) Unstable or severe angina
(2) Recent Ml, between 7 to 30days
b) Decompensated heart failure
143
SOCCA Residents Guide 2013
c)
Significant arrhythmias
(1) Bradyarrhythmias
(a) Symptomatic bradycardia
(b) High-grade atrio-ventricular block (AVB)
(c) Mobitz II-AVB
(d) 3rd degree AVB
(2) Tachyarrhthymias
(a) New-onset ventricular tachycardia
(b) Supraventricular tachycardia
(c) Arrhythmia
(d) New-onset Atrial fibrillation
d) Severe valvular disease
(1) AS, valve area < 1 cm2
(2) MS, symptomatic
3. Surgical risk
a) Low risk surgery: ambulatory, cataract, breast, superficial, endoscopic surgery
b) Intermediate and high risk surgery require further risk stratification
4. Metabolic equivalents
a) Predict functional capacity and risk of cardiopulmonary complications perioperatively
b) METs > 4, for example, climb a flight of stairs, walk up a hill, or greater physical
exertion
5. Goldman Revised Cardiac Risk Factors
a) History of ischemic heart disease
b) History of heart failure
c) History of cerebrovascular disease
d) Insulin dependent diabetes mellitus
e) Renal insufiiciency, creatinine > 2.0 mg/dL
6. Cardiac risk reduction
a) End result of preoperative risk stratification algorithm should help determine whether a
patient:
b) Goes to operating room without further intervention
c) Goes to operating room with HR control
d) Is postponed for further stress testing, only if it will change perioperative management.
B. 2007 ACC/ AHA Percutaneous Coronary Intervention (PCI) patients who require noncardiac
surgery
1. Significant myocardial infarction risk if early surgery post PCI and inadequate antiplatelet
therapy.
2. PCI type
a) Balloon angioplasty
b) Baremetal stent (BMS)
c) Drug eluting stent (DES)
3. Preoperative plan with recent PCI
a) Proceed to operating room for non-urgent surgery:
(1) Balloon angioplasty > 14 days
(2) BMS > 30 – 45 days with aspirin
(3) DES > 365 days with aspirin
VIII. Signs & Symptoms
A. Chest pain described as a pressure sensation, fullness, or squeezing in the mid-thorax
B. Referred pain to jaw, dentition, shoulder, arm, and back
C. Dyspnea or shortness of breath
D. Epigastric discomfort with or without nausea and vomiting
E. Diaphoresis or sweating
F. Syncope or near-syncope
G. Acute impairment of cognitive function
H. More common in early morning or around time of physical activity
I. Asymptomatic or silent MI, increased likelihood in diabetes mellitus
144
SOCCA Residents Guide 2013
IX.
Diagnosis
A. Electrocardiography
1. ST-segment elevation
2. Nonspecific ST and/or T-wave changes
B. Blood Tests
1. Release of specific enzymes and cell wall proteins of myocardial cells that become ischemic
or infarcted
2. CK-MB, troponin-I (TnI)
C. Echocardiography
1. Regional wall motion abnormalities (RWMA) detected may correlate with specific coronary
vasculature lesion
2. Hemodynamic parameters assessed non-invasively
3. Without recent documented echocardiography evaluation, acute versus chronic RWMA may
be difficult to distinguish perioperatively.
D. Concerns regarding diagnosis of PMI
1. Pain may be masked by analgesics
2. Telemetry misses significant ST-changes
3. 12-lead ECG only diagnostic 50% time
4. Usually NSTEMI
E. Serum biomarkers
1. CK-MB, less sensitive and specific in PMI
2. TnI and TnT, markers of choice, very sensitive to cardiac damage.
a) May be elevated in severe CHF, PE, cardiac contusion, renal failure
X.
Therapy
A. Goals
1. Restore coronary blood flow
2. Limit myocardial damage
B. Antiplatelet agents
1. Aspirin (ASA)
a) Immediate therapy 160-325 mg upon signs or symptoms of MI
b) Prevents platelet aggregation, adhesion and cohesion at disruption site
c) Greatly reduces AMI mortality
d) Long-term use once MI diagnosed
2. Clopidogrel (Plavix) and Ticlopidine (Ticlid)
a) Thienopyridine class
b) When prescribed with ASA, considered dual-antiplatelet therapy (DAT)
c) DAT important for maintaining coronary patency after PCI BMS and DES
C. Oxygen
1. Maximize Hb oxygen carrying capacity
2. Though considered standard care, no published studies demonstrate improved morbidity and
mortality with supplemental oxygen
D. Nitrates
1. Improves myocardial oxygen demand and supply imbalance
2. Improves subendocardial perfusion
3. Reduce preload and afterload
4. Used in the setting of Ml, acute CHF, persistent myocardial ischemia, stable angina
5. Effective in first 48hrs of Ml but no long-term mortality advantages
6. Avoid using in acute inferior wall MI (II, III, aVF) or acute right coronary artery lesion since
may cause acute hypotension
E. Beta-blockers
1. Recommended in MI, early
2. Reduces mortality in AMI setting
a) 28% reduction mortality if used in first week post Ml
b) Indefinite use
3. Decrease myocardial oxygen demand by lowering heart rate and contractility
4. Increases myocardial oxygen supply by increasing diastolic time
5. Antiarrhythogenic, decreases VF threshold
145
SOCCA Residents Guide 2013
6. Antagonizes adrenergic effects of catecholamines
F. Unfractionated heparin
1. Inhibits additional formation and propagation of thrombus
2. Requires partial thromboplastin time (PTT) laboratory testing
G. Glycoprotein-2b/3a (G2b3a) antagonists
1. Antagonize platelet G2b3a-receptors, thereby inhibiting platelet aggregation
2. Use during PCI reduces mortality, reinfarction, and need for further revascularization
procedures
3. Abciximab (ReoPro), eptifibatide (Integrilin), tirofiban (Aggrastat)
H. ACE inhibitors
1. Recommended in AMI patients within first 24 hours
2. Reduces afterload via vasodilitation
3. Continued if Ml patients have:
a) CHF
b) LVEF < 40%
c) HTN
4. Relative contraindication if hypotensive or with declining renal function
I. HMG-CoA reductase inhibitors or statins
1. Newer evidence that all Ml patients should be on statin therapy regardless of HDL/LDL levels
[Schwartz et al. Am J Cardiol 2005]
2. Statins can reduce circulating markers of inflammation within days post AMI
3. Improve coronary endothelium function
4. Reversal of prothrombotic states
5. Reduces atherosclerotic plaque volume
6. ln a review of 6 randomized, controlled trials, high intensity statin therapy (atorvastatin
8Omg) reduced early recurrent ischemic events compared to moderate therapy (40mg) or
placebo
J. Fibrinolytics
1. Indicated for Ml and ST elevation > 0.1mV in 2 contiguous leads or new bundle branch block
2. Restore coronary blood flow in 50 – 80% of cases
3. Best when door to needle time is less than 30 minutes
4. Runs risk of significant bleeding in PMI patients
K. Percutaneous coronary intervention (PCl)
1. “Door to balloon " goal of less than 90 minutes
2. Restores coronary flow 90 – 95% of Ml patients
3. Better than fibrinolysis in short term mortality, bleeding rates, and reinfarction rates
4. Significant improvement in cardiogenic shock
5. Stents reduce need for subsequent large vessel revascularization
6. PCI preferable to fibrinoiysis for PMI after recent noncardiac surgery given lower risk of
major bleeding
7. Berger et al. with 48 postop patients for PCI
a) 65% survival rate much better than those untreated
b) No significant surgical site bleeding in cath lab
c) Pts with sudden onset ST elevation from acute thrombotic occlusion did best with
immediate intervention (PCI vs CABG) compared to no intervention in postop pt.
L. Surgical Revascularization (CABG)
1. Urgent if failed PCI and unstable pt
2. Must have anatomy amenable for grafting
3. Urgent if anatomical complications of Ml
a) Ventricular septal defect
b) Free wall rupture
c) Acute mitral regurgitation
4. Emergency procedure riskier than elective
a) 3-7 days post Ml similar risks to elective
5. Elective CABG improves survival in pts with
a) Left Main Coronary Disease
b) 3 Vessel Disease
c) 2 Vessel Disease not amenable to PCI
146
SOCCA Residents Guide 2013
M. Current controversy exists over use of PCI vs CABG (see Serruys PW et. al for more details)
Discussion
PMI is a known risk in patients with known CAD or cardiac risk factors undergoing noncardiac surgery.
Proper preoperative cardiac risk evaluation of such patients can guide the anesthesiologist to obtain further
cardiology evaluation, pursue perioperative rate control, and/or obtain further preoperative noninvasive
stress testing. Early PMI recognition and intervention, both medical and interventional, improve outcomes.
This chapter is a revision chapter authored by Ruben Azocar, MD.
Figure 27-1. Preoperative cardiac evaluation and care algorithm, adapted from ACC/AHA 2007 Guidelines
on Perioperative Cardiovascular Evaluation and Care of Noncardiac surgery.
Bibliography
1.
2.
3.
4.
Practice alert for the perioperative management of patients with coronary artery stents: a report by the
American Society of Anesthesiologists Committee on Standards and Practice Parameters. Anesthesiology
2009;110:22-3.
Adesanya AO, de Lemos JA, Greilich NB, Whitten CW. Management of perioperative myocardial infarction
in noncardiac surgical patients. Chest 2006;130:584-96.
Berger PB, Bellot V, Bell MR, et al. An immediate invasive strategy for the treatment of acute myocardial
infarction early after noncardiac surgery. Am J Cardiol 2001;87:1100-2, A6, A9.
Fleisher LA, Beckman JA, Brown KA, et al. ACC/AHA 2007 Guidelines on Perioperative Cardiovascular
Evaluation and Care for Noncardiac Surgery: Executive Summary: A Report of the American College of
Cardiology/American Heart Association Task Force on Practice Guidelines (Writing Committee to Revise
the 2002 Guidelines on Perioperative Cardiovascular Evaluation for Noncardiac Surgery) Developed in
Collaboration With the American Society of Echocardiography, American Society of Nuclear Cardiology,
Heart Rhythm Society, Society of Cardiovascular Anesthesiologists, Society for Cardiovascular
Angiography and Interventions, Society for Vascular Medicine and Biology, and Society for Vascular
Surgery. J Am Coll Cardiol 2007;50:1707-32.
147
SOCCA Residents Guide 2013
5.
6.
7.
8.
Acute Myocardial Infarction. The Cleveland Clinic, 2009. (Accessed August, 2010, at http://
www.clevelandclinicmeded.com/medicalpubs/diseasemanagement/cardiology/acute-myocardialinfarction/.)
Kopecky SL. Effect of beta blockers, particularly carvedilol, on reducing the risk of events after acute
myocardial infarction. Am J Cardiol 2006;98:1115-9.
Schwartz GG, Olsson AG. The case for intensive statin therapy after acute coronary syndromes. Am J
Cardiol 2005;96:45F-53F.
Serruys PW, Morice MC, Kappetein AP, et al. Percutaneous coronary intervention versus coronary-artery
bypass grafting for severe coronary artery disease. N Engl J Med 2009;360:961-72.
Questions
27-1. Perioperative myocardial infarction can be characterized by the following EXCEPT:
A. lt is usually silent
B. lt is usually NSTEMI
C. lt is often transmural
D. Enzymes values might be difficult to interpret
27-2. Provision of ACE inhibitors is recommended after acute Ml in the following situations EXCEPT:
A. To decrease afterload
B. In patients developing renal failure
C. Patients with low LVEF
D. Patient with HTN
27-3. In
A.
B.
C.
D.
relation to beta-blockers, the following statements are true EXCEPT:
They should be stopped after the acute phase of ischemia is resolved
Decrease incidence of ventricular fibrillation
Decrease myocardial oxygen consumption
Improve oxygen delivery
27-4. The goals of managing the patient with coronary ischemia include the following EXCEPT:
A. Restore coronary blood flow
B. Minimize extent of infarct
C. Decrease oxygen delivery
D. Decrease oxygen consumption
27-5. ln managing patient with coronary stents, the following statement are true EXCEPT:
A. lt is important to know the type of stent
B. The length of time since stent placement is an important piece of information
C. Once the patient has a stent, the risk of a new coronary event is close to zero
D. Management of antiplatelet therapy is of paramount importance to avoid complications
27-6. Patient risk factors for PMI include:
A. Uncontrolled hypertension
B. Age > 65
C. Presence of LBBB on EKG
D. Renal failure
148
SOCCA Residents Guide 2013
28. Valvular Heart Disease
Sriharsha D Subramanya, MD. and Jose Diaz-Gomez, MD
A 62 year old man was admitted to ICU with acute abdomen. CT abdomen
showed perforated appendicular mass. Additional history from the patient
was consistent with progressive worsening of shortness of breath over a
period of two years. He also had chest pain in the past but was able to
continue with normal activities. He had passed out twice in the past year.
Physical exam revealed a ejection systolic murmur in aortic area that was
radiating to the neck. A transthoracic echocardiogram showed dynamic
hypertrophied left ventricle and Doppler demonstrated severe aortic stenosis
with gradient of 70 mmHg. Patient is to undergo emergent laparotomy in the
setting of chronic decompensated valvular disorder.
Valvular heart disease in critically ill patients may present as acute decompensated heart failure due to
valvular dysfuntion secondary to ischemia or as chronic decompensation secondary to increased metabolic
demands. Patients with previously asymptomatic or compeansated stenotic lesions may acutely deteriorate
triggered by sepsis, anemia,pregnancy or hemorrhage.
The following discussion aims at providing basic understanding of the causes, diagnostic and therapeutic
interventions for valvular heart disorders that are frequently encountered in critically ill patients.
I.
Aortic Stenosis (AS)
A. Pathophysiology
1. AS results from thickening calcification and/ or fusion of aortic valve leaflets that produce
obstruction of left ventricular outflow tract. In younger patients AS develops on congenitally
bicuspid valves whereas in older patients, degenerative changes are more common.
2. Impaired cusp opening leads to pressure overload, compensatory LVH, reduced ventricular
compliance.
3. With excessive degree of LVH, wall stress is low and heart becomes hyperdynamic with high
EF. This finding portends worse prognosis.
B. Symptoms
1. Angina
2. Exertional dyspnea
3. Syncope
4. Congestive heart failure
C. Diagnosis
1. Physical findings of soft ejection murmur, diminished aortic component of S2, pulsus parvus
et tardus, brachio-radial delay
2. ECG: LV hypertrophy, strain pattern- T wave inversion & ST depression
3. Doppler Echocardiography to assess severity by measuring maximum jet velocity & mean
transvalvular gradient that allows calculation of aortic valve area. Direct measurement by
planimetry.
4. Left heart catheterization to calculate transvalvular gradient & valve area
D. Management
1. Medical: goal is to maintain cardiac output while preventing volume overload and pulmonary
edema. Only a bridge to surgical or percutaneous intervention.
2. Surgical: percutaneous balloon valvuloplasty used for palliation, can cause severe AI.
Transcatheter aortic valve implantation for critical aortic stenosis in patients not deemed
surgical candidates. Traditional open-heart surgery for most cases.
3. Minor changes in heart rate, preload and afterload can affect cardiac output significantly.
Invasive monitoring is essential to achieve the following goals. Relatively slow heart rate;
adequate intravascular volume status and systemic vascular resistance (diastolic arterial
pressure).
4. Perioperative hemodynamic goals for patients with AS:
149
SOCCA Residents Guide 2013
a)
5.
II.
Maintain sinus rhythm and avoid tachycardia (60-70 beats per minute). Patients with
concomitant aortic regurgitation might tolerate higher heart rates (80 -90 beats per
minute). Atrial contraction maximizes left ventricle preload. Thus immediate
cardioversion should be used in the setting of supraventicular arrhythmias causing
hemodynamic instability. Appropriate post-operative analgesia is important to prevent
tachycardia.
b) Avoid hypovolemia. It is difficult to assess an appropriate volume status with pulmonary
artery catheter because wedge pressure underestimates preload (decreased ventricular
compliance). Central venous pressure trend, IVC variability assessment, and judicious
fluid balance calculation might guide appropriate fluid administration.
c) Avoid acute arterial hypotension. It might precipitate ischemia and cardiac arrest. Alpha
agonist agents (phenylephrine) is the agent of choice in the setting of arterial hypotension
since it does maintain diastolic filling time associated with stable heart rate. If patient has
underlying decreased cardiac output (EF< 40%), norepinephrine might be advantageous
because mild to moderate increase in contractility associated beta-1 activity as well as
preservation of systemic vascular resistance. Patients receiving mechanical ventilation
need careful titration of sedatives and analgesic agents since they can aggravate arterial
hypotension or precipitate severe bradycardia - less than 40 beats per minute (propofol,
dexmedetomidine). The combination of potent opioids and these agents make even more
challenging postoperative sedation-analgesia management because their additive
hemodynamic effect can be detrimental at any time. It is mandatory to treat pain first and
then reassess patient for requirement of hypnotics to facilitate mechanically ventilatory
support and comfort.
Treatment of acute decompensation: myocardial ischemia is the most frequent cause followed
by atrial arrhythmias and congestive heart failure. Medical management should include
betablockers if there is no concomitant acute heart failure. Intra-aortic balloon pump
placement is indicated in state of shock or refractory myocardial ischemia as well. Point of
care echocardiography might be helpful to assess wall motion abnormalities and associated
LV function in this scenario. Patients suffering acute pulmonary edema secondary to
congestive heart failure present a particular dilemma for hemodynamic pharmacological
interevntion. Although the utilization of vasodilators have been considered contraindicated,
recent publications have showed the hemodynamic benefits titrating low doses of sodium
nitroprusside (10-150 mcg/min) in this patient population.
Aortic Regurgitation (AR)
A. Pathophysiology
1. Abnormalities of aortic valve leaflets( calcific degeneration,bicuspid valves, destruction from
endocarditis)
2. Aortic root dilation (idiopathic,aortic dissection)
3. Acute AR: from endocarditis or type A dissection leading to acute decompensation
4. Chronic AR: chronic pressure and volume overload causes progressive ventricular dilation.
B. Diagnosis
1. Physical findings include diastolic murmur, narrow pulse pressure, marfanoid features.
Classical signs of de musset, traube, Quincke and Corrigans pulse unreliable in acute
decompensation
2. Echocardiography
3. Thickened valve leaflets, flail leaflets, prolapsed, vegetation, aortic root dilation
4. Transesophageal echo for suspected thoracic aortic dissection
5. Regurgitant jet across the aortic valve on color flow Doppler
C. Management
1. Symptomatic patients with severe AR to undergo surgery irrespective of LV size & function.
Patients with severe chronic AR need medical optimization as feasible. Perioperative goals are
to decrease the regurgitant volume and maximize forward systemic flow. Thus relatively fast
heart rate (90-100 beats per minute) decrease the time in diastole and regurgitant volume
subsequently.
2. Endocarditis with hemodynamic compromise should promp urgent surgery
3. Asymptomatic patients should be followed closely. Surgery indicated with earliest signs of
decompensation. Chronic mild AR in the perioperative period is well tolerated.
150
SOCCA Residents Guide 2013
4.
5.
6.
III.
Severe AR from annuloaortic ectasia with enlarged aortic root- aorta be replaced if it exceeds
4.5 cm or less in case of Marfans.
IABP contraindicated.
Perioperative hemodynamic goals for patients with AS:
a) Avoid bradycardia and unnecessary use of vasopressors in any episode of arterial
hypotension.
b) Moderate decrease of afterload. If patient is receiving mechanical ventilation, utilization
of sedative agents such as propofol (afterload reduction) may be advantageous.
c) Avoid fluid overload. Patients with underlying severe AI and recovering from general
anesthesia can present pulmonary edema due to aggressive intraoperative fluid
resuscitation and afterload increase (postoperative pain and dissipation of inhalational
agents) in the postoperative period. Pulmonary artery catheter might be desirable in this
setting. However, wedge pressure overstimates LV filling pressure due to premature
mitral valve closure in early diastole. Postoperative fluid management may be guided as
it was described above in management of AS.
d) Treatment of acute decompensation: associated clinical conditions to acute AR iin
patients with native valve include aortic dissection and endocarditis. Cardiac tamponade
might case shock in cases of aortic dissection. Inodilators such as dobutamine and
milrinone are reasonable choices if patient is not hypotensive. Carefully titration of
betablocker is accepted if control of arterial hypertension is needed.
Mitral Stenosis (MS)
A. Pathophysiology
1. Exclusively consequence of rheumatic fever
2. Calcific stenosis is rare
3. Left atrial myxoma may mimic mitral stenosis
4. Rare causes- congenital, malignant carcinoid, lupus, amyloidosis
B. Diagnosis
1. Slowly progressive disease
2. Acute decompensation with increased hemodynamic demands
3. Pregnancy – heart failure is common presentation
4. Pulmonary edema triggered by atrial fibrillation
5. Physical findings- diastolic rumble with opening snap , ECG with AF
6. ECHO – to assess leaflet mobility, thickeneing, calcification.
C. Management
1. Rate control for AF and prolong diastolic filling time
2. Hemodynamic monitoring and ventilatory support in decompensated CHF
3. Intervention for patient in NYHA class III or IV with valve area < 1.5 cm2 or class II
symptoms when critical valve area < 1.0 cm2
4. Percutaneous ballon mitral valvuloplasty procedure of choice for moderate to sever MS if
valve morphology is favourable
5. Mitral valve replacement is indicated if the valve leaflets are calcified and fibrotic or with
significant subvalvular fusion
6. Perioperative hemodynamic goals for patients with AS:
a) Maintain sinus rhythm. Supraventricular tachyarrhythmias may precipitate pulmonary
edema and cardiovascular collapse. Up to 50% of patients with chronic MS develop atrial
fibrillation.
b) Maintain normal cardiac contractility and systemic vascular resistance since these
patients have a fixed cardiac output. The agent of choice in case of arterial hypotension is
phenylephrine because perfusion pressure must be assured and relative bradycardia
secondary to baroreceptor reflex might be beneficial in this clinical scenario.
c) Identify patients with pulmonary hypertension as a consequence of MS. This subgroup of
patients need better management of all factors that worsen pulmonary hypertension
(hypoxemia, hypercapnia, academia, and hypothermia). Some of these patients might
present right ventricular dysfunction. Inotropic support (epinephrine, milrinone or
dobutamine) and judicious fluid management are important in this setting. Patient who
needed nitric oxide as part of perioperative management for severe pulmonary
hypertension might present rebound pulmonary hypertension if it is discontinued
151
SOCCA Residents Guide 2013
abruptly. The use of PAC is potentially helpful to guide the effect of interventions on
pulmonary hypertension.
d) Fluid management in patients with MS mandates carefully monitoring. Suboptimal fluid
resuscitation decreases cardiac output dramatically and excessive administration of fluids
will precipitate pulmonary edema and acute right ventricular failure.
e) Anticoagulation needs to be resumed after risk of postoperative bleeding has decreased
(usually > 48-72h) in those patients who had preoperative indication (especially
intermittent atrial fibrillation, large left atria and spontaneous echo contrast at
echocardiogram) due to increased risk for intracavitary thrombus.
IV. Mitral Regurgitation (MR)
A. Pathophysiology
1. MR can result from abnormalities of annulus (dilatation), valve leaflets (myxomatous change,
leaflet damage from endocarditis, shrinkage from rheumatic disease), chordate tendinae
rupture or elongation, papillary muscle rupture or elongation.
2. Acute MR- from ischemia or infarction causes acute LV overload causing cardiogenic shock
(> 50% of patients with MI present some degree of MR).
3. Chronic MR- progressive increase in LV compliance followed by increase in LVEDV as LV
dilates
B. Diagnosis
1. High degree of clinical suspicion
2. Holosystolic murmur at apex radiating to axilla , lung rales, peripheral edema
3. ECHO- TEE is best technique to determine degree and nature of MR. Also asesse status of
LV function & provides an estimate of PA pressures
4. Blood cultures if suspicion of endocarditis
C. Management
1. For chronic MR- treat underlying cause such as fluid overlaod, anemia or infection &
optimize preload with diuretics and vasodilators
2. For acute MR- supportive with careful pre and afterload management, afterload reduction
with IABP
3. Coronary revascularization for regional wall motion abnormalities causing mitral valve
apparatus dysfunction
4. Surgery
a) Mitral valve repair for leaflet prolapsed
b) Valve resection and replacement for endocarditis with heart failure to avoid further
structural damage
c) Concomitant MAZE for paroxysmal AF
d) Mitral valve replacement is indicated when satisfactory repair cannot be accomplished
5. Perioperative hemodynamic goals for patients with MR: hemodymamic goals are similar to
AR patients. However the left ventricule has lower afterload in comparison to AR. Thus left
atrial dilation and pulmonary hypertension occurs progressively. Mitral regurgitation
deteriorates with afterload increase.
6. Acute decompensation: the pharmacological agents of choice are similar to AR (milrinone or
dobutamine). In contrast, the advantage of using IABP in this setting is unquestionable.
Diuretics might be considered for treatment of pulmonary edema
V.
Tricuspid Valve Disease
A. Pathophysiology
1. Tricuspid stenosis is very rare, associated with mitral stenosis secondary to rheumatic disease
2. Tricuspid regurgitation is most commonly functional in nature as a consequence of advanced
mitral valve disease leading to pulmonary hypertension, RV dilatation and tricuspid annular
dilatation. Thus, normal pulmonary arterial pressures are suggestive of structural tricuspid
valve disease or primary right ventricular dysfunction.
B. Diagnosis
1. Clinical findings include systolic murmur that increases with inspiration, prominent jugular
pulsation, occasionally pulsatile liver.
2. ECHO is confirmatory
C. Management
152
SOCCA Residents Guide 2013
1.
2.
3.
Tricuspid stenosis- surgery indicated for class III-IV symptoms
Tricuspid regurgitation- repair is indicated for severe TR with mitral valve disease that
requires mitral surgery. Replacement for cases not amenable for repair.
Perioperative hemodynamic goals for patients with TR and right ventricular dysfunction:
a) Maintain appropriate preload: a reasonable goal can be to follow closely the trend of
CVP and maintain it around 15 mmHg. Fluid overload can aggravate TR and RV
dysfunction.
b) Maintain adequate RV myocardial perfusion pressure: the RV receives perfusion both in
systole and diastole. It is important to avoid hypotension so myocardial ischemia is
prevented.
c) Decrease RV afterload: manipulation of all variables that affect pulmonary vascular
resistance is helpful to assure better right ventricular performance (PaO2, PCO2, pH,
hypothermia, unnecessary use of vasopressors)
d) Maintain normal to high heart rates (> 80’s-90’s beats per minute): and treat
bradyarryhtmias aggressively: low heart rates are inappropriate for state of shock and
regurgitation tends to be worse in patients with lower heart rates.
e) Increase RV contractility: either use of epinephrine or milrinone in combination with
norepinephrine are adequate choices for inotropic/vasopressor support in severe RV
dysfunction.
Table 28-1: Perioperative hemodynamic considerations of most
common valvular heart disease
Condition Heart rate Contractility
Preload
Afterload
Goals
AS
60-70
MS
Normal
AR
90 - 100
MR
90 – 100
TR
80-100
Normal to
increased
Normal
Normal to
increased
Normal to
increased
Increased
Increased
Increased
Normal to
increased
Increased
Increased
Increased
Decreased
Increased
Decreased
Maintain Sinus
rhythm
Maintain Sinus
rhythm
Decreased
Associated LV
dysfunction
Pulmonary
HTN
Agents
Phenylephrine
Norepinephrine
Dobutamine
Milrinone
Epinephrine
(inotropic dose)
Epinephrine or
Milrinone
+ Norepinephrine
KEY POINTS
Understanding the physiopathology of major heart valve diseases is crucial to prevent perioperative
complication and optimize postoperative management within the ICU.
Implementation of invasive monitoring and pharmacological interventions allow a tight hemodynamic
control of each lesion or combination of them.
Other perioperative factors including analgesia, oxygen therapy, fluid resuscitation, anemia,
hypercoagulability, and nosocomial infections deserve continuous reassessment since they may contribute
to hemodynamic instability in patients with valve heart disease.
153
SOCCA Residents Guide 2013
Table 28-2: Ten perioperative factors to achieve hemodynamic stability
in valvular heart disease
Adequate invasive hemodynamic monitoring
Optimal postoperative pain management
Correction of anemia
Avoidance of hypercoagulability
Reassessment of intravascular volume shifts
Consideration of hemodynamic effects of neuraxial anesthesia/analgesia
Aggressive treatment of atrial fibrillation
Appropriate recognition and prompt treatment of nosocomial infections
Continuation of oxygen therapy throughout postoperative period
Use of beta-blockers as needed in myocardial ischemia
References
1.
2.
3.
4.
5.
6.
7.
Bonow RO, Carabello BA, Chatterjee K et al 2008 focused update incorporated in to ACC/AHA 2006
guidelines for management of pts with valvular heart disease
Moore RA, Martin DE. Anesthetic management for treatment of Valvular heart disease
Mohan SB, Stouffer GA. Timing of surgery in Aortic stenosis. Cardiovascular med 2006
Carabello BA The current therapy for Mitral regurgitation. J Am Coll of Card 2008
Gillinov Am, Blackstone EH, Nowicki ER Valve repair vs. replacement for degenerative mitral valve
disease. J Thorac Cardiovas surg 2008
Manual of peri-operative care in Adult Cardiac Surgery. Robert Bojar
Khot UN, Novaro GM, Popovic ZB, et al: Nitroprusside in critically ill patients with left ventricular
dysfunction nad aortic stenosis. N Engl J Med 348:1756-1763, 2003
QUESTIONS
28-1 A 78 year-old male with recent diagnosis of atrial fibrillation. He is admitted to the ICU after having an episode of
aspiration during induction of anesthesia for a small bowel resection. His echocardiogra1m shows severe mitral stenosis.
What of the following perioperative conditions would be MOST important predictive factor for complication in the ICU:
A. Endocarditis
B. Requirement inotropic support
C. Pulmonary hypertension
D. Decrease of afterload
E. Inappropriate bradycardia
28-2 Which of the following treatments would be LEAST useful in treatment of the acute mitral regurgitation after
myocardial infarction following a aorto-bifemoral bypass
A. Milrinone infusion
B. Intra-aortic ballon pump
C. Epinephrine infusion
D. Sodium nitroprusside
154
SOCCA Residents Guide 2013
28-3 Which of the following heart valve disorders if patient has a normal ejection fraction might represent worse
perioperative outcome if the EKG shows P wave flattening, widening of the QRS complex, peaked T wave?
A. Tricuspid regurgitation
B. Mitral regurgitation
C. Aortic stenosis
D. Pumonary regurgitation
155
SOCCA Residents Guide 2013
29. Systemic Inflammatory Response Syndrome (SIRS), Sepsis
and Multiple Organ Dysfunction Syndrome (MODS)
Shiva Birdi MD, Marc J. Popovich MD
A 67 year-old morbidly obese male with a 4 day history of fever, chills and
severe abdominal pain secondary to a strangulated umbilical hernia is
admitted to the intensive care unit status post exploratory laparotomy,
resection of necrotic small bowel and washout. During the operation, the
patient receives large volume fluid resuscitation and intermittent
vasopressors and upon admission to the ICU he remains sedated, intubated
and on a low dose norepinephrine infusion. Over the course of next 48 hrs,
he develops increasing ventilator requirements secondary to hypoxemia, as
well as worsening hypotension, oliguria and lactic acidosis in spite of
aggressive fluid resuscitation, broad-spectrum antibiotics and multiple
vasopressor infusions.
OUTLINE:
After reading this chapter you should be able to:
• Define the continuum of the sepsis from systemic inflammatory response to multiple organ dysfunction.
• Discuss the epidemiology and pathogenesis of SIRS, sepsis and multiple organ dysfunction syndrome
• Provide the therapeutic approach for management of sepsis and multiple organ dysfunction syndrome
• Discuss future directions in diagnosis and management of septic patients.
DEFINITIONS:
The first “consensus conference” to define the continuum of sepsis was conducted in 1991 by the American
College of Chest Physicians (ACCP) and Society of Critical Care Medicine (SCCM). Prior to this
conference, there was no established framework to define the systemic inflammatory response to infection.
The goal of the conference was to help provide broad definitions that would standardize the classification
as well as improve the study, diagnosis and treatment of sepsis.
Their statement, which was subsequently published in 1992, introduced the term “SIRS” or systemic
inflammatory response syndrome. Based on systemic activation of the immune response, SIRS was
defined as presence of at least two of the following criteria:
•
•
•
•
Body Temperature >38в„ѓ or < 36в„ѓ
Heart Rate > 90 beats per minute
Respiratory Rate > 20 breaths per minute or PaCO2 < 32 torr
White blood cell count > 12,000/mm3 or < 4,000/mm3 or >10% bands
Infection was defined as a pathological process caused by invasion of normally sterile tissue, fluid or body
cavity by pathogenic microorganisms. Sepsis was defined as SIRS (as defined above) in the presence of
infection. Severe Sepsis was defined as sepsis associated with organ dysfunction, hypotension or
abnormalities associated with hypoperfusion (including but not limited to lactic acidosis, encephalopathy
and oliguria). Septic Shock was defined as arterial hypotension (SBP<90 torr or a drop > 40 torr in absence
of another etiology) in spite of other causes of hypotension and in spite of adequate fluid resuscitation.
These definitions were most recently revisited in 2001 at the SCCM / ESIM / ACCP/ ATS / SIS
International Sepsis Definitions Conference. Overall, they were largely upheld as clinically useful and
relevant. However, the panel suggested that the SIRS criteria proposed in the 1992 statement was too
sensitive and non-specific. As a result, they suggest a more clinically relevant approach to the diagnosis of
sepsis using an expanded list of signs and symptoms might better assist the bedside clinician in diagnosis.
The underlying principles identified by the first conference were maintained but the new criteria were
based on several systemic manifestations of sepsis. (Table 1).
156
SOCCA Residents Guide 2013
In addition, the panel provided novel insight into a hypothetical model for the staging of sepsis termed
PIRO (Predisposing conditions, the nature and extent of Infection, nature and magnitude of host Response,
degree of concomitant Organ dysfunction) (Table 2). Using these four staging categories would allow better
stratification by assessment of the baseline risk of adverse outcomes as well as assessment of patients’
potential response to therapy. However, in spite of these favorable attributes, the PIRO system has not been
directly tested in clinical trial designs.
Multiple Organ Dysfunction Syndrome (MODS) is defined as the presence of altered organ dysfunction in
multiple systems such that homeostasis cannot be achieved without intervention. MODS may result from
conditions that cause a systemic derangement in the normal physiological balance in the body such as
sepsis, trauma, massive acute blood loss, severe burns or hemorrhagic pancreatitis. MODS represents the
final stage in the continuum of disease progression in the septic patient.
PATHOPHYSIOLOGY:
EPIDEMIOLOGY
Sepsis is the second leading cause of death in non-cardiac ICU patients. It is also the 10th leading cause of
all-cause mortality in the United States. The economic and social burden from sepsis is overwhelming.
Recent estimates of overall cost of sepsis near $17 billion. Even the patients that survive suffer from a
markedly reduced quality of life. Until the early 1990s, the true incidence of sepsis was difficult to
accurately quantify. Mortality rates raged from 15% for patients with sepsis to 40-60% for patients in
septic shock. Much of this was due to the lack of agreement on consensus definitions in the medical
community. Recent data from a large-scale epidemiological study provides some clarity. Using ICD-9-CM
codes, Martin et al. reviewed 750 million admissions over a 22-year period and found the annual incidence
of sepsis to be 660,000 cases per year. They also found that since 1979 the annualized incidence of sepsis
has risen near three-fold. As a result, the number of deaths (total mortality) from sepsis also increased
proportionally. However, they did find that the in-hospital mortality rate declined from 28% to 18% over
these two decades.
Demographic data suggest men are at greater risk than women. Black men are at greater risk of death than
white men. The degree and presence of organ failure (i.e. severe sepsis, septic shock, MODS) has an
additive effect on mortality. Septic patients that develop greater than three failing organs have a nearly fourto-five fold greater risk of dying when compared to patients without organ failure (70% to 15%,
respectively). Several prediction models (APACHE, MODS, MPM, etc.) have been developed to quantify
degree of organ failure as well as risk of morbidity and mortality. However, their utility is limited to use in
research trials for matching cohorts and subsequent scientific study.
In spite of therapeutic progress, it appears that the burden of sepsis will continue to worsen. Clinicians are
faced with an aging population with more complex medical diseases along with an increasing number of
immuno-compromised patients in an era of increasing organ transplantation and invasive procedures. The
organisms too are evolving near the limits of our current antimicrobial therapy with the emergence of
multi-drug resistant (MRDO) colonies.
In order to continue to fight this disease we must understand the pathophysiology and clinical patterns such
that we can act early and prevent the cascade towards multiple organ failure.
PATHOGENESIS
The Cytokine Hypothesis
Cytokines are mediators that are produced and secreted by inflammatory as well as endothelial cells. They
bind to receptors on the cell surface to activate intracellular signaling systems, often leading to production
of new proteins. There is a subset of cytokines, called pro-inflammatory cytokines, which are believed to be
the mediators involved in sepsis. The most notable of these are interleukin-1 (IL-1) and tumor necrosis
factor-alpha (TNF-О±), which are released from monocytes and macrophages. When released into
circulation, both these mediators lead to widespread microvascular and endothelial injury. In animal
models, they lead to a hemodynamic state that is essentially indistinguishable from septic shock when
injected directly into bloodstream of animal models.
157
SOCCA Residents Guide 2013
Hence it is hypothesized that such cytokines are released in response to infection or endotoxin leading to an
overproduction of pro-inflammatory. This results in prolonged and uncontrollable vasodilation and damage
to endogenous tissue by primed macrophages and neutrophils. TNF-О± activates production of IL-1 and IL-6
that further perpetuate the injury. It would seem reasonable that introducing inhibitors directed towards
these mediators would halt the disease and improve outcomes in septic subjects. Unfortunately, this has not
been shown in human and animal trials as both antibodies towards TNF-О± and IL-6 receptor antagonists
have failed to show a mortality benefit. Even anti-endotoxin therapies have not improved outcomes in
large trials. This likely shows that sepsis is a systemic perturbance in which targeting a single point or
mediator is unlikely to succeed in halting the inflammatory cascade.
There also appears to be a role of the compensatory anti-inflammatory response system (CARS) in the
pathogenesis of sepsis. Following the initial inflammatory response, CARS leads to production of antiinflammatory cytokines like IL-10, Il-4, PGE2, inhibitors of TNFО± and IL-1. This essentially creates an
immunosuppresed that places the patient at a higher risk of death and further organ dysfunction.
Microcirculatory Hypothesis
This hypothesis suggests that sepsis causes pan-microcirculatory failure leading to a low flow state in
which the end organ tissue is deprived of oxygen and nutrient supply. This low flow state may be secondary
to hypotension, hypovolemia or reflect capillary vascular congestion caused by microemboli. It is believed
that the endothelium may be directly affected and perpetuate the cell-mediated damage process.
Reperfusion of tissues also is an important component of the injury process as a result of free radical
production.
Gut Hypothesis
This hypothesis suggests that the occult translocation of bacteria or endotoxin form the gut into the
circulation mediates the disease. This exposure to intestinal flora leads to activation of cytokine pathways
(as described above), causing systemic damage. In experimental models, multiple insults are required for
such translocation to occur.
Two-Hit Hypothesis
An initial insult (“hit”) primes the patient, however a second “hit” is required to precipitate MODS. The
initial insult may act by activating the immune system, disrupting the GI mucosal barrier facilitating
translocation or free radical formation in the gut.
Transcriptional Hypothesis
Genetic factors are certainly important in the susceptibility to sepsis. In addition, it is hypothesized that
they may also play a role in the pathogenesis of disease. The loss of ability to express certain cell specific
proteins leads to cellular de-differentiation and an impairment in function. In animal models, loss of organ
specific transcription has been shown in hepatocytes and pneumocytes. Accelerated apoptosis or
programmed cell death, is a pathologic event mediated by heat shock proteins that may be pivotal in the
development of acute organ failure.
The multiple hypotheses above reflect the scientific study of both the molecular mechanisms of sepsis and
the macroscopic appearance of the disease. In spite of the variety on the proposed mechanisms it is clear
that sepsis is an inflammatory process that effects the entire body’s organ systems. However each organ is
independently susceptible and has its own pattern of disease. We will present these clinical patterns of
sepsis and then discuss organ specific patterns of dysfunction. As you will notice, certain organs are
certainly more susceptible to injury than others.
CLINICAL PATTERNS:
There are essentially two basic clinical disease patterns resulting from sepsis:
A. Primary respiratory pattern with MODS as a terminal event
1. Development of cardiopulmonary dysfunction (ARDS) without involvement of other organ
systems within two to five days following insult
2. Triggers for this pattern of MODS most often involve direct lung injury or infection (i.e.
pneumonia, aspiration, isolated pulmonary trauma, respiratory failure in COPD)
3. Continued cardiopulmonary dysfunction for about three weeks
4. After three weeks, either
a) Resolution of pulmonary dysfunction
158
SOCCA Residents Guide 2013
b) Rapid development of dysfunction in other organ systems, most often kidney and liver
c) Process continues for about one more week with patients either, recovering (60%) or
expiring (40%)
B. MODS as a primary disease entity
1. Development of cardiopulmonary dysfunction (ARDS) without involvement of other organ
systems within two to five days following insult
2. Progressive organ dysfunction, most often kidney and liver, within three to five days
3. Continued dysfunction for about three weeks with patients either, recovering (30%) or
continued dysfunction in other systems leading to death (70%)
PATHOPHYSIOLOGY OF ORGAN DYSFUNCTION:
By definition, patients with sepsis present with signs of systemic inflammation as a result of infection. This
inflammation is characterized by pan-microcirculatory and endothelial injury that can effect all organ
systems in the human body.
Pulmonary
Pulmonary dysfunction can vary from tachypnea and hypoxemia to acute lung injury and severe ARDS.
The degree of disease may be worse in patients with underlying lung disease such as COPD. This also may
eventually lead to chronic respiratory failure and prolonged mechanical ventilation.
Cardiovascular
The cardiovascular manifestations of sepsis are characteristically arterial hypotension, tachycardia and
systemic vasodilation. The hemodynamic profile of sepsis is classically described as hyperdynamic because
of a high cardiac output state in the setting of low systemic vascular resistance (vasodilation). In presence
of hypovolemia, this vasodilation leads to septic shock with arterial hypotension (<65 mm Hg) in spite of
adequate fluid resuscitation. Both arterial and venous vasodilation contribute to the decreased preload in
septic shock. This vasodilation may be refractory to vasopressors due to a defect in endothelial and vascular
smooth muscle responses to catecholamines, perhaps as a result of overproduction of nitric oxide.
Additionally, the microvascular injury in sepsis leads to global capillary leak that causing further reduction
in preload.
Myocardial depression can be seen as reflected by decreased stroke volume, decreased ejection fraction,
and increased filling pressures. This is mediated by inflammatory cytokines such as TNF-О±, IL-1 as well as
nitric oxide (NO). Both the right and left ventricular function may be compromised. Direct endotoxin
mediated myocardial depression is also believed to play a role.
The shock profile of sepsis is also described as “distributive.” This refers to a state in which global and
tissue hypoperfusion exists because of redistribution of blood flow to the tissues. As a result of
hypotension, the blood flow is shunted away from non-vital to vital organs (i.e. splanchnic hypoperfusion).
In addition, microvascular injury, congestion and thrombosis all limit blood flow to tissues.
Renal
The kidney is especially sensitive to the deleterious effects from systemic inflammation and “redistribution” of blood flow. The clinical hallmarks of renal dysfunction in sepsis are a reduction in
creatinine clearance and oliguria. The initial pathologic changes are typical of acute tubular injury
including basement membrane disruption, patchy necrosis, interstitial edema, and formation of tubular
casts. Injury to the distal collecting system manifests itself in the inability to concentrate urine. Decreased
excretion of potassium and acid resulting in tubular acidosis can also occur. Often, this acute kidney injury
progresses to failure and may require renal replacement therapy.
Gastrointestinal
Redistribution of blood flow to “vital” organs (heart and brain) places the gut at increased risk of injury.
Resultant splanchnic ischemia, especially in setting of septic shock, may lead to catastrophic outcomes. In
addition, intestinal ischemia and bowel wall edema may occur secondary to capillary leak. This can lead to
decreased gut motility and adynamic ileus. Sepsis-related-stress can cause mucosal breakdown and
ischemia that can lead to gastrointestinal bleeding. This rapid injury can also lead to malabsorption states
placing patients at increased risk of malnutrition.
159
SOCCA Residents Guide 2013
Hepatic
Hypovolemia from increased membrane permeability and splanchnic pooling cause a reduction in hepatic
blood flow. Microvascular injury from microthrombi leads to decreased sinusoidal blood flow. Liver
dysfunction may present with mild elevation in transaminases and a clinical picture of cholestasis. The
degree of liver injury may be severe and can even cause massive transaminitis similar to “shock liver.” The
metabolic function of the liver can also be effected. Inhibition of the cytochrome p-450 pathway can inhibit
drug metabolism and cytokine biotransformation. Synthesis of new proteins can be greatly reduced. Other
laboratory abnormalities include elevated prothrombin time and increased serum bilirubin.
Metabolic
Sepsis is a catabolic state. In addition to the reduction in hepatic protein synthesis, there is also a
stimulation of protein catabolism. This is achieved by IL-1, which mediates hypoalbuminemia as protein
synthesis shifts form albumin to acute phase reactant proteins. High levels of endogenous catecholamines
inhibit insulin release as well as increase gluconeogenesis causing elevated blood glucose levels. Failure to
clear exogenous long chain fatty acids also results in ectopic fat deposition. Amino acid deamination in the
Krebs cycle may also result in pre-renal azotemia contributing to the protein load on already injured
kidneys.
At the cellular level, there is evidence of “cytopathic hypoxia.” Cytopathic hypoxia occurs when there is an
impaired utilization of delivered oxygen. Thus, even in the setting of adequate flow to an organ, the cells
are unable to adequately utilize the oxygen for aerobic respiration and energy production. This
mitochondrial dysfunction is though to play a key role in the organ dysfunction in sepsis.
Central Nervous System
Most common abnormality is encephalopathy that may vary from mild confusion and agitation to a
comatosed state. Certainly, the degree of encephalopathy correlates to the severity of the underlying
disease. It is also an independent predictor of mortality from sepsis.
Hematologic/Coagulation
Sepsis can lead to bone marrow suppression. Patients can develop coagulopathy and thrombocytopenia
secondary to decreased production in the marrow as well as from the consumption of factors within the
microemboli in circulation. Findings are usually subclinical with mild elevation in bleeding parameters
however, in severe cases, fatal disseminated intravascular coagulation (DIC) can occur.
As we have seen, sepsis is a systemic disease. The initial insult, whether it is from pneumonia or a rusty
nail penetrating the foot, can lead to a cascade of inflammatory, intracellular and microvascular events that
may cause a dysfunction of nearly every organ in the body. Certain organs such as the kidney, liver and gut
are especially at risk since the homeostatic mechanisms in the body lead to redistribution of blood flow
away from them towards the more “vital” organs. As we will see in the next section, there are several
strategies to management of a septic patient that can help attenuate some of this injury however certain
therapies can place patients at increased risk of worsening organ dysfunction.
MANAGEMENT:
An international group of experts, from 11 recognized organizations published the first guidelines for the
management of patients with severe sepsis and septic shock in 2004. These Surviving Sepsis Campaign
(SSC) guidelines were aimed towards bedside clinicians to help improve patient outcomes and increase
awareness of sepsis. Recently, in 2008, these guidelines were revised using a new methodology system to
include the latest evidence based medicine. These guidelines, although left to the discretion of the clinician,
form much of the basis of management of patients with severe sepsis and septic shock.
In general, the goal is to provide early and effective resuscitation to stabilize the hemodynamic system all
the while allowing antibiotic therapies and source control to eradicate the infection. Once a patient
develops signs of severe sepsis, septic shock or MODS, it is recommended that the patient be transferred to
a setting where experts in critical care medicine can provide further care.
Initiation of invasive hemodynamic monitoring is recommended in patients with evidence of sepsis-
160
SOCCA Residents Guide 2013
induced hypotension. Standard non-invasive hemodynamic monitors (pulse-oxymetry and oscillometric
blood pressure recordings) can be largely inaccurate in hypotensive states (i.e. shock). Admission to the
intensive care unit can optimize the setting for placement of such devices. Placement of an arterial catheter
for monitoring for hemodynamics and to follow serial arterial blood gas measurements is recommended.
INITIAL RESUSCITATION (first 6 hours)
aka Early Goal Directed Therapy (EGDT)
After placement of appropriate hemodynamic monitors, resuscitation should begin immediately in patients
with persistent hypotension in spite of fluid administration or a serum lactate > 4 mmol/L. The resuscitation
should not be delayed for admission to a critical care unit.
In 2001, Rivers et al. published a landmark randomized control trial of patients with severe sepsis or septic
shock admitted to the emergency room who received EGDT for 6 hrs from enrollment (which was <1 hr of
presentation). The resuscitation was guided with the use of mean arterial pressure (MAP), central venous
pressure (CVP), and urine output. In addition, central venous oxygenation was measured on a continuous
basis (SVO2) in the treatment group. In general, a low SVO2 (<70%) represented decreased oxygen
delivery to the tissues and was a surrogate for mixed-venous oxygenation (CvO2) in patients without a
pulmonary artery (PA) catheter.
The four main goals of early resuscitation were:
• MAP ≥65 mmHg
• Rapid fluid administration to CVP 8-12 mm Hg (higher 12-15 mm Hg if mechanical ventilated or
pre-existing decrease ventricular compliance)
• Urine output ≥ 0.5 ml/kg/hr
• If target SVO2 of ≥ 70% was not achieved with above and further fluid infusion then,
• PRBC transfusion was given to Hct ≥ 30% and/or,
• Dobutamine infusion started to target SVO2 ≥70%
The SSC guidelines adopted this therapeutic strategy since the patients in the EGTD group had significant
reduction in 28-day mortality when compared to patients receiving standard care.
ANTIBIOTIC THERAPY AND SOURCE CONTROL
As soon as sepsis is suspected, appropriate cultures should be drawn prior to administration of antibiotic
therapy, if possible. Early appropriate antibiotic therapy has been shown to improve patient survival. If preexisting catheters are present then one of the cultures should be drawn from the device. Initially, broadspectrum antibiotics should be started depending on the local susceptibility patterns and with good
penetration into presumed source. Coverage for both gram-positive and gram-negative organisms is
recommended. Once the infection is identified, the antibiotic therapy should be streamlined. Daily
evaluation for possible discontinuation of antibiotic therapy should be performed. Consider stopping
treatment after 72 hrs if no source is identified to prevent risk of super-infection or increasing resistance.
Radiographic studies (CT, XR, MRI) should be performed early to identify a source of infection, if safe to
do so. Local sources of infection, such as an abscess, should be controlled and removal of infected devices
should be done as rapidly as possible. Surgical debridement should be performed early if possible. Most
common sources of infection in septic patients are pulmonary, urinary tract, intra-abdominal, vascular
access devices and sinusitis. Some uncommon sources include acalculous cholecystitis, fungal infections,
vaginitis/endometriosis, and Clostridium difficile colitis among others. It is important to note that in over
50% of patients, no source of sepsis can be identified.
HEMODYNAMIC OPTIMIZATION
Fluid Resuscitation
Goals of fluid resuscitation are to maintain MAP ≥ 65 and CVP ≥ 8 (≥ 12 if on MV). Aggressive fluid
challenges with either crystalloid (1000 cc) or colloid (300 to 500 cc) should be given rapidly to any
patients with evidence of decreased perfusion. Several fluid boluses may be needed in patients with septic
shock. Fluid resuscitation techniques should be continued as long as hemodynamic improvement is being
achieved.
161
SOCCA Residents Guide 2013
Vasopressors
Administration of vasopressors should be considered if hypotension persists in spite of aggressive fluid
resuscitation. Initial vasopressor of choice is either norepinephrine or dopamine. Norepinephrine, however,
may provide an appropriate hemodynamic profile with less exacerbation of tachycardia than dopamine.
Dopamine may be preferred in a bradycardic patient with septic shock. Pure alpha-agonists like
phenylephrine would support hemodynamics but may lead to overt peripheral vasoconstriction that may be
deleterious to cardiac output and splanchnic perfusion. Vasopressin (0.03 units/min) may be added to
norepinephrine to help achieve hemodynamic goals. The benefit of vasopressin in the presence of a
“relative vasopressin deficiency” in later septic shock remains unclear.
The use of low dose dopamine for renal protection alone is no longer standard of care or recommended by
SSC guidelines. Also of note, use of bicarbonate therapy purely to optimize hemodynamics or vasopressor
responsiveness is of no benefit. This notion has been debunked and should removed from critical care
practice.
Inotropic Therapy
Cardiac output may also be decreased in sepsis secondary to sepsis-induced myocardial depression or in a
patient with pre-existing myocardial dysfunction. If there is evidence of impaired cardiac output, consider
inotropic support (dobutamine is preferred). If inotropic infusion is started, it is recommended to monitor
cardiac output either by placement of a PA catheter or via surrogates of cardiac output (i.e. SVO2, end
organ function, tissue oxygenation parameters). Inotropes should not be titrated to supranormal cardiac
output levels. This practice has been shown in multiple large RCTs to be of no clinical benefit and likely to
place patients at increased risk.
OTHER SUPPORTIVE THERAPIES
Steroids
Use of corticosteroids continues to be a topic of contentious debate in the critical care community. Septic
shock induces a state of relative adrenal insufficiency as evidenced by a post ACTH cortisol increase of
<10 mcg/dL in such patients. However, an ACTH stimulation test (250mcg) is not required to diagnose this
insufficiency. Hence it is recommended that septic patients who are poorly responsive to both fluid
resuscitation and vasopressors receive hydrocortisone supplementation. The dose of hydrocortisone should
be ≤ 300 mg/day. Much higher doses increase risk of secondary infections and may in fact worsen patient
outcomes. The CORTICUS trial is the largest multicenter RCT to date to study use of steroids in severe
sepsis and septic shock. This trial failed to show a mortality benefit in patients treated with steroids
however did show a faster resolution of shock in treated patients. In spite of this finding, the recent SSC
guidelines support use of steroids patients with septic shock unresponsive to fluids and vasopressors. Given
the evidence, the therapy should be weaned when the vasopressors are no longer required and should not be
used for treatment of sepsis in the absence of shock.
Recombinant Activated Protein C (rhAPC)
Protein C is a Vitamin-K dependent factor that is synthesized in the liver. It is activated by binding to
thrombomodulin on the endothelial surface. Once activated, APC can reduce the production of thrombin
(anti-coagulant) as well as decrease production of pro-inflammatory cytokines (anti-inflammatory). Based
on results of large RCTs, initial SSC guidelines recommended that rhAPC should be considered in patients
with septic shock requiring vasopressors or in patients with three or more organ dysfunction (APACHE II ≥
25). Since APC also stimulates endogenous fibrinolysis (anti-fibrinolytic), its use is contraindicated in
patients who are considered to have a high risk of bleeding. Serious bleeding complications can occur from
use of rhAPC including intracranial hemorrhage. Because of this as well as a lack of efficacy in broader
clinical use, it has been withdrawn from the market by the manufacturer.
Blood Product Administration
Transfusion of red blood cells is certainly appropriate in the setting of massive active bleeding or severe
anemia with evidence of decreased oxygen delivery (organ ischemia, lactic acidosis). In septic shock, even
without evidence of above, transfusion can be reasonably considered if the Hb <7 g/dL up to a range of 7-9
162
SOCCA Residents Guide 2013
g/dL. In general, transfusion triggers should be avoided and the decision to treat should be individualized.
Transfusion of blood products (FFP, platelets, cryoprecipitate) should only be considered if there is ongoing
active bleeding or prior to an intervention (surgery, vascular catheter placement). Confirmatory laboratory
testing is recommended to guide therapy.
Glycemic Control and Intensive Insulin Therapy
Hyperglycemia commonly occurs in septic patients. In addition, the normal homeostatic mechanism for
glycemic control is altered and inadequate. Over the past decade there has been a paradigm shift in terms of
how “tightly” the elevated glucose should be controlled. Initial recommendations considered
“tight” (80-110 mg/dL) control to be most beneficial to critically ill patients with severe sepsis. However,
based on a recent largest-to-date RCT (NICE-SUGAR), the risk of hypoglycemic complications in tightly
controlled patients is too high to be considered safe. As a result, insulin therapy should be titrated to blood
glucose levels <150 mg/dL when levels exceed 180 g/dL.
Mechanical Ventilation
Septic patients who are mechanically ventilated should always have the head of bed elevated > 30 degrees
to prevent risk of aspiration and reduce the risk of ventilator associated pneumonia. If ALI/ARDS is
present, a lung protective ventilation technique (tidal volume 4-6 cc/kg with plateau pressures < 30 cm
H2O) should be employed. If plausible, a daily sedation interruption associated with a spontaneous
breathing trial should be performed to assess for candidacy of extubation.
Prophylactic Therapy
Stress ulcer prophylaxis should be given to all patients with severe sepsis or septic shock. The use of either
H2 receptor blockers or proton pump inhibitors reduces the risk of gastrointestinal hemorrhage. Also,
enteral nutrition should be used whenever possible as this may reduce bacterial translocation in the GI tract.
Deep Vein Thrombosis prophylaxis should be provided in the form of either mechanical (compression
devices) or pharmacological (LMWH/ SQ Heparin) to all septic patients.
In addition, use of prophylactic therapies should be monitored for necessity on a daily basis.
Table 29-1: Diagnostic Criteria for Sepsis
Infectiona
documented or suspected
AND some of the following
General
Fever (core temperature > 38.3в„ѓ)
Hypothermia (core temperature < 36в„ѓ)
Heart rate > 90 or over 2 SD above normal for age
Tachypnea
Altered mental status
Significant edema or positive fluid balance (>20ml/kg in 24 hours)
Hyperglycemia (glucose > 120 md/dL or 7.7 mmol/L) in absence of
diabetes
Inflammatory
Leukocytosis (WBC > 12.0/mL)
Leukopenia WBC < 4.0/mL)
163
SOCCA Residents Guide 2013
Inflammatory
Table 29-1: Diagnostic Criteria for Sepsis
Normal WBC with over 10% immature forms
Plasma C-reactive Protein (CRP) > 2 SD over normal
Plasma procalcitonin > 2 SD over normal
Hemodynamic
Arterial hypotension (SBP <90 or MAP <70 or decrease in > 40 from
baseline (adults) or > 2 SD below normal for age
SVO2 > 70% (adults onlyb)
Cardiac Index > 3.5L/min/M2 (adults onlyb)
Organ dysfunction
Arterial hypoxemia (PaO2/FiO2 < 300)
Acute oliguria (Urine < 0.5ml/kg/hr for at least 2 hours)
Creatinine increase >0.5 mg/dL
Coagulation abnormalities (INR > 1.5 or PTT > 60 sec)
Ileus (absent bowel sounds)
Thrombocytopenia (platelets < 100)
Hyperbilirubinemia (total bilirubin > 4.0 mg/dL or 70 mmol/L)
Tissue perfusion
Hyperlactemia (lactate > 1 mmol/L)
Decreased capillary refilling or mottling
WBC: white blood cells, SBP: systolic blood pressure, MAP: mean arterial pressure, SVO2:
mixed venous oxygen saturation, INR: international normalized ratio, PTT: partial
thromboplastin time, SD: standard deviation
a: Infection defined as a pathologic process induced by a microorganism
b: SVO2 > 70% and CI > 3.5 are normal in children (normal ranges 75-80% and 3.5-5.5
respectively) so these should not be used as diagnostic criteria
c: Pediatric criteria include signs of infection, altered temperature (> 38.5в„ѓ or < 35.0в„ѓ),
tachycardia (although may be absent in hypothermia), and at least one of following organ
dysfunction criteria: hypoxemia, altered mental status, hyperlactemia or bounding pulses.
Adapted from Levy MM, Fink MP, Marshall JC, et al. 2001 SCCM/ESICM/ACCP/ATS/SIS.
International Sepsis Definitions Conference. Crit Care Med. 2003; 31(4):1250-1256.
Table 29-2 PIRO Model of
Sepsis Syndrome
P = predisposition of patients to
respond in different ways to
infection, including genetics and
preexisting disease
164
SOCCA Residents Guide 2013
Table 29-2 PIRO Model of
Sepsis Syndrome
I = infection, culture and sensitivity
profiles of the infecting
microorganism(s)
R = response: physiologic response
with signs and symptoms of sepsis
may vary, as may serum levels of
inflammatory markers
O = organ dysfunction: qualtifying
failing organs by number or with a
scoring system (i.e., MODS, SOFA)
FUTURE DIRECTIONS:
Recent research points towards techniques for early detection and diagnosis of sepsis. Some of the current
focus is on the serologic detection of certain biochemical markers of sepsis such as IL-6, adrenomodulin,
soluble CD14, soluble endothelial cell/leukocyte adhesion molecule 1, CRP, and procalcitonin. Using these
markers in conjunction with clinical data may potentially provide new therapeutic strategies. Theoretically,
this early intervention may prevent progression of towards severe sepsis or MODS leading to improved
survival in these critically ill patients. However, due to lack of adequate, good quality clinical trials, routine
clinical use of biochemical studies is not currently supported.
Research into the utility of steroids, rhAPC and anti-thrombin as therapies for treating sepsis is still
ongoing. At this point, however, we do not have strong evidence that these lead to improved patient
outcomes in all septic patients. There are subsets of patients that do show some beneficial response to
treatment.
SEPSIS BUNDLES AND CHECKLISTS
In order to facilitate the adaptation of the current guidelines and recommendations, the concept of a
treatment “bundle” has emerged. Largely propelled by national quality organizations (IHI) and the SSC, the
goal of the bundle is to incorporate all the current evidence based guidelines and interventions into a
simple, portable format that can be cloned and implemented at critical care units across the nation’s
hospitals. The IHI sepsis and severe sepsis bundles have had widespread adoption in the United States.
Use of checklists in the ICU has caught national attention in recent years. A medical checklist in its
simplest form is a quick, efficient and relevant team discussion of important treatment goals and the daily
patient care plan. A prime example specifically related to sepsis, is the CLABSI (central line associated
blood stream infection) checklist that has been a key focus in intensive care units across the US.
Bundles and checklists provide a protocol driven system that includes EGDT, appropriate interventions and
prophylactic regimens delivered in a timely manner to septic patients. Although management should always
be individualized to specific patient populations and organ dysfunctions, these quality systems help ensure
that the best evidence based care is provided to all patients.
DISCUSSION:
Sepsis is associated with high morbidity and mortality in critically ill patients. The continuum of disease
stretches from mild signs of systemic inflammation to multiple system organ failure and septic shock.
Currently, the annual rate of septic disease is increasing. An aging population, in addition to increased rates
165
SOCCA Residents Guide 2013
of organ transplantation and immuno-suppression is bringing sicker patients into the health system. All the
while, liberal use of antibiotics and ever-evolving microorganisms are leading to more virulent multi-drug
resistant strains. It is prudent that as clinicians we fully understand the pathophysiology of this disease and
treat it in a thorough, systematic manner. Treatment should be initiated immediately upon suspicion of
sepsis with EGDT and broad-spectrum antibiotics while looking for the source of infection. In the setting of
septic shock, aggressive fluid resuscitation and vasopressor therapy should be initiated with the goal of
optimizing oxygen delivery. Consideration should be given for administration of steroids for vasopressorresistant shock. Supportive and prophylactic regimens help to prevent further damage and may help
improve patient outcomes. Future research in the field may provide better therapeutic and diagnostic tools
that allow us to detect early sepsis and treat the disease before the inflammatory cascade causes irreversible
damage.
This chapter is a revision of the original chapter authored by Sean M. Quinn, M.D. and Miguel A. Cobas, M.D.
REFERENCES:
1.
2.
3.
4.
5.
6.
7.
8.
Members of the American College of Chest Physicians/Society of Critical Care Medicine Consensus
Conference Committee (1992): Definitions of sepsis and organ failure and guidelines for the use of
innovative therapies in sepsis. Crit Care Med. 1992; 20:864-874.
Levy MM, Fink MP, Marshall JC, et al. 2001 SCCM/ESICM/ACCP/ATS/SIS International Sepsis
Definitions Conference. Crit Care Med. 2003; 31:1250-1256.
Martin GS, et al. The Epidemiology of sepsis in the United States from 1979 through 2000. N Engl J
Med. 2003; 348:1546-54.
Hotchkiss RS, Karl IE. The pathophysiology and treatment of sepsis. N Engl J Med. 2003; 348:138-150.
Rivers E, Nguyen B, Havstad S, et al. Early goal-directed therapy in treatment of severe sepsis and septic
shock. N Engl J Med. 2001; 345:1368-1377.
Dellinger RP, Levy MM, Carlet JM, et al. Surviving Sepsis Campaign: International guidelines for
management of severe sepsis and septic shock: 2008;Crit Care Med. 2008; 36:296-327.
Sprung CL, Annane D, Keh D, et al. Hydrocortisone therapy for patients with septic shock. N Engl J Med.
2008; 358:111-124.
NICE-SUGAR Study Investigators. Intensive versus conventional glucose control in critically ill patients.
N Engl J Med. 2009; 360:1283-1297.
QUESTIONS (K-Type):
Please choose the single best answer for the question below. The options are:
A.
B.
C.
D.
E.
1, 2 and 3 only
1 and 3 only
2 and 4 only
4 only
All are correct
29.1 An 84 year-old woman is admitted to the emergency room with fever, chills and light-headedness. On exam, she
has marked erythema on her left thigh with evidence of subcutaneous crepitus. She is able to speak but is in obvious
pain. Her vitals are as follows: temp 38.9 C, RR 20, SpO2 98% on 3L NC, HR 118, BP 88/45. Intravenous access has been
established. The next most appropriate step is:
1. Administer 24 mcg/kg/hr infusion of rhAPC
2. Bolus 2000 cc of NS IV rapidly
3. Start a heparin infusion
4. Administer ceftriaxone and vancomycin IV after blood cultures obtained
29.2 Management
1.
2.
3.
4.
of acute lung injury/ARDS secondary from severe sepsis includes:
Keep plateau airway pressure < 30 cm H20
Keep peak airway pressure <30 cm H20
Use a tidal volume of 6 cc/kg
Initiate neuromuscular blockade for 48 hr after onset of disease
166
SOCCA Residents Guide 2013
29.3 A 56 year-old man is admitted to the intensive care unit with ischemic colitis. He is intubated, mechanically
ventilated and is sedated. His vitals are as follows: Tm 38 C, RR 14, HR 110, BP 70/40, SpO2 100%. His most recent CBC
reveals: WBC 18,000, Hb 6.8 g/dL, Hct 21%, platelets 75,000. Bright red blood is seen coming out from the rectal tube.
Appropriate steps in his management include:
1.
2.
3.
4.
Transfuse 2 units of packed red blood cells
Place an arterial catheter
Start norepinephrine infusion and titrate to keep MAP > 65 mm Hg
Place a pulmonary artery catheter and start dobutamine to target CI > 3
167
SOCCA Residents Guide 2013
30. Infections and Antibiotic Therapy in the Intensive Care Unit
Ronald Pauldine, MD
A 36 year old man was admitted with a 40% total body surface area full
thickness burn and inhalation injury. He is now 5 days s/p burn excision and
grafting. He is intubated and has indwelling central venous, arterial and
urinary catheters. He has developed fever, increased WBC, and a moderate
decrease in blood pressure.
INTRODUCTION
Infection can be defined as the isolation of a predominant pathogen from a site in the body that is normally
sterile. Importantly, infection contributes to more than half of all deaths in intensive care unit patients.
However, most infected patients do not go on to develop sepsis. The sepsis syndrome and related issues are
discussed elsewhere in this text. The relationship between infection, inflammation, and sepsis is complex.
Host factors are extremely important in defining an individual patient’s response to infection. Most
infections observed in ICU patients will fall into one of 3 categories. Patients will be admitted for primary
infections, develop nosocomial infections or infections will develop in immune-compromised patients.
Infection is often suspected in the setting of fever and leukocytosis. Localizing signs and symptoms such
as erythema, swelling, drainage, pain or cough may be present. However, the diagnosis of true infection
may be difficult due to non-specific physical findings, lack of characteristic findings in some patient
populations, overlap of findings with a number of non-infectious processes and confounding culture results
often seen with colonization. Accurate diagnosis is a cornerstone in ensuring proper antimicrobial therapy.
Indiscriminate antibiotic usage is a significant problem in promoting selection of antibiotic resistant
organisms which are then potentially transmitted to other patients in the ICU.
I.
General considerations for ICU infections
A. Serious infections in ICU patients are associated with high morbidity and mortality
1. Infection is the most common cause of death in critically ill patients, particularly when
involved in initiation and maintenance of multisystem organ failure
B. Endemic nosocomial organisms will colonize most patients within 72 hours of hospitalization and
are very important when choosing empiric therapy. It is important to be familiar with your ICU
antibiogram
C. Nosocomial infections may be caused by multiple drug resistant organisms
D. Infection in the ICU can be difficult to diagnose, as critically ill patients may or may not be able to
mount the expected host response or have underlying conditions such as trauma or burns that may
cause non-specific alterations in thermoregulation, heart rate, respiratory rate and WBC without
the presence of infection.
E. As a general rule cultures should be obtained before initiation of empiric therapy
F. Empiric therapy should be initiated with broad spectrum agents that have activity against
commonly encountered organisms in the ICU then tailored to specific agents based on the culture
results
G. Early appropriate antibiotic therapy is associated with improved survival
H. Source control (mechanical removal of the source of infection if appropriate) is a governing
principle of treatment. Examples include drainage of abscesses, removal of infected hardware,
decompression of biliary obstruction, etc.
II.
Risk Factors
A. Critically ill patients are at increased risk for serious infections due to factors related to their
underlying condition and interventions commonly employed in the ICU. These factors include:
1. Severity of illness
2. Airway instrumentation and mechanical ventilation
3. Bladder catheterization
4. Intravascular catheters
5. Multiple, broad spectrum, long-term antibiotic usage
6. Chronic, pre-existing illness
7. Medications: steroids, other immunosuppressants, chemotherapeutic agents
168
SOCCA Residents Guide 2013
8. Endemic resistant organisms
B. Many of the risk factors are likely markers of severity of illness and not necessarily independent
determinants.
III.
Commonly encountered patterns of infection in the ICU (GPC = Gram positive cocci, GNR = Gram
negative rods)
A. Pneumonia
1. Pneumonia may be classified by the clinical setting where it occurs. This has important
implications for the likely causative organisms.
a) Community-acquired pneumonia
(1) First positive bacterial culture within 48hrs of admission
(2) Common pathogens – Strep pneumoniae, H. flu, M. catarrhalis, M. pneumoniae,
Legionella, Chlamydia, MRSA, Influenza
b) Healthcare-associated pneumonia
(1) First positive bacterial culture within 48hrs of admission AND transferred from
another hospital OR reciving outpatient therapy for hemodialysis, wound care or
intravenous treatment OR prior hospitalization ≥3 days within 90 days of positive
culture OR immunocompromised
c) Hospital-acquired pneumonia
(1) First positive bacterial culture 48hrs after admission
d) Ventilator-associated (VAP)
(1) Mechanically ventilated patients with first positive bacterial culture after 48hrs of
admission or tracheal intubation
(2) Causative organisms for healthcare associated, hospital acquired, and VAP include
– MRSA, Psudomonas aeruginosa, Klebsiella pneumoniae, Acinetobacter spp.
Stenotrophomonas spp. Legionella pneumophila
2. Diagnosis
a) Microbiologically confirmed
(1) Abnormal CXR (new or progressive infiltrate), high clinical suspicion, (clinical
pulmonary infection score (CPIS) ≥6 , recovery of a likely pulmonary pathogen via
uncontaminated specimen, in high concentration on semi-quantitative culture, or
positive serology. CPIS system considers 6 elements graded from 0-2pts each
including tracheal secretions, radiographic infiltrates, fever, leukocytosis,PaO2/
FiO2 ratio, and microbiology. CPIS is often unreliable in trauma and burn patients.
b) Probable
(1) Abnormal CXR, high clinical suspicion (CPIS≥6), no microbiological or
serological confirmation (pathogens recovered below diagnostic threshold)
c) Possible
(1) Abnormal CXR of uncertain cause, low to moderate clinical suspicion but positive
microbiological or serological criteria
3. Treatment–antibiotics and pulmonary toilet
B. Bloodstream
1. Clinical setting and Diagnosis
a) Primary
(1) Unknown origin
(2) Blood culture positive for a recognized pathogen OR
(3) 2 or more blood cultures positive for common skin contaminant AND
(4) Cultured organism is not related to an infection at another site
(5) Catheter-related (CRBSI)
(6) suspected if local signs (erythema, swelling, drainage) present at insertion site OR
(7) postive culture of removed catheter tip OR
(8) positive blood cultures for same organism drawn simultaneously from suspected
catheter and peripheral blood: catheter is likely source if cultures turn positive 2 or
more hours before peripheral sample
b) Secondary
(1) related to an established infection at another site
(2) Blood culture positive for a recognized pathogen related to an infection with the
same organism at another site
169
SOCCA Residents Guide 2013
C.
D.
E.
F.
G.
(3) Organisms – GPCs, fungi, GNRs
(4) Treatment – Antibiotics, removal of catheter if suspected as source
C. Infective Endocarditits
1. Diagnostic Criteria
a) Modified Duke
b) Organisms – most frequently S. aureus, but B-hemolytic strep, enterococci, GNRs and
candida also seen
2. Treatment – Antibiotics
Intra-abdominal
1. Clinical Setting and Classification
a) Primary peritonitis – (spontaneous bacterial peritonitis) infection of peritoneal fluid in the
absence of gastrointestinal perforation, abscess or other localized GI tract infection
b) Secondary peritonitis – Infection of the peritoneal space following perforation, abscess,
ischemic necrosis or penetrating injury of the abdomen
c) Tertiary peritonitis – persistent intra-abdominal inflammation related to nosocomial
pathogens after an episode of secondary peritonitis
2. Organisms – GNR, anaerobes, enterococci, occasionally fungi
3. Diagnosis – history of intraabdominal operation or trauma, physical examination, CT
scanning or ultrasound
4. Treatment–drainage (operative or percutaneous), antibiotics
Urinary Tract
1. Clinical Setting
a) Catheter-associated Urinary Tract Infection
(1) most common nosocomial infection (up to 50% of patients with indwelling catheter
≥ 5days develop bacteriuria or candiduria
(2) silent catheter-associated baceriuria represents a significant reservoir for resistant
organisms
(3) rarely symptomatic and rarely a source of severe sepsis
(4) difficult to discern colonization from infection
b) Urosepsis
(1) Usually community acquired in association with pyelonephritis
(2) May be associated with obstruction or instrumentation of the urinary tract
2. Organisms – enteric GNRs, entrococcus, pseudomonas, candida
3. Diagnosis - culture threshold for “infection” is unclear – but > 105 cfu/mL is associated with
infection
4. Treatment – removal of catheter if possible, catheter exchange for bacteriuria, antibiotics for
infection, decompression for obstructive etiology
Skin and Soft Tissue
1. Clinical Setting
a) Surgical Site Infections
b) Cellulitis
c) Necrotizing Fasciitis
d) Abcesses
2. Organisms – Most are polymicrobial, GPCs, anaerobes, GNRs
3. Diagnosis
a) Clinical exam
b) Imaging (soft tissue stranding, fluid collections, gas)
c) Gram Stain and Culture of infected material
4. Treatment
a) Drainage and debridement if indicated
b) Antibiotics
Central Nervous system Infections
1. Clinical Setting
a) Meningitis
b) Encephalitis
c) Brain Abscess
2. Organisms
a) Meningitis - Pneumococcus, Neisseria meningitides, Haemophilus influenzae, Listeria,
170
SOCCA Residents Guide 2013
Cryptococcus
b) Encephalitis – Herpes Simplex Virus, Varicella Zoster Virus
3. Diagnosis
a) Cerebrospinal Fluid Analysis
b) Gram Stain
c) Antigen testing
4. Treatment – Antibiotics, add dexamethasone for adult patients with pneumococcal meningitis
H. Septic thrombophlebitis
1. Organisms–GPC, GNR
2. Diagnosis–history, physical examination, culture
3. Treatment–remove catheter, antibiotics, anti-coagulation if in deep vein, occasionally excision
or lysis
I. Sinusitis
1. Organisms–GNR, GPC
2. Diagnosis–physical examination, CT scan, aspiration
3. Treatment–remove or relocate nasoenteric or naso-tracheal tubes, decongestants, antibiotics,
rarely drainage
IV.
Considerations for the prevention of antibiotic resistance
A. Infection Control Practices
1. Hand washing – Use of alcohol based hand sanitizer enhances compliance but some
organisms (e.g. C. diff.) are resistant and mandate use of soap and water when providing care
for affected patients
2. Standard Precautions
3. Full barrier techniques for inserting vascular devices
4. Ventilator “Bundles” for prevention of VAP
5. Antibiotic Stewardship
a) Rational Use of Antibiotics
b) Effective Therapy
c) Appropriate Duration
d) De-escalation when appropriate
e) Antibiotic Utilization Programs
B. Mechanisms of Antibiotic Resistance
1. Genetic Mutation
2. Selection
3. Genetic Exchange
a) transformation
b) conjugation
c) transduction
C. Examples
1. Production of inactivating enzymes
a) ОІ-lactamase and extended-spectrum ОІ-lactamase
2. Efflux pumps
3. Altered cell wall synthesis
4. Altered binding proteins
5. Altered channel proteins
V.
Special Considerations:
Emerging organisms and changing patterns of disease provide considerable challenges in diagnosis,
management and prevention of several infectious disease processes. Cases may be sporadic but have
the potential to occur as endemic or epidemic outbreaks. The ability to provide sufficient critical care
resources figures prominently in such events. Mass casualty scenarios and acts of bioterrorism also
have the potential to overwhelm healthcare infrastructure and consume scarce resources.
A. Emerging Infections
1. Viral Diseases
a) Influenza
b) Seasonal
c) Influenza A (Avian)
171
SOCCA Residents Guide 2013
d) West Nile Virus
e) HIV
2. Bacterial Diseases
a) Methicillin Resistant Staphyloccus Aureus (MRSA)
b) Vancomycin Resistant Enterococcus (VRE)
c) Multi-drug Resistant GNRs
(1) Acinetobacter
(2) E. coli
(3) Klebsiella
(4) Enterobacter
d) Clostridium Difficile Colitis
(1) Increasing frequency and severity of infection have been related to toxigenic strain
NAPI/BI/027 that is associated with unsuppressed and unregulated toxin
production.
(2) Increasing rates of treatment failure
(3) Diagnosis based on screening for presence of antigen (sensitive but not specific)
then confirming presence of toxin by tissue culture or ELISA.
(4) Treatment – Stop offending antibiotics if able. Oral vancomycin or metronidazole
but resistance to metronidazole is increasing. Oral vancomycin is the agent of
choice for severe infection. IV vancomycin is not effective.
(5) In preventing transmission it is important to remember that alcohol based hand
cleansers are ineffective in killing spores. All contacts must perform a vigorous
scrub with soap and water.
3. Fungal Diseases
a) There is increasing resistance to fluconazole.
(1) This is of primary concern for non-albicans species of Candida
b) Molds
B. Bioterrorism
1. Preparedness
a) Familiarity with your institutional Disaster/Mass Casualty Plan is important
b) Local Incident Command System
c) National Disaster Medical System
2. Be familiar with possible presentation, precautions and treatment for likely organisms
a) Effective weaponization is difficult but feasible
b) High priority agents are characterized by easy dissemination, person to person
transmission, lethality and impact on public health system. They include:
(1) Smallpox
(2) Anthrax
(3) Plague
(4) Botulism
(5) Tularemia
(6) Hemorrhagic Fever Viruses
VI.
Considerations in Dosing Antibiotics in Critical Illness
A. Pharmacokinetics
1. The volume of distribution for many agents is altered in disease states such as sepsis and
major burn injury where Vd can be greatly increased and underdosing may occur
2. Pharmacokinetic parameters may change frequently in critically ill patients. Alterations in
clearance for example, may create increased likelihood of toxicity
3. When possible, serum levels should be followed and dosing optimized
B. Pharmacodynamics
1. Aminoglycoside antibiotics exhibit concentration dependent killing where effective killing of
a susceptible organism is related to the maximal concentration of agent achieved.
2. Aminoglycosides exhibit a significant post-antibiotic effect where bacterial growth remains
suppressed well after serum levels fall below the MIC.
3. Beta-lactams exhibit time dependent killing where effective killing of susceptible organisms
is related to the time the organism is exposed to concentrations of agent above the minimum
inhibitory concentration (MIC).
172
SOCCA Residents Guide 2013
4.
5.
Killing of susceptible bacteria by vancomycin is related to area under the concentration curve
over the minimum inhibitory concentration for the pathogen treated.
These factors influence decisions in dosing that maximize efficacy while limiting toxicity
Antibiotics commonly used in the intensive care unit - Table 31-1
Table 31-1: Antibiotics commonly used in the ICU
Aztreonam
Spectrum:
GNR (generally somewhat less broad than animoglycosides)
Toxicity:
Minimal cross reactivity in penicillin allergic patients
Cephalosporins,
1st Generation
(cephalexin,
cefazolin)
Spectrum:
GPC, some GNR
Cephalosporins,
2nd generation
(cefoxitin,
cefuroxime)
Spectrum:
GPC, many GNR. Cefotixin also covers anaerobes including
B. fragilis. Cefuroxime also covers H. influenzae
Cephalosporins,
3rd generation
(cefotaxime,
ceftriaxone,
ceftazidime)
Spectrum:
most GNRs. Ceftazidime also covers some P. aeruginosa
Cephalosporins,
4th generation
(cefepime)
Spectrum:
GNR including P. aeruginosa, GPC
Toxicity:
for all cephalosporins, 10% cross-reactivity with penicillin
allergy
Spectrum:
GNR, GPC (some Streptococcus species resistant)
Toxicity:
rare, impairs cartilage growth
Metabolism:
Moxifloxacin undergoes hepatic metabolism and biliary
excretion, does not concentrate in urine and not effective for
UTI
Spectrum:
anaerobes, GPC
Toxicity:
C. difficile colitis
Spectrum:
GNR, including P. aeruginosa, A. baumanii, E. coli, some
enterobacter species, H. influenzae, B. pertussis, Legionella,
Salmonella, Shigella, Stenotrophomonas. Resistant species
include Burkhoderia cepacia, Serratia, M. catagghalis,
Proteus, Providentia and Morganella.
Toxicity:
renal failure and neuropathy
Note:
this old drug is finding resurgent utility is some MDRs
Quinolones
(ciprofloxacin,
levofloxacin,
moxifloxacin)
Clindamycin
Colistin
(polymixin)
173
SOCCA Residents Guide 2013
Table 31-1: Antibiotics commonly used in the ICU
Daptomycin (a
cyclic lipopeptide)
Spectrum:
GPCs
Toxicity:
myopathy (monitor CPK), peripheral neuropathy
Note:
may be preferred for MRSA endocarditis
Spectrum:
GPC, H. influenzae, Mycoplasma species, Chlamydia species.
Clarithromycin is also useful against H. pylori
Toxicity:
nausea, vomiting, diarrhea
Note:
bacteriostatic
Aminoglycosides
(gentamicin,
tobramycin,
amikacin)
Spectrum:
GNR, synergy against some GPC
Toxicity:
nephrotoxic, ototoxic, particularly with prolonged courses
Carbapenems
(imipenem,
meropenem,
doripenem)
Spectrum:
GPC, GNR including P. aeruginosa, anaerobes
Toxicity:
seizures (imipenem), minimal cross-reactivity in penicillin
allergy
Linezolid
Spectrum:
GPC, including MRSA
Toxicity:
thrombocytopenia
Note:
may be preferred for MRSA pneumonia
Spectrum:
anaerobes, E. histolytica (amebic abscess), Trichomonas
species, C. difficile
Toxicity:
nausea and vomiting (especially in combination with alcohol)
Spectrum:
GPC, Clostridia, Syphillis, oral anaerobes (i.e.,
Peptostreptococcus); some GNR with ampicillin/amoxacillin;
usually not good against Enterococcus; improved anaerobic
effect in combination with a пЃў-lactamase inhibitor (i.e.,
Piperacillin/Tazobactam)
Toxicity:
wide range, including hypersensitivity, rash, diarrhea,
urticaria, neurotoxicity, superinfection (overgrowth)
Note:
decreasing efficacy with prevalence of пЃў-lactamase
producing bacteria
Spectrum:
GPC, no activity against E. faecalis
Toxicity:
arthralgia, myalgia, hyperbilirubinemia, phlebitis
Spectrum:
many GPC, GNR, including MRSA and other resistent species.
Not effective against Pseudomonas or Proteus species
Toxicity:
diarrhea, nausea/vomiting
Macrolides
(erythromycin,
clarithromycin,
azithromycin)
Metronidazole
Penicillins
Quinupristin/
Dalfopristin
Tigecycline
174
SOCCA Residents Guide 2013
Table 31-1: Antibiotics commonly used in the ICU
Note:
like tetracycline, adversely affects teeth/bone growth
Spectrum:
some GPC, some GNR, Chlamydia species, Pneumocystis
carinii, Legionella species, S. maltophila
Toxicity:
rash, toxic epidermal necrolysis (Stevens-Johnson syndrome)
Spectrum:
GPC (including MRSA), Clostridia species (including C.
difficile)
Toxicity:
face and neck rash (red-man syndrome), ototoxicity,
nephrotoxicity (particularly with longer course)
Note:
Appropriate loading dose, high trough serum levels to
maintain adequate tissue levels and prevent emergence of
resistant organisms
Amphotericin B,
Amphotericin-Lipid
Complex,
Liposomal
Amphotericin
Spectrum:
fungus
Toxicity:
nephrotoxic, hypokalemia, hypersensitivity reactions
Note:
toxicity decreased with lipid formulations
Echinocandins
(caspofungin,
micafungin)
Spectrum:
Candida species
Toxicity:
well tolerated
Note:
dosage adjustment required in hepatic failure, not
concentrated in urine
Spectrum:
C. albicans, Cryptococcus neoformans. Increaseing
resistance among non-albicans Candida species
Toxicity:
visual disturbances, rash, fever, elevated liver enzymes with
voriconazole
Note:
voriconazole preferred for Aspergillus species
Trimethroprim/
sulfamethoxazole
(TMP/SMX)
Vancomycin
Azoles
(fluconazole,
voriconazole)
References:
1.
2.
3.
4.
5.
American Thoracic Society, Infectious Diseases Society of America. Guidelines for the management of
adults with hospital-acquired, ventilator-associated, and healthcare-associated pneumonia. (2005)
American Journal of Respiratory and Critical Care Medicine. 171, 388-416.
Bagshaw SM, Laupland KB. (2006) Epidemiology of intensive care unit-acquired urinary tract infections.
Current Opinion in Infectious Disease. 19, 67-71.
Bartlett JG. (2006) Narrative review: The new epidemic of clostridium difficile-associated enteric disease.
Annals of Internal Medicine. 145, 758-764.
O'Grady NP, Barie PS, Bartlett JG, Bleck T, Carroll K, Kalil AC, et al. (2008) Guidelines for evaluation of
new fever in critically ill adult patients: 2008 update from the american college of critical care medicine
and the infectious diseases society of america. Critical Care Medicine. 36, 1330-1349.
Roberts JA, Lipman J. (2006) Antibacterial dosing in intensive care: Pharmacokinetics, degree of disease
and pharmacodynamics of sepsis. Clincal Pharmacokinetics. 45, 755-773.
175
SOCCA Residents Guide 2013
Questions
30-1 How long does it take most patients to become colonized with nosocomial flora in the intensive care unit?
A. 6 hours
B. 72 hours
C. 1 week
D. 2 weeks
30-2 Which are the two most common organisms causing nosocomial pneumonia?
A. E. coli and B. fragilis
B. C. albicans and E. aerogenes
C. S. aureus and P. aeruginosa
D. S. pneumoniae and H. influenzae
30-3 What minimal level of bacteriuria is indicative of infection?
A. 10 colony forming units (CFU)/ ml
B. 100 CFU/ ml
C. 10,000 CFU/ ml
D. 100,000 CFU / ml
30.4 What is the first line treatment of an infected intravascular catheter?
A.
B.
C.
D.
Local line care
Removal of catheter
Antibiotics
Antiseptic through the catheter
30.5 What measures have been demonstrated to decrease the incidence of CRBSI?
A.
B.
C.
D.
Routine rewiring of central venous catheters every 5 days
Use of prophylactic antibiotics prior to insertion of the catheter
Use of alcohol skin prep prior to insertion
Routine use of full barrier precautions during vascular access procedures
176
SOCCA Residents Guide 2013
31. Antibiotic Prophylaxis
Ronald W. Pauldine, M.D.
A 69 year old man with a past medical history of hypertension, diabetes mellitus and
obesity is admitted to the intensive care unit with a symptomatic 8 cm juxtarenal
abdominal aortic aneurysm. The vascular surgeon does not believe the problem can
be treated with an endovascular approach and the patient is being prepared for
urgent open repair.
INTRODUCTION:
Surgical site infections (SSI) complicate 2-5% of all surgical procedures performed in the United States.
SSIs are associated with increased mortality, hospital length of stay and health care cost. Prophylactic
antibiotic administration is an important part of multiple strategies directed at decreasing the risk of SSI.
Surgical site infection is defined as infection occurring in a surgical incision or organ or organ spaces
opened or manipulated during surgery. These infections should not be confused with infections occurring
in traumatic wounds. Antibiotic administration should result in effective drug levels during the procedure
and for a short time afterwards. Patients receiving pre-operative antibiotics generally do not require
additional coverage for endocarditis prophylaxis.
I.
General Principles of Prophylactic Antibiotic Use
A. Prophylaxis should be used for all elective procedures that enter a hollow viscus or result in
placement of intravascular prostheses or total joint prosthesis.
B. Treat with agents that are safe and have activity against the most likely pathogens for a given
procedure.
C. Administration of the antibiotic should result in peak concentrations at the time of incision.
1. For most antibiotics this means that administration should occur within one hour of skin
incision. Therefore it is inappropriate to order antibiotic administration “on call.”
2. For medications requiring longer infusion times such as vancomycin and floroquinolones,
infusion should begin 60-120 minutes before skin incision to ensure the infusion is complete
before incision is made.
D. Therapeutic levels of the chosen antibiotic should be maintained in the serum and tissue for the
duration of procedure. This may necessitate redosing.\
1. Redosing is based on blood loss and effective antibiotic half life.
a) For procedures over 4 hours in duration or procedures involving major blood loss
redosing should occur every 1-2 half-lives of the chosen antibiotic in patients with
normal renal function.
II. Risk of Infection
A. The risk of SSI increases in relation to number of risk factors present. Risk factors include patient
related and procedure related variables.
1. Patient Related Factors.
a) Age
b) Poorly controlled diabetes mellitus
c) Obesity
(1) Important consideration in antibiotic dosing to ensure adequate serum levels.
d) Tobacco use
e) Treatment with immunosuppressive medications
f) Duration of preoperative hospitalization
2. Procedure Related Factors
a) Wound Class
(1) Clean: Elective procedures with no violation of colonized or inflamed sites, good
sterile technique, procedure restricted to integumentary and musculoskeletal soft
tissues.
(a) Infection risk: 2.1% -- based on data from the National Nosocomial Infections
Surveillance System (NNIS).
(2) Clean-contaminated: Procedures where a hollow viscus has been opened under
controlled circumstances (such as elective resection of the colon).
177
SOCCA Residents Guide 2013
(a) Infection risk:3.3%
(3) Contaminated: Procedures where bacteria have been in contact with a normally
sterile body cavity, but for an insufficient period of time for infection to become
established (such as penetrating abdominal trauma or traumatic wounds <4 hours
old.
(a) Infection risk: 6.4%
(4) Dirty: procedure performed to address established infection. Example include
Traumatic wounds >4 hours old or with foreign body, devitalized tissue, or fecal
contamination, perforated viscus or any procedure where pus is encountered.
(a) Infection risk:7.1%
b) Duration of Surgery
c) Shaving of Hair
(1) If hair removal is necessary clippers should be used.
(2) Interval between shaving and procedure is also important.
d) Intraoperative Hypoxia
e) Hypothermia
(1) This stresses the importance of active warming in procedures where it is not
contraindicated.
III. General Recommendations for Prophylaxis
A. Primary goal is the source most likely to cause an infection in the surgical setting.
B. Skin flora are the most common sources. Use of a first generation cephalosporin, such as
cefazolin, is generally appropriate. Second line choices, i.e., if the patient has allergies to
cephalosporins, include Clindamycin or Vancomycin.
C. Local resistance patterns should be taken into consideration.
Table 31-1: Most Common Organisms Causing SSIs
Organism
% Infections
Staphylococcus aureus
20
Coagulase-negative staphylococci
14
Enterococci
12
Pseudomona aeruginosa
8
Escherichia coli
8
Enterobacter sp.
7
Proteus mirabilis
3
Streptococci
3
Klebsiella pneumoniae
3
Candida albicans
2
Data from NNIS
IV. Case Specific Issues
A. Cardiac Surgery
1. Careful attention must be given to redosing in procedures lasting > 400 min.
B. GI Surgery
1. Esophageal procedures with obstruction
2. Gastroduodenal procedures in setting of decreased motility or gastric acidity including
obstruction, hemorrhage, malignancy, morbid obesity, H2-blocker or PPI therapy.
3. Biliary procedures with risk factors including age >70, acute cholecystitis, obstruction or
known common duct stones.
4. Colorectal surgery
5. Perforated appendicitis (Treatment)
178
SOCCA Residents Guide 2013
C. GU Surgery
1. Usually not needed if urine is sterile
2. Beneficial for TURP, transrectal biopsy, and procedures involving placement of prosthetic
material
D. Gynecology and Obstetrics
1. Hysterectomy
2. Cesarean section (both pre-incision and after cord clamp are acceptable)
E. Head and Neck Surgery
1. Procedures involving incision of oral or pharyngeal mucosa
F. Neurosurgery
1. Craniotomy
2. Spine Fusion or instrumentation (placement of foreign material)
G. Orthopedic
1. Total Joint Replacement
2. Operative repair of closed fractures
3. Ensure administration prior to inflation of tourniquet
H. Thoracic (non-cardiac)
1. Lung resection
I. Vascular Surgery
1. Abdominal Aorta Repair (open or endovascular)
2. Lower extremity revascularization involving incision in the groin
3. Amputation of ischemic lower extremity
4. Placement of prosthetic grafts
Table 31-2: Common bacteria by operative site
Site
Organisms
Head and Neck
GPC, enteric GNR, anaerobes
Esophagus, stomach, duodenum
Biliary tree
Sterile (unless obstructed or acid-suppressed), GPC, enteric
GNR
GNR, enterococci, clostridia
Small bowel
Sterile (unless obstructed), GPC, enteric GNR
Large bowel
GNR, anaerobes, enterococci
Genito-urinary system
GNR, enterococci
Gynecologic and Obstetric
Enteric GNR, anaerobes, Gp B Strep, enterococci
Thoracic (Non-cardiac)
GPC, enteric GNR
Skin (includes coverage for Neuro,
GPC
Orthopedic, and Vascular Surgery)
GPC: Gram positive cocci, GNR: Gram negative rods
*Note: patients who have been hospitalized for more than 48 hours have a high rate of colonization at all
sites with nosocomial organisms, primarily GNRs.
Discussion
The patient described in the case has multiple risk factors for surgical site infection and these factors must
be considered in appropriately dosing and maintaining effective tissue concentrations of antimicrobials.
The patient’s obesity may necessitate higher doses due to an increased volume of distribution. The choice
of antibiotic should be directed at treating normal skin flora and of course must be administered within one
hour prior to incision. The surgical plan involves placement of intravascular prosthetic graft material. The
procedure may be long due to technical difficulty and may be associated with significant blood loss, all
leading to the need for vigilance in antibiotic redosing.
179
SOCCA Residents Guide 2013
Table 31-3: Commonly used antibiotics, general spectrum and intraoperative dosing
Antibiotic
Spectrum
Dosing interval
Cefazolin
Skin flora
Q 2-5 hrs
Cefoxitin
Skin and colonic flora (inc. anaerobes)
Q 2-3 hrs
Ampicillin/Sulbactam
Skin flora, Enterococcus spp., anaerobes
Q 2-4 hrs
Ciprofloxacin
GNR, Some staph spp.
Q 4-8 hrs
Gentamicin
GI, GU flora (GNR)
Q 6-8 hrs
Clindamycin
Skin flora, anaerobes
Q 3-6 hrs
Metronidazole
Anaerobes
Q 6-8 hrs
Vancomycin
Skin Flora (all GPC)
Q 6-12 hrs
References
1.
2.
3.
Anderson, DJ (2011). Surgical Site Infections. Infectious Disease Clinics of North America, 25, 135-153
Mauermann WJ and Nemergut EC. (2006). The Anesthesiologist’s Role in the Prevention of Surgical Site
Infections. Anesthesiology, 105, 413-421
National Nosocomial Infections Surveillance (NNIS) System Report, data summary from January 1992
through June 2004, issued October 2004. American Journal of Infection Control, 32, 470-485.
Questions
32.1. When should prophylactic antibiotics be first administered?
A.
B.
C.
D.
The night before operation
On call to the operating room
Within one hour of skin incision
In the recovery room
32.2. Under most circumstances, for how long should prophylactic antibiotics be administered?
A.
B.
C.
D.
≤24 hours
3 days
7 days
Until skin staples are removed
32.3 What is the expected wound infection rate for a clean wound?
A.
B.
C.
D.
0%
1-5%
5-10%
10-20%
32.3 How often should cefazolin be redosed during an operation in the absence of severe blood loss?
A.
B.
C.
D.
Q
Q
Q
Q
30 mins
2-4 hours
6 hours
8 hours
180
SOCCA Residents Guide 2013
32. Management of the Immunocompromised
Patient
Zdravka Zafirova, M.D., Jennifer Hofer, M.D.
A 27 year old man with acute myelocytic leukemia has received a bone
marrow transplant 1 month ago. He is admitted to the ICU with BP 87/38, HR
124, productive cough, respiratory distress and O2 saturation 91% in 100 %
face mask. Complete blood count reveals WBC 0.9 x 103/mm3, Hgb 7.8 gm/
dL.
Immunocompromised patients pose unique medical and social challenges to the health care team as a result
of altered organ function and immune system response to illness. A precarious balance exists between
immunosuppressive therapy and increasing infection risk. Management considerations for these patients
include the differential diagnosis, evaluation, and treatment of respiratory failure, shock, and end-organ
failure along with focus on immunosuppression and infection issues. The management of these patients is
based on an understanding of the mechanisms of their pathology and use of diagnostic and therapeutic
interventions adapted to the unique presentations of critical illness in the immunocompromised host. A
multidisciplinary approach to the care of these patients with collaboration among providers with a wide
range of expertise is key to improvement of outcomes.
Pathophysiology
The immune system is complex, composed of multiple components involved in responding to pathogens.
The different components include innate immunity, cellular subsets, immunoglobulins, cytokines, and
complement. Innate immunity includes different types of barriers: mechanical, chemical and biological.
Cellular subsets include NK cells, phagocytes, macrophages, T cells, and B cells. Immunoglobulins range
from IgA to IgM.
Many immune system disorders exist which predispose patients to infection. Different immune system
abnormalities make patients more susceptible to some types of pathogens over others.
Humoral immunity disorders include congenital, chemotherapy, and chronic lymphocytic leukemia.
Congenital disorders include agammaglobulinemia, selective immunoglobulin deficiency, hyper IgM
syndrome, and common variable immune deficiency. Various chemotherapy agents affect humoral
immunity including azathioprine, cyclophosphamide, and methotrexate.
Disorders of cell-mediated immunity includes congenital, immunosuppression and chemotherapy,
lymphoma, protein-calorie malnutrition, and viral illnesses.
Complement deficiencies may be associated with pyogenic infections with encapsulated organisms.
Autoimmune diseases such as multiple myeloma and systemic lupus erythematosus, along with certain
bacterial organisms, are associated with deficiencies of the complement regulatory proteins.
Neutropenia and phagocytic dysfunction can be associated with disorders that are congenital, autoimmune,
infectious, or secondary to leukemia and lymphoproliferative disorders, total body radiation, or associated
organisms. Congenital disorders include chronic granulomatous disease, myeloperoxidase deficiency, and
chemotactic disorders. Drug induced includes bone marrow suppressions, antibody and complementmediated destruction. Culprit chemotherapy medications include adramycin, ARA-C, and
cyclophosphamide. Other medications include procainamide, antithyroid drugs, sulfasalazine,
phenothiazine, semisynthetic penicillins, non-steroidal anti-inflammatory agents, aminopyrine derivatives,
benzodiazepines, barbiturates, gold compounds, and sulfonamides.
Combined immunodeficiency may be congenial or acquired. Congenital causes include severe combined
immune deficiency, Wiskott-Aldrich, and Ataxia-telangiectasia. Acquired causes are lymphoproliferative
disorders, chemotherapy, and infections.
HIV has become more prevalent, and treatments more extensive which has helped to prolong the life-
181
SOCCA Residents Guide 2013
expectancy of patients with this disease. The epidemiology of HIV focuses on demographics, and
classification of HIV infection, how it is acquired, risks to health care workers and post-exposure
prophylaxis, the need for universal precautions, and the natural history of HIV infection. The diagnosis of
HIV and AIDS depends on history and physical exam, the definition of AIDS, and diagnosis using serology
testing and laboratory evaluation.
Multiple clinical manifestations and co-existing diseases are associated with HIV infection. These affect
every organ system and include the acute retroviral syndrome, chronic HIV infection, HIV wasting
syndrome, cutaneous complications, central and peripheral neuromuscular system complications, along
with ocular, cardiovascular, pulmonary (pulmonary hypertension), hematologic, renal, endocrine,
autoimmune/rheumatic, psychiatric, and gastrointestinal complications including oropharyngeal,
hepatobiliary, and pancreatic manifestations.
Infections are a significant cause of morbidity and mortality in the immunocompromised host.
Patients undergoing bone marrow transplantation are at risk for a variety of complications. There are
combined risks of ablative chemotherapy and suppression of cell-mediated/humoral immunity, along with
pre-bone marrow engraftment infections. Engraftment complications can be divided into three main time
frames; early (0-30 days), intermediate (30-90 days) and late (> 90 days).
Early post-marrow engraftment complications may be therapy-related toxicity including mucositis, cystitis,
interstitial pneumonitis, alveolar hemorrhage, veno-occlusive disease, or myocarditis. Graft failure may
occur, as well as infection secondary to barrier injury or neutropenia making the patient susceptible to
Gram-negative bacteremia, Gram-positive bacteremia, fungal infections, HSV viremia, and respiratory
viruses.
Intermediate post-marrow engraftment complications include acute graft-versus-host disease (GVHD),
infections due to T cell dysfunction and hypogammaglobulinemia making the patient at risk for Aspergillus
spp. Candida spp., Pneumocystis carinii, and CMV infections.
Late post-marrow engraftment complications include chronic GVHD, hypothyroidism, malignancy, and
infection, often with Streptococcus pneumonia, Haemophilus influenza, PCP, CMV, and Varicella-zoster.
Therapeutic interventions for bone marrow transplantation complications include immunosuppression for
GVHD, infection prophylaxis and therapy, and supportive therapy and management of complications.
Many issues surround solid organ transplantion. These include preoperative organ system dysfunction,
strategies for immunosuppression including induction and maintenance therapy, the diagnosis and therapy
of acute, chronic, and recurrent organ rejection, and infection.
Early infection complications from day 0-30 after transplant are general site specific, such as from a
surgical wound, respiratory tract, bacteremia from vascular access, or urinary. Other sources for infection
include preexisting recipient pathogens, donor-transmitted hepatitis, HIV, and nosocomial infections, along
with HSV and hepatitis B. Intermediate infections occur from day 30-180 and include lingering pathogens,
and opportunistic infections including CMV, EBV, varicella-zoster, Pneumocystis, Aspergillus,
Toxoplasma, and Listeria. Late infections from >120-180 days are attributed to community-acquired
respiratory viral and bacterial infections, hepatitis B and C, CMV, Listeria, and Cryptococcus.
Complications of chronic immunosuppression surround solid organ transplantation. These complications
include bone marrow suppression and immunomodulation, organ system toxicity including renal failure,
neurotoxicity, hypertension, metabolic abnormalities, diabetes mellitus, hyperlipidemia, and osteoporosis,
along with malignancy.
Management
Multiple immunosuppressive agents exist for transplant recipients. Corticosteroid administration is
common. Other drugs include alkylating agents such as chlorambucil and cyclophosphamide,
antiproliferative agents Azathioprine and Mycophenolate, mTOR inhibits sirolimus and everolimus,
calcineurin inhibitors cyclosporine and tacrolimus, and antibodies that are monoclonal (OKT-3), polyclonal
(thymoglobulin), and anti-interleukin-2 receptor antibodies. Transplant recipients with active infections
182
SOCCA Residents Guide 2013
require modification of immunosuppression regimens.
Strict infection control is imperative for immunocompromised patients in the ICU. This includes
preoperative evaluation and treatment of infection, proper hand washing techniques, isolation precautions,
universal healthcare worker precautions, and environmental/endemic nosocomial infection surveillance.
The preoperative evaluation should focus on sites of chronic or latent infection, and include treatment and
prophylaxis or overt clinical infection, or latent viral or fungal infection. Isolation requirements may be
category-specific vs. disease-specific, may require use of barrier protection, or use of a single, HEPAfiltered, positive-air-pressure sealed room.
Modes of antibiotic therapy include therapeutic, prophylactic, empiric, and preemptive. Therapeutic
antimicrobial therapy focuses on treatment of established infection. Prophylactic therapy involves
administration of antimicrobials to an entire patient population to prevent infection. Empiric therapy is
treatment of suspected infection with fixed antimicrobial regimen, while preemptive therapy is
administration of antimicrobials to a select population at risk based on epidemiology, clinical or laboratory
markers.
Antimicrobial prophylaxis may be perioperative or post-transplant. Perioperative management is based on
surgical site, and presence of preexisting pathogen. Post-transplant prophylaxis is based on timing,
preexisting pathogen in donor or recipient, and risk for opportunistic infection.
Many drug-resistant pathogens have evolved requiring different medicine regimens. These drug-resistant
pathogens include methicillin resistant Staphylococcus aureus, vancomycin resistant Enterococcus,
multidrug resistant Pseudomonas aeruginosa, multidrug resistant enteric Gram-negative pathogens, and
other pathogens with limited antimicrobial drug availability such as Stenotrophomonas, and Acinetobacter.
Immunization is also included in antimicrobial therapy to prevent increased infectious risk.
A range of treatments exist for HIV including highly-active antiretroviral therapy (HAART), nucleoside
reverse transcriptase inhibitors (NRTIs, non-nucleoside reverse transcriptase inhibitors (NNRTIs, protease
inhibitor (PIs), fusion inhibits, entry inhibitors, and integrase inhibitor. Initiation of therapy is common,
along with resistance testing when adverse effects and treatment failure occurs
Table 32-1: Types of immune system disorders and the
associated pathogens
Immune System Disorder
Humoral Immunity
Associated Pathogens
Enteric Gram-negative bacilli
Haemophilus influenzae
Neisseria meningitides
Streptococcus pneumoniae
Staphylococcus spp.
Mycoplasma pneumoniae
Giardia entovirus
183
SOCCA Residents Guide 2013
Table 32-1: Types of immune system disorders and the
associated pathogens
Cell-Mediated Immunity
Cytomegalovirus (CMV)
Epstein-Barr Virus (EBV)
Human Immunodeficiency Virus (HIV)
Varicella zoster
Herpes Simplex virus
Candida
Aspergillus
Nocardia
Coccidiomycosis
Cryotpcoccus neoformans
Histoplasmosis
Legionella
Listeria monocytogenes
Pneumocystis carinii
Toxoplasma gondii
Mycobecteria spp
Salmonella spp
Complement
Neisseria gonorrheae
Neisseria meningitides
Streptococcus
Neutropenia and Phagocytic
Varicella zoster
Measles
Rubella
CMV
EBV
Hepatitis
HIV
Parvovirus
Influenza
Aspergillus
Mucor
Candida
Enteric GNRs
Pseudomonas aeruginosa
Staphylococcus aureus
Burkholderia cepacia
Nocardia
Mycobacterium
Serratia
Combined immunideficiency
EBV
CMV
HIV
Note: pathogens may have effects in more than one type of disorder
184
SOCCA Residents Guide 2013
Table 32-2: Antimicrobial agents: targets and drug classes
TARGETS
Drugs
Antibacterial
•Penicillins
•Cepahlosporins
•Beta-lactam inhibitor
•Monobactams
•Aminoglycosides
•Macrolides/ketolides
•Fluoroquinolones
•Lincosamides
•Trimethoprim-Sulfamethoxazole
•Glycopeptides
•Streptogramins
•Oxazolidinones
•Lipopeptides
•Glycylcyclines
Antifungal
•Amphotericin B
•Fluconazole
•Itraconazole
•Voriconazole
Antiparasitic
Antiviral
Acyclovir
•Pentamidine
Gancyclovir
•Sulfadiazine/
Pyrethiamine/Folinic
Acid
Table 32-3: Medical complications and issues in HIV patients
Bacterial
Fungal
Infections Infections
and targets
•CNS
•Mycoses
•GI
•Respiratory
•Mycobacteri
al
•Syphilis
Parasitic
infections
Viral Infections
Malignancies
(AIDS defining)
Other Issues
•Cryptosporidium •Cytomegalovirus •Kaposi’s sarcoma •Other malignancy
•Isospora belli •Epstien-Barr virus •Non-Hodgkin’s
•Perinatal transmission
•Pneumocystis •Herpes simplex I Lymphoma
•Pain management
•Toxoplasma
•Herpes simplex II •Hodgkins’s disease •Prophylaxis against
opportunistic infections
•Papovavirus
•Cervical cancer
•JC virus
•Confidentiality issues
•Hepatitis B and C
•Counseling and testing
•Voluntary vs. mandatory
partner notification
Future directions
Future directions in the management of the immunocompromised patient focus on adoptive immunotherapy
and vaccination strategies. The goal is to use understanding of infection immunobiology to design modified
T cells to provide enhanced protective immunity. This cellular immunotherapy has the potential to prevent
and treat common infections that often plague the immunocompromised patient.
Discussion:
Immunosuppressed state can develop as a result of multitude of etiologies such as congenital and acquired
immunodeficiency syndromes as well as immunosuppressive therapy in the setting of organ transplantation.
The immunosuppressed patients are vulnerable to infections with the usual pathogens as well as
opportunistic agents and multidrug resistant organisms. These patients present a diagnostic and therapeutic
challenge due to impaired immune response and abnormal clinical presentations; the diagnosis of infections
may be delayed and typical symptoms and signs may be absent even in the face of severe infection and
sepsis. Other complications related to their immunosuppressed state may develop and the management of
these patients is based on vigilance in their assessment and prompt initiation of aggressive supportive and
antimicrobial therapy.
References
1.
2.
3.
Fishman JA. Infection in solid-organ transplant recipients. N Engl J Med 2007; 357: 2601-2614
Eggimann P, Pittet D. Infection control in the ICU. Chest 2001; 120:2059-2093.
Masur H. Critically ill immunosuppressed host. In: Critical Care Medicine (2nd ed) (Parillo J, Dellinger
RP; eds. 1). St Louis: Mosby, Inc 2002, 1089-1108.
185
SOCCA Residents Guide 2013
4.
5.
6.
7.
Kusane S, Krystofiak S. Infection control issues after bone marrow transplantation. Curr Opin Infect Dis
2001; 14: 427-431.
Khardori N. Antibiotics- past, present, and future. Med Clin N Am 2006; 90:1049-1076.
Fischchl MA, Richman DD, Grieco MH, et al. The efficacy of Azidothymidine (AZT) in the treatment of
patients with AIDS and AIDS-related complex; a double-blind, placebo-controlled trial. N Eng J Med
1987; 317: 187-192. A landmark article describing the first major breakthrough in the battle against the
HIV infection.
Einsele H. Antigen-specific T cells for the treatment of infections after transplantation. Hematol J. 2003;
4:10-17.
Questions
32.1
Early post-marrow engraftment complications include
1. Interstitial pneumonitis
2. Alveolar hemorrhage
3. Graft failure
4. GVHD
32.2
Humoral
1.
2.
3.
4.
immunity disorders can be a result of
Chronic lymphocytic leukemia
Multiple myeloma
Selective immunoglobulin deficiency
Systemic lupus erythematosus
186
SOCCA Residents Guide 2013
33. Neurologic Critical Care
Bobby L. Tsang, MD; Ihab Dorotta, MD
A 40 year-old male was brought into the ED after being ejected from a
vehicle at high speed. He was given a GCS score of 6 at the scene (1 eye, 1
verbal, 4 motor) and intubated for airway protection. CT scan done on
arrival showed subarachnoid hemorrhage, subdural hematoma, and a basilar
skull fracture. Medical and surgical interventions were started immediately,
including SDH evacuation, placement of an external ventricular drain (EVD)
with intracranial pressure (ICP) monitoring, and hyperosmolar therapy. He
was subsequently admitted to the neurosurgical ICU for continued therapy
and monitoring.
Neurophysiology/pathology
A. Cerebral Perfusion
1. Cerebral perfusion pressure = MAP – ICP (or CVP if higher)
2. Cerebral blood flow - largely determined by the brain’s metabolic demands (coupling), PaO2,
PaCO2, temperature, viscosity, and MAP
B. Brain Injury
1. The brain is ill-adapted to respond to neurologic insults
2. Direct injury to brain structures can lead to deficit of that area
3. In addition, the cranium is a non-compliant structure – any increase in volume (eg. mass,
edema, bleeding) can lead to a quick rise in intra-cranial pressures
4. Sustained elevated ICPs (>20 mmHg) lead to poor neurologic outcomes and death.
5. Secondary injury can occur with hypoxia, hypotension, hypercarbia, hyperthemia,
hyperglycemia, and hypoglycemia contribute to poor outcomes.
C. General goals of management in a neurologic patient
1. Maintain CPP by keeping MAPs and ICPs normal
2. Treat underlying and reversible causes
3. Avoid secondary insults
Neuromonitoring and studies
A. History and Physical – full neurologic exam, glasgow coma scale (GCS)
B. Imaging –
1. head CT/CTA
2. MRI/MRA
3. Angiography
4. FMRI,
5. PET scan
6. Carotid ultrasound
C. ICP monitoring
1. Monitoring only: Subdural bolt, epidural sensor
2. Monitoring and therapeutic (CSF drainage): Venticulostomies (EVD)
3. Indications
a) GCS ≤ 8 and abnormal CT findings
b) CGS ≤ 8 and normal CT if
c) > 40 years old
d) Systolic BP < 90 mm Hg
e) Posturing
4. Complications
a) Bleeding
b) Ventriculitis
c) Malplacement
187
SOCCA Residents Guide 2013
D.
E.
F.
G.
H.
I.
J.
K.
L.
Arterial catheter – determination of PaO2, PaCO2, MAPs, and SVV
Continuous EEG (recognition of seizures, and encephalopathy patterns)
Transcranial doppler Ultrasonography (TCD): Vasospasm recognition
Lumbar puncture: Diagnostic CSF examination as well as therapeutic CSF drainage
Brain oxygenation and metabolism monitors
Cerebral oximetry (NIRS)
Jugular venous oxygen saturation (SjvO2)
Microdialysis
Electromyography and evoked potentials.
Common neurologic pathologies
A. Intracranial hypertension – treat underlying medical causes such as seizures, hyperthemia, venous
obstruction, tumors
1. Medical treatment
a) Treat hypoxia – maintain PO2 of at least 100
b) Treat hypercarbia – maintain approx. 35
c) Treat hypotension – maintain euvolemia, transfuse, pressors as needed
d) Keep head of bed elevated
e) Temperature regulation (avoid hyperthermia)
f) Keep sedated and possibly paralyzed
g) Hyperosmotic therapy with mannitol and/or hypertonic saline.
h) Hyperventilate to PaCO2 of 30 if elevated ICPs refractory to other therapeutic modalities
i) High-dose barbiturate coma
j) Induced hypothermia
2. Surgical treatment
a) Ventriculostomy with EVD
b) Lumbar drain
c) Decompressive craniectomy
B. Cerebrovascular disease
1. Ischemic CVA (85% of strokes)
a) Causes – embolic, thrombotic, hypoperfusion, cryptogenic
b) Diagnosis – H&P, CT/MRI, TTE/TEE, carotid ultrasound, holter monitor, angiogram,
hypercoagulable states
c) Treatment:
(1) Thrombolysis within 3-6 hours of symptoms
(2) Blood Pressure control
(3) Anti-platelets
(4) Statins
(5) Intracranial HTN: as detailed above
(6) Supportive
2. Hemorrhagic CVA (15% of strokes)
a) Causes – Hypertension, aneurysm, AVM, coagulopathies, trauma
b) Diagnosis:
(1) H&P (headache, sudden neurological dysfunction)
(2) Head CT/CTA
(3) Angiography
(4) Coagulation studies
c) The ICH score:
(1) GCS
(2) IVH
(3) Age
(4) Infratentorial location
(5) Hematoma Volume
d) Treatment:
(1) Elevated blood Pressure control
(2) Coagulopathy reversal
(3) Hemostatic agents (factor VII)
188
SOCCA Residents Guide 2013
(4) Surgical decompression (if ICP an issue), supportive
e) Supportive ICU care
3. Subarachnoid hemorrhage
a) SAH grading scale
(1) Fisher grade (CT findings): indicates likelihood of complications
(2) Hunt/Hess (signs and symptoms) gives prognostication value
b) Causes
(1) saccular aneurysm rupture (>75%)
(2) traumatic
(3) AVM
(4) Cocaine
(5) Vasculitis
c) Diagnosis
(1) H&P (sudden severe HA, nausea, LOC, seizures)
(2) Head CT/CTA
(3) Angiography
(4) Lumbar Puncture
d) Treatment
(1) Supportive
(2) Aneurysm clipping or coiling
(3) Prevention and management of secondary complications
e) Complications
(1) Early
(a) Rebleeding – up to 1/3 of patients, greatest during 1st 24 hours.
(b) Control elevated blood pressure
(c) Antifibrinolytics
(d) Intracranial HTN : as detailed above
(e) Seizures
(f) Myocardial Complications
(g) Arrhythmias, ST depression, peaked T waves, prolonged QT
(h) Self limited, Potential role for Beta Blockers
(2) Late
(a) Vasospasm:
(b) Prophylaxis: Nimodipine, Statins
(c) Surveillence: Transcranial Doppler Ultrasonography
(d) Treatment:
(e) “triple H” therapy (hypertensive, hypervolemia, hemodilution)
(f) Intra-arterial vasodilators/Stents
(g) CCB, statins
(h) Metabolic disturbances – cerebral salt wasting, SIADH
C. Traumatic brain injury (TBI)
1. Leading cause of morbidity and mortality between 15 and 45 year olds
2. Primary injury – traumatic forces to the brain
a) Contact – skull fractures, contusions, epidural, subdural hematoma, SAH
b) Inertial – acceleration and deceleration, rotational/angular acceleration, contrecoup
contusions, SDH, diffuse axonal injury
c) Secondary and other associated injuries – elevated ICPs, cerebral edema, brain
herniation, hypoxia, hypotension, hypercarbia, hyperthemia, hyperglycemia
3. Diagnosis – H&P, Head CT, and MRI
4. Management:
a) Supportive, prevention or minimizing secondary injuries
b) Intracranial HTN, as detailed above
c) Reversal Of coagulopathy
d) Seizure prophylaxis
D. Status epilepticus
1. Prolonged seizure episode at least 5-7 minutes in duration
2. Etiology – infection, metabolic, drug effect, CVA, tumor, trauma, anoxic brain
3. Diagnosis – H&P, continuous EEG, CT
189
SOCCA Residents Guide 2013
4.
Management:
a) Airway protection
b) Dextrose and thiamine
c) Anti-convulsants (dilantin, phenobarbital, propofol, benzodiazepines)
d) Hemodynamic support
e) Treatment of the cause
5. Complications – rhabdomyolysis, hyperthermia, cerebral edema
E. Spinal cord injury (SCI)
1. Etiology: motor vehicle, falls, assault, sports related, spine surgery, aortic surgery (AAA,
TAA)
2. Diagnosis:
a) Determine level of injury
b) Complete vs incomplete
3. Radiological:
a) plain films
b) CT
c) MRI
4. Management:
a) supportive
b) Induced hypertension
c) C/T/L precautions
d) Steroids (controversial)
e) Surgical Decompression/Stabilization
f) Hypothermic protocol (investigational)
5. Spinal shock:
a) Bradycardia, hypotension, hypothermia.
b) Dopamine pressor of choice for treatment of bradycardia and hypotension. Pacemaker
may be required.
c) Higher levels (usually above T6) associated with autonomic hyper-reflexia
F. Brain death
1. Unresponsive coma:
a) GCS 3T
b) No eye movement or motor responses to noxious stimuli
c) Spinal cord reflexes may be present
2. Apnea:
a) no spontaneous respiration with a documented PaCO2 > 60 (assuming no medication
effects are on board like narcotics or paralytics)
3. Absent brainstem reflexes:
a) unreactive pupils
b) No oculocephalic, oculovestibular reflexes
c) No corneal, cough,or gag reflex
4. Additional tests – cerebral angiography, EEG, TCD, SSEP, brainstem evoked potentials
G. Cardiopulmonary arrest
1. CPR:
a) CAB, no longer ABC! Early, high-quality cardiac compressions and early defibrillation
critical to higher success rate to return of spontaneous circulation
b) Post-cardiac arrest monitoring and resuscitative efforts
(1) Temperature – mild hypothermia (33-34 oC x 24 hours)
(2) Cardiac – Support blood pressure with fluids/pressors
(3) Respiratory – Supportive on ventilator
(4) Endocrine – avoid hypo- and especially hyperglycemia
(5) Fluids/electrolytes/nutrition – maintain euvolemia, good urine output, monitor
electrolytes
(6) Heme – monitor for coagulopathies (DIC)
(7) Infectious – prophylactic antibiotic not indicated
(8) Continuous EEG
2. Outcomes
a) 44% of in-hospital cardiac arrest with ROSC eventually have withdrawal of treatment
190
SOCCA Residents Guide 2013
H. Guillain-Barre’ syndrome
1. Etiology/pathology:
a) preceding infection
b) Immune mediated demyelinaion of cranial and peripheral nerves
2. Diagnosis:
a) Ascending paralysis (Legs weaker than arms and symmetric)
b) CSF analysis
c) EMG and nerve conduction studies
3. Complications:
a) Dysautonomia
b) EKG changes and arrhythmias
c) Syndrome of inappropriate antidiuretic hormone
d) Acute colonic pseudo-obstruction
e) Hypo and hypertension
f) Respiratory Failure
4. Management:
a) Respiratory monitoring
b) Intubation and mechanical ventilation
c) Intravenous immunoglobulin
d) Plasma Exchange
e) Supportive:
(1) Pain
(2) Ileus
(3) Blood pressure
(4) Arrhythmia
I. Myasthenia Gravis:
1. Etiology:
a) Acetylcholine receptor autoantibodies
2. Diagnosis:
a) Regional weakness and fatigue
b) modified Osserman classification
c) EMG
d) Edrophonium or neostigmine test
e) Serology
3. Management:
a) Myasthenic crisis:
(1) Cholinergic drugs
(2) Respiratory monitoring
(3) Intubation and mechanical ventilation
(4) Intravenous immunoglobulin
(5) Plasma Exchange
(6) Corticosteroids
(7) Thymectomy
This chapter is a revision of the original chapter authored by Khaldoun Faris, MD and Faraz A. Syed, DO
References:
1.
2.
3.
4.
The American Heart Association/American Stroke Association Stroke Council, Clinical Cardiology
Council, Cardiovascular Radiology and Intervention Council, and the Atherosclerotic Peripheral Vascular
Disease and Quality of Care Outcomes in Research Interdisciplinary Working Groups: the American
Academy of Neurology affirms the value of this guideline as an educational tool for neurologists. Stroke
2007;38:1655-711.
Morgenstern LB, Hemphill JC, 3rd, Anderson C, et al. Guidelines for the Management of Spontaneous
Intracerebral Hemorrhage. A Guideline for Healthcare Professionals From the American Heart
Association/American Stroke Association. Stroke.
Suarez et al. Aneurysmal subarachnoid hemorrhage. N Engl J Med (2006) vol. 354 (4) pp. 387-96
Wijdicks EFM, Sharbrough FW. New-onset seizures in critically ill patients. Neurology
1993;43:1042-1044.
191
SOCCA Residents Guide 2013
5.
6.
Wijdicks EF, Varelas PN, Gronseth GS, Greer DM; American Academy of Neurology. Evidence-based
guideline update: determining brain death in adults: report of the Quality Standards Subcommittee of
the American Academy of Neurology. Neurology 2010;74:1911-8.
Mayer SA, Chong J: Critical care management of increased intracranial pressure. J Intensive Care Med
2002;17:55-67.
Questions
33.1 Which of the following patient is an ICP monitoring device NOT indicated?
A. 30yo s/p MVA with eye opening to voice, localizes pain, and has incomprehensible speech, CT scan significant
for subdural hematoma and subarachnoid hemorrhage. Vitals 85/52, HR 120, RR 30, SaO2 99%
B. 50yo intubated on arrival to ED without any motor movement to pain. CT scan normal. BP 89/42. HR 88, RR
14, SaO2 100%
C. 29yo admitted for ACOM aneurysm rupture with SAH and intraventricular hemorrhage. BP 188/98, HR 50, RR
18, SaO2 92%. Patient was seen posturing, eyes closed to painful stimulus, and non-verbal.
D. All cases above should have ICP monitoring
33.2 All
A.
B.
C.
D.
E.
of the following affect cerebral blood flow (CBF) except:
PaO2
PaCO2
SVR
Mean arterial pressure
Brain oxygen demand
33.3 A 34 year old previously healthy male is admitted to the ED after being found down at home. Upon arrival, he was
intubated and sedated. CT scan showed large SAH secondary to an aneurysmal bleed with midline shift. An EVD was
immediately placed, which showed ICPs in the 40s. When admitted to the neurosurgical ICU, his vitals were: HR 95, BP
200/98, RR 12, SpO2 99% on mechanical ventilation. Which of the following should be used with caution in this patient?
A. Labetalol
B. Nicardipine
C. 7.5% hypertonic saline
D. Hydralazine
E. Mannitol
33.4 Which of the following EKG findings is relatively common in SAH patients?
A. ST depression
B. ST elevation
C. Prolonged PR interval
D. Widened QRS
E. Right bundle branch block
33.5 A 21yo female was admitted to the ICU for presumed cryptococcus meningitis and sepsis. A pulmonary artery
catheter was placed in addition to an EVD for likely elevated ICPs. Hemodynamic measures are as follows: PA 35/12
(mean 24), PCWP 8, BP 88/42 (mean 57), CVP 16, ICP 18. Which of the following is the correct cerebral perfusion
pressure?
A. 41
B. 40
C. 39
D. 6
E. 8
192
SOCCA Residents Guide 2013
34. Management of Increased Intracranial
Pressure (ICP)
Lauryn R. Rochlen, MD
A 31 year-old female with no significant past medical history presents to the
emergency department following a MVA. GCS at the scene was 14. While in
the emergency room, she becomes unresponsive with a GCS of 7. Her BP =
160/90, HR= 54, RR= 12, SpO2=99% on RA. She is intubated for airway
protection. A head CT obtained reveals extensive bilateral frontal lobe
contusions, intraventricular hemorrhage, cerebral edema and compression of
the lateral ventricles.
Outline:
I.
Physiology
A. Monro-Kellie Doctrine(1)
1. Cranium is a rigid compartment
a) Normal ICP: Adults=7-15mmHg; Children=3-7mmHg
2. Determinants of ICP
a) Brain parenchyma
b) CSF
c) Intracranial Blood
3. Increase in volume of one component or addition of a mass past a threshold volume will
increase intracranial pressure
4. Above a threshold pressure, small increases in volume will result in more profound increases
in ICP and may result in herniation
5. Strategic foundation for management of increased ICP
B. Elastance & Compliance
1. Compliance: Change in Volume for a given change in Pressure (О”V/О”P)
2. Elastance: Change in Pressure for a given change in Volume (О”P/О”V)
a) More physiologically accurate description of intracranial mechanics
b) Flat lower portion: compensatory reserve; ICP remains low despite increase in
intracranial volume
c) Steep rise: low compensatory reserve; large increase in ICP with small increases or
intracranial volume
d) High levels of ICP: curve plateaus again; arterial is at maximal dilation and may even
start to collapse; CPP is low and ICP may even equal MAP
C. Cerebral Blood flow determinants
1. Blood pressure
a) Autoregulation intact between MAP 50-150mmHg
b) Injured brain results in malfunction of autoregulation
c) Chronic hypertension shifts autoregulatory curve to the right
2. PaCO2
a) Linear relation between PCO2 20-80mmHg
3. PaO2
a) Begin to see increased CBF when PaO2<50mmHg
4. CMRO2(cerebral metabolic rate of oxygen)
a) Increased metabolic activity results in increased blood flow
D. Cerebral Perfusion Pressure
1. Indirect measure of cerebral blood flow(CBF)
2. CPP=MAP-ICP
3. Recommend maintaining CPP>50-70mmHg
II.
Etiology of increased ICP
A. Cytotoxic edema
1. Direct neuronal injury
193
SOCCA Residents Guide 2013
2. Sodium influx results in increased intracellular brain water
B. Vasogenic edema
1. Increased extracellular brain water derived from vasculature
2. Transependymal edema(Hydrocephalus)
3. Shifting of fluid from the ventricular system in the brain interstitium
4. Occurs due to obstruction of normal CSF outflow pathways
C. Osmotic edema
1. Rapid or profound decrease in serum osmolarity
2. Venous obstruction
3. Increased brain volume
4. Increased blood volume
D. Mass effect
III.
Neuroimaging
A. Non-contrast Head CT
1. Obtain if increased ICP is suspected
2. Findings that support elevated ICP
3. Cerebral edema
4. Compression of basal cisterns
5. Hydrocephalus
6. Midline shift
7. Mass effect
B. MRI
1. Not appropriate in setting of acute increased ICP
2. More accurate assessment of brain water content or underlying lesions
IV.
ICP Monitors
A. Rationale for placement
1. Optimization of CPP
2. Warns of abnormal values before change in neurologic status or if unable to follow neurologic
exam
B. Indications for placement
1. Standard
a) Moderate to severe brain injury and GCS<8
b) Moderate to severe brain injury and hypotensive(SBP<90mmHg) or intubated
2. Other
a) Subarachnoid hemorrhage with symptomatic hydrocephalus, massive hemispheric
strokes, diffuse cerebral edema, fulminant liver failure, neuroimaging signs of elevated
ICP
C. Intraventricular
1. Inserted into one of the lateral ventricles
2. Advantages
a) Gold standard
b) Allows for therapeutic/diagnostic draining of CSF
c) Re-zeroing possible
3. Disadvantages
a) Risk of infection, tissue damage during placement, hematoma formation
b) Placement can be challenging as it is placed blindly
D. Intraparenchymal
1. Directly measures brain tissue pressure using sensors placed in cortical gray matter
2. Advantages
a) Low infection rate
3. Disadvantages
a) Risk of tissue damage during placement
b) Cannot drain CSF
c) Cannot be recalibrated once placed
d) Drifts from zero
E. Subdural bolt
194
SOCCA Residents Guide 2013
1.
2.
Hollow screw threaded into skull and placed in subdural space
Advantages
a) Low infection rate
b) No brain penetration
3. Disadvantages
a) Limited accuracy
b) High failure rate
F. Epidural
1. Pressure sensor membrane in contact with dura
2. Advantages
a) Low infection rate
b) Easy to insert
c) No penetration of dura
3. Disadvantages
a) Limited accuracy
V.
Management of ICP
A. Indications
1. Prevention of secondary injury
a) Initial insult causes cell injury and necrosis which results in cerebral edema, leading to
increased ICP
b) When ICP is increased, perfusion to area surrounding injury is at risk of ischemia and
further injury
2. Decrease in CPP due to increased ICP
B. Goals
1. Maintain CPP
2. Maintain oxygen delivery
3. Reduce CMRO2 and CBF, thereby reducing cerebral blood volume(CBV)
4. Prevent worsening of injury/edema
C. Prophylactic measures
1. Avoid hypercarbia and hypoxemia
2. Avoid prophylactic hyperventilation
3. Maintain normoglycemia
4. Maintain normothermia
5. Avoid hypotension/hypovolemia
6. Sedation and analgesia
7. Head of bed elevated >30В°
8. Avoid head turning
9. Low level of PEEP
10. Low tidal volume ventilation
11. Seizure prophylaxis
D. Treatment measures
1. Acute interventions
a) Non-pharmacologic
(1) Hyperventilation
(a) Vasoconstricts cerebral arterioles and reduces CBV
(b) May contribute to cerebral ischemia
(2) Hypothermia
(a) Reduces CMRO2 and CBF
(b) Increased risk of infection
(c) Requires sedation and mechanical ventilation
(3) CSF drainage
(a) Reduces CSF volume
(4) Decompressive craniectomy
(a) Reduces mass effect and CBV
(5) Risk of surgical morbidity
2. Pharmacologic
a) Mannitol
195
SOCCA Residents Guide 2013
(1) Osmotic and vasoconstrictive effects reduce CBV
(2) May see rebound effect with prolonged use
(3) Risk of hypovolemia and renal failure
(4) Dose range=0.5 – 1.5g/kg bolus
(5) Titrate to serum osmolality>310
(6) Peak effect is 20-60 minutes
(7) Duration of action is 4-6 hours
b) Hypertonic Saline
(1) Osmotic and vasoconstrictive effects reduce CBV
(2) May see rebound effect with prolonged use
(3) Risk of hypervolemia, dilutional coagulopathy
(4) Shown to be more effective than mannitol for acutely reducing ICP
c) Steroids
(1) May reduce vasogenic edema associate with tumors
(2) Shown to increase mortality in acute brain injury patients
d) Propofol
(1) Reduces CMRO2 and CBF
(2) Risk of hypotension
(3) Risk of propofol infusion syndrome and hypertriglyceridemia
(4) Titrate to EEG
e) Barbiturates
(1) Reduces CMRO2 and CBF
(2) Risk of hypotension
(3) Risk of multi-system organ dysfunction
(4) Long half-life
(5) No difference between thiopental and pentobarbital
(6) Titrate to EEG
f) Neuromuscular blockade
(1) Abolishes muscle activity and Valsalva which results in increased ICP
(2) Main disadvantages include loss of neurologic exam, risk of critical illness
myopathy, and requirement for mechanical ventilation
Figure 34-1. Intracranial pressure-volume curve
196
SOCCA Residents Guide 2013
Figure 34-2. Cerebral perfusion pressure (CPP) in relation to oxygenation, ventilation,
and ICP.
Discussion:
Elevated intracranial pressures due to cerebral injury or intracranial mass are a result of the physiologic
principles of the Monro-Kellie doctrine and can lead to irreversible cell damage and dysfunction. To
prevent secondary injury and prolonged ischemia, it is paramount to treat sustained increases in ICP. ICP
monitors can help guide therapy. It is best to start treatment with less invasive approaches, such as
hyperventilation and ensuring proper head positioning. If ICP remains refractory, more invasive therapies
may need to be instituted, such as barbiturate coma or decompressive hemicraniectomy.
References:
1.
2.
3.
4.
Bershad EM, Humphreis WE, 3rd, Suarez JI. Intracranial hypertension. Semin Neurol. 2008 Nov;28(5):
690-702.
Steiner LA, Andrews PJ. Monitoring the injured brain: ICP and CBF. Br J Anaesth. 2006 Jul;97(1):26-38.
Meyer MJ, Megyesi J, Meythaler J, Murie-Fernandez M, Aubut JA, Foley N, et al. Acute management of
acquired brain injury part II: an evidence-based review of pharmacological interventions. Brain Inj.24(5):
706-21.
Meyer MJ, Megyesi J, Meythaler J, Murie-Fernandez M, Aubut JA, Foley N, et al. Acute management of
acquired brain injury part I: an evidence-based review of non-pharmacological interventions. Brain Inj.
24(5):694-705.
197
SOCCA Residents Guide 2013
Questions:
K-type:
A=1,2,3
B=1,3
C=2,4
D=4
E=all of the above
34-1. Which of the
1.
2.
3.
4.
following are TRUE regarding cerebral autoregulation:
Arterial CO2 is an important factor
CBF is maintained over a wide MAP range
Chronic hypertension results in altered parameters of autoregulation
Autoregulation is intact in injured areas of brain
34.2. Which of the
1.
2.
3.
4.
following are indications for placement of an ICP monitor:
Moderate-Severe brain injury and GCS<8
Mild-Moderate brain injury
Moderate-Severe brain injry and hypotension
Intracranial tumor with peri-tumor edema
34.3. Pharmocologic interventions for treatment of elevated ICP due to traumatic brain injury include all EXCEPT:
1. Propofol
2. Mannitol
3. Barbiturates
4. Corticosteroids
198
SOCCA Residents Guide 2013
35. Traumatic Brain Injury
Scott Wolf, MD
A 35 year-old male is admitted to the neurocritical care unit after having
been an unrestrained passenger in a motor vehicle crash. He does not open
his eyes to painful stimuli, he flexes his extremities in response to pain, and
his speech is unintelligible. His is not intubated and is wearing a hard
cervical collar.
I.
Epidemiology
It is estimated that 1.1-1.5 million Americans are treated for traumatic brain injury (TBI) each year.
A. Over 200,000 patients per year are admitted to hospitals for non-fatal TBI
B. Approximately 50,000 Americans die of TBI each year
C. Of survivors, over 40% will develop long-term disability from TBI
D. The major risk factors for TBI in the United States include age, gender, and low socioeconomic
E. TBI occurs more commonly in those younger than 10 and older than 70 years old, and is twice as
frequent in males than females
F. The leading causes of TBI in the United States are falls, motor-vehicle collisions, and assaults.
G. The total lifetime costs of all TBI cases in the year 2000 exceeds $60.0 billion, with productivity
losses exceeding $50.0 billion and a per patient lifetime cost of $44,000.
II.
Classification
A. Traumatic brain injury (TBI) can be classified according to mechanism, severity, or morphology see table 35-1.
B. The most common scoring system for the severity of TBI is based on the Glasgow Coma Score
(GCS) - see table 35-2.
1. Mild TBI: GCS 13-15
2. Moderate TBI: GCS 9-12
3. Severe TBI: GCS 8 or below
C. Anesthesiologists and critical care physicians are most likely going to encounter patients with
severe TBI because of their need for definitive airway control, mechanical ventilatory support,
invasive hemodynamic monitoring and support, operative intervention, and ICU admission.
III.
Primary and secondary assessment
A. The primary assessment should be the same as that for all trauma patients beginning with an
assessment of airway, breathing, and circulation
B. A thorough secondary assessment should be done to avoid missing injuries.
IV.
Neurological injury
The outcome from traumatic brain injury is governed by the degree of primary and secondary injury.
A. Primary injury describes the neurological injury sustained at the time of the initial trauma,
resulting from:
1. Shearing
2. Compression
3. Disruption of membranes
4. Axonal injury
B. Secondary injury describes the injury to tissue not directly involved in the initial insult. It occurs
due to a cascade of cellular events which evolves over time and includes but is not limited to:
1. Ischemia-reperfusion
2. Accumulation of excitiotoxins
3. Inflammation
4. Oxidant injury and oxygen free radical formation
5. Apoptosis
C. The primary injury is generally considered not modifiable and irreversible
D. The goal of treatment of severe traumatic brain injury is to prevent secondary injury by avoiding
modifiable risk factors, and usually includes neurosurgical intervention, hemodynamic support,
199
SOCCA Residents Guide 2013
ventilatory support, control of intracranial pressure (ICP), pharmacologic or adjuvant
neuroprotection, and treatment of medical complications.
V.
Primary injury
A. Numerous types of injuries can occur at the time of the initial insult and are described here:
1. Skull fractures
a) May be associated with either penetrating or closed injuries
b) Are linear, depressed, or comminuted
c) Suggest that significant force was involved
d) Depressed skull fractures have the highest risk of underlying parenchyma injury and
dural tears
e) Temporal bone fractures crossing the middle meningeal artery are associated with
epidural hematomas
f) Basilar and occipital bone fractures indicate high-intensity impact and are associated with
facial, acoustic, and vestibular nerve dysfunction, periorbital ecchymoses, Battle’s sign,
and hemotympanum
2. Epidural hematoma
a) Most commonly associated with temporal bone fractures damaging the middle meningeal
artery
b) Arterial bleeding causing hematoma between the skull and dura mater which appear
biconvex on CT scan
c) May have a lucent period in between initial loss of consciousness and rapid deterioration
d) Respond well to prompt intervention and evacuation
3. Subdural hematoma
a) Defined by the presence of blood between the dura mater and the pia-arachnoid mater
b) Acute subdural hematomas are usually due to brain laceration or disruption of bridging
veins between the cortex and dural sinuses
c) Brain compression from the hematoma together with brain edema can cause significant
elevations in intracranial pressure
4. Intracerebral hematoma
a) Occur most commonly in the frontal and temporal lobes where contusion injury is
common
b) Often occurs as delayed phenomenon causing secondary hemorrhage known as a
“blossoming” contusion
c) Extravasated blood may incite other secondary injury processes including inflammation,
edema, mass effect, increased ICP, oxidative stress and cell death
5. Subarachnoid hemorrhage
a) Occurs commonly in TBI but rarely requires neurosurgical intervention
b) Intraventricular extension may necessitate CSF drainage via a ventriculostomy catheter to
prevent outflow obstruction and hydrocephalus
6. Diffuse axonal injury (DAI)
a) DAI is frequently associated with deceleration injury in which shearing forces
causes widespread disruption of axonal fibers and myelin sheaths
b) CT findings show small diffuse petechial hemorrhages in the white matter without the
presence of gross structural lesions
c) May cause acute or prolonged loss of consciousness
d) In infants DAI is the predominant injury in shaken baby syndrome
7. Cerebral edema
a) Occurs commonly in contusion injury
b) Brain edema may lead to elevated ICP and jeopardize cerebral perfusion
c) Does not respond to steroid therapy, and treatment of severe traumatic brain injury with
steroids is not recommended
d) Patients should be kept euvolemic and normosmolar to slightly hyperosmolar to mitigate
brain swelling and edema
e) Mannitol titrated to a serum osmolarity of 310-320 mOsm/kg or hypertonic saline titrated
to a serum sodium of 145-155 meq/L are suitable options
f) Decompressive craniectomy with creation of a large dural flap may be necessary to
control ICP
200
SOCCA Residents Guide 2013
VI.
Secondary Injury
A. Secondary injury describes the injury to tissues not directly involved in the initial insult but
become injured as a result of a cascade of cellular events including ischemia-reperfusion,
inflammation, accumulation of excitotoxins, oxidant injury, and apoptosis
B. Unlike primary injury, secondary injury is thought to be modifiable and preventable provided that
adequate oxygen delivery to cells and tissues is maintained by ventilatory and hemodynamic
support, control of ICP, and control of complicating medical conditions
C. The treatment of severe TBI is primarily focused on preventing and treating secondary brain injury
VII. Treatment to prevent secondary injury
A. Airway Management
1. Patients with severe head injury may also present with craniofacial trauma or injury to the
cervical spine making endotracheal intubation difficult
2. Early securement of the airway is encouraged, especially patients with GCS < 8 or patients
with severe head and neck injuries
3. 5-10% of severe TBI patients have concomitant cervical spine injury, and half of those have a
cervical spinal cord injury
B. Ventilatory management
1. Maintain the PaO2 at a minimum of 60-70 mmHg
2. Adding PEEP to improve oxygenation does not adversely affect ICP until the PEEP is greater
than or equal to 20cm H2O
3. The PaCO2 should be maintained normal to slightly below normal
4. The indiscriminate use of hyperventilation should be avoided
5. Hyperventilation should only be utilized to treat acute increases in ICP, but once the ICP is
normalized, normocapnia should be resumed
6. Be careful of exacerbating hypotension through the use of mechanical ventilation
C. Hemodynamic management
1. Reduced cerebral perfusion pressure due to intracranial hypertension or inadequate blood
pressure contributes to a fall in cerebral blood flow
2. Adequate intravascular volume should be restored by administration of isotonic crystalloid or
colloid solutions
3. Maintain adequate mean arterial pressure (MAP) and cerebral perfusion pressure (CPP),
where CPP = MAP – ICP; a CPP greater than 60 mmHg is preferred
4. Avoid any episode of hypotension with a systolic blood pressure less than 90 mmHg
D. Control of ICP
1. According to the Monro-Kellie doctrine, the total volume of intracranial contents is constant
2. The cranium is a rigid non-expansible container
3. Eventually, small increases in the volume of CSF, blood, or parenchyma swelling will cause
significant increases in intracranial pressure
4. As ICP increases, the CPP decreases
5. Every effort must be made to decrease the ICP to less than 20 mmHg
6. Strategies to decrease ICP may include:
a) Elevation of the head of bed to 30 degrees
b) Maintain the head and neck in neutral position to allow venous drainage
c) Judicious administration of sedation and analgesia to avoid pain and agitation
d) Placement of intraventricular drain to allow CSF drainage
e) Evacuation of hematoma formation early
f) Decompressive craniectomy to allow for brain swelling and edema
g) Administration of hypertonic saline or mannitol as osmotic therapy to limit cerebral
edema
h) Intubation and hyperventilation to decrease cerebral blood flow
i) Barbiturate coma
j) Avoidance of hyperthermia
k) Induced hypothermia
E. Treatment of complicating medical conditions (seizure, anemia, hyperglycemia, coagulopathy,
electrolyte abnormalities)
1. Patients with severe traumatic brain injury have a higher incidence of seizures in the long
201
SOCCA Residents Guide 2013
term compared to the general population
Seizures should be treated with antiepileptic drugs and supportive care should they occur
Some practitioners favor the prophylactic use of antiepileptic drugs for the first 7 days post
injury
4. Patients with severe traumatic brain injury should maintain a hemoglobin of 10 g/dL or a
hematocrit of 30%, at least in the acute phase
5. Strict control of blood glucose levels to avoid hyperglycemia greater than 110-140 mg/dL and
avoidance of hypoglycemia < 90 mg/dL
6. Coagulopathy should be rapidly corrected with the use of iv vitamin K and through blood
component therapy to achieve a normal PT/INR, PTT, and a platelet count greater than
100,000
F. Hypothermia
1. Patients that are mildly hypothermic at presentation have an improved prognosis compared to
patients that are hyperthermic or normothermic
2. Whether inducing hypothermia improves outcome remains controversial
3. Recent animal studies showing improvement with induced mild hypothermia has renewed
interest in this treatment
4. Animal studies have primarily focused on short-term histopathological outcomes, as
compared to human studies that have focused on long-term functional outcomes
5. Hypothermia can be used to treat refractory increases in ICP, however that has not been
shown to improve mortality
G. Pharmacologic neuroprotection and experimental therapies
1. Numerous studies dedicated to pharmacologic cerebroprotection have been attempted
2. These studies have included the use of calcium channel blockers, steroids, antiinflammatories, magnesium, glutamate receptor blockers, and nitric oxide inhibitors
3. None of these have been shown to be beneficial in human trials
2.
3.
Table 35-1: Classification of Traumatic Brain Injury
Mechanism
Severity
Morphology
Blunt
High velocity (automobile collision)
Low velocity (fall, assault)
Penetrating
Gunshot wounds
Other penetrating injuries
Mild
GCS Score 14-15
Moderate
GCS Score 9-13
Severe
GCS Score 3-8
Skull Fractures
Vault
Linear vs stellate
Depressed vs non-depressed
Open vs closed
Basilar
With or without CSF leak
With or without VII nerve palsy
Intracranial
Lesions
Focal
Epidural
Subdural
202
SOCCA Residents Guide 2013
Intracranial
Focal
Lesions
Table 35-1: Classification of Traumatic Brain Injury
Intracerebral
Diffuse
Concussion
Multiple contusions
Hypoxic/ischemic injury
Table 35-2: Glasgow Coma Scale
Motor
Verbal
Eye
6 - Obeys commands fully
5 - Alert and oriented
4 - Spontaneous eye opening
5 - Localizes to noxious stimuli
4 - Confused, yet coherent
speech
3 - Eyes open to speech
4 - Withdraws from noxious
stimuli
3 - Inappropriate words and
jumbled phrases consisting of
words
2 - Eyes open to pain
3 - Abnormal flexion, i.e.,
decorticate posturing
2 - Incomprehensible sounds
1 - No eye opening
2 - Extensor response, i.e.,
decerebrate posturing
1 - No sounds
1 - No response
Add the numbers from M-V-E to get the total score. Minimum is 3, maximum is 15
References:
1.
2.
3.
4.
5.
6.
Corrigan, John: The Epidemiology of Traumatic Brain Injury. J Head Trauma Rehabil 2010; Vol 25, No 2:
72-80.
Parikh, Samir, Koch, Marcella, Narayan, Raj: Traumatic Brain Injury. Int Anesthesiol Clin 2007;
45:119-35.
Aarabi, Bizhan, Simard, J Marc: Traumatic Brain Injury. Curr Opin Crit Care 2009; 15: 548-553.
Young, Neil et al: Ventilator Strategies for Patients with Acute Brain Injury. Curr Opin Crit Care 2010; 16:
45-52.
Li, Lucia et al: The Surgical Approach to the Management of Increased Intracranial Pressure After
Traumatic Brain Injury. Anesth Analg 2010; 111: 736-748.
Badjatia, Neeraj: Hyperthermia and Fever Control in Brain Injury. Crit Care Med 2009; Vol 37, No 7:
250-257.
203
SOCCA Residents Guide 2013
36. Renal Protection
Daniel R Brown, M.D., Ph.D.
A 74-year-old woman with a history of hypertension and diabetes mellitus
presents for urgent coronary artery revascularization. Her preoperative
serum creatinine is 2.2 mg/dL. Urine output is low post-operatively.
Pathophysiology
Acute kidney injury (AKI) is a associated with significant morbidity and mortality. The RIFLE criteria
attempt to standardize the classification of acute renal failure (ARF). RIFLE stands for Risk of renal
dysfunction; Injury to the kidney; Failure of kidney function; Loss of kidney function; and End-stage
kidney disease. This scheme is based on serum creatine or glomerular filtration rate and urine output.
More recently, the RIFLE criteria have been modified to classify AKI into three stages. The three stages of
AKI correspond to the Risk, Injury and Failure categories of the RIFLE scheme. It is important to
recognize that the AKI stages have limited validation in terms of risk but represent a standardized means by
which to communicate the degree of renal impairment, a factor which was not present in the medical
literature previously and which limits interpretation of much of the literature in this field. Also of note is
that serum creatinine and urine output not sensitive or specific markers of AKI. Significant changes in
renal function may occur prior to changes in commonly used renal biomarkers (e.g. serum creatinine).
Possible biomarkers of AKI include neutrophil gelatinase-associated lipocalin (NGAL) and cystatin C.
Causes of AKI are generally divided into prerenal, intrinsic renal and postrenal causes (Figure 1). Prerenal
AKI is secondary to absolute or relative renal hypoperfusion. Such hypoperfusion, if not corrected, will
result in ischemic acute tubular necrosis (ATN). Intrinsic causes of AKI may occur at various anatomic
locations within the kidney and injury may be secondary to a variety of factors including intrarenal
autoregulation, inflammation and toxin exposure. Postrenal AKI are caused by obstruction of the urine
collection system. ATN is the most common cause of AKI. The pathogenesis of ATN is multifactorial and
appears to involve both ischemia and inflammation.
Identifying risk factors for AKI is difficult secondary to the previous lack of standardized definitions for
AKI and ARF. Commonly cited risk factors include elevations in baseline serum creatinine or blood urea
nitrogen (BUN), prior renal dysfunction, toxin exposure, disease states associated with hypotension such as
sepsis and cardiogenic shock, and certain procedures such as vascular and cardiac surgery.
Management
While there is general agreement that hypovolemia should be avoided, �adequate’ intravascular volume is
difficult to define. Recommendations to maintain MAP > 65-70 mmHg, CVP 10-15 mmHg, PAWP 10-15
mmHg and UO > 0.5 cc/kg/hr are widespread but represent expert opinion and need to be balanced with the
potential adverse effects related to excessive fluid resuscitation. The ideal fluid for resuscitation is also
unclear. Aggressive normal saline may lead to hyperchloremic metabolic acidosis. Some reports have
suggested an association between administration of hydroxyethyl starch solutions and increased risk of AKI
and death though this remains controversial.
The optimal MAP to maintain renal perfusion is not known. Autoregulation of renal blood flow occurs
over a wide variety of systemic pressures though how this impacts the low MAP that still results in
acceptable renal perfusion is also unclear. Of note, increased intraabdominal pressure either during
laparoscopic procedures or in the setting of intraabdominal hypertension, such as following emergent
abdominal surgery, also may contribute to decreased renal perfusion. The exact threshold for
intraabdominal hypertension to impact renal perfusion is not known but concerns are generally raised when
intraabdominal pressures exceed 20-30 mmHg.
Many medications can potentially cause or exacerbate AKI. Antihypertensive drugs may limit the patient’s
ability to compensate for hypotension and result in prerenal insult. Furthermore, intrarenal autoregulation
may also be impacted and promote AKI. Antibiotics may be associated with interstitial nephritis. NSAIDs
alter prostaglandin synthesis and may alter renal blood flow and are also associated with interstitial
nephritis. Radiologic contrast dye administration is associated with ATN. Multiple studies have
204
SOCCA Residents Guide 2013
investigated potential prophylactic strategies to mitigate this risk. Pre- and post procedure hydration
appears to decrease risk while the administration of sodium bicarbonate and N-acetylcysteine is less clear
as there are both negative and positive results using these two interventions. Aprotinin, a serine protease
inhibitor used to limit bleeding in certain procedures, appears to increase the risk of AKI.
�Renal’ dose dopamine does not protect patients from developing AKI however and does have potential
deleterious side effects. Diuretic administration does not appear to change the frequency of renal
replacement therapy in the setting of AKI but has been associated with increased mortality. Fenoldopam
mesylate, a dopamine-1 receptor agonist used in the management of hypertensive emergencies, has been
studied in various populations at risk for AKI. A meta-analysis suggested that fenoldopam may be
associated with reduced incidence of AKI and mortality in high risk patients though larger trials are
necessary to validate these observations.
Discussion
AKI is a common condition in the intensive care unit and is associated with significant morbidity and
mortality. Recent AKI definitions should allow for better communication between clinicians as well as
investigative study designs while new biomarkers are needed to allow for earlier diagnosis of AKI. Causes
of AKI can be divided into pre-, intra- and postrenal etiologies. Hallmarks of management include
maintaining adequate intravascular volume and renal perfusion while avoiding nephrotoxins. Currently
there are no approved pharmacologic therapies for treatment or prevention of AKI though specific therapies
and targeted patient populations are actively being pursued.
References
1.
2.
3.
4.
5.
Bellomo R, et al. Acute renal failure- definition, outcome measures, animal models, fluid therapy and
information technology needs: the Second International Conference of the Acute Dialysis Quality
Initiative (ADQI) Group. Critical Care 8(4): R204-R212, 2004.
Jones DR and HT Lee. Perioperative renal protection. Best Practice & Research Clinical Anesthesiology
22(1): 193-208, 2008.
Lameire N, BW Van Biesen and R Vanholder. Acute renal failure. Lancet 365: 417-30, 2005.
Landoni G, et al. Benficial impact of fenoldopam in critically ill patients with or at risk for acute renal
failure: a meta-analysis of randomized clinical trials. Am J Kidney Dis 49: 56-68, 2007.
Mehta RL, et al. Acute kidney injury network: report of an initiative to improve outcomes in acute kidney
injury. Critical Care 11(2): R31, 2007.
Table 36-1: Causes of AKI and associated treatment
Prerenal
Intrarenal
Postrenal
Causes
Hypovolemia
(relative or absolute)
Hypotension
(relative or absolute)
Urine collection system
obstruction
Treatment
Fluid resuscitation
Hemodynamic optimization
Inflammation
Intrarenal ischemia
Direct drug toxicity
Endogenous toxins
(e.g. myoglobin)
Intraabdominal
hypertension
Avoid or limit exposure
Relief of obstruction
205
SOCCA Residents Guide 2013
QUESTIONS:
36.1 Which of the following factors are used in the RIFLE criteria for classifying acute renal failure?
1.
2.
3.
4.
Serum creatinine
Glomerular filtration rate
Urine output
Age
36.2 Which of the following statements concerning fluid management and AKI are true?
1.
2.
3.
4.
Colloids are superior to crystalloids
Fluid management is best guided by urine output
Normal saline is hypotonic compared to plasma
Normal saline administration may result in a hyperchloremic metabolic acidosis
36.3 Which of the following statements concerning NSAIDs and AKI are true?
1.
2.
3.
4.
NSAIDs can contribute to ischemic ATN and acute interstitial nephritis
NSAIDs alter intrarenal prostaglandin synthesis and autoregulation
Chronic NSAID use predisposes patients to developing chronic kidney disease
NSAIDs should be held in the perioperative period in patients with normal renal function for fear of
causing AKI
206
SOCCA Residents Guide 2013
37. Renal Replacement Therapy
Shahriar Shayan M.D.
68 yr male emergently undergoes an exploratory laparatomy for acute
peritonitis. He had recently had an urgent type A dissection repair (3 weeks
prior). His post-operative course was complicated by prolonged mechanical
ventilation, ventilator associated pneumonia and a persistent respiratory
failure as a result necessitating tracheostomy and J-tube placement for
malnutrition. He eventually made progress and was discharged to a low
acuity rehabilitation center for further physiotherapy. On the morning of
admission he was found to have an increased abdominal distension, and
mental status changes. Radiological studies confirmed the diagnosis of a
dislodged J tube.
During his post-laparatomy course in ICU, he remained tachycardic and
hypotensive, requiring vasopressor and a urine output of < 0.3 cc/kg/hr for
> 12hrs.
I.
Pathophysiology
Renal failure is common in ICU. Up to 25% of all critically ill patient may have acute kidney injury
(AKI)1 and about 4 % would go on further to require renal replacement therapy (RRT). The mortality
of AKI requiring RRT, independent of any other co-morbid factor is about 60%2. Pathophysiology
involved in AKI is multifactorial, and related to hypoperfusion leading to ischemia and direct
nephrotoxic effect exerted by medications and agent (i.e. contrast dye).
II.
Indications
A. Fluid Overload: as a result of oliguric/anuric AKI, either as result of direct injury or pre-ernal
causes such as acute decompensated heart failure.
B. Metabolic derangement
C. Hyerkalemia: as a result of intra-abdominal infections, or Rhabdomyolysis.
D. Hypo/hypernatremia.
E. Metabolic acidosis: refractory metabolic acidosis (typically pH < 7.2) resulting in hemodynamic
instability.
F. Uremia: BUN > 100, or any uremia related complications such as pleuritis, pericarditis, uremic
encephalopathy, coagulopathy (mainly platelet dysfunction).
G. Drug overdose.
H. Sepsis: controversial.
III.
Physiological Principles
There are different modalities of RRT. Regardless of the type of RRT solute clearance and fluid
removal are achieved through either diffusion or convection.
A. Principle of Diffusion (Figure 38-1):
1. Movement of solutes from an area of HIGH
concentration to an area of LOW
concentration across of a semi-permeable
membrane.
2. Blood and dialysate are moving in
opposite directions (a counter current
fashion), maintaining a maximal
concentration gradient.
3. The efficiency in clearance of solute will
depend on the rate of the dialysate flow
and the membrane permeability.
4. Accumulation of medium to large size
molecules within the membrane will
likely occur over time, adversely affecting the efficiency
207
SOCCA Residents Guide 2013
of the diffusion.
B. Principle of Convection (Figure 38-2):
1. Movement of solute occurs along with fluid
across the semi-permeable membrane
according to PRESSURE gradient
(filtrate).
2. Blood and dialysate are moving in the
same direction.
3. The rate of filtrate formation depends
on the hydrostatic pressure across the
membrane.
4. The overall efficiency of solute removal
is limited by blood flow.
C. Principle of Solute Removal:
1. The rate of removal of a solute (X) is
determined by the coefficient of
clearance of that solute (K) over the time period (t) and
its volume of distribution (V). This relationship is expressed in the following equation
Kt/V=1
2. Theoretically the attempt should be made to remove the solute in its entire volume of
distribution.
Table 37-1: Molecular Size of Dialysates
IV.
Large
Medium
Small
Albumin - 50,000
Vitamin B12 - 1300
Urea - 60
B2 Microglobulin - 11m000
Glucose - 180
Potassium - 35
Creatinine - 110
Phosphorus - 30
Phosphate - 80
Sodium - 20
Modality
A. Hemofiltration (HF):
1. Also known as ultrafiltration (UF) or plasma ultrafiltration (PUF).
2. Uses convection principle.
3. Appropriate for fluid and byproducts removal in state of BODY volume overload.
4. Semi-permeable membrane used in UF is much less permissive compared to HF thereby
allowing only very small molecule to be cleared by UF.
5. Subtypes
a) Continuous Veno-Venous Hemofilration (CVVH):
(1) Porous semi-permeable membrane (medium to large size molecules).
(2) Ideal to use in hemodynamically unstable patient to rapidly obtain a metabolic and
acid-base balance.
(3) Rate limiting step is the blood flow.
(4) As the filtrate formation is high the blood gets rapidly concentrated as it moves
from the proximal to distal end, hence potentially causing filter thrombosis.
(5) Therefore it needs anticoagulation (limited to circuit); citrate vs. unfractionated
heparin.
(6) Also the blood returning to patient needs to be diluted with replacement solutions
prior to re-infusion.
(7) Efficacy of cytokine removal in sepsis postulated, however clear lack of clinical
evidence. The actual mechanism of cytokine removal is filter adsorption.
(8) It needs placement of a large bore dialysis catheter in femoral or subclavian venous
208
SOCCA Residents Guide 2013
territories.
(9) Electrolytes (particularly inonized calcium) need to be monitored.
b) Slow Continuous Ultra-filtration (SCUF):
(1) It utilizes the same principles as above.
(2) The semi-permeable membrane is very small, effectively allowing large volume
removal.
(3) Ideal for acute heart failure with pulmonary congestion unresponsive to diuretic
therapy.
(4) It may be carried out with large bore peripheral intravenous catheter, depending on
the technology available.
B. Dialysis:
1. Also known as hemodialysis (HD).
2. Uses diffusion principle.
3. It may be intermittent vs. continuous therapy
4. Much more efficient to remove small to medium solutes as a result of the counter-current
flow. The higher the hemodialysate flow the higher the concentration gradient across the
semi-permeable membrane, thereby promoting solute removal.
5. It may be accomplished over shorter period of time depending o the technology.
6. Ideal for outpatient setting.
7. It may be performed through temporary indwelling catheter as well as A-V fistulae.
8. Subtypes:
a) Continuous Veno-venous hemodialysis (CVVHD):
(1) The rate limiting step is the blood flow to the machine. If the patient is
hemodynamically stable it could achieve better solute small to medium solute
removal than compared to CVVH, but rarely the case in ICU.
b) Continous Artero-Venous Hemodialysis (CAVH);
(1) Rarely used.
(2) Advantage: It uses the patient blood pressure as the driving force for blood flow.
(3) Disadvantage: indwelling arterial & venous cannula, heparinization, and potential
for bleeding complication.
c) Intermittent Hemodialysis (IHD):
(1) It is reserved for hemodynamically stable patients.
(2) Depending on the hemodyamic parameters it can achieve high blood-dialysate flow
rate, therefore removing byproducts at higher rate.
(3) As the name suggest in can be achieved in matter of hours.
(4) Incidence of inflammatory reaction and thrombocytopenia similar to other modes.
C. Hybrid modes:
1. Combination of diffusive and convective physics.
2. Better overall volume management AND solute removal.
3. Subtypes:
a) Continous Veno-Venous Hemodia-Filtration (CVVHDF).
b) CAVHD.
c) Sustained Low Efficiency Dialysis (SLED):
(1) It is also known as Extended Daily Dialysis.
(2) It is very similar to CVVHDF.
(3) It uses the dual capacity machines.
(4) It is typically prescribed between 6-12 hrs per treatment.
(5) The blood –dialysate flow rate is roughly 70-300 ml/min.
(6) Excellent hemodynamic profiles. Clearly the future of renal support therapy in ICU.
209
SOCCA Residents Guide 2013
Table 37-2 Comparison of IHD and CRRT
Advantage
Disadvantage
IHD
• Very  rapid  removal  of  solute
• Outpa4ent  se6ng  op4on
• Normothermia  maintained
• Does  not  require  an4coagula4on
• Less  expensive  filter  circuit
• More  hemodynamic  instability
• Requires  expert  staff  to  operate
• Nutri4onal  consequences
• Surgical  catheter  placement  or  fistula  crea4on  for  long-­‐term  use
CRRT
• Rapid  volume  removal  and  acidosis  correc4on
• Less  effect  on  hemodynamics
• BeGer  middle  size  molecule  removal
• Nurse  protocol  run
• Clearance  capacity  limited
• Requires  an4coagula4on
• Hypothermia  common
• Needs  a  dedicated  temporary  catheter
• More  expensive  filter  circuit
V.
Renal Failure in ICU and RRT: Current Clinical Observations
A. There are two facts that remain true in all patients with renal failure in ICU:
1. Only a small percentage of patients with renal failure progress to irreversible damage
requiring dialysis.
2. Today, there are truly no methods of predicting the risk factors for developing renal failure in
ICU patients.
B. RRT and sepsis
The utility of RRT in sepsis in the absence or presence of renal failure has been advocated. As a
significant portion of this population has hemodynamic instability and leaky capillary syndrome
hemofiltration using porous membrane has been proposed as an effective mean of eliminating the
circulating cytokines and “evil Humor” that may be responsible for diffuse cellular. There are
currently no solid clinical evidence pointing to any beneficial survival effect of RRT on the
mortality from sepsis, using high vs. regular volume hemofiltration.3
C. Timing of RRT
Indications of RRT initiations were previously outlined. However in many circumstances the there
is no life threatening indication (i.e. hyperkalemia, refractory acidosis), and often the institution of
the RRT is based on clinical judgment and assessment of uremia. As the majority of patients with
diagnosis of ARF in ICU will not require permanent dialytic modality, it is often difficult to
choose an appropriate time to begin RRT. Even though the trends of most retrospective studies are
toward a survival improvement in the early RRT group, there is selection bias and variable
definitions of the uremia in majority of the studies and there is a lack of convincing evidence that
early RRT has a marked survival benefit over late RRT.4
D. RRT dosing
Current level of evidence suggests that the optimal dosing remains between 20-45 ml/kg/hr for
CRRT for majority of the ICU patients. The key to prescribing the right dosage is comprehension
of the clearance of the desired solute (Kt/V > 1) therefore higher clearance intensity may be
appropriate in subset of critically ill patient.5
E. RRT & filter membranes
All RRT modalities use the semi permeable membrane in order to achieve their therapeutic goals.
All filters activate complement cascade and induce varying degree of SIRS, resulting in
thrombocytopenia and hypotension. Biocompatibility is the concept describing the extent of
cytokine and coagulation system activation for each filter. The least biocompatible membrane is
unsubstituted cellulose.6
210
SOCCA Residents Guide 2013
References:
1.
2.
3.
4.
5.
6.
Schunemann HJ, Jaeschke R, Cook DJ, Bria WF, El-Solh AA, Ernst A, Fahy BF, Gould MK, Horan KL,
Krishnan JA, Manthous CA, Maurer JR, McNicholas WT, Oxman AD, Rubenfeld G, Turino GM, Guyatt G:
An official ATS statement: grading the quality of evidence and strength of recommendations in ATS
guidelines and recommendations. Am J Respir Crit Care Med 2006; 174: 605-14
Uchino S, Kellum JA, Bellomo R, Doig GS, Morimatsu H, Morgera S, Schetz M, Tan I, Bouman C, Macedo
E, Gibney N, Tolwani A, Ronco C: Acute renal failure in critically ill patients: a multinational, multicenter
study. JAMA 2005; 294: 813-8
Cole L, Bellomo R, Hart G, Journois D, Davenport P, Tipping P, Ronco C: A phase II randomized,
controlled trial of continuous hemofiltration in sepsis. Crit Care Med 2002; 30: 100-6
Bouman CS, Oudemans-Van Straaten HM, Tijssen JG, Zandstra DF, Kesecioglu J: Effects of early highvolume continuous venovenous hemofiltration on survival and recovery of renal function in intensive care
patients with acute renal failure: a prospective, randomized trial. Crit Care Med 2002; 30: 2205-11
Palevsky PM, Zhang JH, O'Connor TZ, Chertow GM, Crowley ST, Choudhury D, Finkel K, Kellum JA,
Paganini E, Schein RM, Smith MW, Swanson KM, Thompson BT, Vijayan A, Watnick S, Star RA, Peduzzi
P: Intensity of renal support in critically ill patients with acute kidney injury. N Engl J Med 2008; 359:
7-20
Brochard L, Abroug F, Brenner M, Broccard AF, Danner RL, Ferrer M, Laghi F, Magder S, Papazian L,
Pelosi P, Polderman KH: An Official ATS/ERS/ESICM/SCCM/SRLF Statement: Prevention and
Management of Acute Renal Failure in the ICU Patient: an international consensus conference in
intensive care medicine. Am J Respir Crit Care Med; 181: 1128-55
Questions:
37.1 Acute kidney injury in critically ill:
A. It is relatively infrequent entity.
B. It is better treated with every other day diffusion dialysis
C. It is associated with mortality between 40-65%.
D. It has no impact in the ICU or hospital stay.
37.2 The
A.
B.
C.
D.
37.3 All
A.
B.
C.
D.
following are true with regard to RRT EXCEPT:
May be of benefit in septic patient.
Better hemodynamic control and fluid balance are achieved.
I t may lead to hypothermia.
It is better than intermittent hemodialysis in critically ill.
following statements in relation to RRT are true EXCEPT:
It is less efficient that IHD.
The cost per therapy is less than IHD.
Hypocalcemia may be a common side effect.
There is a need for anticoagulation.
211
SOCCA Residents Guide 2013
38. Toxicology and Support of Patients with Drug
Overdoses
Matthew D. Koff M.D. M.S
A 26-year-old male is found down at home by his girlfriend after they had an
argument over the phone. EMS was called and transported the patient to a
local emergency department. He is spontaneously breathing with a nonrebreather mask over his face collapsing the reservoir bag and breathing.at a
rate of 40 breaths/min, HR is 120, BP is 138/82, He has an altered level of
consciousness. A finger stick in the ED shows glucose of 186. ECG shows
sinus tachycardia with normal QRS intervals and with a normal axis. An ABG
performed shows a pH of 6.9 pCO2 21 pO2 420 HCO3 5 and an anion gap of
22. A serum osmolarity, aspirin level and acetaminophen is pending. How
should this patient be immediately managed?
INTRODUCTION:
Critically ill patients who are admitted to the intensive care unit with drug-related events test the
understanding and application of pharmacology and pathophysiology and medical history assessment.
Patients can present with drug toxicity secondary to side-effects, allergic reactions, or overdoses.
Additionally, toxic reaction to commonly encountered plants and animals can produce life-threatening
reactions. Occasionally, some patients in the ICU and even the OR will develop drug toxicity as a sequel to
iatrogenically administered medications. The initial step in management, irrespective of initiating factors,
is most often that of supportive care, which includes control of airway patency, oxygenation/ventilation,
and maintenance of circulation. Following initial assessment, these patients may require monitoring and
physiologic support in the ICU. The following is an outline of important topics for management of a
patient suffering from a toxicological emergency. Use of internet based resources such as Micromedex and
Clinical Pharmacology on-line is helpful to guide current treatment and management. Contact with the
local poison center can also offer immediate assistance and should be readily utilized.
I.
General management
A. Basic life support with careful and continuous evaluation of airway protection
B. Neurologic evaluation
1. Glasgow coma scale
C. Antagonist administration
1. Glucose
2. Naloxone
3. Thiamine
4. Flumazenil
5. Fomepazole
D. Laboratory evaluation
1. Acid-base disorders
2. Co-oximetry
3. Osmolar Gap
4. Toxicology screens
5. Electrocardiogram
E. Prevention of further absorption
1. Gastric lavage rarely used unless immediately after life threatening ingestion
2. Activated charcoal with cathartics
single and multidose for specific ingestions
3. Emetics rarely indicated and used.
4. Whole bowel irrigation for extended release medications ie. welbutrin
F. Enhanced elimination
G. Solute diuresis
H. Alkaline diuresis
212
SOCCA Residents Guide 2013
I.
J.
Hemodialysis
Charcoal/resin hemoperfusion
II.
Pharmacokinetics
A. Absorption
B. Metabolism
C. Distribution
D. Excretion
III.
Substances
A. Alcohol
1. Ethanol
a) Wernicke’s encephalopathy
b) Withdrawal/Delirium Tremens
c) Korsakoff’s psychosis
d) Alcoholic Ketoacidosis
2. Methanol
3. Ethylene Glycol
4. Isopropanol
B. Sedative-hypnotics
1. Barbiturates
a) Receptor physiology
b) Thio/oxybarbiturates
2. Gamma Hydroxy Butyrate (GHB)
C. Benzodiazepines
1. Ultra-short acting vs. short-acting vs. long-acting
2. Receptor physiology
3. Withdrawal
4. Antagonism
D. Cyclic Antidepressants
1. Classes
2. Cardiovascular toxicity
3. Treatment
4. CNS toxicity
5. Monitoring
E. Narcotics
1. Receptors
2. Physical exam
3. Drugs
4. Withdrawal
5. Antagonists
6. Agonist/antagonists
F. Salicylates
1. Acid-base disorders
2. Coagulopathy
3. ARDS
4. Fever
G. Acetaminophen
1. Rumack-Matthew nomogram
2. Hepatotoxicity
a) NAPQI
b) Glutathione
c) N-acetylcysteine
H. Cocaine
1. Organ Effects
a) Central nervous system effects
b) Cardiovascular effects
c) Respiratory effects
213
SOCCA Residents Guide 2013
d) Gastrointestinal effects
e) Obstetrical considerations
2. Treatment
a) Benzodiazepines
b) Beta and Alpha blockade
c) Symptholytics
I. Cyanide
1. Nitroprusside
a) Physiochemistry
b) Dose
c) Acid-Base effects
d) Mixed venous saturation
e) Thiocyanate
f) Treatment
J. Smoke inhalation
1. Carbon monoxide
a) Non-invasive co-oximetry
K. Iatrogenic
1. Malignant hyperthermia
2. Neuroleptic malignant syndrome
3. Chemotherapeutics
4. Radiation effects
5. Pro-arrhythmic drugs
6. Renal toxins
7. Hepatotoxins
8. Dermatologic prescriptions
L. Pediatric (One pill or mouthful can kill a 10 kg child)
1. Iron.
2. Chloroquine, quinine, quinidine and hydroxychloroquine
3. Clonidine
4. Sulfonylureas.
5. Tricyclic antidepressants
6. Lindane
7. Diphenoxylate/atropine
8. Beta blockers.
9. Theophylline
10. Calcium channel blockers
11. Camphor
12. Oil of wintergreen/salicylates
13. Ethylene glycol
14. Nose sprays and eyedrops
15. Benzocaine
16. Opioids, ie. methadone
READING LIST:
1.
2.
3.
4.
5.
6.
7.
Hall JB, Schmidt GA, Wood LDH, editors. Principles of critical care. Toxicology in Adults Third Edition
New York: McGaw-Hill, Inc.; 2005 . pp. 1499-1545.
Hall JB, Schmidt GA, Wood LDH, editors. Principles of critical care. Critical Care Pharmacology Third
Edition New York: McGaw-Hill, Inc.; 2005 . pp. 1547-1571.
Mokhlesi B, Leiken JB, Murray P, Corbridge TC. Adult toxicology in critical care: part I: general approach
to the intoxicated patient. Chest 2003; 123(2):577-592.
Mokhlesi B, Leikin JB, Murray P, Corbridge TC. Adult toxicology in critical care: Part II: specific
poisonings. Chest 2003; 123(3):897-922.
Baumgartner GR, Rowen RC. Clonidine vs. chlordiazepoxide in the management o acute alcohol
withdrawal syndrome. Arch Intern Med 1987;147:1223.
Hakim AM, Pappius HM. Sequence of metabolic, clinical, and histological events in experimental
thiamine deficiency. Ann Neurol 1983;13:365.
Jacobsen D, McMartin KE. Methanol and ethylene glycol poisonings. Mechanism of toxicity, clinical
course, diagnosis, and treatment. Med Toxicol 1986;1:309.
214
SOCCA Residents Guide 2013
8.
9.
10.
11.
12.
13.
Brent J, McMartin K, Phillips S, Aaron C, Kulig K. Fomepizole for the treatment of methanol poisoning. N
Engl J Med 2001; 344(6):424-429.
Brent J, McMartin K, Phillips S, Burkhart KK, Donovan JW, Wells M et al. Fomepizole for the treatment
of ethylene glycol poisoning. Methylpyrazole for Toxic Alcohols Study Group. N Engl J Med 1999; 340(11):
832-838.
Skolnick P. The (gamma)-aminobutyric acid A (GABA) receptor complex and hepatic encephalopathy:
some recent advances. Ann Intern Med 1989;110:532.
Khantzian EJ, McKenna GJ. Acute toxic and withdrawal reactions associated with drug use and abuse.
Ann Intern Med 1979;90:361.
Frommer DA, Kulig KW, Marx JA, Rumack B. Tricyclic antidepressant overdose: a review. JAMA
1987;257:521.
Henry K – Deadly Ingestions. Pediatr Clin North Am - 01-APR-2006; 53(2): 293-315
QUESTIONS:
38.1 Signs of cyanide toxicity include all EXCEPT which of the following:
A. Metabolic acidosis
B. Tachycardia
C. Seizures
D. Decreased mixed venous saturation
38.2 A 31-year-old patient is brought to the emergency department after being found at home, unconscious, lying next
to a bottle of Tylox. Upon arrival to the ED, she is given naloxone and glucose and promptly awakens. No other
abnormalities are noted. What other drug should be most likely administered to this patient?
A. Flumazenil
B. Pyridostigmine
C. Acetylcysteine
D. Ipecac
38.3 Isopropyl alcohol ingestion leads to an anion gap metabolic acidosis.
A. True
B. False
38.4 A comatose patient is admitted to the ICU and responds to sternal rub with withdrawal. Neurologic evaluation
shows increased deep tendon reflexes. Her ECG is interpreted as sinus tachycardia with first degree heart lock and
prolonged QRS and QTc with frequent ventricular ectopic beats. The most likely cause of her symptoms is:
A. Aspirin
B. Heroin
C. Diazepam
D. Acetaminophen
E. Nortriptyline
38.5 Prior to administering glucose to a patient with alcohol intoxication, what drug should be administered?
A. Folate
B. Magnesium
C. Multivitamins
D. Thiamine
E. Clonidine
38.6 The benzodiazepine receptor is an integral part of the serotonin receptor complex.
A. True
B. False
38.7 The most common symptom associated with an aspirin overdose in pediatric patients is:
A. Headache
B. Respiratory acidosis
C. Fever
D. Respiratory alkalosis
215
SOCCA Residents Guide 2013
39. Solid Organ Transplantation
Zdravka Zafirova, M.D., Jennifer Hofer M.D.
A 53 y/o M with alcoholic cirrhosis now post-operative day #1 after
orthotopic liver transplant remains intubated requiring mechanical ventilation
with FiO2 of 80%. He is febrile with a temperature of 38.8, blood pressure
88/43 mmHg, HR 109, pulmonary artery pressure 51/26 mmHg, and ALT/
AST 1760/1975 IU/L. Intraoperatively he received intravenous
thymoglobulin and methylprednisolone for immunosuppression.
Introduction
The success rate of solid organ transplantation has increased dramatically over recent years due to advances
in understanding immune mechanisms and the development of new immunosuppressive strategies to
increase survival. The transplant patient does remain a challenge to the clinician due to complications of
their underlying disease processes, the transplant procedure and side effects of immunosuppressive therapy.
The comprehensive expertise of the critical care physician is pivotal in managing these patients with the
goal of improving their life expectancy and quality of life.
Management considerations for patients receiving organ transplants include organ rejection diagnosis,
therapy and prevention, administration of immunosuppression therapy as well as management of
complications related to the transplantation and the induction of immunocompromised state. The
differential diagnosis, evaluation, and treatment of various medical problems such as respiratory failure,
hemodynamic alterations, shock, end organ dysfunction and infections present additional challenges in
these patients.
Etiology and pathophysiology
I.
Heart transplant.
Multiple indications and contraindications for heart transplantation exist. Indications include: severe heart
failure NYHA class III-IV, uncontrolled on maximal medial therapy, requiring continuous intravenous
inotropes; hypertrophic cardiomyopathy, congenital heart disease, or valvular heart disease; severe
coronary artery disease untreatable by medical therapy or revascularization; refractory ventricular
arrhythmias not controlled with medical or device therapy.
Contraindications are divided into two groups; absolute and relative. Absolute contraindications include:
malignancy with high recurrence potential; irreversible severe pulmonary hypertension; active infection
(including HIV); irreversible severe neurologic, hepatic, renal, pulmonary dysfunction, or peripheral
vascular disease; severe mental or psychological defects or substance abuse. Relative contraindications are
advanced age, unfavorable socioeconomic factors and active systemic illness that would limit long-term
survival.
Patients are listed for transplant based on different categories with status 1A being highest on the list to
receive an organ. Status 1A patients include those with 1) mechanical devices (VAD for < 30 days, ECMO,
IABP, ventilator support); 2) VADs> 30 days with device related complications; 3) high dose single
inotropes with pulmonary artery catheter, dobutamine at 7.5mcg/kg/min, or milrinone at 0.5mcg/kg/min;
multiple inotropes with pulmonary artery catheter. Status 1B patients include those with VAD for > 30
days, inotropic support (regardless of location of patient-ICU without PA catheter, floor or home). Status 2
patients are all the other patients actively listed for heart transplant.
The preoperative evaluation requires the consideration of cardiac function including the patient’s
medications and devices (VAD, intraaortic balloon counterpulsation (IABP), pacemaker/AICD), along with
attention to the function of different organ systems (pulmonary, CNS, renal) and assessment of coagulation
status and infectious issues.
Other perioperative issues for the cardiac transplant patient include post-operative coagulopathy,
immunosuppression (induction, maintenance, need for rescue), and infection control with antimicrobial
prophylaxis and therapy, strict antisepsis in care for the patient and devices, and isolation as indication.
A variety of complications are encountered in the cardiac transplant patient. These include right ventricular
failure requiring inotropic support, control of pulmonary vascular resistance with use of selective inhaled or
systemic pulmonary vasodilators, and respiratory complications. Rejection is diagnosed in the basis of the
clinical presentation, biochemical markers, imaging (echocardiography or cardiac MRI), and
endomyocardial biopsy. Hyperacute, acute cellular, acute humoral (vascular), or chronic rejection is
216
SOCCA Residents Guide 2013
encountered. Other complications include primary graft dysfunction, coronary artery vasculopathy,
hypertension, respiratory failure, CNS complications, renal insufficiency and complications secondary to
immunosuppression. Graft coronary artery disease is related to the donor and recipient risk factors such as
hypertension, hyperlipidemia, age, gender and preexisting disease; however, association with CMV
infection and episodes of acute rejection is also evident.
II. Lung transplant.
Indications for lung transplantation include pulmonary vascular disease and obstructive and restrictive lung
disease. Examples of pulmonary vascular disease are primary pulmonary hypertension, cyanotic congenital
heart disease, and Eisenmenger’s syndrome. Obstructive lung disease includes emphysema, and alpha1antitrypsin deficiency, while restrictive lung disease includes idiopathic pulmonary fibrosis, sarcoidosis,
and occupational lung disease. Other indications for lung transplant are cystic fibrosis, bilateral
bronchiectasis, and lymphangioleiomyomatosis.
The lung allocation score (LAS) is used to prioritize wait-list candidates based on a weighted combination
of predicted risk of death on the wait list (wait-list urgency measure )and a predicted likelihood of survival
following transplant. It assigns a score from 0 to 100 to all patients over 12 years old awaiting
transplantation. LAS includes wait-list urgency measure factors such as FVC, PA systolic pressure, O2
required at rest (L/min), age at offer, body mass index (BMI), NYHA functional status, diagnosis, sixminute walk distance <150 feet, continuous mechanical ventilation and diabetes, as well as posttransplant
survival measure and survival benefit factors such as FVC, PCW mean pressure ≥ 20 mmHg, continuous
mechanical ventilation, age at transplant, serum creatinine (mg/dL), NYHA functional status and diagnosis.
Preoperative evaluation focuses on the pulmonary and cardiovascular systems. Management is guided by
subjective evaluation of the patient’s clinical status, combined with objective imaging and function testing
including CXR, CT scans, pulmonary function tests, cardiorespiratory exercise test, EKG’s,
echocardiography, cardiac catheterization, and the patient’s preoperative medical therapy regimen.
A multitude of potential complications surround lung transplantation. These can be divided into three main
categories: pulmonary, cardiovascular, and renal. Pulmonary complications involve the airway such as
anastomotic dehiscence, strictures, obstruction, or the lung parenchyma such as reperfusion injury, acute
and chronic rejection and bronchiolitis obliterans syndrome. Primary graft dysfunction is an acute lung
injury resulting from ischemia-reperfusion insult, which occurs in the first 72 hours and increases
significantly morbidity and mortality. Early recognition and supportive therapy are vital. Acute rejection
depending on its severity can present with mild symptoms or acute respiratory distress. It requires prompt
diagnosis with bronchoscopic confirmation, differentiation from other causes of respiratory failure and
aggressive therapy with high-dose immunosuppression. Other complications include hemorrhage, acute and
chronic native lung hyperinflation, pneumothorax, air leak, effusion, lung cancer, diaphragmatic
dysfunction and phrenic nerve injury. Cardiovascular complications include shock, pulmonary embolism,
and arrhythmias. Renal complications include infection and posttransplantation lymphoproliferative
disease. Infections, including pre- and post-transplant infections are a major challenge and require
aggressive antimicrobial prophylaxis and therapy. Pretransplant infections with highly resistant bacteria
such as Pseudomonas, Stenotrophomonas and Mycobacteria affect posttransplant outcomes and are
challenging to treat.
III. Kidney transplant.
Multiple causes for renal transplantation exist, along with multiple co-morbidities that complicate the
perioperative period. These include cardiovascular (hypertension, coronary artery disease, and heart
failure), pulmonary (pleural effusions), metabolic and electrolyte abnormalities requiring perioperative
renal replacement therapy, endocrine pathology (diabetes mellitus, hyperparathyroidism), and
hemodynamic abnormalities (platelet dysfunction and anemia).
Complications of renal transplant are related to renal dysfunction and center on the assessment and
treatment of renal failure, associated metabolic derangements and respiratory failure and shock. Graft
dysfunction may be due to long ischemic time causing acute tubular necrosis, or may be a result of
rejection, drug toxicity, vascular thrombosis, or anastomotic leaks. A renal ultrasound with Doppler
evaluation of the blood flow, drug levels, and urine electrolytes or fluid challenges often help to distinguish
one complication from another.
IV. Liver transplant.
Indications for hepatic transplantation include chronic or progressive hepatic failure and also conditions
with normal hepatic function. Cirrhosis is one of the leading causes of liver transplantation. Causes include
217
SOCCA Residents Guide 2013
chronic hepatitis, primary biliary cirrhosis and alcoholic liver disease. Other chronic pathologies leading to
transplant are primary sclerosing cholangitis, fulminant viral hepatitis/hepatic failure, alpha-1 antitrypsin
deficiency, Budd-Chiari syndrome, Wilson’s disease, drug/toxin related, or idiopathic. Conditions with
normal liver function requiring transplantation include hemophilia, protein C deficiency, and oxalosis.
The MELD score, established in 2002, provides a numerical value to rank patients for organ allocation
based on the severity of their disease. The MELD score is based on bilirubin, INR, creatinine, and an
etiology score (score of 0 if cholestatic or alcoholic, 1 if other). There are fallacies with the MELD score. It
was designed from TIPS outcomes, and then adopted into organ transplantation. A negative score is
possible. The MELD score does not address the patient with multiple etiology hepatic failure, nor does it
account for the variability in serum creatinine measurement in the setting of hyperbilirubinemia which may
affect the score.
The modified MELD score ranges from 6-40. It avoids unfair advantage of intrinsic renal disease.
Modifiers include using the time on the waiting list as a tiebreaker. Also, special conditions such as acute
liver failure (fulminant failure, early graft failure from graft primary non-function, hepatic artery
thrombosis), hepatocellular carcinoma (tumor burden, predicted survival), hepatopulmonary syndrome
(PaO2 < 60 mm Hg on RA), and metabolic diseases (familial amyloidosis, primary oxaluria) factor into the
modified MELD score.
Preoperative care of the liver transplant patient focuses on dysfunction of many organs and systems. The
central nervous system requires evaluation of mental status, presence of cerebral edema, and hepatic
encephalopathy. The cardiovascular system may be complicated by low systemic vascular resistance, low
blood pressure and increased cardiac output. Portopulmonary hypertension or the hepatopulmonary
syndrome may complicate the respiratory system, while the hepatorenal syndrome may implicate the renal
system. Metabolic abnormalities include hypoglycemia and metabolic acidosis. Coagulopathy, with
elevated INR, and thrombocytopenia and platelet dysfunction are common finding, as are gastrointestinal
complications including ascites, esophageal varices, and hyperbilirubinemia. Immune dysfunction
including infection and sepsis is also common.
Complications of liver transplant may affect every organ system. There may be hemodynamic instability or
hypertension, pulmonary edema and pleural effusions, and pulmonary hypertension. Neurological
complications include seizures, encephalopathy, cerebral edema, central pontine myelinolysis, CNS
infections, or intracranial hemorrhage. Graft dysfunction may be characterized by biliary system failure
due to leaks or obstruction, graft rejection, or thrombosis, while renal dysfunction is often secondary to
nephrotoxicity of immunosuppressive drugs or perioperative hypotension/ischemia causing acute tubular
necrosis. Bleeding, infection, and nutritional and metabolic complications are other perioperative issues
requiring aggressive attention and intervention
V. Intestinal and pancreas transplant
Multiple indications for intestinal transplant exist including congenital abnormalities, ischemia, thrombosis,
obstruction, or inflammation. More specifically, diagnoses that may indicate the need for intestinal
transplant include gastroschisis, jejunoileal atresia, midgut volvulus, necrotizing enterocolitis, resection for
intestinal or large abdominal tumor, mesenteric thrombosis, and bowel resection for IBD, trauma or
obstruction.
Indications for pancreas transplant generally are associated with co-morbidities of diabetes mellitus.
Intestinal and pancreatic transplant can be a combined procedure not uncommonly.
Management
I.
Heart transplant
Perioperative management continues into the operating room beginning with the type of surgical procedure
(biatrial or bicaval orthotopic, heterotopic), combined with the patients pre-operative assessment which
helps determine the anesthetic plan including choice of intraoperative monitoring devices (invasive arterial,
central venous and pulmonary artery pressure, echocardiography), respiratory management, hemodynamic
support (inotropes, rhythm control), fluid, blood, and coagulation replacement, and expected physiologic
alterations secondary to cardiopulmonary bypass.
Postoperative management includes ventilatory support with the goal to optimize oxygenation, ventilation,
and hemodynamics. Ventilatory therapy choices include volume control vs. pressure control, PEEP,
noninvasive ventilation strategies, inhaled bronchodilators and pulmonary vasodilator therapy. The patient’s
baseline pulmonary function combined with perioperative lung injury further guide postoperative
pulmonary management.
The denervated transplanted heart guides post-operative hemodynamic therapy. Attention to perfusion by
218
SOCCA Residents Guide 2013
maintaining preload, contractility, and rhythm, combined with monitoring filling pressures, pulse pressure
variation, cardiac output by thermodilution, mixed venous oxygen saturation, and echocardiography guide
therapeutic choices. Hemodynamic support includes inotropes, vasopressors, and vasodilators, or
implantation of an IABP.
II. Lung transplant
Surgery may involve a single-lung, double-lung, or lung-heart transplant, and may require cardiopulmonary
bypass. Specific anesthetic considerations include choosing regional or intravenous route for postoperative
pain control, intraoperative invasive pressure monitoring, respiratory management with double lumen ETT
and different ventilation modes, hemodynamic support and fluid and blood product strategy.
Postoperative management focuses on lung mechanics and gas exchange, chest therapy, ventilation modes
and weaning strategy, and determining if bronchoscopy and biopsy are needed. The fluid management is
challenging in the postoperative period with difficulty achieving balance between the administration of IV
fluids or dieresis. Attention to immunosuppression and infection control, with inhaled and systemic
antimicrobial therapy is important.
III. Kidney transplant
Intraoperative considerations depend on the type of surgery (cadaveric vs. living-related, orthotopic vs.
heterotopic), and the anesthetic plan requires consideration of altered drug pharmacokinetics. The support
of hemodynamics and filling pressures is critical and necessitates invasive monitors. Accurate measurement
of urine output, fluid management and electrolyte corrections are essential.
Postoperative care focuses on the evaluation and treatment of hypo- and hypertension, fluid replacement
and correction of acid-base abnormalities and electrolytes, and providing immunosuppression and infection
control.
IV. Liver transplant
Intraoperative considerations require knowledge of the type of transplant techniques involved such as
orthotopic or living-related donor, end to side anastomosis of donor hepatic vein, or venovenous bypass.
There are three main phases to hepatic transplantation - the preanhepatic phase, the anhepatic phase, and
the reperfusion. While significant hemodynamic and metabolic disturbances should be anticipated during
any of those phases, the time of reperfusion is critical and requires prompt assessment and intervention to
support hemodynamic, acid-base and metabolic instability and treat the reperfusion syndrome.
For hepatic transplants, the anesthetic management considerations include aspiration risk, pharmacokinetic
alterations of medications (volume of distribution, metabolism, protein binding, and encephalopathy),
possibly elevated ICP, need for invasive monitoring (arterial, venous and pulmonary artery pressures and
cardiac output) and fluid and blood management.
Postoperative care focuses on respiratory support and weaning, circulatory support, coagulopathy
management, monitoring graft function, immunosuppression, and infection control and prophylaxis.
Respiratory support and weaning requires managing different modes of ventilation, monitoring oxygen
delivery and consumption (SVO2), and evaluation of perioperative lung injury (TRALI, sepsis).
Coagulopathy requires frequent monitoring of coagulation tests and treatment with factor and fibrinogen
replacement with products including plasma, cryoprecipitate, recombinant factor VII, antifibrinolytics,
DDAVP, and platelets.
Graft function is evaluated by biochemical markers, ultrasound, angiography, liver biopsy, and
determination of early poor function vs. primary non-function.
V. Intestinal and pancreas transplant
The surgical technique involves placement of a graft, and drainage of exocrine secretions. Perioperative
considerations focus on fluid and electrolyte management, metabolic abnormalities, and nutritional issues;
lability of the blood glucose frequently poses a challenge. Postoperative care is complicated by preexisting co-morbidities, graft dysfunction and rejection and infection. Monitoring of graft function, and
blood glucose and urinary amylase, parenteral nutrition and glucose control with dextrose-containing
solutions and insulin infusion are essential.
Immunosuppression.
I.
II.
Multiple modes of immunosuppression exist. Typically, there is an induction phase, followed by
maintenance of immunosuppression, and rescue therapy if necessary.
Induction immunosuppression involves intense prophylactic therapy at the time of transplantation that
219
SOCCA Residents Guide 2013
is short term but with high toxicity potential. It allows for decreases in acute rejection, although there
is higher incidence of late rejection with induction therapy. It is believed that solid organ donation
after brain death, trauma, reperfusion, or ischemia causes increased antigen presentation on the donor
organ leading to an exaggerated immune response in the recipient that predisposes to rejection if
induction therapy is not implemented. The administration of polyclonal and monoclonal antibodies is
the mainstay of induction therapy.
III. Maintenance immunosuppression typically involves a triple drug regimen. This includes a calcineurin
inhibitor, an anti-metabolic agent, and steroids.
IV. Rescue immunosuppression is intense therapy which is administered in response to rejection. It is
chronically intolerable due to high doses and toxicity potential.
V. Drugs:
A. Antithymocyte and antilymphocyte polyclonal antibodies such as thymoglobulin and Atgam result
in lymphopenia via complement mediated cell lysis. Thymoglobulin is also associated with
cytokine release syndrome, along with thrombocytopenia, leucopenia, serum sickness, nephritis,
and post-transplantation lymphoproliferative disease.
B. Daclizumab and basiliximab are monoclonal anti-CD25 antibodies that target the IL-2 receptor on
activated T cells. Unlike OKT3 and thymoglobulin, toxicity is minimal.
C. OKT3 is a murine monoclonal antibody directed against the CD3 complex, resulting in depletion
of CD3 T cells. The associated toxicity is the cytokine release syndrome which includes fever,
rigors, headache, dyspnea, pulmonary edema, and GI side effects, along with post-transplantation
lymphoproliferative disease. Patients are often treated with histamine blockers and steroids prior
to OKT3 administration to decrease the severity of the cytokine release syndrome.
D. Alemtuzumab is a monoclonal antibody against the CD52 cell surface marker on mature
lymphocytes. The drug was taken off the market in late 2012, and is expected to be re-released
with a new trade name and new indications.
E. Calcineurin inhibitors include cyclosporine (CSA) and tacrolimus (TAC). These agents inhibit
IL-2 and T cell activation by inhibiting calcineurin (a calcium-dependent phosphatase ), and act at
cyclophilin and FK binding protein receptors. Associated dose-dependent toxicities with these
medicines include nephrotoxicity (vasoconstriction of the afferent renal arteriole), neurotoxicity
(tremor, headache, seizures), diabetes, hypertension, lipid abnormalities, gingival hyperplasia,
osteoporosis, alopecia (tacrolimus) or hirsutism (CSA), and electrolyte abnormalities including
hyperkalemia and hypomagnesemia and rarely hemolytic-uremic syndrome.
F. Anti-metabolite agents which include inhibitors of purine and pyrimidine synthesis are
mycophenolate mofetil (MMF) and azathioprine (AZA). Mycophenolate mofetil acts by inhibiting
inosine monophosphate dehydrogenase and guanosine monophosphate synthesis, and by exerting
a cytostatic effect on T and B lymphocytes. Associated toxicities include GI toxicity (diarrhea,
nausea, abdominal pain, pancreatitis, cholestasis), hematologic (leucopenia, thrombocytopenia,
anemia), teratogenic effects and progressive multifocal encephalopathy. Azathioprine is a prodrug
that is converted to purine analog which is incorporated into DNA thus inhibiting the proliferation
of T and B cells.
G. Steroids (prednisone, prednisolone and methyprednisolone) are part of the triple therapy for
maintenance immunosuppression as well as rescue for rejection. Their mechanism of action
includes lymphotoxicity, alteration of lymphocyte distribution, and inhibition of IL-1 production
by macrophages. Toxicity includes adrenal suppression, hypertension, diabetes, acne, obesity,
hyperlipidemia, osteoporosis, and cataracts.
H. Inhibitors of mammalian target of rapamycin (mTOR) are sirolimus (SIR) and everolimus
(EVER). They inhibit mTOR or FK binding protein, a protein that regulates cell growth,
proliferation and survival, thus inhibiting T cell activation and proliferation as well as B cells and
may exert antifibroproliferative action. Fatal airway anastomotic dehiscence early after lung
transplant with sirolimus necessitates postponement of the therapy until after clear healing of the
anastomosis and discontinuation of therapy 4 weeks before and after major operations. Doserelated toxicities include interstitial pneumonitis, organizing pneumonia, alveolar hemorrhage,
vasculitis, dislipidemia, abnormal liver function, neutropenia and thrombocytopenia.
220
SOCCA Residents Guide 2013
Table 39-1. Immunosuppression therapy agents, their mechanism of action and side effects
Agent/Mode of
administration
Corticosteroids – PO, IV
Mechanism of Action
Side Effects
lymphotoxicity, alteration of lymphocyte
adrenal suppression, hypertension, diabetes, acne,
distribution, inhibition of IL-1 production by
obesity, hyperlipidemia, osteoporosis, cataracts
macrophages
Cyclosporin A (CSA) – PO, IV Inhibition of calcineurin, binding to cyclophilin nephrotoxicity, neurotoxicity (tremor, headache,
seizures), diabetes, hypertension, lipid
abnormalities, gingival hyperplasia, osteoporosis, or
hirsutism, hyperkalemia, hypomagnesemia,
hemolytic-uremic syndrome
Tacrolimus (TAC) – PO,
Inhibition of calcineurin, binding to FK binding nephrotoxicity, neurotoxicity, diabetes, hypertension,
sublingual, IV
protein
lipid abnormalities, osteoporosis, alopecia,
hyperkalemia, hypomagnesemia
Azathioprine (AZA) - PO, IV
Inhibition of purine synthesis
leukopenia, thrombocytopenia, anemia, GI toxicity,
pancreatitis, cholestatic hepatitis
mycophenolate mofetil (MMF) – Inhibition of purine synthesis (inosine
leukopenia, thrombocytopenia, anemia, nausea,
PO, IV
monophosphate dehydrogenase and
diarrhea, abdominal pain, progressive multifocal
guanosine monophosphate synthesis)
encephalopathy
Sirolimus (SIR) - PO
Inhibition of mTOR or FK binding protein, T Fatal airway anastomotic dehiscence early after
and B cell activation and proliferation
lung transplant, interstitial pneumonitis, organizing
pneumonia, alveolar hemorrhage, vasculitis,
dislipidemia, abnormal liver function, neutropenia,
thrombocytopenia.
Everolimus (EVER) – PO
Daclizumab,
Basiliximab - IV
Muromonab (OKT3) - IV
Thymoglobulin, Atgam - IV
Alemtuzumab - IV
Inhibition of mTOR, growth factor-stimulated Similar to SIR
cell proliferation
Monoclonal anti-CD25 antibodies targeting the minimal
IL-2 receptor
Monoclonal antibody against the CD3
Cytokine-release syndrome, GI, neurotoxicity,
complex, resulting in depletion of CD3 T cells hypertension, neutropenia
Antithymocyte and antilymphocyte polyclonal Cytokine-release syndrome, GI, neurotoxicity,
antibody depleting Tcells via Fc receptorhypertension, neutropenia, thrombocytopenia
dependent mechanisms
Monoclonal antibody against the CD52 cell
minimal
surface marker on mature lymphocytes
Future directions
A. Xenotransplantation
B. Organ preservation, ischemia-reperfusion injury attenuation, molecular markers for prediction of
primary graft dysfunction
C. Immunosuppression -new strategies include inhibition of T cell proliferation, interference with
cell-surface molecules regulating immune cell interactions, inhibition of signaling mechanisms,
and altered trafficking and recruitment of immune cells responsible for rejection
D. Rejection – improved diagnosis, new biomarkers
Conclusion
The progress of the medical knowledge and the expansion of the transplantation of solid organs has resulted
in increased patient survival and improved organ allocation. Perioperative complications, rejection, graft
dysfunction, multiorgan failure and infections with opportunistic as well as highly resistant organisms
continue to present challenges to the care of these patients and contribute to morbidity and mortality. The
advances in immunosuppression therapy have improved patient survival and graft function. However, the
toxicity of the immunosuppression agents affects the therapeutic benefits and contributes to morbidity and
mortality. Multidrug regimens such as triple therapy improve graft survival and decrease the complications
related to the use of these drugs.
References
1.
Chinen J, Buckley RH. Transplantation immunology: Solid organ and bone marrow. J Allergy Clin
Immunol 2010; 125: S324-335.
221
SOCCA Residents Guide 2013
2.
3.
4.
5.
6.
7.
Hirose R, Vinenti F. Immunosuppression: Today, Tomorrow, and Withdrawal. Semin Liver Dis 2006:
26:201-210
Khan SA et al. Acute Liver Failure: a Review. Clin Liver Dis 2006; 10:239-258. Review of acute liver
failure management.
Fishman JA. Infection in Solid-Organ Transplant Recipient. NEJM 2007; 357:2601-2614.
Lindenfeld J et al. Drug Therapy in the Heart Transplant Recipient Part I: Cardiac Rejection and
Immunosuppressive Drugs. Circulation. 2004; 110:3734-3740.
Lindenfeld J et al. Drug Therapy in the Heart Transplant Recipient Part III: Common Medical Problems.
Circulation. 2005; 111:113-117.
Bhorade, SM, Stern E. Immunosuppression for Lung Transplantation. Proc Am Thorac Soc 2009; 6:
47-53.
Questions
39.1 Which of the following drugs or drug classes are administered for maintenance immunosuppression in transplant
patients?
A. Steroids
B. Calcineurin inhibitors
C. Anti-metabolite agents
D. Antithymocyte antibodies
39.2 Which of the following parameters is/are not included in the MELD score?
A. Bilirubin
B. Intrinsic renal disease
C. INR
D. Ascites
39.3 Which of the following immunosuppression agents can induce neutropenia?
A. Mycophenolate mofetil
B. Tacrolimus
C. Sirolimus
D. Hydrocortisone
222
SOCCA Residents Guide 2013
40. Organ Donation and Procurement in the ICU
Jocelyn A. Park, MD
35 year old intubated male with fixed and dilated pupils is admitted to the
ICU after a single vehicle unhelmeted motorcycle accident. One day later,
after aggressive medical therapy and monitoring, he remains with unreactive
pupils. The ICU team discusses the patient’s poor prognosis with the family
and contacts the organ donation coordinator that brain death determination
is being undertaken.
Outline
I.
Potential Donors and Consent
A. There are more than 100,000 people on the waiting list for transplants in the United States in 2010,
with over 85,000 of these awaiting kidney transplant. There is a shortage of donors which is
exacerbated by the fact that only 15-20% of potential candidates become donors and of these 25%
have perfusion events which make them unsuitable for donation.
B. There are very few absolute exclusion criteria for donors. However, they do include HIV infecton,
human T-cell leukemia-lymphoma virus, systemic viral disease, prion related disease, herpetic
meningoencephalitis, active malignant disease with exceptions of non-melanoma skin cancers and
certain primary brain tumors. Most criteria are relative and should be evaluated in concert with
the organ donation team.
C. The diagnosis of brain death requires a total lack of function of the whole brain including the brain
stem. Each protocol for the declaration of brain death varies by hospital but includes a
documented pathology of irreversible comatose state without drug intoxication, neuromuscular
blockers, metabolic abnormalities, hypothermia or hypotension. On exam there must be no
spontaneous movement, absent pupillary, corneal, occulocephalic, cough and gag reflexes and
finally an apnea test must be performed.
II. Medical Management
A. All previous routine nursing care should be continued and aggressive hemodynamic optimization
as dictated by management pathways should be implemented.
B. Cardiovascular
C. Brain death strongly affects the cardiovascular system through a variety of mechanisms including
decreased sympathetic outflow manifesting as hemodynamic instability in the potential donor.
D. The goal of management is to preserve organ function by optimizing hemodynamic parameters
and various algorithms are available to achieve these goals. The most common complication is
hypotension but arrythmias, cardiac arrest and pulmonary edema may all be seen.
E. Respiratory
F. The maintenance of relatively normal respiratory parameters should allow for an increase in the
percentage of this organ which are successfully transplanted. Early bronchoscopy and pulmonary
hygiene should be considered.
G. Endocrine-hypothalamic-pituitary dysfunction
H. Most commonly, donors may exhibit fluid and electrolyte derangements from destruction of the
posterior pituitary causing a shortage of vasopressin, however this must be differentiated from
iatrogenic polyuria. Donors also require tight monitoring of their blood glucose levels and a
biphasic curve in blood glucose levels may be seen. They can also exhibit dysfunction of the
anterior pituitary gland exhibiting with hypothyroidism and depression of glucocorticoid release.
I. Coagulopathies are commonly encountered in the donor population and are a consequence of both
primary injury and exacerbation by the release of tissue factors from the dying brain. They are
exhibited by an elevated prothrombin time and thrombocytopenia.
J. Hypothermia is due to central dysregulation and can be worsened by iatrogenic factors. It may
exacerbate dysfunction of other organ systems in the potential donor and core temperature should
be maintained at greater than 35 В°C.
Discussion
There is a large discrepancy between the number of people waiting on transplant lists and the number of
available organs in the United States. There are very few absolute exclusion criteria to donation and all
223
SOCCA Residents Guide 2013
potential donors should be evaluated by the organ donation team. Once the decision to donate has been
confirmed meticulous attention to the hemodynamic status of the donor will allow for the greatest success
of the future transplants.
References:
1.
2.
3.
4.
5.
6.
Critical Pathway for the Adult Organ Donor. United Network for Organ Sharing. 27 February, 2008.
http://www.unos.org/SharedContentDocuments/Critical_Pathway_DCD_Donor.pdf
Grenvik, A., Darby, J., Broznick, B., “Organ Transplantation: A Review of Problems and Concerns.”
Critical Care. Ed. J. civetta, 3rd ed. Philadelphia: Lipincott-Raven, 1997. 1289-1299.
Heffron, T., “Care of the Multiorgan Donor.” Principles of Critical Care. Ed. J. Halle. 3rd ed. New York:
McGraw-Hill, 1998. 1345-1350.
Jenkins, D., Reilly, P., Schwab, W., “Improving the Approach to Organ Donation: A Review” World J Surg.
23:644-649, 1999.
Powner, D., DeJoya, G., Darby, J., “Brain Death-Definition, Determination, and Physiologic Effects on
Donor Organs.” Textbook of Critical Care. Ed. A. Grenvik. 4th ed. Philadelphia: W.B. Saunders, 2000.
1894-1899.
Wood, K, Becker, B., McCartney, J, D’Alessandro, Al, Coursin, D., “Care of the potenial organ donor.”
NEJM 2004; 351: 2730-9.
Questions
40.1 A 65 year old man who has been declared brain dead after a trauma and is awiting donor harvesting has a urine
output of 1000ml/hr for the past two hours with hypotension and a central venous pressure 6. His serum sodium is
152mEq/L. He responded minimally to a single one liter fluid bolus. What is the next most appropriate treatment.
A. Vasopressin infusion
B. IV fluid bolus with D5W
C. Insulin Infusion
D. Transfuse one unit of prbc
E. Free water via NG tube
40.2 The same patient then begins to ooze from his catheter insertion sites and his urine becomes blood-tinged. A
coagulation panel was sent to the laboratory. Would the Prothrombin time be:
A. normal
B. elevated
C. low
40.3 The most common complication seen in patients after brain death is
A. hypercoaguability
B. hypothermia
C. hypotension
D. cardiac arrest
224
SOCCA Residents Guide 2013
41. Trauma Management
Edgar J. Pierre M.D., Shawn M. Cantie M.D.
Introduction
When a trauma patient arrives to the emergency room or the resuscitation bay the initial moments should be
devoted to obtaining the most basic information about the overall condition: stable, unstable, dying or dead.
The primary survey involves rapid evaluation and stabilization of the functions that are crucial to survival
(ABCDE): Airway patency, Breathing, Circulation with hemorrhage control, brief neurologic exam
(Disability) and complete removal of the patient’s clothing (Exposure). The immediate care required by
most severely injured patients is best managed with a multidisciplinary team. The Advanced Trauma Life
Support (ATLS) Course developed by the Committee on Trauma of the American College of Surgeons
helps physicians maximize their resuscitative efforts and avoid missing life-threatening injuries by using an
organized approach in trauma care.
Initial assessment
The goal for the trauma patient is to re-establish blood volume and control hemorrhage. Approximately
60% of all hospital deaths related to trauma occur during the first hour. In trauma patients whose injuries
are deemed to be life threatening, a rapid systemic approach to their initial assessment and resuscitation
will improve survival. The initial assessment of any trauma victim consists of the primary, secondary, and
tertiary surveys.
I.
Primary Survey: This initial phase consists of following the ABCDE algorithm for the resuscitation
of trauma patients.
A. Airway
1. Your first responsibility is to establish and maintain the airway.
2. Endotracheal intubation is the most common form of establishing an adequate airway.
3. In patients with a GCS <9 maintaining a patent airway is critical.
4. Management of the airway:
5. The primary objective of airway assessment is to determine whether there is an immediate
need to secure the airway. The primary goals of airway intervention are to relieve or prevent
airway obstruction, to secure the unprotected airway from aspiration, to provide adequate gas
exchange, and to maintain cervical spine stabilization. The trauma patient is almost always
considered a full stomach. This requires an induction technique before direct laryngoscopy
known as a rapid sequence induction (RSI). RSI is the near-simultaneous administration of an
induction agent (midazolam, etomidate) and a paralyzing dose of a neuromuscular blocking
agent (succinylcholine or rocuronium). The goal of RSI is to obtain a secure airway while
avoiding complications such as vomiting, aspiration, cardiac arrhythmias, or the reflex
sympathetic response caused by laryngoscopy.
6. Trauma patients may also present with cervical spine injuries, making direct laryngoscopy
more difficult. All patients with blunt trauma are at risk for cervical spine injury. A patient that
is awake and does not complain of neck pain is unlikely to have a cervical spine injury. If a
cervical spine injury is suspected any excessive neck hyperextension must be avoided.
7. In-line neck immobilization is applied. In this technique, an assistant places their hands on the
sides of the patients’ head and holds down their occiput to minimize any head rotation. Direct
laryngoscopy is then performed with care not to hyperextend the neck.
8. Acute trauma patients may also present with facial or neck injuries preventing the use of
direct laryngoscopy. Although each treatment should be individualized to each patient’s
situation, the use of the fiberoptic bronchoscope and cricothyrotomy should be considered. All
clinicians caring for the trauma patient must have the ability to provide immediate and
effective airway protection and ventilation.
B. Breathing
1. After the airway is secured, breathing should be assessed. This involves a physical
examination of the chest, including palpitation for crepitus or chest wall deformity, and
auscultation. Chest wall defects create open pneumothoraces and are potentially rapidly lethal.
Large sucking chest wounds can allow rapid equilibration of pleural and atmospheric
pressure, preventing lung inflation and alveolar ventilation and causing death by asphyxia.
2. Pneumothorax and Hemothorax
a) Simple pneumothorax is a collection of air arising from leakage of air from an injured
225
SOCCA Residents Guide 2013
lung into the pleural space. An open pneumothorax arises when air enters the thoracic
cavity from an open chest wound, with equalization of pressure between the thorax and
the atmosphere. Treatment of pneumothorax depends on the type. In a patient with a
small simple pneumothorax, supplemental oxygen may be administered to enhance
reabsorption of the pneumothorax. Patient with larger pneumothoraces may be treated
either with standard tube thoracostomy or, in select patients, a “pig-tail” catheter.
b) Hemothorax is the presence of blood in the pleural space, can arise from either thoracic
or abdominal sources. Thoracic sources include: bleeding from injured lung parenchyma;
lacerated intercostals or mammary arteries; the heart or great vessels. An abdominal
source occurs in the setting of diaphragmatic injury with associated abdominal injury (i.e.
spleen or liver).Treatment goal is the same as in a pneumothoraces, to re-expand the lung
with a tube thoracostomy.
c) Tension pneumothorax
(1) Most common life-threatening injury identified during the breathing assessment
(2) Typically occurs with a parynchemal lung injury in which air cannot escape the
pleural space
(3) Increasing pressure within the pleural space causes inadequate ventilation
(4) With increased pressure in the mediastinum, mediastinal structures shift to the
contralateral side causing tension pneumothorax
(5) The result is a lower cardiac output and hypotension
(6) Diagnosis is based on clinical findings because there is often insufficient time to
obtain a chest radiograph
(7) The quickest way to relieve a tension pneumothorax is to insert a 14 or 16 gauge
IV catheter into the second intercostal space at the mid-clavicular line. A sudden
rush of air confirms the diagnosis of tension pneumothorax
(8) A chest tube should be placed once the diagnosis is made followed by a chest
radiograph to confirm the location of the tube
3. Rib Fractures and Flail chest
a) Presents with signs of respiratory difficulty requiring immediate therapy
b) Caused by multiple rib fractures in three or more adjacent ribs that leads to paradoxical
chest wall movements
c) Problems associated with flail chest include: rib fractures, pneumothorax, pain, and
pulmonary contusion. This can lead to severe hypoxemia
d) Failure to provide sufficient analgesia has been shown to result in hypoventilation,
increased atelectasis, lobar collapse and respiratory failure.
e) Pain management in chest injury consists of non-steroidal anti-inflammatory drugs such
as ketorolac, patient controlled analgesia with morphine and epidural catheter placement
with fentanyl or preservative-free morphine.
C. Circulation
1. Restoration of an adequate circulating intravascular volume is the third step in the ABCDE
algorithm.
2. Signs of inadequate intravascular volume include:
a) Pale, cool extremities, delayed capillary refill, clammy skin, tachycardia, decreased pulse
pressure, hypotension, cyanosis, stupor to coma.
3. Decreased cardiac output, variable tachycardia, oliguria, and decreased capillary pressure
defines the initial phase of hemorrhagic shock. In many trauma patients, the cause of
hypovolemia is secondary to hemorrhage.
a) Determining the quality, rate, and regularity of the pulse can also assess adequate
perfusion.
b) The pulse is a sensitive indicator of hypovolemia: the blood pressure remains normal
even with the loss of up to 30% of the blood volume
c) It is often appropriate to assume that hypovolemia is present if there is a pulse rate faster
than 120 bpm in an adult
4. Altered mental status can be used as an indicator to identify the hypovolemic patient. Signs
maybe unreliable because patients may have central nervous system injury, be intoxicated
with drugs or alcohol, or have some degree of hypoxemia that may contribute to change in
mental status
D. Resuscitation
226
SOCCA Residents Guide 2013
1.
The restoration of circulating intravascular volume is best accomplished by the use of
intravenous fluids and blood transfusions when necessary
2. For the initial resuscitation large-bore peripheral intravenous catheters are usually sufficient
(14-16 gauge x2). When necessary, a 7-9 F introducer in either the internal jugular or
subclavian vein can be placed.
3. The placement of the catheters should be dictated by the location of the injuries
a) In patients with upper torso injuries, catheters may be placed in the lower extremities
b) In patients with abdominal trauma, however, it may not be appropriate to place IV
catheters in the lower extremities
4. Fluid resuscitation
a) The initial fluid of choice in trauma patients is Lactated Ringer’s or Normal Saline
solution. A bolus of 10-20 ml/kg that is roughly equivalent to one liter is usually given.
Normal saline is favored over Lactated Ringers for the treatment of severe head injuries
due to its higher sodium content, which will theoretically reduce intracerebral swelling.
Despite this concern, there is no literature supporting the use of normal saline over
Lactated Ringer in this setting
a) It is generally accepted that a patient in shock who fails to respond adequately to 2 liters
of crystalloid is in need of blood products. If type-specific blood is not available initially
and the patient is unresponsive to crystalloid infusions then the use of uncrossed Onegative blood is required. Once type-specific blood is available then the use of
uncrossed O-negative blood is discontinued. Usage of blood must be monitored closely in
order to have blood available at all times for the hemorrhagic trauma patient
b) Colloid solutions are expensive and are not necessary, as an adequate restoration of
intravascular volume can be accomplished with the use of crystalloid solutions or blood
products
c) Hypertonic Saline has osmotic properties that result in influx of fluid into the
intravascular space. This occurs without causing swelling of red blood cells or
endothelium
d) Rapid infusion systems are available for massive amounts of blood loss. It is important to
warm all IV fluids and blood products prior to transfusion as hypothermia can cause
multiple problems that can make the resuscitation of trauma patients inadequate and
much more difficult
5. Adequacy of resuscitation: There is no one measurement that consistently indicates
completion of resuscitation. Indicators may include:
a) Normalization of vital signs (BP and pulse)
b) Base deficit as determined by arterial blood gas analysis
E. Disability
1. A quick neurologic examination is appropriate. It is important to first evaluate the pupillary
size and reactivity, motor and sensory responsiveness, and level of consciousness.
F. Exposure
1. The complete assessment of the trauma patient involves a thorough examination for occult
injury. Patients should have all garments removed to look for non-obvious injuries.
2. Patients should be rolled with care to protect the spine so that the patient’s back can be
examined. After the patients have been examined thoroughly, cover the patient to prevent
hypothermia.
II. Secondary Survey:
The goal of the secondary survey is to follow up the primary survey with a complete head to toe
examination while there is an ongoing reevaluation of the adequacy of resuscitation.
A. Initial Therapy
1. Therapy and diagnosis should be carried out simultaneously. Prioritize treatment and
management to the greatest threat to life or limb and the stability of the patient.
2. Airway maintenance is crucial, because re-intubation may be difficult owing to injuries and
airway edema.
3. Blood and crystalloid solutions are both necessary to resuscitate severely injured patients.
4. Myocardial contusion is significant only if it results in arrhythmias or hemodynamic
instability.
5. Rewarming should be the initial treatment for most coagulopathic trauma patients because of
hypothermia-induced thrombocytopenia.
227
SOCCA Residents Guide 2013
6.
The first step in limiting blood loss in patients with pelvic fractures is application of an
external fixator. If a patient continues to bleed, patient should undergo angiography and
embolization of any bleeding sites that are identified.
B. Role of Sonogram for Trauma
1. The FAST exam (Focused Assessment with Sonography in Trauma) begins with the
examination of the pericardial area. Mild pressure on the transducer below the xiphoid toward
the pericardial sac is required to visualize the heart. An anechoic band between the heart and
the pericardial/diaphragmatic interface detects hemopericardium.
2. The abdominal right upper quadrant is then visualized by placing the transducer in the right
mid to posterior axillary line, 11th intercostal space, in both longitudinal and transverse planes
to visualize the right subdiaphramatic and hepatorenal interface. An anechoic band between
the liver and kidney identifies blood in the area.
3. The abdominal left upper quadrant is examined by directing the transducer between the 10th
and 11th ribs in the posterior axillary line. The right-sub-diaphragmatic and splenorenal
spaces are examined in two planes for free fluid, detected again by an anechoic band
separating the two organs.
4. Transducer is then placed transversely just above the symphysis pubis and directed inferiorly
looking for a coronal view of the bladder. This is ideally done before bladder catherization to
allow for a distended bladder.
5. Using these four transducer positions, the pericardium, and five dependent abdominal regions
are examined for free fluid: right subdiaphramatic space, hepatorenal interface (Morrison’s
pouch), left subdiaphramatic space, splenorenal interface and the pelvis.
III. Specific Traumatic Problems:
A. Undetected hemorrhage
1. This injury is usually overlooked during the primary and secondary survey. Abdominal
sources of hemorrhage must be addressed. Ultrasound, CT scan of the abdomen and
diagnostic peritoneal lavage (DPL) are the most common methods used to evaluate abdominal
trauma. Ultrasound is preferred for the unstable patient and CT scan is often reserved for
evaluation of hemodynamically stable patients. The best technique to rule out abdominal
injury is the exploratory laparotomy.
B. Cardiac Tamponade
1. Patients may present with hypotension, tachycardia, and an elevated central venous pressure.
Beck’s triad (distant heart sounds, distended neck veins, and hypotension) is often not present
in the acute setting. Beck’s triad is only present in approximately 10% of patients.
Kussmaul’s sign, described as jugular venous distention upon inspiration, is another classic
sign attributed to pericardial tamponade, however it is rarely seen because of decreased rightsided filling.
C. Acidosis/ Hypothermia
1. Present in massively injured and resuscitated patients. Acidosis can result in persistent
hypotension despite adequate resuscitation. Primary treatment is fluid resuscitation and
restoration of blood volume with fluids and blood products and improvement of oxygen
transport.
D. Blunt Cardiac Injury
1. Formerly known as “myocardial contusion,” it is usually caused by blunt chest trauma and is
frequently produced by a deceleration injury in a motor vehicle accident.
2. Cardiac contusion presents as a spectrum of injuries including direct myocardial damage,
injury to coronary arteries, rupture of the cardiac free wall and septum. Injuries to cardiac
valves including rupture are rare but can occur.
3. Cardiac contusion can clinically present as angina, difficulty breathing, and chest wall
ecchymoses, arrhythmias, and heat failure. The most common arrhythmia is sinus tachycardia,
which is a common nonspecific finding in the trauma victim.
4. Studies used to diagnose cardiac contusion include: 12 lead ECG, troponin I levels and
echocardiogram. Some common findings on 12 lead ECG include: arrhythmias, conduction
delays, and ST-T wave changes. Treatment options will vary according to the specific
diagnoses.
E. Intra-abdominal Hypertension
1. Elevated Intra-abdominal pressure carries significant complications. The most common cause
is hemo-peritoneum, which typically occurs postoperatively. Other causes include: peritoneal
228
SOCCA Residents Guide 2013
tissue edema, fluid overload secondary to hemorrhagic or septic shock, retroperitoneal
hematoma, abdominal packing from hemorrhage and closure of abdomen under tension.
2. Intra-abdominal pressure (IAP) can be measured using an indwelling Foley catheter. The
tubing is clamped just distal to the aspiration port. Abdominal compartment syndrome is
defined by sustained or repeated IAP > 25 mm Hg in association with new onset single or
multiple organ failure.
3. Manifestations of elevated intra-abdominal pressures
a) Cardiac output may decrease dramatically because of a decrease in venous return and
elevated systemic vascular resistance.
b) Peak airway pressures rise to and there is a marked decrease in tidal volume secondary to
decreases in compliance.
c) Renal failure develops precipitously and is thought to be the result of a decrease in
cardiac output as well as direct renal compression.
F. Massive Blood Transfusion
1. Defined as a replacement of more than one blood volume, which is usually greater than 10
units packed red blood cells. Complications that can arise include:
2. Thrombocytopenia, which results from dilution and in part by consumption.
a) Hypothermia-induced platelet dysfunction leads to coagulopathy despite adequate
platelets numbers and can only be corrected by re-warming the patient
b) In actively bleeding patients, platelets should be maintained to a level greater than
75,000/cubic millimeters.
3. Hyperkalemia
a) Potassium leaks out of red blood cells during storage
b) In a unit of packed red blood cells there is a high concentration of potassium; it is usually
such a small volume that the potassium is rapidly diluted in the patient’s circulation.
4. Hypothermia
a) The definition of hypothermia is a core body temperature of less than 35 degrees Celsius.
Heat loss or production can be a result of evaporation, radiation, conduction and
convection. Acute trauma patients have numerous sources for potential heat loss.
Exposure on the field and further exposure per ATLS protocols causes further heat losses.
b) The incidence of hypothermia in trauma patients during resuscitation has been described
to be as high as 66%. The majority of heat loss occurs in the resuscitation bay.
c) Hypothermia in the trauma patient is poor prognostic indicator and it has been shown to
be independently predictive of mortality. Treatment of hypothermia is prevention.
d) First step is to obtain an accurate core temperature. Esophageal or bladder temperature is
most reliable. Given the high rate of heat loss with infusion with infusion of room
temperature fluids, all resuscitation solutions should be warmed. Operating room
temperatures elevated to minimize losses due to conduction and radiation.
5. Citrate toxicity
a) Binding to calcium uses citrate as an anticoagulant in banked blood and works. Normally
citrate is metabolized in the liver to bicarbonate. In a massive transfusion, the citrate
continues to circulate and binds to calcium.
b) This results in severe decrease in ionized calcium that may result in myocardial
dysfunction and a decrease in vascular tone.
c) Arrhythmias and prolonged QT intervals can also be observed
d) Additional calcium may be administered as necessary to maintain hemodynamic stability
6. Decrease in 2,3 DPG
a) The 2, 3 DPG levels in stored blood decline but in stable patients these levels are restored
within 24 hours.
b) A shift in the oxyhemoglobin dissociation curve as a result of hypothermia and a decrease
in 2, 3 DPG levels increases the affinity of hemoglobin for oxygen.
G. Pelvic Fractures:
1. Pelvic fractures constitute 3-5% of all fractures, account for 1 in every 1000 hospital
admissions and are the third leading injury in victims of a motor vehicle crash.
2. Morbidity rates range from 33% to 75%. Open pelvic fractures carry a high mortality rate (>
50%) and require operative intervention to prevent the development of multiple organ failure
and sepsis.
3. Pelvic fractures are often associated with significant blood loss and hemodynamic instability.
229
SOCCA Residents Guide 2013
It is essential to rule out significant Intraabdominal injury. Abdominal CT scan with contrast
may be used in stable patients.
H. The most common associated injuries include urethral and bladder injuries. If an anorectal injury
is found, the patient must have a diverting colostomy. Vaginal lacerations must be repaired after
thorough irrigation and debridement.
I. Early reduction and fixation of pelvic fractures reduces the incidence of hemorrhagic shock and
sepsis. The potential sources of hemorrhage in pelvic fractures include: fracture sites, deep pelvic
arterial bleeds, and major arterial injuries.
J. The most important predictor of outcome is hemodynamic status. The mortality rate of stable
patients is reported to be 3.4%, and the mortality of unstable patients reaches 42%.
K. Different therapeutic interventions for pelvic fractures
1. Angiographic embolization
2. Has become part of the first line of treatment for pelvic hemorrhagic associated with severe
blunt pelvic fractures such as open-book or wind-swept pelvic fractures.
3. An external binder or external fixator is placed to restore the conformation of the pelvis to
allow for tamponade of pelvic venous bleeding. This is accomplished by restoring the normal
pelvic circumference by decreasing anterior widening.
4. A more definitive stabilization requires open reduction and internal fixation of both the
anterior and posterior elements.
5. Posterior component fractures that do not stabilize with external fixation or application of the
MAST may have lacerated deep pelvic arteries. These arteries must be located by
angiography. Indications include:
a) More than 6 units of blood transfused in 24 hours.
b) Expanding pelvic hematoma visualized on CT scan or during surgery.
c) Open pelvic fracture
d) Major pelvic fracture in a patient undergoing angiography for associated injuries.
e) 85-95% success rate in controlling hemorrhage can be achieved.
L. Traumatic Brain Injury:
1. Despite improvements in diagnosis, monitoring and treatment, traumatic brain injury remains
a major cause of disability and mortality. Traumatic brain injury is classified as penetrating or
closed.
2. Closed traumatic brain injury result from sudden cranial impact or angular cranial
acceleration. Traumatic brain injury accounts for 40% of all deaths from acute injuries. The
most important feature of the initial neurologic examination, the Glasgow Coma Scale, is
designed to identify rapidly the severity of the injury. The scale comprises three tests: eye,
verbal and motor responses. The three values separately as well as their sum are considered.
The lowest possible GCS (the sum) is 3 (deep coma or death), whilst the highest is 15 (fully
awake person).
3. It is the single most important factor in determining the outcome of various form of trauma.
Clinical factors associated with poor outcome with head injury include:
a) Midline shift on CT scan
b) SBP < 90 mmHg
c) Intracranial pressure (ICP) > 15 mmHg
d) Age > 55 years
e) Glasgow Coma Scale (GCS) < 8
4. Patients with mild to moderate injuries may receive skull and neck X-rays to check for bone
fractures. For moderate to severe cases, the gold standard imaging test is a computed
tomography (CT) scan, which creates a series of cross-sectional X-ray images of the head and
brain and can show bone fractures as well as the presence of hemorrhage, hematomas,
contusions, brain tissue swelling.
5. Primary injury is that injury which occurs at the time of the traumatic incident and includes
brain lacerations or other mechanical injuries to the brain at the moment of impact. After
impact, the brain continues to be injured by various mechanisms including cerebral edema,
intracranial hematomas, ischemia from hypotension, and hypoxia from inadequate ventilation.
These are all considered secondary injuries. Prevention of secondary injury is key.
6. Primary concerns include insuring proper oxygen supply, maintaining adequate blood flow,
and controlling blood pressure. Since many head-injured patients may also have spinal cord
injuries, the patient is placed on a backboard and in a neck restraint to prevent further injury
230
SOCCA Residents Guide 2013
to the head and spinal cord.
The second important consideration after avoidance of hypotension during resuscitation of the
head –injured patient is the prevention of hypoxia. While the optimal PaO2 level in the head
injured patient has not been determined, available data suggest that level of <60 mm Hg is
associated with poor outcome.
8. ICP is controlled with a variety of modalities. Hyperventilation reduces ICP by reducing
cerebral blood volume with increased cerebral vascular tone and induction off hypocapnia.
Hyperventilation can cause vasoconstriction independent of the metabolic demands of the
brain.
9. Hyperventilation may therefore, reduce blood flow to the brain even if that reduction results
in an ischemic injury. Increasing evidence indicates that hyperventilation is an ICP treatment
with high cost, the threat of ischemia.
10. Current management therefore:
a) Includes use of less toxic means of reducing ICP, for example drainage of cerebrospinal
fluid (CSF) through ventricular drains should be started early with aggressive use of
sedation, muscle relaxants, and administration of mannitol before resorting to
hyperventilation.
b) Drainage of CSF and use of mannitol may be employed to control ICP and to optimize
cerebral perfusion pressure (CPP). Drainage of CSF may be the first choice for the
treatment of increased ICP. Mannitol is an osmotic diuretic given as a bolus that develops
an osmotic gradient between the blood and the brain.
c) Mannitol may also act by improving cerebral blood flow through reduction in hematocrit
and viscosity. Mannitol cannot be given to the hypotensive patients, as it will magnify
shock states. Optimal modalities to prevent secondary brain injury focus on maintenance
of optimal CPP with the lowest possible ICP consistent with avoidance of cerebral
ischemia.
7.
Questions
41.1 What is the treatment of tension pneumothorax?
A. Two large bore IVs and resuscitate with Lactated Ringers Solution
B. Immediate CXR and supplemental oxygen
C. Immediate decompression of the chest with an 14 gauge angiocath placed in the 2nd intercostals space at the
mid-clavicular line
D. Intubate and paralyze the patient
41.2 A patient with on going bleeding and with a hematocrit of 18%, what type of blood would you gives if the type and
cross-match are not completed?
A. AB Rh- positive
B. O Rh- positive
C. AB Rh-negative
D. O Rh-negative
41.3 What is the treatment of hemorrhagic shock?
A. Open thoracotomy
B. Phenylephrine 1mg IV
C. Administration of fluids and blood
D. Administration of Hypertonic Saline
41.4 What is the potential risk of mannitol in patients with traumatic brain injury?
A. Decrease in intracranial pressure
B. Slight increase in intracranial pressure
C. Hypertension
D. Coagulopathy
231
SOCCA Residents Guide 2013
41.5 What are the different modalities to decrease intracranial pressure?
A. Elevated head of bed to 45 degrees
B. Hyperventilation for normocarbia
C. Drainage of cerebral spinal fluid
D. Administration of mannitol
E. All of the above
41.6 The most common arrhythmia after blunt cardiac injury is:
A. Ventricular fibrillation
B. Sinus tachycardia
C. Sinus Bradycardia
D. Multifocal atrial fibrillation
41.7 Four hours after an exploratory laparotomy for splenectomy and pelvic fracture, the patient is intubated with high
peak airway pressures and low tidal volumes. Urine output has decreased in the past two hours and the patient is
hypotensive. On physical exam the abdomen is distended. What is the mostly cause of these findings?
A. Cardiac tamponade
B. Pulmonary edema
C. Bronchospasm
D. Abdominal compartment syndrome
41.8 How is hypothermia defined?
A. As a body temperature of less than 35 degrees Celsius
B. Shivering post-operatively
C. Increase in urine output and thrombocytopenia
D. As a body temperature of less than 38 degrees Celsius
232
SOCCA Residents Guide 2013
42. Obstetric Critical Care
Gustavo Angaramo, M.D
Patient is a 29-year-old female, approximately 31 weeks gestation, who
appears to be in moderate respiratory distress, suddenly, she begins to have
a seizure. She is noticed to be hypertensive and is bleeding from intravenous
sites.
INTRODUCTION
Over the past 45 years in the United States, pregnancy associated mortality rates have declined from
50/100,000 live births to less than 10/100,000 live births. This is due to a decline in deaths from direct
obstetric causes: eclampsia, hemorrhage, infection, and cardiac disease. The most prevalent causes of death
now are due to trauma, pulmonary embolism (PE), and poor antenatal care. As illustrated in the case above,
a working knowledge of the uniqueness of these processes is essential to avoid morbidity. Goals for
obstetric knowledge fundamentals include: (1) an understanding of how disease processes impact
pregnancy and vice versa, (2) an ability to assess the severity of the disease and evaluate the need for
patient transfer to a high-risk care area, and (3) an understanding of the anesthetic management of a patient
for vaginal or cesarean delivery.
MATERNAL PHYSIOLOGY
I.
II.
III.
IV.
V.
VI.
Cardiac output increases during the first trimester and peaks in the second trimester to almost 50%
above normal.
Blood volume increase in the first trimester and peaks at 140% of the non pregnant state by the third.
Red cell mass and total blood volume increase; however, red cell mass does not increase at the same
rate as plasma volume, which causes a modest dilutional anemia.
Respiratory rate and tidal volume increase in the second and third trimesters.
Total lung capacity and functional residual capacity decrease in pregnancy.
Uterine blood flow is directly related to maternal blood pressure.
DISORDERS OF PREGNANCY
I.
Hypertensive Disorders of Pregnancy
A. Knowledge
1. Classification of hypertensive disorders during pregnancy: preeclampsia/eclampsia, chronic
hypertension, chronic hypertension with superimposed pregnancy induced hypertension.
2. Preeclampsia: hypertension that occurs after the 20th week of pregnancy with proteinuria,
greater than 2 g per day and peripheral edema. It occurs in 6-8% of all pregnancies and is
associated with end organ failure.
3. Eclampsia: onset of seizures that are unrelated to a known seizure disrder or anatomic focus in
hypertensive patients.
4. Epidemiology of preeclampsia–risk factors: adolescent or elderly primigravidas, high body
mass, chronic hypertension, chronic renal disease, sickle cell anemia, collagen vascular
disease, family history.
5. Pathophysiology of preeclampsia: arteriolar vasospasm seems to be the cause. Inciting
factors: failure of prostacycline mediated uterine vasodilatation, high thromboxane levels
from endothelial damage, and increased sensitivity to angiotensin II.
a) Recognition of sequelae:
(1) Seizures
(2) Coagulopathy
(3) Hepatic dysfunction
(4) HELLP syndrome: involves hemolysis, elevated liver function and low platelets.
6. Medical/obstetric management of preeclampsia
a) Term vs. preterm fetus
b) Mild vs. severe disease
c) Assessment of fetal well being
d) Seizure prophylaxis; magnesium sulfate effects
233
SOCCA Residents Guide 2013
e) Antihypertensive therapy: hydralazine, alpha methyldopa, labetalol.
f) Management of oliguria
g) Indications for invasive monitoring
7. Anesthetic selection for and management of the preeclamptic parturient
a) Labor and vaginal delivery
b) Abdominal delivery–non-urgent
c) Abdominal delivery–urgent
B. Skills:
1. Ability to diagnose the parturient with preeclampsia
2. Ability to identify the patient requiring invasive monitoring
3. Ability to effectively treat the patient with an eclamptic seizure
II. Morbid Obesity
A. Understanding of effects of morbid obesity on pulmonary and cardiac physiology В± pregnancy
B. Ability to perform successful neuroaxial anesthesia techniques in morbidly obese patients
III. Respiratory Disease
A. Asthma
B. ARDS
IV. Cardiac Disease
A. Knowledge
understand when invasive monitors are needed for delivery and postpartum care
1. Congenital heart disease
a) Left-to-right shunts
b) Right-to-left shunts (Tetrology of Fallot)
c) Pulmonary hypertension (Eisenmenger’s Syndrome)
d) Coarctation of aorta
2. IHSS
3. Ischemic heart disease
4. Valvular heart disease
a) Aortic stenosis
b) Aortic insufficiency
c) Mitral stenosis
d) Mitral regurgitation
5. Peripartum cardiomyopathy
V. Endocrine Disease
A. Knowledge:
1. Diabetes mellitus
2. Thyroid disease
3. Hyperthyroidism
4. Hypothyroidism
B. Skills:
1. Ability to manage glucose control in the parturient during cesarean or vaginal delivery
VI. Hematologic and Coagulation Disorders
A. Anemia
B. Coagulation disorders
VII. Neurologic Disease
A. Multiple sclerosis
B. Spinal cord injury
C. Myasthenia gravis
D. Seizure disorders
E. Subarachnoid hemorrhage or vascular malformations
VIII. Substance Abuse and HIV Infections
A. Substance abuse
1. Ethanol abuse
2. Opioid abuse and barbiturate use
3. Cocaine abuse
B. HIV infection
IX. Miscellaneous Disorders
A. Renal Disease
234
SOCCA Residents Guide 2013
B.
C.
D.
E.
F.
G.
H.
I.
J.
K.
Liver Disease (e.g., acute fatty liver of pregnancy)
Musculoskeletal disorders
Scoliosis
Rheumatoid arthritis
Spina bifida
Autoimmune Disorders
Prior back surgery, including Harrington rods placement
Ante partum and postpartum hemorrhage
Amniotic fluid embolism
Blunt and penetrating trauma
REFERENCES
1.
2.
3.
4.
5.
6.
Writer D. Hypertensive disorders. Obstetric anesthesia, 3rd edition, 2004: 846-882.
Irwin R., Rippe J. Manual of Intensive Care Medicine, 4th edition, 2006: 669-673.
Gutsche B, Cheek T. Anesthetic considerations in preeclampsia-eclampsia. Anesthesia for obstetrics, 4th
edition, 2004: 305-336.
Lechner R. The critically ill obstetric patient. Handbook of critical care pain management, 1994.
Birnbach D, Browne I. Anesthesia for obstetrics. Miller’s Anesthesia, 6th edition, 2005: 2329-2333.
Cheek T, Samuels P.Pregnancy induced hypertension. Anesthetic and obstetric management of high risk
pregnancy, 3rd edition, 2004: 386-411.
QUESTIONS:
42.1 Which of the following is true regarding hypertensive disease of pregnancy?
A. It affects 6-8% of all pregnancies.
B. It affects multiparous patients approximately 3 to 1.
C. It accounts for almost 60% of all maternal deaths in the United States.
D. The incidence has fallen dramatically since the discovery of its etiology.
E. It is restricted, almost exclusively, to older parturients.
42.2 Which of the following would indicate a diagnosis of severe preeclampsia?
A. A diastolic blood pressure of at least 100 mmHg on two occasions at least 6 hours apart.
B. More than 300 mg protein in a 24-hour urine collection.
C. A systolic blood pressure of at least 140 mmHg on two occasions at least 6 hours apart.
D. Urine output of less than 300 mL in a 24-hour period.
E. A diastolic blood pressure of at least 100 mmHg AND a systolic blood pressure of at least 140 mmHg on two
occasions at least 6 hours apart.
42.3 The cardiovascular changes associated with hypertensive disease of pregnancy include all of the following EXCEPT:
A. An increased sensitivity to endogenous pressor hormones.
B. Blood volume may be reduced 30-40% in women with severe disease.
C. There is a poor correlation between CVP and PCWP in many of these patients.
D. The colloid oncotic pressure usually increases 4-6 mmHg postpartum in women with severe disease.
E. The majority of patients have a hyperdynamic myocardium.
42.4 Which of the following statements regarding preeclampsia are true?
A. Thrombocytopenia occurs in 2-4% of women with preeclampsia or eclampsia.
B. The classic manifestation of severe preeclampsia include severe headache, visual disturbances, CNS
hypoexcitability, and hyporeflexia.
C. Pulmonary edema remains the leading cause of death in women with severe preeclampsia.
D. Levels of angiotensinogen, angiotensin I, angiotensin II, and aldosterone increase markedly in preeclamptic
women, which aids in the diagnosis as levels usually fall in normal pregnancy.
E. The GFR is, on average, 25% below that in normal pregnancy.
235
SOCCA Residents Guide 2013
42.5 Which of the following statements regarding the management of PIH are true?
A. MgSO4 exerts a peripheral effect at the neuromuscular junction.
B. Hydralazine reduces systolic blood pressure primarily as a result of its myocardial depressant effects.
C. ACE inhibitors are indicated when hydralazine fails to adequately control maternal blood pressure in the
antepartum period.
D. Sodium nitroprusside is a powerful smooth muscle vasodilator but, since it results in the release of nitric oxide,
it is contraindicated during pregnancy.
236
List of Figures and Tables
Figure 4-1
Figure 4-2
Figure 4-3
Table 5-1
Table 5-2
Table 6-1
Table 6-2
Figure 7-1
Figure 7-2
Table 8-1
Table 8-2
Figure 9-1
Table 11-1
Table 13-1
Table 13-2
Figure 13-1
Figure 17-1
Figure 18-1
Figure 18-2
Figure 18-3
Figure 18-4
Figure 18-5
Figure 19-1
Figure 19-2
Figure 19-3
Question 19-4
Table 21-1
Figure 21-1
Figure 26-1
Table 26-1
Figure 27-1
Table 28-1
Table 28-2
Figure 29-1
Figure 29-2
Table 31-1
Table 31-2
Rhythm Strip: ventricular fibrillation
CPR
Rhythm Strip: sinus w/ PVCs
Patient Transfer Checklist
Departure Checklist
Drugs Used for Sedation
Subjective sedation assessment scales
Visual Analog Scale
Numeric Rating Scale for Pain
NMBs by mechanism of action and duration
Drugs that interact with NMBs
Acid-Base
Commonly used methods to assess intravascular volume and adequacy of resuscita
Rules for Predicting Probability of Embolism
Current Recommendations for Prophylaxis
Algorithm: Suspected PE
Waveforms PA Catheter
Flush Test
Arterial Waveform
Pulse Pressure Variation
Pressure Variation with Respiration
Systolic Pressure Variation
Echo: RV Thrombus
Echo: Dilated LA/MS
Echo: Aortic Dissection
Echo: Pericardial Effusion
Bedside Tracheostomy Pre-procedural Checklist
Components of a Tracheostomy Tube
“funny looking strip”
CHADS2 Score
AAC/AHA 2007 Guidelines
Perioperative hemodynamic considerations of most common valvular heart disease
Ten perioperative factors to achieve hemodynamic stability in valvular heart disease
Diagnostic Criteria for Sepsis
PIRO Model of Sepsis
Most common organisms causing SSIs
Common bacteria by operative site
Table 31-3
Table 32-1
Table 32-2
Table 32-3
Figure 34-1
Figure 34-2
Table 35-1
Table 35-2
Table 36-1
Commonly used antibiotics, spectrum and dosing
Types of immune system disorders and the associated pathogens
Antimicrobial agents: targets and drug classes
Medical complications and issues in HIV patients
Intracranial pressure-volume curve
Cerebral blood flow in relation to oxygenation, ventilation, and ICP.
Classification of TBI
Glasgow Coma Scale (GCS)
Causes of AKI and associated treatment
Answers
Chapter 1
1.1 (D)
1.2 (D)
1.3 (C)
1.4 (D)
Chapter 2
2.1 (E)
2.2 (A)
2.3 (C)
Chapter 3
3.1 (A)
3.2 (B)
3.3 (D)
3.4 (C)
3.5 (C)
Chapter 4
4.1 (D)
4.2 (B)
4.3 (A)
Chapter 5
5.1 (D) Oxygen is administered via Venturi mask and two large bore peripheral intravenous accesses obtained for
fluid administration. A thoracostomy tube was placed in the right chest. Repeat arterial blood gases showed pH
7.40, pO2 152 and pCO2 38 and vitals signs showed Pulse 99, BP 110/58 and RR 20.
5.2 (A) The method of transport should always take into consideration the urgency, mobilization time, geographical
factors, weather and traffic conditions. Road transfer is satisfactory for this patient since the level one trauma center
is only 30 minutes away.
5.3 (C) Under the COBRA/EMTALA statute, hospitals must examine and stabilize all patients regardless of ability
to pay. Physicians must also certify that the benefits of transfer to an outside facility outweigh the risks and the
receiving facility must have the personnel/space and agree to accept transfer.
Chapter 6
6.1 (C)
6.2 (B)
6.3 (C)
6.4 (A)
Answer for question in the beginning of the chapter is haloperidol
Chapter 7
7.1
7.2
7.3
7.4
7.5
(B)
(C)
(D)
(B)
(C)
Chapter 8
(no answers given)
8.1 (C)
8.2 (C)
8.3 (B)
8.4 (C)
8.5 (A)
Chapter 9
9.1. [D] The most common acid-base abnormality associated with salicylate intoxication is a respiratory alkalosis
caused by direct stimulation of the medullary respiratory center. A pure metabolic acidosis due to salicylate
intoxication is unusual. Even in severe salicylate intoxication, it is more common to see a mixed disorder secondary
to the anion gap metabolic acidosis due to both salicylic acid and formation of lactate.
9.2. [C] Winter’s formula is used to estimate the expected drop in PaCO2 that occurs as a result of respiratory
compensation for a metabolic acidosis. The formula is less accurate for mild acidosis with a bicarbonate level > 20
mmol/L. The formula is as follows:
Expected PaCO2 (mmHg) = [(1.5 x HCO3-) + 8]
9.3. [E] In contrast to acute acidemia, acute alkalemia reduces cerebral blood flow, increases the affinity of oxygen
for hemoglobin and increases calcium binding to proteins.
9.4. [E] Depending on the etiology of the acidosis, bicarbonate therapy remains controversial. It is generally
acceptable to administer alkali when the pH falls below 7.2 due to a metabolic acidosis to prevent arrhythmias and
cardiovascular collapse. However, it may have harmful consequences as listed in the question above and therapy
with alkali should be monitored closely.
9.5 [B] The anion gap is the difference between the major cations (Na+ and K+) and major anions (HCO3- and Cl-)
which is normally 12 + or – 4 when potassium is included. The difference between these measured ions is typically
due to the other normal anions in circulation, mainly albumin and phosphate. To a much lesser extent, sulfates and
lactate also contribute to the anion gap, but their contribution is offset by other proteins and ions. Therefore, the
anion gap is used to narrow the differential diagnosis of a metabolic acidosis, but is limited in the critically ill patient
by changes in albumin and phosphates to the point that it has been recommended that the “normal” range for the
anion gap be adjusted for each patient by the following formula:
“adjusted” anion gap = 2 x [albumin(g/dL)] + 0.5 x [phosphate(mg/dL)]
Chapter 10
10.1
10.2
10.3
10.4
(E)
(C)
(B)
(D)
Chapter 11
11.1. [B]
11.2. [A]
11.3. [D]
11.4. [D]
11.5. [D]
Chapter 12
12.1 (E)
12.2 (E)
12.3 (D)
12.4 (A)
12.5 (D)
Chapter 13
13.1. [B] The primary goal is full anticoagulation for patients in which a high index of suspicion for PE exists. The
diagnostic study chosen will be partly dependent on institutional strengths.
13.2. [E] A high probability V/Q scan is accepted as evidence of PE in greater than 90% of patients, and no further
studies are needed. Exceptions to the above rule would include patients who have previously experienced a PE and
who have not received a V/Q scan or baseline comparison. With low clinical suspicion, only 56% of patients with a
high-probability V/Q scan will have a PE. Warfarin therapy should be initiated within 24 hours of the diagnosis of
DVT.
13.3. [C] Normal lower extremity studies, of course, don’t preclude DVT or PE. Depending on the patient’s
hemodynamic and pulmonary vasculature response, hypoxemia may or may not be present with clinically significant
PE. Hormone replacement is not associated with an increased risk of DVT, although pharmacologic estrogen
therapy (i.e., oral contraceptives) is.
Chapter 14
14.1
14.2
14.3
14.4
14.5
(E)
(B)
(C)
(D)
(A)
Chapter 15
(no answers given)
15.1 (D)
15.2 (C)
15.3 (C)
15.4 (B)
15.5 (C)
Chapter 16
16.1 (B)
16.2 (E)
16.3 (D)
Chapter 17
17.1 (C) Recent studies looking at patients with ARDS did not demonstrate improved outcomes in patients who
were managed with PACs.
17.2 (A) Pulmonary hypertension will be reflected in an elevated PAP, but it does not affect PCWP.
17.3 (B) Cyanide toxicity and early stage sepsis are associated with an elevated SvO2 due to the decrease in
oxygen extraction that occurs during those pathologic states.
Chapter 18
18.1: (D) This implies that the contractility of the ventricle is poor. If the patient is not in failure and excessive
preload is not a factor then the only drug that independently increases the contractility is milrinone.
18.2: (B) The only reliable value really is mean arterial pressure!
18.3: (C) End expiration would be the answer so much so that early methods mandated that apnea be used to mimic
end expiratory conditions and calculate stroke volume variations.
18.4: (A) It is an indicator of volume responsiveness and to be specific mostly preload!
Chapter 19
19-1 (D) Patients with cervical instability are at risk for spinal cord injury during TEE probe insertion into the
esophagus and therefore a TTE would be the preferred exam.
19-2 (C) PEEP and morbid obesity increase the distance from the chest wall to the heart resulting in poor
image quality. Similarly a recent sternotomy with dressings and wires will make adequate image acquisition
difficult. Surveillance of the left atrium for clot is best accomplished with TEE allowing for visualization of the
left atrial appendage. Patients with hepatic cirrhosis that is severe are at increased likelihood of having
esophageal varices which are a relative contraindication to a TEE exam.
19-3 (E) The ability of ultrasound to aid in diagnosis of different types of pathophysiology is ever increasing.
Ultrasound is able to aid in diagnosis of all of the listed disease states. Diagnosis of pulmonary embolism can
be made with direct lung ultrasound showing an area of lung infarction or vascular ultrasound of lower
extremities can be used to demonstrate the presence of deep venous thrombosis.
19-4 (D) The image demonstrates a large pericardial effusion which may be causing the patients hypotension.
It should be noted that the left ventricle appears hypertrophied.
Chapter 20
20.1 (D) In this scenario of a patient who you are unable to intubate and is desaturating, it would be wisest to
abandon your rapid sequence induction plans and attempt to mask ventilate the patient to determine your ability to
ventilate and guide you through the difficult airway algorithm. Placement of an LMA would be an appropriate
answer once your attempts at mask ventilation fail.
20.2 (A) Although cricoid pressure may prove helpful in preventing aspiration, it is not a foolproof method to
prevention of aspiration of gastric contents. Cricoid pressure should be maintained until confirmation of
endotracheal placement of the ETT and inflation of the cuff.
20.3 (C) An awake fiberoptic through the mouth is a safe way to proceed with intubation for this patient. Due to the
history of face trauma, a nasal intubation in contraindicated in this patient secondary to possibility of skull fracture.
Vecuronium is not an ideal choice in this patient as they should be considered a full stomach and a candidate for
rapid sequence and, with the history of facial trauma, he may be a difficult intubation and a long acting muscle
relaxant is not ideal.
20.4 (D) This patient should not be discharged home. Soot in the airway and singed nasal hairs suggest that she
may have sustained inhalational injury. This patient is at risk for impending airway edema and airway obstruction.
Succinylcholine is not contraindicated in this case as the burn occurred 2 hours prior. Succinylcholine remains an
appropriate choice for rapid sequence induction within the first 24hours after sustaining a major burn injury. After
24 hours, burn victims may have up-regulation of extrajunctional acetylcholine receptors which predisposes to a
hyperkalemic response with succinylcholine.
20.5 (C) Pregnant patient are at risk for aspiration and should always receive a rapid sequence induction. The
gravid uterus increases intragastric pressure and decreases the angle of the gastroesophageal junction, facilitating
reflux of gastric contents. In addition, elevated gastrin levels from placenta secretion result in more acidic gastric
contents. Diabetic patients may be at risk for autonomic neuropathy and delayed gastric emptying, however, this is
unlikely in a newly diagnosed diabetic with no evidence of end-organ damage (retinopathy, nephropathy, peripheral
neuropathy). If intubation is necessary in a gravid patient, smaller endotracheal tubes are needed due to airway
edema and capillary engorgement.
Chapter 21
21.1
21.2
21.3
21.4
21.5
(E)
(A)
(C)
(D)
(A)
Chapter 22
22-1 (A)
22-2 (D)
22-3 (D)
Chapter 23
23.1 (E)
23.2 (D)
23.3 (B)
Chapter 24
24-1 (E) Esteban et al. (JAMA 2002; 287:345–355) showed that there is much delay in the weaning process and
that patients may indeed spend between 40-50% of the time on the ventilator weaning. With prolonged mechanical
ventilation there are associated risks such as ventilator associated pneumonia, malnutrition, muscle wasting, etc.
Cooper et al. (Crit Care Med 2004;32:2247–2253) estimated that mechanical ventilation costs about $2,000 per day
and may consume up to 37% of intensive care unit resources. The incidence of unplanned extubation ranges 0.3–
16% and in most cases (83%), the unplanned extubation is initiated by the patient, while 17% are accidental (Epstein
SK. Decision to extubate. Intensive Care Med 2002;28:535–546). The decision to attempt weaning is based on the
clinician’s assessment that the patient is hemodynamically stable, awake, the disease process has been adequately
treated, and indices of minimal mechanical ventilatory support are present; but the actual process of weaning begins
with and SBT or low level PSV.
24-2 (A) Electrolyte abnormalities (hypokalemia, hypophosphatemia, hypomagnesemia) could absolutely be one
cause. Increased sputum production could be a sign of a new infection and would warrant a chest x-ray, sputum
cultures would also be warranted in the setting of a new fever. This may be another reason for the patient’s failure.
Excessive sedation is often problematic and should be investigated, especially when attempting an SBT. Respiratory
therapists, nursing staff, and clinicians should have a plan/protocol in place when SBTs are to take place in order for
sedation to be turned off or at an appropriate dose for patient participation. It is unlikely that there are lingering
effects of neuromuscular blocking agents 10 days post-intubation.
24-3 (B) The decision to wean is often subjective and based on clinical experience, but there are helpful guidelines
as well. The underlying illness must be treated and/or improving. There must be cardiovascular stability, optimal
fluid balance and electrolyte replacement. Sedation should be minimized or discontinued. Weaning is likely to fail if
the patient is confused, agitated or unable to cough. In terms of respiratory function, the following criteria are
usually followed: Respiratory rate < 25 breaths/minute, Tidal volume >5ml/kg, Vital capacity >10ml/kg, minute
volume <10 l/min, FiO2<0.4, and PaO2 >60 mmHg.
Chapter 25
25.1 (A) The patient is in cardiogenic shock and needs inotropic support
25.2 (A) He is showing signs of right heart failure. Treatment of RV failure included 1) inotropic support 2)
improve RV perfusion pressure and 3) decrease RV after load.
25.3 (A) He is now showing signs of septic shock and needs a vasoconstrictor to maintain adequate blood pressure.
Chapter 26
26-1 (A) Since he is asymptomatic and hemodynamically stable, rate control is the most appropriate next step.
Since his EF is 23% amiodarone is the best choice.
26-2 (D) He is unstable due to the tachycardia (most likely due atrial fibrillation) and synchronized cardioversion is
the most appropriate treatment.
26-3 (C) Tachycardia without a pulse should be defibrillated as soon as possible.
Chapter 27
27.1
27.2
27.3
27.4
27.5
27.6
C()
(B)
A()
(C)
(C)
(D)
Chapter 28
28-1 (C)
28-2 (C)
28-3 (B)
Chapter 29
29.1 (C)
29.2 (B)
29.3 (A)
Chapter 30
30.1 [B] The most common site of colonization is the oropharynx/airway. Differentiating between organisms
causing infection vs. colonization is difficult.
30.2 [C] E-coli and B. fragilis are the most common organisms cultured from intraabdominal abscesses. S.
pneumonia and H. influenzae cause community acquired pneumonias. Although C. albicans is frequently isolated
from the airway, it rarely causes pneumonia in patients who are not severely immunosuppressed (neutropenic
leukemic, bone marrow transplant recipients).
30.3 [D] If 10,000 CFU/ ml are isolated, repeat culture may be warranted.
30.4 [B] Proven line infections require replacement of the catheter to a new site unless absolutely impossible.
Routine changing of catheters without evidence of infection is not warranted.
30.5 [D] Full barrier precautions and meticulous aseptic technique are important actions during insertion of central
venous catheters.
Discussion: The patient in the case presentation is clearly at risk for a nosocomial infection. He is intubated, has
numerous other indwelling devices and has a large wound. All are potential sources of infection. A thorough
examination, appropriate imaging and culture work-up should be pursued. In the setting of new or increased fever,
leukocytosis and hemodynamic instability serious infection is likely. Specific management considerations for sepsis
are reviewed elsewhere in the text. In this setting decisions must be made regarding removal and replacement of
indwelling catheters, diagnostic procedures to detect a possible ventilator associated pneumonia as well as initiation
of empiric broad spectrum antibiotic treatment. Antibiotic coverage must include treatment of pathogens and
resistance patterns commonly encountered in the patient’s hospital location. Principles of empiric therapy and
antibiotic stewardship should be employed including tailoring or de-escalation of antimicrobials based on culture
results if possible.
Chapter 31
31.1 [C] Common exceptions include evidence of perforation (as in case sited) or possible ongoing infection
(cholecystitis or appendicitis). Nonetheless, even in these cases antibiotics can generally be given for one day or
less.
31.2 [A] In fact, prophylaxis for the actual duration of the operation only (no postoperative doses) may be
adequate, though this is unproven.
31.3 [B] Despite all efforts, there remains a low but real wound infection rate even under ideal circumstances.
These infections are probably due to bacteria found in hair follicles, sweat glands, etc., that may be out of reach of
both skin cleansers and systemic antibiotics.
31.4 [B] Based on studies of both serum and tissue (colon) levels. This issue may be related to why longer
operations have consistently higher wound infection rates even when other variables are controlled.
Chapter 32
32.1 (A)
32.2 (B)
Chapter 33
33.1 (A)
33.2
33.3
33.4
33.5
(C)
(D)
(A)
(C)
Chapter 34
34.1 (A)
34.2 (B)
34.3 (D)
Chapter 35
(no questions)
Chapter 36
36.1 (A)
36.2 (D)
36.3 (A)
Chapter 37
37.1 (C)
37.2 (D)
37.3 (B)
Chapter 38
38.1 [D] Cyanide, by blocking the cytochrome c in the electron transport chain, prevents the utilization of oxygen
and consequently the mixed venous saturation increases (Reference 1)
38.2 [C] Reference 1
38.3. [B] Isopropyl is converted to ketones, not ketoacids, so although the patient’s breath has the aroma of
ketones, he does not exhibit a metabolic acidosis (Reference 7).
38.4. [E] Reference 12.
38.5. [D] Reference 6.
38.6. [B] Reference 10.
38.7. [C] Reference 13
Chapter 39
39.1 (A)
39.2 (C)
39.3 (B)
Chapter 40
40.1 (A)
40.2 (B)
40.3 (B)
Chapter 41
41.1 (C)
41.2
41.3
41.4
41.5
41.6
41.7
41.8
(D)
(C)
(D)
(E)
(B)
(D)
(A)
Chapter 42
42.1 [A] Most (85%) cases of preeclampsia affect women in their first pregnancy (primipara), rather than
multiparous. Although young patients have a higher incidence of the disease, older parturient may present with
more severe disease. Preeclampsia accounts for 20% of maternal deaths in the United States. The etiology remains
unknown.
42.2 [D] A urine output of less than 400 mL in a 24-hour period is sufficient to diagnose severe preeclampsia. The
diagnosis of severe preeclampsia would be indicated if any ONE of the following features were present: (1) systolic
BP of at least 160 mmHg, or (2) diastolic BP of at least 110 mmHg, or (3) greater than 6 gm proteinuria in a 24-hour
urine collection.
42.3 [D] The colloid oncotic pressure (COP) is reduced in normal pregnancy and is often decreased further in
preeclamptic patients. The COP can be expected to fall slightly, not increase, postpartum.
42.4 [E] Thrombocytopenia occurs in 15-30% of women with preeclampsia or eclampsia. The classic
manifestations of severe preeclampsia include several headache, visual disturbances, CNS hyperexcitability, and
hyperreflexia; cerebral hemorrhage is the leading cause of death. Angiotensinogen, angiotensin I, angiotensin II,
and aldosterone levels decrease markedly in preeclamptic women.
42.5 [A] Hydralazine is not a myocardial depressant; it has direct smooth muscle relaxing properties. ACE
inhibitors are contraindicated in the antepartum period because of potentially lethal fetal and neonatal sequelae.
Sodium nitroprusside indeed results in the release of nitric oxide and is useful in the antepartum period, primarily for
short-term use to control the pressor response to tracheal intubation.